Download as pdf or txt
Download as pdf or txt
You are on page 1of 227

Department of Distance and Continuing Education

University of Delhi
nwjLFk ,oa lrr~ f'k{kk foHkkx
fnYyh fo'ofo|ky;

B.A.
B.A.
(Hons.)
(Hons.)
Political
Economics
Science
Semester-II
Semester-I
Discipline Specific
Course Core Course (DSC-5)
Credits-4
Course Credit-4
INTERMEDIATE MATHEMATICAL
METHODS FOR
UNDERSTANDING ECONOMICS
POLITICAL THEORY
(Department of Economics)

As As
perper
thethe
UGCF-2022
UGCF andand
National
National
Education
Education
Policy
Policy
2020
2020
Intermediate Mathematical Methods for Economics

Editorial Board
Prof. J. Khuntia, Devender

Content Writers
N.Shradha Varma, Devender, Ashish Kumar Garg

Academic Coordinator
Deekshant Awasthi

© Department of Distance and Continuing Education


ISBN: 978-81-19169-61-0
1st edition: 2023
E-mail: ddceprinting@col.du.ac.in
economics@col.du.ac.in

Published by:
Department of Distance and Continuing Education under
the aegis of Campus of Open Learning/School of Open Learning,
University of Delhi, Delhi-110 007

Printed by:
School of Open Learning, University of Delhi

© Department of Distance & Continuing Education, Campus of Open Learning,


School of Open Learning, University of Delhi
Intermediate Mathematical Methods for Economics

• Corrections/Modifications/Suggestions proposed by Statutory Body, DU/Stakeholder/s in the Self


Learning Material (SLM) will be incorporated in the next edition. However, these
corrections/modifications/suggestions will be uploaded on the website https://sol.du.ac.in. Any
feedback or suggestions can be sent to the email- feedbackslm@col.du.ac.in

© Department of Distance & Continuing Education, Campus of Open Learning,


School of Open Learning, University of Delhi
Intermediate Mathematical Methods for Economics

INTERMEDIATE MATHEMATICAL METHODS FOR ECONOMICS


Study Material: Lesson 1-12

TABLE OF CONTENT

Name of Lesson Content Writers Page No


LESSON 1 Vector Spaces And Linear N.Shradha Varma 1-10
Transformations
LESSON 2 System of Linear Equations N.Shradha Varma 11-19
LESSON 3 Eigenvalues and Eigenvectors N.Shradha Varma 20-26
LESSON 4 Matrices & Determinant Devender 27-44
LESSON 5 Diagonalization and Spectral Theorem N.Shradha Varma 45-52
LESSON 6 Differentiability of Function of Two Ashish Kumar Garg 53-79
Variable
LESSON 7 Secibd Irder Deruvatuves Ashish Kumar Garg 80-106
LESSON 8 Implicit Functions Theorem Ashish Kumar Garg 107-122
LESSON 9 Homogeneous Functions Ashish Kumar Garg 123-150
LESSON 10 Convex Sets and their Properties Ashish Kumar Garg 151-172
LESSON 11 Convex and Quasiconvex Functions Ashish Kumar Garg 173-188
LESSON 12 Unconstrained Optimization Ashish Kumar Garg 189-222

About Contributors
Contributor's Name Designation
N.Shradha Varma Assistant Professor, Department of Economics, Maitreyi College,
University of Delhi
Devender Assistant Professor, Department of Economics, School of Open
Learning, University of Delhi
Ashish Kumar Garg Assistant Professor, Ramjas College, University of Delhi

© Department of Distance & Continuing Education, Campus of Open Learning,


School of Open Learning, University of Delhi
Intermediate Mathematical Methods for Economics

LESSON-1

VECTOR SPACES AND LINEAR TRANSFORMATIONS

STRUCTURE
1.1 Learning Objectives
1.2 Introduction
1.3 Vectors
1.3.1 Writing a Vector
1.3.2 Graphical Representation of Vectors
1.3.3 Norm
1.3.4 Orthogonality
1.4 Vector Spaces
1.4.1 Properties of Vectors
1.4.2 Further properties of Vectors
1.5 Linear Transformations
1.5.1 Forms of Linear Transformations
1.6 Summary
1.7 Answers to Intext Questions
1.8 Self-Assessment Questions
1.9 References
1.1 LEARNING OBJECTIVES
After studying this lesson, the students are expected to learn about vectors and sketching the
vectors in a cartesian plane. They will also understand the relevance of vectors in different
fields with examples. The idea of properties of vectors is also taught so that they can apply in
real life examples. Linear transformations is another area in which students are given the
understanding.

1.2 INTRODUCTION
In mathematics and physics, the concept of Vectors and Vector Spaces play an important role.
Vectors are basically the geometric objects that have magnitude and direction also. In physics,

1|Page

© Department of Distance & Continuing Education, Campus of Open Learning,


School of Open Learning, University of Delhi
B.A. (Hons.) Economics

vectors represent concepts like displacement, force, velocity etc. Vectors play an important role
in other related mathematical concepts like matrices.
Vector generally represent the set of numbers of finite sequence. We can calculate the
magnitude of a vector and determine its direction. Generally, vector can be represented
geometrically in cartesian plane by a directed line segment.
In simple words, it is a line segment with a fixed starting point and directed towards a direction
with an arrow. A vector has a point with x and y coordinates respectively in a cartesian plane.
Length of the vector is the magnitude of the vector which is the distance between two points
and direction refers to the movement from one point to another.

1.3 VECTORS
In mathematics and physics, vector space is the space where vectors are presented in one space
with more than one vectors graphically represented in the form of addition of vectors or some
other combination of vectors. The concept of vector spaces is very fundamental for linear
algebra as it helps in understanding the structure of system of linear equations.
Basically, vector space involves more than one vector combined through addition or
multiplication operations with the help of scalars.
Scalar is any mathematical number or in other words, a constant. It is generally considered as
a real number.
1.3.1 Writing a Vector
In general, we call an ordered set of numbers
ⱱ = {a1, a2, a3, a4,.., an}
a vector of order n,
where a1, a2, a3, a4,..,an are called the components of the vector ⱱ.
When the components are written in a row this is called a row vector and it is denoted by ⱱ'.
When the components are written in a column, it is called a column vector and written ⱱ.
23
For instance: ⱱ'= {1,2,3,4} is a row vector and ⱱ = {35} is a column vector
41
The combined presentation of row vector(s) and column vector(s) helps in making a matrix.
2|Page

© Department of Distance & Continuing Education, Campus of Open Learning,


School of Open Learning, University of Delhi
Intermediate Mathematical Methods for Economics

1.3.2 Graphical Representation of Vectors


Let us first consider a vector of order 2, say ⱱ' = {a1, a2}
Note that this is an ordered pair of numbers and thus can be shown geometrically as follows:
Let us draw a two-dimensional space with coordinates X1 and X2, and define our unit points E1
and E2.
If ⱱ' = {3, 2}
we find a point on X1 that will be three units away from 0 and designate it 3.
We likewise find a point on X2 that will be two units from 0 and designate it 2.
Then, a point P (3, 2) will be determined in the plane. The directed segment OP is the vector
ⱱ' = {a1, a2} = {3, 2}
It will also be noted that ⱱ1 = {3, 0} is the vector OA and ⱱ2 = {0, 2} is the vector OB.
Also, ⱱ3= {0, 0} = 0 is called the zero vector or null vector, and in terms of our graph, it will
be the origin 0.

1.3.3 Norm
The length of the vector is referred to as the vector norm or the vector's magnitude. The length
of a vector is a nonnegative number that describes the extent of the vector in space, and is
sometimes referred to as the vector's magnitude or the norm.

3|Page

© Department of Distance & Continuing Education, Campus of Open Learning,


School of Open Learning, University of Delhi
B.A. (Hons.) Economics

Norm of a vector x is denoted as: ‖x‖. The norm of a vector is a measure of its distance from
the origin in the vector space.
The norm of a vector is simply the square root of the sum of each component squared.
Example 1: Find the norm of the following vector.
A=[3,4,5]
The norm of a vector is simply the square root of the sum of each component squared.

∥A∥=√32 + 42 + 52 = √9 + 16 + 25 = √50 = 5√2.


1.3.4 Orthogonality
We say that two vectors are orthogonal if they are perpendicular to each other. i.e., the dot
product of the two vectors is zero. Definition. We say that a set of vectors {v1, v2, ..., vn} are
mutually orthogonal if every pair of vectors is orthogonal.
Two vectors x, y in Rn are orthogonal or perpendicular if x · y = 0. Notation: x ⊥ y means x ·
y = 0.
Definition: the dot product of two vectors is given by x.y where x= (x1, x2, x3) and y = (y1, y2,
y3)
x.y = (x1. y1 + x2. y2 + x3. y3)
When this dot product is equal to 0 (zero value) then we call these vectors as orthogonal to
each other.

INTEXT QUESTIONS

1. Find the scalar product of these two vectors:


(a) v = {2,3,4}
(b) u = {-3,-5,-7}
2. Calculate the value of norm of these two vectors:
(a) A = {1,2,-1}
(b) B = {2,3,0}
(c) C = {3,4,-5}
3. Check if any of these vectors is orthogonal to each other:
(a) x = {3,6,-1}
4|Page

© Department of Distance & Continuing Education, Campus of Open Learning,


School of Open Learning, University of Delhi
Intermediate Mathematical Methods for Economics

(b) y = {2,3,4}
(c) z = {0,0,0}

1.4 VECTOR SPACES


Vector space always comprised of vectors of various kinds. This space comprises of these
vectors with the associative and commutative law of addition of vectors and the associative
and distributive law of multiplication of vectors by scalars.
Mathematically, a vector space has vectors and scalars combined with binary operations of
addition and multiplication. Basically, vectors of different kinds in one space are considered as
vector space.
1.4.1 Properties of Vectors
Following list of properties of vectors play a fundamental role in linear algebra.
Let u, v, w be three vectors in the plane and let c, d
be two scalars.
1. closure under addition: u + v is a vector in the plane
2. Commutative property of addition: u + v = v + u
3. Associate property of addition: (u + v) + w = u + (v + w)
4. Additive identity: (u + 0) = u
5. Additive inverse: u + (−1) u = 0
6. closure under scalar multiplication: c(u) is a vector in the plane.
7. Distributive property of scalar multiplication: c (u + v) = cu + cv
8. Distributive property of scalar multiplication: (c + d) u = cu + du
9. Associate property of scalar multiplication: c(du) = (cd)u
10. Multiplicative identity property: 1(u) = u
1.4.2 Further properties of Vectors
The discussion of vectors in plane can now be extended to a discussion of vectors in n−space.

5|Page

© Department of Distance & Continuing Education, Campus of Open Learning,


School of Open Learning, University of Delhi
B.A. (Hons.) Economics

A vector in n−space is represented by an ordered n−tuple (x1, x2, . . ., xn).


Let u = (u1, u2, . . ., un) and v = (v1, v2, . . ., vn) be
vectors in Rn.
The sum of these two vectors is defined as the vector u + v = (u1 + v1, u2 + v2, . . ., un + vn).
For a scalar c, define scalar multiplications, as the vector cu = (cu1, cu2, . . ., cun).
Also, we define negative of u as the vector
−u = (−1) (u1, u2, . . ., un) = (−u1, −u2, . . ., −un)
The difference
u − v = u + (−v) = (u1 − v1, u2 − v2, . . ., un − vn).
Let v be a vector in Rn and let c be a scalar. Then,
1. v + 0 = v.
(Because of this property, 0 is called the additive identity in Rn.)
2. v + (−v) = 0.
(Because of this property, −v is called the additive inverse of v.)
Further, the additive inverse of v is unique.
3. 0v = 0.
Here the 0 on left side is the scalar zero and the bold 0 is the vector zero in Rn.
4. If cv = 0, then c = 0 or v = 0.
5. −(−v) = v.
Example 1: In E5, let v=(-1,0,2,3,-6), w=(1,0,-2,-3,6), u=(3,7,-6,2,-1) and z=(0,0,0,0,0).
Calculate (a) u+v (b) v+u (c) u+z (d) 3u (e) -7v (f) 3u+7v (g) v+w (h) 0u and (i) 2z.
Solution:
(a) u+v = (3,7, -6,2, -1) + (-1,0,2,3,-6)
=(3+(-1), 7+0, (-6)+2, 2+3, (-1)+(-6))

6|Page

© Department of Distance & Continuing Education, Campus of Open Learning,


School of Open Learning, University of Delhi
Intermediate Mathematical Methods for Economics

= (2,7, -4,5,-7)
(b) v+u = (-1,0,2,3, -6) + (3,7, -6,2, -1) = (2,7,-4,5,-7)
(c) u+z = (3,7, -6,2, -1) + (0,0,0,00) = (3,7,-6,2,-1).
(d) 3u = 3 (3,7, -6,2,-1) = (9,21,-18,6,-3)
(e) -7v = -7(-1,0,2,3, -6) = (7,0,-14,-21,42)
(f) 3u+7v = 3(3,7, -6,2, -1) + 7(-1,0,2,3,-6)
= (2,21, -4,27, -45)
(g) v+w = (-1,0,2,3, -6) + (1,0, -2, -3,6)
= (0,0,0,0,0)
(h) 0u = 0 (3,7, -6,2,-1) = (0,0,0,0,0)
(i) 2z = 2(0,0,0,0,0) = (0,0,0,0,0)

INTEXT QUESTIONS

4. Sketch the following vectors and show the addition of these vectors.
v = {2,3}
w = {3,1}

1.5 LINEAR TRANSFORMATIONS


Analysis of linear functions and conversion to such kind of functional expressions is the main
objective of linear algebra.
Let V and W be real vector spaces (their dimensions can be different),
and let T be a function with domain V and range in W( writtenT : V →
W . We say T is a linear transformation if
(a) For all x, y Є V, T (x + y) = T(x)+ T (y) (T is additive).
(b) For all x Є V, r Є R, T (rx) = rT(x) (T is homogeneous).
If V and W are complex vector spaces, the definition is the same except in(b), r Є C. If V = W,
then T can be called a linear operator.

7|Page

© Department of Distance & Continuing Education, Campus of Open Learning,


School of Open Learning, University of Delhi
B.A. (Hons.) Economics

Example 2: Let V = W = E. Define T (x) = nx, here n is a fixed real number. Show that T is
a linear transformation.
Solution: We must show that T is additive and homogeneous.
1.5.1 Forms of Linear Transformations
For theadditivity, we let x and y be in E and calculate
T (x + y) = n(x + y) = nx + ny
T(x)+ T (y) = nx + ny
Since T (x + y) = T (x) + T(y), we know that T is additive. Also, T is homogeneous
since T(rx) = n(rx) = (nr)x = r(nx)= rT(x)
Thus, T is a linear transformation.
Example 3: Let V = M n 1 and W = M m 1 . Let M be an m × n real matrix.
Define T : V → W by
T (X) = MX
T is linear because by matrix algebra.
T(X + Y ) = M(X + Y ) = MX + MY
T (cX) = M(cX) = c(MX)

INTEXT QUESTIONS

5. Can we have the linear transformation of the sum of these vectors?


V1 = {2,3,4}
V2 = {3,0,1}

1.6 SUMMARY
In this lesson, we have studied the idea of vector and vector spaces where students
have understood the existence of vectors and how vectors play a crucial role in
determining concepts like displacement, distance etc. Linear transformations help in
creating the concept of dependency among vectors. Students have understood the
properties of vectors and of varied forms of linear transformations.

8|Page

© Department of Distance & Continuing Education, Campus of Open Learning,


School of Open Learning, University of Delhi
Intermediate Mathematical Methods for Economics

1.7 ANSWERS TO INTEXT QUESTIONS


1. Scalar Product u.v = {(2*-3), (3*-5), (4*-7)} = {-6, -15, -21}
2. Norm

(a) ||A|| = √12 + 22 + (−1)2 = √1 + 4 + 1 = √6

(b) ||B|| = √22 + 32 + 02 = √4 + 9 = √13

(c) ||C|| = √32 + 42 + (−5)2 = √9 + 16 + 25 =√50 = 5√2

3. (a) x.z = {(3*0), (6*0), (-1*0) = {0,0,0}


(b) y.z = {(2*0), (3*0), (4*0)} = {0,0,0}
Hence, x and z are orthogonal to each other and y and z are orthogonal to each other.

4.
5. v1 + v2 = {5,3,5} Linear transformation of this: e.g. 2(v1+v2) = {2*5, 2*3, 2*5} =
{10,6,10}

1.8 SELF-ASSESSMENT QUESTIONS


1. Let x and y be vectors in En. Define x-y as x+(-y). Setting x = (x1, x2, x3,….,xn) and y
= (y1,y2,y3,…..,yn), write out vector x-y.

9|Page

© Department of Distance & Continuing Education, Campus of Open Learning,


School of Open Learning, University of Delhi
B.A. (Hons.) Economics

2. Find constants c1 and c2 satisfy


(3,5) = c1(1, -2) + c2(2, -3)
3. State commutative laws, associative laws, and distributive laws in your own words.
4. Consider the system of equations AX = 0, where A is in Cnn. Show that the set of all
solutions of AX=0 is a vector space under the usual operations.
5. Suppose that a set of vectors S1= {v1, v2, v3} is a spanning set of a subspace V in R3. Is
it possible that S2={v1}is a spanning set for V?

1.9 REFERENCES
• Sydsaeter, K., Hammond, P. (2002). Mathematics for economics analysis. Pearson
Education.

• Hoy, M., Livernois, J., McKenna, C., Rees, R., Stengos, T, (2001). Mathematics for
Economics, Prentice-Hall India.

10 | P a g e

© Department of Distance & Continuing Education, Campus of Open Learning,


School of Open Learning, University of Delhi
Intermediate Mathematical Methods for Economics

LESSON-2

SYSTEM OF LINEAR EQUATIONS

STRUCTURE
2.1 Learning Objectives
2.2 Introduction
2.3 Types of solutions
2.4 Degrees of freedom
2.5 Redundant Equations
2.6 Homogeneous System
2.6.1 Properties of Homogeneous System
2.7 Summary
2.8 Answers to Intext Questions
2.9 Self-Assessment Questions
2.10References

2.1 LEARNING OBJECTIVES


In this lesson, students will be able to understand the system of linear equations and their
importance in real world. The way of writing the system of different linear equations with
various types of solutions possible in such system. They will also learn about the concept of
degrees of freedom. They will understand homogeneous system of equations and the existence
of solutions.

2.2 INTRODUCTION
An equation in the unknowns x,y,z,... is called linear if both sides of the equation are a sum of
(constant) multiples of x,y,z,..., plus an optional constant.
A system of linear equations is a collection of several linear equations, like
x+5y+3z=6 (2.1)
2x−3y+z=1
3x+y−z=−2.
11 | P a g e

© Department of Distance & Continuing Education, Campus of Open Learning,


School of Open Learning, University of Delhi
B.A. (Hons.) Economics

Linear equations play a vital role in linear algebra and in many fields like economics, physics
etc. Expressing any information with the set of linear equations helps in understanding the
concept much easier and thus solving the system of linear equations provides the values of the
variable.
Finding solution to the linear equations is important as it gives us the value(s) of variable which
makes the entire system of linear equations valid and possible.

2.3 TYPES OF SOLUTIONS


Solution of system of linear equations
• A solution of a system of equations is a list of numbers x,y,z,... that make all of the
equations true simultaneously.
• The solution set of a system of equations is the collection of all solutions.
• Solving the system means finding all solutions with formulas involving some number
of parameters.
NOTE: A system of linear equations need not have a solution. For example, there do not exist
numbers x and y making the following two equations true simultaneously:
5x+2y=3 (2.2)
5x+2y=−3.
In this case, the solution set is empty.
Types of solutions:
1. Consistent
2. Inconsistent
If a system of linear equations has a solution, then it is termed as consistent and if it does not
have any solution possible then it is called inconsistent.
Basically, A solution of a system of equations in n variables is a list of n numbers.
Depending on the kinds of solutions, we can have the possibility of three types of solutions,
namely, Unique, Infinite and No solution.

12 | P a g e

© Department of Distance & Continuing Education, Campus of Open Learning,


School of Open Learning, University of Delhi
Intermediate Mathematical Methods for Economics

Whenever a system of linear equations has a single solution means single and distinct values
of each of the variables of that system then we call that as existence of unique solution.
If the system of linear equations has more than one solution; multiple values of each of the
variables which satisfies the system of linear equations, this is the case of multiple solutions.
And if the system of linear equations doesn’t have any values of the variables possible, then
we call it as the case of no solution. (Case of 2.2).
Example1: For the following system of linear equations, determine if the solution is
consistent or not:
x+ 2y -4z = 2
2x+ 6y-z = 0
x-y-z=3
Once we start solving these equations, we must use either the method of substitution or matrix
algebra where we assign matrices to coefficients, variables, and constants. This requires
detailed understanding of matrices so as of now, we restrict ourselves to method of simple
linear algebra and end up solving all three equations for the values of three variables.
From 3rd equation, we can have x= 3+y+z
Then substitute this value of x into 1st and 2nd equation so that we have
3+y+z+2y-4z = 2
2(3+y+z) +6y -z = 0
Then solving these two equations for two variables; y and z will give us the required solutions
of y and z and finally, substitute the respective values of y and z will give us the value of x.
Finally, one can analyse if the solution is unique, infinitely many or no solution or in other
words, if it is consistent or inconsistent.

INTEXT QUESTIONS

1. Solve the following system of linear equations:


x−7y=−11
5x+2y=−18

13 | P a g e

© Department of Distance & Continuing Education, Campus of Open Learning,


School of Open Learning, University of Delhi
B.A. (Hons.) Economics

2. Check if the following system of linear equation is having consistent solution or not:
7x−8y=−12
−4x+2y=3
3. Find the solution to the following system of 3 linear equations:
x+y+z=6
3x – 2y – z = 4
2x + 3y – 2z = 2

2.4 DEGREES OF FREEDOM


The concept of linear equations is built on the idea that the number of independent variables
define the number of independent equations and if that is not true then we may have serious
issue in deducing the solution to that system.
For example: x+2y+3z = 1 (2.3)
2x+3y-z=2
In this scenario, we have major problem in terms of number of variables not being equal to
number of equations. As we can see that there are only two equations while the number of
variables is three so this brings in the idea of degrees of freedom.
The degrees of freedom for a given problem are the number of independent problem variables
which must be specified to uniquely determine a solution.
For example:
degree of freedom in (2.3) is 3-1 (number of variables -1) or (n-1) = 2.

2.5 REDUNDANT EQUATIONS


In a system of linear equations, the number of equations which are redundant would determine
the type of solution.
Not all systems of equations have a unique solution as in the equations below.
x+y=4
2x + 2y = 8

14 | P a g e

© Department of Distance & Continuing Education, Campus of Open Learning,


School of Open Learning, University of Delhi
Intermediate Mathematical Methods for Economics

The second equation is just the first multiplied by two. Any values of x and y will therefore
satisfy both equations. In this context, the second equation is redundant as it gives no additional
information to the first.
Redundant equations are basically the number of equations in which the variables are not
independently determined.
Degrees of freedom = n-k where n represents the number of equations in the system and k is
the number of dependent variables or number of dependent equations.

2.6 HOMOGENEOUS SYSTEM OF EQUATIONS


A linear system of the form
a11x1+a12x2+……………. +a1nxn=0
a21x1+a22x2+a23x3+……. +a2nxn=0
.
.
.
am1x1+am2x2+am3x3+…..+amnxn=0
This is a system in 'n' unknowns (x₁, x₂, ..., xₙ), and in each equation, the constant term is 0.
When we solve these systems using matrices (by writing augmented matrix), there is no change
in the last column (that is made up of zeros) though when row operations are applied.
A homogeneous system may have two types of solutions: trivial solutions and nontrivial
solutions. Since there is no constant term present in the homogeneous systems, (x₁, x₂, ..., xₙ)
= (0, 0, ..., 0) is obviously a solution to the system and is called the trivial solution (the most
obvious solution).
Notice that homogeneous systems are always consistent. This is because all the variables can
be set equal to zero to satisfy all the equations. This special solution, (0,0,…,0), as we have
called it the trivial solution.
Whenever a homogeneous system has a nontrivial solution, it has infinitely many solutions.
2.6.1 Properties of Homogeneous System
Here are the properties of a homogeneous system of linear equations:
• It always has at least one solution that is called a trivial solution where the value of each
variable is 0.

15 | P a g e

© Department of Distance & Continuing Education, Campus of Open Learning,


School of Open Learning, University of Delhi
B.A. (Hons.) Economics

• If a and b are two solutions of a homogeneous system, then their sum a + b is also a
solution.
• If a is a solution, then ka is also a solution, where k is a scalar.
• A zero vector is always a solution of the homogeneous system.

INTEXT QUESTIONS

1. Solve the following system of homogenous equations.


(i) 3x + 2 y + 7z = 0, 4x - 3y - 2z = 0, 5x + 9 y + 23z = 0
(ii) 2x + 3y - z = 0, x - y - 2z = 0, 3x + y + 3z = 0

2.7 SUMMARY
In this lesson, student has understood how to solve a system of equations and check the
existence of different forms of solutions, namely, unique, many and no solutions. They would
have understood the relevance of degrees of freedom while using the system of equations. The
idea of redundant equations is also made them understood in a system of linear equations.
Homogeneous system of linear equations was also taught along with its possible solution.

2.8 ANSWERS TO INTEXT QUESTIONS


1. x−7y=−11⇒x=7y−11
5x+2y=−18
5(7y−11) +2y=−18
5(7y−11)+2y=−18
35y−55+2y=−18
37y=37
Thus, y=1
So, x=7(1)−11=−4
2. 7x−8y=−12
−4x+2y=3
−4x+2y=3
16 | P a g e

© Department of Distance & Continuing Education, Campus of Open Learning,


School of Open Learning, University of Delhi
Intermediate Mathematical Methods for Economics

2y=4x+3

⇒y=2x+3/2
7x−8y=−12
7x−8(2x+3/2) =−12
7x−8(2x+3/2) =−12
7x−16x−12=−12
−9x=0
x=0
y=2(0) +3/2=3/2
So, this system of equations is consistent.
3. let x = 6-y-z
Substitute x in eq (2) & (3)
3(6-y-z) -2y -z = 4
18-3y-3z-2y-z=4
18-5y-4z=4
14=5y+4z
And 2(6-y-z) + 3y-2z = 0
12 +y-4z = 0
y= 4z-12
So, 14= 5(4z-12) + 4z
14 = 20z – 60 +4z
74 = 24z
z= 37/12

17 | P a g e

© Department of Distance & Continuing Education, Campus of Open Learning,


School of Open Learning, University of Delhi
B.A. (Hons.) Economics

where y=4z-12 gives y = 1/3 after substituting the value of z


and x= 31/12.
4. Let z = k
So, 3x + 2y = -7k
4x – 3y = 2k
Now using the substitution method, we get x = -k and y = -2k
Hence, this homogeneous system has x=-k, y=-2k and z=k
5. Let z = k
So, 2x+3y = k
x-y = 2k
Now using the substitution method, we get y= -3k/5 and x = 7k/5

2.9 SELF-ASSESSMENT QUESTIONS


1. How many solutions does the following system has? Find them all.
4x - 3y - 2z = 0
5x + 9y + 23z = 0
3x + 2y + 7z = 0
2. Which of the following is/are a homogeneous system?
(a) x + y - 1 = 0
2x - 3y = 0
(b) 5x + 3y + z = 0
3x - y = 0
x - 2y + z = 0
3. For the following exercises, determine whether the ordered pair (-1,1) is a solution to
the system of equations:
3x-y=4
x+4y=-3

18 | P a g e

© Department of Distance & Continuing Education, Campus of Open Learning,


School of Open Learning, University of Delhi
Intermediate Mathematical Methods for Economics

4. For the following exercises, write a system of equations to solve each problem. Solve
the system of equations.
(a)A factory has a cost of production C(x) = 150x+15,000 and a revenue function R(x)
= 200x. What is the break-even point?
(b)A performer charges C(x) = 50x+10,000 where x is the total number of attendees at
a show. The venue charges ₹75 per ticket. After how many people buy tickets does the venue
break even, and what is the value of the total tickets sold at that point?
5. Find the equation of the line PQ, where P has coordinates (7, -6) and Q has coordinates
(-3, 2). (ii) Find the point of intersection of PQ and the line 2x - 3y + 1 = 0.

2.10 REFERENCES
• Sydsaeter, K., Hammond, P. (2002). Mathematics for economics analysis. Pearson
Education.

• Hoy, M., Livernois, J., McKenna, C., Rees, R., Stengos, T, (2001). Mathematics for
Economics, Prentice-Hall India

19 | P a g e

© Department of Distance & Continuing Education, Campus of Open Learning,


School of Open Learning, University of Delhi
B.A. (Hons.) Economics

LESSON-3

EIGENVALUES AND EIGENVECTORS

STRUCTURE
3.1 Learning Objectives
3.2 Introduction
3.3 Eigenvalues
3.3.1 Properties of eigenvalues
3.4 Eigenvectors
3.4.1 Characteristic Equations
3.5 Summary
3.6 Answers to Intext Questions
3.7 Self-Assessment Questions
3.8 References

3.1 LEARNING OUTCOMES


This lesson will help the students to learn about eigenvalues and eigenvectors. The concept of
eigenvector has a special space in the area of spectral theory. Students will have the exposure
to the various properties of eigenvalues and their existence in different special cases. They will
learn about characteristic equations and the reason of why we call eigenvalues as characteristic
values.

3.2 INTRODUCTION
Eigenvalues and eigenvectors have a special place in the spectral theory which talks about
casting quadratic forms in infinitely many variables. Basically, Spectral Theory can be said to
refer to the study of eigenvalues and eigenvectors of a matrix. It is of fundamental importance
in many areas for instance physics, chemistry, mathematics etc. These mainly feature in the
area of analysis of linear transformations. We can use vector algebra to understand the concept
of eigenvalues. In matrix algebra, it has its main advantage in matrix diagonalization.

20 | P a g e

© Department of Distance & Continuing Education, Campus of Open Learning,


School of Open Learning, University of Delhi
Intermediate Mathematical Methods for Economics

3.3 EIGENVALUES
If we have a vector from any vector space then we can have a scalar multiplication of that
vector possible and at the same time, we can create a linear transformation of that and if under
any circumstances, these two are equal then we claim to have eigenvector and the
corresponding eigenvalue.
If T is said to be a linear transformation from a vector space V and v is a nonzero vector in V,
then v is an eigenvector of T if T(v) is a scalar multiple of v. This can be written as
T(v) = k v
where k is a scalar.
In the case of matrices, when A is a square matrix and AX=kX where k is some scalar and X
being a vector. When this equation holds for some X and k, we call the scalar k
an eigenvalue of A.
The set of all eigenvalues of an n×n matrix A is denoted by σ(A)and is referred to as
the spectrum of A.
1.3.1 Properties of Eigenvalues
Let A be a matrix with eigenvalues λ1, λ2,…., λn.
The following are the properties of eigenvalues.
➢ The trace of A, defined as the sum of its diagonal elements, is also the sum of all
eigenvalues,

tr (A) = ∑𝑛𝑖=1 𝑎𝑖𝑖 = ∑𝑛𝑖=1 𝜆𝑖

➢ The determinant of A is the product of all its eigenvalues.

det (A) = ∏𝑛𝑖=1 𝜆𝑖 = 𝜆1 𝜆2 … . . 𝜆𝑛

➢ The eigenvalues of the kth power of A; that is the eigenvalues of Ak, for any positive
integer k, are

𝜆1𝑘 𝜆𝑘2 𝜆𝑘3 ……………..𝜆𝑘𝑛


➢ The matrix A is invertible if and only if every eigenvalue is nonzero.
➢ If A is invertible, then the eigenvalues of A-1 are

21 | P a g e

© Department of Distance & Continuing Education, Campus of Open Learning,


School of Open Learning, University of Delhi
B.A. (Hons.) Economics

1 1 1
, 𝜆 , ……., 𝜆
𝜆1 2 𝑛

and each eigenvalue’s geometric multiplicity coincides.


➢ If A is unitary, every eigenvalue has absolute value |λi| = 1.
➢ If A is a n×n matrix and {λ1, λ2,…., λk} are its eigenvalues, then the eigenvalues of
the matrix I + A (where I is the identity matrix) are {λ1+ 1, λ2+1,…., λk+1}.
Special Note on eigenvalues: Eigenvalue can be zero and every singular matrix has a 0
eigenvalue.

INTEXT QUESTIONS

1. If A is an n×n matrix and c is a nonzero constant, compare the eigenvalues of A and


cA.
2. If A is an invertible n×n matrix, compare the eigenvalues of A and A-1
3. Suppose A is an n×n matrix and it satisfies Am = A for some m a positive integer
larger than 1. Show that if λ is an eigenvalue of A then |λ| equals either 0 or 1.

3.4 EIGENVECTORS
Taking the same idea as before, when AX=λX for some X≠0 where X being a vector, then we
call such X an eigenvector of the matrix A.
The eigenvectors of A are basically associated to an eigenvalue.
Hence, if λ1 is an eigenvalue of A where we have AX=λ1X1, we can consider X1 as the
eigenvector.
It is important to note precisely that in order to be an eigenvector, X1 must be nonzero.
Geometrically, the eigenvectors of any matrix are those vectors for which multiplication
by that matrix would result in a vector in the same direction or opposite direction to X. Since
the zero vector has no direction, therefore, we cannot have this possible for the zero vector. So,
0 (zero vector) is never allowed to be an eigenvector.
Suppose X satisfies the below equation. Then
AX−λX=0
(A−λI) X=0 (X≠0)
22 | P a g e

© Department of Distance & Continuing Education, Campus of Open Learning,


School of Open Learning, University of Delhi
Intermediate Mathematical Methods for Economics

(λI−A) X=0
Hence, when we are looking for eigenvectors, we are looking for nontrivial solutions to this
homogeneous system of equations.
The solutions to a homogeneous system of equations include not only the basic solutions, and
the possible linear combinations of those basic solutions.
In this context, we call the basic solutions of the equation (λI−A) X=0 as the basic
eigenvectors. It follows that any (nonzero) linear combination of basic eigenvectors is again
an eigenvector.
1.4.1 Characteristic Equations
Also, to note that if a matrix is not invertible, then its determinant is equal to 0. Therefore, we
can conclude that
det(λI−A) =0
The expression det(λI−A) is a polynomial called the characteristic polynomial of A,
and det(λI−A) =0 is called the characteristic equation.
For this reason, we may also refer to the eigenvalues of A as characteristic values.
Definition says Let A be n×n matrix with characteristic polynomial given by det(λI−A).
Then, the multiplicity of an eigenvalue λ of A is the number of times λ occurs as a root of that
characteristic polynomial.
Let A be n×n matrix.
We have to find the eigenvalues λ of A by solving the equation det(λI−A) =0
(characteristic polynomial).
1. Then for each λ, find the basic eigenvectors X≠0 by finding the basic solutions
to (λI−A) X=0.

INTEXT QUESTIONS

4. Let A and B be invertible n×n matrices which commute. That is, AB=BA. Suppose X is an
eigenvector of B. Show that then AX must also be an eigenvector for B.

5. Show that if AX=λX and AY=λY, then whenever k,p are scalars,
A(kX+pY)=λ(kX+pY)
Does this imply that kX+pY is an eigenvector? Explain.
Example 1: Find the eigenvalues and eigenvectors of the following matrix.

23 | P a g e

© Department of Distance & Continuing Education, Campus of Open Learning,


School of Open Learning, University of Delhi
B.A. (Hons.) Economics

Solution:

Example 2: Consider the matrix

for some variable ‘a’. Find all values of ‘a’ which will prove that A has eigenvalues 0, 3, and
−3.
Solution:
Let p (t) be the characteristic polynomial of A, i.e., let p (t) = det (A − tI) = 0. By expanding
along the second column of A − tI, we can obtain the equation

= (3 − t) [(−2 −t) (−1 − t) − 4] + 2[(−2 − t) a + 5]


= (3 − t) (2 + t + 2t + t2 −4) + 2 (−2a − ta + 5)
= (3 − t) (t2 + 3t − 2) + (−4a −2ta + 10)
= 3t2 + 9t − 6 − t3 − 3t2 + 2t − 4a − 2ta + 10

24 | P a g e

© Department of Distance & Continuing Education, Campus of Open Learning,


School of Open Learning, University of Delhi
Intermediate Mathematical Methods for Economics

= −t3 + 11t − 2ta + 4 − 4a


= −t3 + (11 − 2a) t + 4 − 4a
For the eigenvalues of A to be 0, 3 and −3, the characteristic polynomial p (t) must have roots
at t = 0, 3, −3.
This implies p (t) = –t (t − 3) (t + 3) =–t(t2 − 9) = –t3 + 9t
Therefore, −t3 + (11 − 2a) t + 4 − 4a = −t3 + 9t.
For this equation to hold, the constant terms on the left and right-hand sides of the above
equation must be equal. This means that 4 − 4a = 0, which implies a = 1.
Hence, A has eigenvalues 0, 3, −3 precisely when a = 1.

1.5 SUMMARY
This lesson has helped the students to learn about eigenvalues and their working. They have
also been taught how to solve for eigenvalues and to consider the existence of such values
under special cases. Eigenvectors were also discussed in this lesson and they were supported
by the concept of characteristic equations and values. Students had the good exposure of
various examples to understand these concepts in detail.

1.6 ANSWERS TO INTEXT QUESTIONS


1. When AX=λX then cAX=cλX and so the eigenvalues of cA are just cλ where λ is an
eigenvalue of A.
2. In the case of inverse, A-1λX = AA-1X = X so A-1X = λ-1X. Thus, the eigenvalues of
A-1 are just λ-1 where λ is an eigenvalue of A.
3. Let X be the eigenvector. Then AmX=λmX, AmX=AX=λX and so λm=λ. Hence
if λ≠0, then λm−1=1and so |λ|=1|.
4. BAX=ABX=AλX=λAX. Assumption BX=λX.
5. The formula follows from properties of matrix multiplications. However, this vector
might not be an eigenvector because it might be equal 0 and eigenvectors cannot
equal 0.

25 | P a g e

© Department of Distance & Continuing Education, Campus of Open Learning,


School of Open Learning, University of Delhi
B.A. (Hons.) Economics

1.7 SELF-ASSESSMENT QUESTIONS


1. Find the eigenvalues and eigenvectors of the following matrix:
1 90 0
[0 −2 0 ]
3 89 −2
One eigenvalue is 1.
2. Is it possible for a non-zero matrix to have only 0 as an eigenvalue?
3. Find the eigenvalues and eigenvectors of the following matrix:
−6 −92 12
[0 0 0]
−2 −31 4
One eigenvalue is -2.
4. Find the eigenvalues and eigenvectors of the following matrix:
9 2 8
[ 2 −6 −2]
−8 2 −5
One eigenvalue is -3.
5. Find the eigenvalues and eigenvectors of the following matrix:
6 76 16
[−2 −21 −4]
2 64 17
One eigenvalue is -2.

1.8 REFERENCES
• Sydsaeter, K., Hammond, P. (2002). Mathematics for economics analysis. Pearson
Education.

• Hoy, M., Livernois, J., McKenna, C., Rees, R., Stengos, T, (2001). Mathematics for
Economics, Prentice-Hall India.

26 | P a g e

© Department of Distance & Continuing Education, Campus of Open Learning,


School of Open Learning, University of Delhi
Intermediate Mathematical Methods for Economics

LESSON-4

MATRICES & DETERMINANT

STRUCTURE
4.1 Learning Objective
4.2 Introduction
4.3 Definition of A Matrix
4.4 Equality of Matrices
4.5 Addition of Matrices
4.6 Multiplication of A Matrix by A Solar
4.7 Multiplication of Matrices
4.8 Transpose of A Matrix
4.9 Determinant
4.10 Intext questions
4.11 Symmetric and Skew-Symmetric Matrices
4.12 Self- assessment Questions
4.13 Solutions to Intext Questions
4.14 References

4.1 LEARNING OBJECTIVE


After reading this lesson you will be able to understand

• What is a matrix

• Basic operations of matrices like addition, multiplication and Matrix transpose

• Symmetric and Skew-Symmetric Matrices

4.2 INTRODUCTION
You are already familiar with addition and multiplication of matrices. We shall now talk about
some important types of matrices such as symmetric and skew-symmetric matrices, Hermitian
and skew-Hermitian matrices etc., elementary operations on a matrix inverse of a matrix, rank
27 | P a g e

© Department of Distance & Continuing Education, Campus of Open Learning,


School of Open Learning, University of Delhi
B.A. (Hons.) Economics

of a matrix, and characteristic equation of a matrix. In the end we shall apply some of these
concepts to solutions of systems of linear equations. However, before we do so, we shall briefly
recapitulate the main facts about addition and multiplication of matrices.

4.3 DEFINITION OF A MATRIX


Let S be any set. A set of mn elements arranged in a rectangular array of m rows and
n column as

 a11 a12 a1n 


 a21 a22 a2 n 
 
 
 am1 am 2 amn 

is called an m × n (“m by n”) matrix over S. A matrix may be represented by the symbols || aij
||, [aij], [aij] or by a single letter such as A. The aij’s in a matrix are called the element of the
matrix. The indices i and j of an element indicate respectively the row and the column in which
the elements aij is located.

Since we shall be dealing only with matrices over the set of complex number therefore,
we shall use the word “matrix” so as to mean “matrix over C” throughout, unless we state to
the contrary.
The 1 × n matrices are called row vectors and the m × l matrices are called column vectors.
The m × n matrix whose elements are 0 is called the null matrix (or zero matrix) of the type m
× n. It is usually denoted by Om × n or simply by O if there is no possibility of confusion.

If the number of rows and the number of columns of a matrix are equal (say each equal to
n) the matrix is said to be a square matrix of order n or an n-row square matrix. The elements
a11 a22, ... amn of a square matrix A are said to constitute the main diagonal of A. A square
matrix in which all the off-diagonal elements are zero is called a diagonal matrix. Thus, an n-
rowed square matrix [aij] is a diagonal matrix if aij = 0 whenever i + j. An n-rowed diagonal
matrix is often written as
dia. [a11, a22, ..., amn]

A diagonal matrix in which all the diagonal elements are equal is called a scalar matrix.
In other words, an n-rowed square matrix [aij] is a scalar matrix if for some number k.

28 | P a g e

© Department of Distance & Continuing Education, Campus of Open Learning,


School of Open Learning, University of Delhi
Intermediate Mathematical Methods for Economics

aij = 
k , when i = j,
0, when i  j.

A scalar matrix in which each diagonal element is unity, is called a unit matrix. Thus, an
n-rowed square matrix [aij] is called a unit matrix if

aij = 
1, whenever i = j,
0, whenever i  j.

The n-rowed unit matrix is usually denoted by In (or simply by I if there is no possibility
of confusion).
The matrix of elements which remain after deleting any number of rows and columns of
a matrix A is called a sub matrix of A.
Illustrations :

0 0 0 0
1.  0 0 0 0  is the 3 × 4 null matrix.
0 0 0 0
 

 3 1 2
2.  5 4 7  is a 3-rowed square matrix. 3, 4, 8 constitute the main diagonal of this
 −1 2 8 
 
matrix.

1 0 0
3.  0 7 0  is a 3-rowed diagonal matrix.
 0 0 −2 
 

1 0 0
4.  0 7 0  is a 3-rowed scalar matrix.
 0 0 −2 
 

1 0 0
5.  0 1 0  is the 3-rowed unit matrix. We denote it by I3.
0 0 1
 

1 8 7
 3 4
6. The matrix   is submatrix of  −2 3 4  because it can be obtained from the
 6 −5 
 1 6 −5 
latter by deleting the first row and the first column.
29 | P a g e

© Department of Distance & Continuing Education, Campus of Open Learning,


School of Open Learning, University of Delhi
B.A. (Hons.) Economics

4.4 EQUALITY OF MATRICES


Two matrices A = [aij] and B = [bij] are said to be equal if (i) they are comparable, i.e., the
number of rows in B is the same as the number or rows A, and the number of columns in B is
the same as the number of columns in A; (ii) aij = bij = for every pair of subscripts i and j.

Thus, for example, the matrices   and  


3 7 1 5 4
  3 6 2  are not comparable; the matrices
8 9  
 −1 2 3  and  4 3 6  are comparable but not equal; the matrices  2 4 7  and
 3 1 0 1 8 9  6 3 −1
     
 4 4 7
  are equal.
 2.3 9 −1

From the definition of equality of matrices, it can be easily verified that if A, B, and C be
any matrices, then.
(i) A = A (reflexivity)
(ii) A = B B = A (symmetry)
(iii) if A = B and B = C, the A = (transitivity)
The above statements (i)—(iii) can be summed up by saying that the relation of equality
in the set of all matrices is an equivlance relation.

4.5 ADDITION OF MATRICES


If A = [aij], and B = [bij] be two matrices of the same type, say m × n, their sum is the m × n
matrix C = [cij], where cij = aij + bij for every pair of subscripts i and j. In other words,

 a11 a12 a1n   b11 b12 b1n 


a a22 a2 n  b b22 b2 n 
If A =  21  B =  21 
   
 a m1 am 2 amn   bm1 bm 2 bmn 
 

 a11 + b11 a12 + b12 a1n + a1n 


then A + B =  a21 + b21 a22 + b22 a2n + b2n 
 
 am1 + am1 am 2 + bm 2 amn + bmn 

−2 1
A = 
3  4 2 5
Illustrations. If  and B =  
4 2 −1  3 0 6
30 | P a g e

© Department of Distance & Continuing Education, Campus of Open Learning,


School of Open Learning, University of Delhi
Intermediate Mathematical Methods for Economics

−2 + 4 1 + 2 3 + 5   2 3 8
A + B =  =
2 + 0 −1 + 6   7 2 5 
then
 4+3

Properties of matrix addition


Addition of matrices, has the following properties:
(i) Addition of matrices is associative. That is, if A, B, and C be matrices of the same
type, then A + (B + C) = (A + B) + C
(ii) Addition of matrices is commutative. That is, if A and B be matrices of the same type,
then A + B = B + A.
(iii) Property of zero matrix. If A be an m × n matrix and 0 denotes the m × n zero matrix,
then A + 0 = 0 + A = A.
(iv) Negative of a matrix. If a be an m × n matrix, there exists an m × n matrix B, called
the negative of the matrix A, such that A + B = B + A = 0.

4.6 MULTPLICATION OF A MATRIX BY A SOLAR


If A = [aij] be an m × n matrix, and k be any complex number, then kA is defined to be the m
× b matrix whose (i, j)th the elements is k aij. The matrix kA is called the scalar multiple of A
by k. The following properties of scalar multiplication are worth nothing :
(i) If A and B are comparable matrices, and k is any complex number, then
k(A + B) = kA + kB.
(ii) If A be any matrix, and k and l be any two complex numbers, then
(k + 1) A = kA + lA.
(iii) If A be any matrix, and k and l be any two complex numbers, then
k(lA) = (kl) A.
(iv) For every matrix A,
1 A =A

4.7 MULTIPLICATION OF MATRICES


Definition 1. Let A = [aij] and B = [bij] be m × n and n × p matrices respectively. The m × p
matrix [cij], where

31 | P a g e

© Department of Distance & Continuing Education, Campus of Open Learning,


School of Open Learning, University of Delhi
B.A. (Hons.) Economics

n
cij = aij, bij + ai2 + b2j + ain bnj =  aij bkj ,
k =1

is called the product of the matrices A and B and is denoted by AB.


The above definition expresses two facts:
(i) We can talk of the product AB if two matrices of and only if the number of columns
of A is equal to the number of rows of B. In the case this condition is satisfied, we
say that A and B are conformable to multiplication.

(ii) If A and B are conformable to multiplication, the (i, j)th element of the matrix AB is
obtained by multiplying the elements of the ith row of A by the corresponding
elements of the jth column of B and adding the products. The sum so obtained is the
desired (ij)th elements of AB.
Properties of matrix multiplication
The following are some of the important properties of matrix multiplication:
(i) Matrix multiplication is associative. That is, if A, B, and C be of suitable sizes for the
products A(BC) and (A + B) C to exist, then A(BC) = (AB) C.
(ii) Matrix multiplication is not commutative. That is, given two matrices A and B, AB =
BA is not always true. It is important to note here that for pair of matrices A and B
several different possibilities arise.
(a) Neither of the products AB and BA exits.
(b) only one of the products AB and BA exist and the other one does not exist.
(c) both AB as well as BA exist but they are of different type.
(d) both AB as well as BA exist and are of the same type, but are not equal
(e) AB = BA.
All the above possibilities do exist for certain pairs of matrices.
The important thing to note is that the phase ‘matrix multiplication is not
commutative’ means that AB is not always equal to BA. It does not exclude the
possibility of AB and BA being equal in some cases.

32 | P a g e

© Department of Distance & Continuing Education, Campus of Open Learning,


School of Open Learning, University of Delhi
Intermediate Mathematical Methods for Economics

(iii) Multiplication of matrices is distributive with respect to addition, i.e.,


A(B + C) = AB + BC
and (B + C)D = BD + CD
where A, B, C and D are of the suitable sizes for the above relations to be meaningful.
(iv) Multiplication by the unit matrix. If A be any m × n matrix, then Im A = A = A In

4.7.1 POSITIVE INTEGRAL POWERS OF SOURCES MATRIX

If A be an n-rowed square matrix, and n be a positive integer, then An is defined by setting A1


= A, Ak + 1 = AkA. By the principle of finite induction this defines An for all positive integer
An.

If A be an n-rowed square matrix and p and q be positive integers, it can be easily shown
that

Ap · Aq = Ap + q, (Ap)q = Apq.

4.8 TRANSPOSE OF A MATRIX


Consider the matrices.

 3 6
3 1 −4 
A =   , B =  1 0 .
6 0 7   −4 7 
 

Matrix A is a 2 × 3 matrix, and matrix B is a 3 × 2 matrix. Also, the first column of B is the
same as the first row of A, and the second columne of B is the same as the second row of A. In
other words, B is the matrix obtained from A by writing the row of A as columns. We say that
the matrix B is the transpose of A.
Defintion 2. If A = [aij] be an m × n matrix, then the n × m matrix B = [bij], such that bij = aji
is called the transpose of A and is deonted by At.
From the above defintion we find that
(i) the transpose of an m × n matrix is an n × m matrix.

(ii) the (i, j)th element of At is the (j, i)th element of A.

33 | P a g e

© Department of Distance & Continuing Education, Campus of Open Learning,


School of Open Learning, University of Delhi
B.A. (Hons.) Economics

4.9 DETERMINANT
The determinant of a matrix is a scalar value that can be calculated from the elements of the
matrix. The determinant is only defined for square matrices (matrices with the same number
of rows and columns).
There are different methods to calculate the determinant of a matrix, but one common way is
to use the cofactor expansion method. Here are the steps:
If the matrix is 1x1, then the determinant is just the value of the single element in the matrix.
If the matrix is 2x2, then the determinant is calculated as follows:
determinant = (a11 x a22) - (a12 x a21)
where a11, a12, a21, and a22 are the elements of the matrix.
For larger matrices, the cofactor expansion method can be used. Choose a row or a column of
the matrix, and for each element in that row or column, calculate the minor, which is the
determinant of the submatrix obtained by deleting the row and column that contain that
element. Multiply each minor by the corresponding element, and then sum the results,
alternating the signs of the terms.
For example, if we choose the first row of a 3x3 matrix, we can calculate the determinant as
follows:
determinant = a11 x minor11 - a12 x minor12 + a13 x minor13
where minor11 is the determinant of the submatrix obtained by deleting the first row and first
column, minor12 is the determinant of the submatrix obtained by deleting the first row and
second column, and minor13 is the determinant of the submatrix obtained by deleting the first
row and third column.
Repeat step 3 for each row or column until the determinant is calculated.
Note that the determinant of a matrix can be used to determine if a matrix is invertible. If the
determinant is zero, then the matrix is not invertible.
Example
here is an example of calculating the determinant of a 3x3 matrix using the cofactor
expansion method:
2 3 1
Consider the matrix 𝐴= 4 −1 2
0 5 −3
34 | P a g e

© Department of Distance & Continuing Education, Campus of Open Learning,


School of Open Learning, University of Delhi
Intermediate Mathematical Methods for Economics

We will calculate the determinant by expanding along the first row:


det = 2 * minor11 - 3 * minor12 + 1 * minor13
where minor11, minor12, and minor13 are the determinants of the submatrices obtained by
deleting the first row and the corresponding column.

To calculate the minors, we first need to find the 2x2 determinants of each of the
submatrices:
−1 2
minor11 = => (-1) (-3) - (2)(5) = -7
5 −3
4 2
minor12 = => (4) (-3) - (2)(0) = -12
0 −3
4 −1
minor13 = => (4)(5) - (-1) (0) = 20
0 5
Now we can substitute the values into the formula for the determinant:
det = 2 * (-7) - 3 * (-12) + 1 * (20) = 34
Therefore, the determinant of the given matrix is 34.

4.10 INTEXT QUESTIONS


3 −1 2   −1 3 5 
Q 1. Let A =   and B =  6 2 1  .
 0 4 7   

Compute(i) At

(ii) (At)t,

(iii) Bt,

(iv) (A + B)t,

(v) At + Bt,

(vi) (3A)t,

35 | P a g e

© Department of Distance & Continuing Education, Campus of Open Learning,


School of Open Learning, University of Delhi
B.A. (Hons.) Economics

(vii) 3At.

Remark. In the above example we find that ( At )t = A, ( A + B)t = At + Bt , and (3A)t = 3At.
These results are only special cases of the following theorem:
Theorem 1. If At and Bt are transposes of A and B respectively, then.

(i) (At)t = A

(ii) C A + Bt = At + Bt, if A and B are comparable.

(iii) kAt = kAt, k being any complex number

Proof. (i) Let A = [aij] be an m × n matrix. Then At is an n × m matrix and (At)t is an m × n


matrix. The matrices (At)t and A are, therefore, comparable.

Also, (i, j)th element of (At)t

= (j, i)th element of At

= (i, j)th element of A

Since the matrices (At)t and A are comparable and their (i, j)th elements are equal, threfore,
(At)t = A.
(ii) Let A = [aij] and B = [bij] be m × n matrices. Since A and B are both m × n matrices,
therefore A + B exists and is an m × n matrix. Consequently (A + B)t is an n × m matrix,
therefore.

Again, At and Bt are both n × m matrices, so that At + Bt also exists and is an n × m matrix.

The matrices (A + B)t and At + Bt are both of the type n × m, and are therefore comparable.

Also, (i, j)th element (A + B)t = (j, i)th element of A + B


= aji + bji

= (j, i)th element of At + (i, j)th element Bt

= (j, i)th element of (A + B)t


36 | P a g e

© Department of Distance & Continuing Education, Campus of Open Learning,


School of Open Learning, University of Delhi
Intermediate Mathematical Methods for Economics

Thus, the matrices (A, B)t and At + Bt are comparable, and their corresponding elements
are equal.

Hence (A + B)t + At + Bt .

(iii) Let A = [aij] be an m × n matrix. kA is an m × n matrix and therefore (kA)t is an n ×


m matrix. Also, At being an n × m matrix, kAt is an n × m matrix. The matrices (kA)t and kAt
are both of type n × m, and are, therefore, comparable. Also (i, j)th element of (kA)t = (j, i)th
element of kA
= kaji

= k[(i, j)th elements of At]

Since the matrices (kA)t and kAt are comparable and their (i, j)th elements are equal,
therefore.

(kAt) = kAt.

Remark. If At = B, the Bt = (At)t = A, i.e., if B is the transpose of A, then A is the transpose


of B.

1 3 4
3 −1 2 
Example 2. If A =   and B =  −2 1 −1 ,
 1 0 −3   0 −4 2 
 

Compute (AB)t and BtAt.

1 3 4
3 −1 2  
Solution. AB =   −2 1 −1
 1 0 −3   0 −4 2 
 

= 
5 0 17 
,
 15 −2 
1

5 1
so that (AB)t =  0 15 
 17 −2 
 

37 | P a g e

© Department of Distance & Continuing Education, Campus of Open Learning,


School of Open Learning, University of Delhi
B.A. (Hons.) Economics

 1 −2 0  3 1
Also, Bt =  3 1 −4 , A =  −1 0  .
 t
 4 −1 2   2 −3 
   

5 1
Therefore, Bt At =  0 15  .
 17 −2 
 

Remark. In the above example (AB)t = BtAt. This is of course, only a particular case of
the general result which we state and prove in the following theorem.
Theorem 2. If A and B be matrices conformable to multiplication, then.

(AB)t = BtAt
Proof. Let A = [aij] and B[bij] be m × n and n × p matrices particularly.

The At = [cij], where cij = aji, is an n × m matrix.

Bt = [cij], where dij = bji, is an p × n matrix. The matrices (AB)t and


BtAt are both of type of p × m and are therefore comparable.

Also (i, j)th elements of (AB)t

= (j, i)th element of (AB)t


n
=  a jk bki
k =1

n
=  a jk bki
k =1

n
=  dij ckj
k =1

= (i, j)th element of BtAt

Since the matrices (AB)t and BtAt are of the same type, and their (ij)th elements are equal,
therefore, (AB)t = BtAt

38 | P a g e

© Department of Distance & Continuing Education, Campus of Open Learning,


School of Open Learning, University of Delhi
Intermediate Mathematical Methods for Economics

4.11 SYMMETRIC AND SKEW-SYMMETRIC MATRICES


Consider the matrices.

3 0 2 0 1 3
A = 0 4 −1 , B =  −1 0 −4 
 2 −1 5   −3 4 0 
   

In matrix A, (1, 2)th element is equal to (2, 1)th element, (1, 3)th element is equal to (3, 1)th
element, and (2, 3)th element is equal to (3, 2)th element. Because of these properties we say
that matrix A is symmetric.

In matrix B, (2, 1)th element is the negative of (1, 2)th element, (3, 1)th element is the negative
of the (1, 3)th element, (3, 2)th element is the negative of the (2, 3)th element, and (1, 1)th
element, (2, 2)th element, and (3, 3)th element are own negatives, (i.e., they are all zero).
Because of these properties we say that matrix B is skew-symmetric.
Symmetric and skew-symmetric matrices play useful (an important) roles in the theory of
matrices.
Definition 3. A square matrix A = [aij] is said to be symmetric if aij = aji for all i and j.

Illustrations 1. The matrices

 4 1 − i 2  a h g 1 0 0
1 − i 3 7  h b f and  0 1 0  are all symmetric.

 2 7 i   g c  0 0 1
 f  

2. The matrices

 0 i 1 + i  0 3 6 − i
 −i 0 −3  and  −3 0 −4  are both skew-symmetric.
 −1 − i 3   −6 + i 4 0 
 0  

3. The matrices

 1 i 1 + i  0 1 −2 
 −i 2 6i and  1 0
 i  are neither symmetric nor skew-symmetric.
 −1 − i 6i   0 
 3  2 −i

39 | P a g e

© Department of Distance & Continuing Education, Campus of Open Learning,


School of Open Learning, University of Delhi
B.A. (Hons.) Economics

In the following theorem we state and prove some basic facts about symmetric and skew-
symmetric matrices.
Theorem 3.

(i) A necessary and sufficient condition for a matrix A to be symmetric is that At = A.

(ii) A necessary and sufficient condition for a matrix A to be skew-symmetric is that At


= –A.
(iii) The diagonal elements of a skew-symmetric matrix are all zero.
Proof. (i) Necessity. Let A = [aij] be a symmetric matrix. Since A is symmetric, it must be
square matrix, say of order n, At is then also of order n, so that At and A are comparable. Also,
(i, j)th element of At = aji = aij = (i, j)th elements of A.

Therefore At = A.

Sufficiently. Let A = [aij] be an m × n matrix such that At = A. Since A is an m × n matrix.,


therefore At is an m × n matrix. Since At and A are equal matrices, they are comparable, so that
n = m, and consequently A is a square matrix. Also, as given, (i, j)th element of At = (i, j)th,
which gives aji = aij.

Since A is a square matrix such that aij = aij for all i and j, therefore A is symmetric.

(ii) Necessity. Let A = [aij] be a skew-symmetric matrix. Since A is skew-symmetric, it


must be a square matrix, say of order n. At is then also of order n, so that At and A are
comparable. Also, (i, j)th element of At = aji = –aij = (i, j)th element of –A threfore At = A.

Sufficiently. Let A = [aij] b an m × n matrix such that At = –A. Since A is m × n matrix,


therefore At is an n × m matrix and –A is an m × n matrix. Since the matries At and –A are
equal, they are comparable, so that n = m, and consequently A is a square matrix. Also (i, j)th
element of At = –[(i, j)th element of A], which gives aji = –Aij.

We shall now state and prove a theorem which assures us that every square matrix can be
expressed uniquely as a sum of a symmetric and skew-symmetric matrix.

40 | P a g e

© Department of Distance & Continuing Education, Campus of Open Learning,


School of Open Learning, University of Delhi
Intermediate Mathematical Methods for Economics

Theorem 4. Every square matrix can be expressed uniquely as the sum of a symmetric and a
skew-symmetric matrix.
Proof. Let A be an n-rowed square matrix.
Let A = X + Y,
where X is an n-rowed symmetric, and Y is an n-rowed skew-symmetric matrix. Taking
the transpose of both sides of (i), we have

At = ( X + Y )t = X t = Y t = X − Y ...
(2)

since X is symmetric and Y is skew-symmetric so that Xt = X and Yt = –Y


From (1) and (2), we get.
1
X= ( A + At ), ...
2
(3)
1
Y= ( A − At ) ...
2

(4)
We have shown that if A is expressible as the sum of a symmetric matrix X and a
skew-symmetric matrix Y, then X and Y must be given by (3) and (4). This establishes the
uniqueness of the part. To demonstrate the existence of a symmetric matrix, X and a skew-
symmetric matrix Y such that A = X + Y, we have only to see that if we write.
1 1
X= ( A + At ), Y = ( A − At ),
2 2
t
1 
then Xt =  ( A + At )  ,
2 

t
1 
=  ( A + At )  ,
2 

1 t
= ( A + A),
2

41 | P a g e

© Department of Distance & Continuing Education, Campus of Open Learning,


School of Open Learning, University of Delhi
B.A. (Hons.) Economics

= X,
t
1 
Yt =  ( A − At ) 
2 

t
1 
=  ( A − At ) 
2 

1 t
= ( A − A)
2

= –Y,
so that X is an n × n symmetric matrix and Y is an n × n skew-symmetric matrix.
Furthermore, X + Y = A, which completes the proof.
Example 3. Express the matrix.

2 6 5
A = 3 1 4
 9 −1 7 
 

as the sum of symmetric and a skew-symmetric matrix.


Solution. Let
2 6 5
 3 1 4  = X + Y,
 9 −1 7 
 

where X is a 3-rowed symmetric matrix and Y is a 3-rowed skew-symmetric matrix.

Taking transposes of both sides (1), and using the facts that (X + Y)t, =, we have
 2 3 6
 6 1 −1  = X – Y.
5 4 7
 

From (1) and (3) we find that


 2 3 6  0 3 −4 
2X =  6 1 −1  2Y =  −3 0 5  ,
5 4 7  4 −5 0 
   

42 | P a g e

© Department of Distance & Continuing Education, Campus of Open Learning,


School of Open Learning, University of Delhi
Intermediate Mathematical Methods for Economics

so that
 9   3 
2 7  0 −2 
2 2
9 3  3 5
X= 1 , Y =  − 0 
2 2  2 2
 3   5 
7 7  2 − 0
 2   2 

Verification. Since Xt = X, Yt = Y, therefore matrix X is symmetric, and the matrix Y is


skew-symmetric. Also, by actual addition we find that X + Y = A.

4.12 SELF-ASSESSMENT QUESTIONS

1. For each of the following matrix’s A, verify that At = A :


 1 2 −1   −1 6 −7   a h g
 2 0 −4  ,  6 3 8  ,  h b f
 −1 −4 3   −7 8 −5   g c 
     f

2. For each of the following matrix’s A, verify that At = –A.


0 i 1  0 3 1 + i   0 −3 2i 
 −i 0 −2  ,  − 3 0 − 4  ,  3 0 4 
 −1 2 0   −1 − i 4 0   −2i −4 0 
  

1 2 −2 
1 
3. If A = 2 1 2  , verify that A At = At = A = I3.
3  2 −2 −1 
 

4. If A be any square matrix, verify that the matrix A + At is symmetric and the matrix A —
At is skew-symmetric.
 3 1 −7 
5. Express the matrix  2 4 8  as X + Y where X is symmetric, and Y is skew-symmetric.
 6 −1 2 
 

4.13 ANSWER TO INTEXT QUESTIONS


3 0
Solution. (i) At =  −1 4 
2 7 

43 | P a g e

© Department of Distance & Continuing Education, Campus of Open Learning,


School of Open Learning, University of Delhi
B.A. (Hons.) Economics

3 −1 2 
(ii) (At)t =  
0 4 7

 −1 6 
(iii) Bt =  3 2 .
 5 1
 
2 6
(iv) (𝐴 + 𝐵)𝑡 = (2 6)
7 8
2 6
𝑡 𝑡
(vi) 𝐴 + 𝐵 = (2 6)
7 8
9 0
(vii) 3𝐴𝑡 = (−3 12).
6 21
9 0
𝑡
(vii) (3𝐴) = (−3 12)
6 21
4.14 REFERENCES

• Sydsaeter, K., Hammond, P. (2002). Mathematics for economics analysis. Pearson


Education.

• Hoy, M., Livernois, J., McKenna, C., Rees, R., Stengos, T, (2001). Mathematics for
Economics, Prentice-Hall India.

44 | P a g e

© Department of Distance & Continuing Education, Campus of Open Learning,


School of Open Learning, University of Delhi
Intermediate Mathematical Methods for Economics

LESSON 5
DIAGONALIZATION AND SPECTRAL THEOREM

STRUCTURE

5.1 Learning Objectives


5.2 Introduction
5.3 Diagonalization
5.3.1 Diagonalization Theorem
5.3.2 Properties of Diagonalizable Matrices
5.3.3 Powers of Diagonalizable Matrices
5.4 Spectral Theorem
5.5 Summary
5.6 Answers to Intext Questions
5.7 Self-Assessment Questions
5.8 References

5.1 LEARNING OBJECTIVES

This lesson will help the students to learn the concept of diagonalization in the matrix system.
They will learn about the basic definition and the process of diagonalization. They would learn
about the theorem and other various associated concepts about the same. They would also
understand computing the powers of a matrix by diagonalization and some discussion on
properties of diagonal matrices. They would also be taught the idea of spectral theorem in this
discussion of diagonal matrices.

5.2 INTRODUCTION

The process of converting a matrix into its diagonal form is known as diagonalization. In a
diagonal matrix, eigenvalues are clearly represented. A Diagonal Matrix is a square matrix in
which all the elements are zero except the principal diagonal elements. It is basically a
construction of a diagonal matrix (with nonzero entries only on the main diagonal) that is
similar to a given matrix. Symmetric matrices are diagonalizable by orthogonal matrices.

5.3 DIAGONALIZATION

To understand this process, let us start with its definition,


45 | P a g e

© Department of Distance & Continuing Education, Campus of Open Learning,


School of Open Learning, University of Delhi
B.A. (Hons.) Economics

Definition: An n×n matrix A is diagonalizable that is, if there exists an


invertible n×n matrix C and a diagonal matrix D such that
A=CDC−1
Any diagonal matrix is D is diagonalizable because it is similar to itself.
For example,
2 0 0 2 0 0
[0 3 0] = 𝐼3 . [0 3 0] 𝐼3−1
0 0 4 0 0 4
Hence, we can say, any diagonal matrix D is diagonalizable, as it is similar to itself.
If any matrix A is diagonalizable then P-1AP is a diagonal matrix, say, D which will further
gives AP = PD so writing P = [a1, a2, a3] (these are basically the diagonal elements of D) as
a block matrix of its column vectors. The column vectors of P are right eigenvectors of A, and
the corresponding diagonal entry is the corresponding eigenvalue. This is the necessary and
sufficient condition for diagonalizability.
5.3.1 Diagonalization Theorem
An n X n matrix A is diagonalizable if and only if A has n linearly independent eigenvectors.
In fact, A = PDP-1, with D a diagonal matrix, if and only if the columns of P are n linearly
independent eigenvectors of A.
In this case, the diagonal entries of D are eigenvalues of A that correspond, respectively, to the
eigenvectors in P.
J
| | | λ1 ⋯ 0
P = (v1 v2 v3 ) and D = [[ ⋮ ⋱ ⋮ ]]
| | | 0 ⋯ λn

where
v1 , v2 , v3 are linearly independent eigenvectors and λ1 , … … . , λn are the corresponding
eigenvalues in the same order.
We can also say that A (nXn) matrix with n distinct eigenvalues is diagonalizable.

46 | P a g e

© Department of Distance & Continuing Education, Campus of Open Learning,


School of Open Learning, University of Delhi
Intermediate Mathematical Methods for Economics

There are generally many different ways to diagonalize a matrix, corresponding to different
orderings of the eigenvalues of that matrix. The important thing is that the eigenvalues and
eigenvectors have to be listed in the same order.
Special Notes:
1. One property of eigenvalues and eigenvectors is that eigenvectors of different
eigenvalues are linearly independent. Therefore, if all eigenvalues of the matrix are
unique the matrix is diagonalizable.
2. Another way to determine whether a matrix can be factored into a diagonal matrix is
by using the algebraic and geometric multiplicities. The algebraic multiplicity is the
number of times an eigenvalue is repeated, and the geometric multiplicity is the
dimension of the nullspace of matrix (A-λI). Thus, if the algebraic multiplicity is equal
to the geometric multiplicity for each eigenvalue, the matrix is diagonalizable.
2 0 0
Example1: Let A = [ 1 4 −1]. Find an invertible matrix P and a diagonal matrix D such
−2 −4 4
that P-1AP = D.
Solution: First, we will find the eigenvalues of A. To do so, we solve det (𝜆𝐼 − 𝐴) as follows.
1 0 0 2 0 0
det (𝜆 [0 1 0] − [ 1 4 −1]) which gives
0 0 1 −2 −4 4
𝜆1 = 2, 𝜆2 = 2, 𝜆3 = 6.
Next, we need to find the eigenvectors. We first find the eigenvectors for 𝜆1 = 2, 𝜆2 = 2.
Solving (2I-A) X = 0 to find the eigenvectors, we find that the eigenvectors are
−2 1
t[ 1 ] + s[0]
0 1
where t, s are scalars.
Hence there are two basic eigenvalues and eigenvectors which are given by:
−2 1
X1 = [ 1 ] , X2 = [0]
0 1

47 | P a g e

© Department of Distance & Continuing Education, Campus of Open Learning,


School of Open Learning, University of Delhi
B.A. (Hons.) Economics

0
Now for 𝜆3 = 6, we have X3 = [ 1 ]
−2
So, we construct P as
−2 1 0
P=[ 1 0 1 ]
0 1 −2
And also, we can find P-1
−1⁄ 1⁄ 1⁄
4 2 4
-1 1
P = ⁄2 1 1⁄
2
1 1⁄ −1⁄
[ ⁄4 2 4]
Thus, the diagonal matrix is given by inserting all the respective matrix in the expression P-
1
AP
2 0 0
P-1AP = [0 2 0]
0 0 6
5.3.2 Properties of Diagonalizable Matrices
The characteristics of this type of matrices are:
If matrix A is diagonalizable, then so is any power of A.
1. Almost all matrices can be diagonalized over a complex environment. Although some
matrices can never be diagonalized.
2. If matrix P is an orthogonal matrix, then matrix A is said to be orthogonally
diagonalizable and, therefore, the equation can be rewritten:
A = PDPt
3. A matrix is diagonalizable by a unitary matrix if and only if it is a normal matrix.
4. Given two diagonalizable matrices, they commute if and only if they can be
diagonalized simultaneously, that is, if they share the same orthonormal basis of
eigenvectors.

IN-TEXT QUESTIONS

1. Find the characteristic polynomial, eigenvalues and

48 | P a g e

© Department of Distance & Continuing Education, Campus of Open Learning,


School of Open Learning, University of Delhi
Intermediate Mathematical Methods for Economics

3 1 1
an invertible 2 matrix P such that P-1AP is a diagonal matrix if A = [−4 −2 −5].
2 2 5
7 𝑎 𝑏
2. For what values of a, b and c is the matrix A = [0 2 𝑐 ] diagonalizable?
0 0 3
3. Suppose that A is a diagonalizable n×n matrix and has only 1 and −1 as eigenvalues.
Show that A2=In, where In is the n×n identity matrix.
5.3.3 Powers of Diagonalizable Matrices
Multiplying diagonal matrices together just multiplies their diagonal entries.
If A= CDC -1, where D is a diagonal matrix, then An= CDnC-1.
𝑥 0 0
A = C[0 𝑦 0] C-1
0 0 𝑧
𝑥𝑛 0 0
An = C[ 0 𝑦 𝑛 0 ]C-1
0 0 𝑧𝑛
1 −6 𝑘
Example 2: Find a formula for [ ] by diagonalizing the matrix.
2 −6
Solution: The eigenvalues are -3 and -2, and the diagonalized form of the matrix is
1 −6 3 2 −3 0 −1 2
[ ]=[ ][ ][ ]
2 −6 2 1 0 −2 2 −3
It follows that
𝑘
1 −6 𝑘 3 2 (−3) 0 −1 2
[ ] =[ ][ 𝑘] [ 2 ]
2 −6 2 1 0 (−2) −3
5 0
Example 3: Let D = [ ]. Compute D2. In general, what is Dk where k is a positive integer.
0 4

5 0 5 0 25 0
Solution: D2 = [ ][ ]=[ ]
0 4 0 4 0 16
𝑘
So, in general, Dk = [5 0]
0 4𝑘

49 | P a g e

© Department of Distance & Continuing Education, Campus of Open Learning,


School of Open Learning, University of Delhi
B.A. (Hons.) Economics

6 −1 1 1
Example 4: Let A = [ ]. Find a formula for Ak, given that A=PDP-1 where P = [ ],
2 3 1 2
5 0 2 −1
D=[ ], P-1 = [ ]
0 4 −1 1
Solution: A2 = (PDP-1) (PDP-1) = PD(P-1P). DP-1
= PDDP-1 = PD2P-1
Also, A3 = A2.A = (PD2P-1) (PDP-1) = PD2(P-1P). DP-1
= PD3P-1
𝑘 𝑘
Thus, Ak = PDkP-1 = [
1 1 5𝑘
][ 0 ] [ 2 −1] = [ 2.5𝑘 − 4 𝑘 −5𝑘 + 4𝑘 ]
1 2 0 4𝑘 −1 1 2.5 − 2.4 −5𝑘 + 2. 4𝑘
IN-TEXT QUESTIONS

4. Suppose that A and P are 3×3 matrices and P is invertible matrix. If P-1AP =
1 2 3
[0 4 5] then find all the eigenvalues of the matrix A2.
0 0 6

1 2
5. Let A = [ ] so diagonalize the matrix A2 – 3A.
4 3
5.4 SPECTRAL THEOREM

The spectral theorem states that every real symmetric matrix is diagonalizable. This theorem
is extremely useful because computations involving a diagonalizable matrix can often be
reduced to much simpler computations involving the corresponding diagonal matrix. The
concept of diagonalization is relatively straightforward for operators on finite-dimensional
vector spaces but requires some modification for operators on infinite-dimensional spaces. In
general, the spectral theorem identifies a class of linear operators that can be modelled
by multiplication operators, which are as simple as one can hope to find.
The spectral theorem provides a sufficient criterion for the existence of a particular canonical
form. Specifically, the spectral theorem states that if M equals the transpose of M,
then M is diagonalizable: there exists an invertible matrix C such that C-1MC is
a diagonal matrix.
A matrix M with entries in R is called symmetric if M=MT. The Spectral theorem states that
any symmetric matrix is diagonalizable.

50 | P a g e

© Department of Distance & Continuing Education, Campus of Open Learning,


School of Open Learning, University of Delhi
Intermediate Mathematical Methods for Economics

5.5 SUMMARY

This lesson was discussing the idea of diagonalization of matrices which involves converting
a matrix into its diagonal form. There are some special circumstances where diagonalization is
not possible and also the diagonalization methods give specific results. With the help of
properties of diagonalizable matrices, students were able to get the detailed analysis of these
kinds of matrices. They were also taught power of a diagonalizable matrix. After that spectral
theorem was discussed in the area of diagonalization.

5.6 ANSWERS TO INTEXT QUESTIONS

1. A has three eigenvalues 𝜆1 = 1, 𝜆2 = 2 and 𝜆3 = 3.


Eigenvectors are the homogeneous systems
(I3 - A) X = 0, (2I3 - A) X = 0 and (3I3 - A) X = 0 have basic solutions as
1 1 0
X1 = [−3] , X2 = [−1] , X3 = [−1], respectively.
1 0 1
1 1 0
So, matrix P = [X1, X2, X3] = [−3 −1 −1] which diagonalizes A.
1 0 1
2. Note that the matrix A is an upper triangular matrix. Hence the eigenvalues of A are
diagonal entries 7,2,3. So, the 3×3matrix A has three distinct eigenvalues. This implies
that A is diagonalizable. Hence, regardless of the values of a,b,c, the matrix A is always
diagonalizable. Thus, a,b,c can take arbitrary values.
3. Since A is diagonalizable, there exists an invertible matrix P such that P-1AP=D,
where D is a diagonal matrix. Since A has only ±1 as eigenvalues, we can choose P so
that the diagonal entries of D are either ±1. Then we have A=PDP-1 and
A2=(PDP-1) (PDP-1) = (PD)(DP−1) =PD2P-1.
Note that D2=I2 and thus we have A2=I2 as required.
4. Let B=P-1AP. Since B is an upper triangular matrix, its eigenvalues are diagonal
entries 1,4,6 (or we can find them easily). Since A and B=P-1AP have the same
eigenvalues, the eigenvalues of A are 1,4,6. It follows that all the eigenvalues of A2
are 12,42,62, that is, 1,16,36.

51 | P a g e

© Department of Distance & Continuing Education, Campus of Open Learning,


School of Open Learning, University of Delhi
B.A. (Hons.) Economics

1
5. For matrix A, eigenvalues are -1 and 5 with corresponding eigenvectors as u = [ ]
−1
1 1 1 −1 1 2 1 1
and v = [ ] So diagonal matrix is D = S-1AS which gives [ ] [ ][ ]
2 −1 2 4 3 −1 2
−1 0
so D = [ ]
0 5
Since D = S-1AS so A = SDS-1
So, A2 = A.A = (SDS-1) (SDS-1) = SD2S-1
Thus, A2-3A = (SD2S-1) – 3(SD2S-1)
= S (D2-3D) S-1.
(−1)2 0 −1 0
Hence, S-1(A2-3A) S = D2-3D = [ ]−3[ ].
0 (5)2 0 5
1 0 3 0 4 0
=[ ]+ [ ]= [ ]
0 25 0 −15 0 10
5.7 SELF-ASSESSMENT QUESTIONS

1. Diagonalize the 2×2 matrix A=[1 2] by finding a nonsingular matrix S and a diagonal
2 1
matrix D such that S-1AS=D.
2. Let A, B be matrices. Show that if A is diagonalizable and if B is similar to A, then B is
diagonalizable.
3. (a) Is every diagonalizable matrix invertible?

(b) Is every invertible matrix diagonalizable?


7 𝑎 𝑏
4. For which values of constants a,b and c is the matrix A= [0 2 𝑐 ] diagonalizable?
0 0 3
5. Suppose that A is a diagonalizable n×n matrix and has only 1and −1 as eigenvalues. Show
that A2=In where In is the n×n identity matrix.

5.8 REFERENCES

• Sydsaeter, K., Hammond, P. (2002). Mathematics for economics analysis. Pearson


Education.
• Hoy, M., Livernois, J., McKenna, C., Rees, R., Stengos, T, (2001). Mathematics for
Economics, Prentice-Hall India.

52 | P a g e

© Department of Distance & Continuing Education, Campus of Open Learning,


School of Open Learning, University of Delhi
Intermediate Mathematical Methods for Economics

LESSON-6

DIFFERENTIABILITY OF FUNCTION OF TWO VARIABLE

STRUCTURE
6.1 Learning Objectives
6.2 Introduction
6.3 Differentiability of Function of Two Variables
6.3.1 Functions of Two Variables
6.3.2 Neighbourhood of Point
6.3.3 Simultaneous and Iterated Limits
6.3.4 Continuity of Function of Two Variable
6.3.5 Partial Derivatives
6.3.6 Differentiability
6.4 Summary
6.5 Glossary
6.6 References
6.7 Suggested Readings
6.1 LEARNING OBJECTIVES

One of the main objectives to learn differentiability of function of two real variable is that
because we use it in economics theory as demand and Income function of two real variable and
other place like data science, statistics, management etc.

6.2 INTRODUCTION

A real valued function of 𝑛-variables is a function 𝑓: 𝐷 → ℝ, where the domain 𝐷 is a subset


of ℝ𝑛 . So: for each (𝑥1 , 𝑥2 , … , 𝑥𝑛 ) in 𝐷, the value of 𝑓 is a real number 𝑓(𝑥1 , 𝑥2 , … , 𝑥𝑛 ). For
example, the volume of a cylinder: 𝑉 = 𝜋𝑟 2 ℎ (i.e., 𝑉 = 𝐹(𝑟, ℎ) ) is a function of two variables.
If 𝑓 is defined by a formula, we usually take the domain 𝐷 to be as large as possible. For
example, if 𝑓 is a function defined by 𝑓(𝑥, 𝑦) = 9 − cos (𝑥) + sin (𝑥 2 + 𝑦 2 ), we have a
function of 2 variables defined for all (𝑥, 𝑦) ∈ ℝ2 . So 𝐷 = ℝ2 . However, if 𝑓 is defined by

53 | P a g e

© Department of Distance & Continuing Education, Campus of Open Learning,


School of Open Learning, University of Delhi
B.A. (Hons.) Economics

1
𝑓(𝑥, 𝑦, 𝑧) =
√𝑥 2 + 𝑦 2 + 𝑧 2

Then 𝑓 is a function of 3 variables, defined whenever √𝑥 2 + 𝑦 2 + 𝑧 2 ≠ 0. This is all


(𝑥, 𝑦, 𝑧) ∈ ℝ3 except for (𝑥, 𝑦, 𝑧) = (0,0,0).
Likewise, a multivariable function of 𝑚-variables is a function 𝑓: 𝐷 → ℝ𝑛 , where the domain
𝐷 is a subset of ℝ𝑚 . So: for each (𝑥1 , 𝑥2 , … , 𝑥𝑛 ) in 𝐷, the value of 𝑓 is a vector
𝑓(𝑥1 , 𝑥2 , … , 𝑥𝑛 ) ∈ ℝ𝑛 .
For example:
1 An object rotating around the origin in the 𝑥𝑦-plane (say at distance 5 from the origin)
will have its position described by the function 𝑓(𝑡) = (5cos (𝑡),5sin (𝑡)). This is a
function from ℝ to 𝐑2 .
2 An object spiralling around the 𝑥 axis (again at distance 5 from the axis) and
travelling at constant speed might have its position given by 𝑓(𝑡) =
(𝑡, 5cos (𝑡),5sin (𝑡)). This function is from ℝ2 to R2 .
6.3 DIFFERENTIABILITY OF FUNCTION OF TWO VARIABLES
In this section we will discuss
• Functions of Two Variables
• Neighbourhood of Point
• Simultaneous and Iterated Limits
• Continuity of Function of Two Variable
• Partial Derivatives
• Differentiability
6.3.1 Function of Two Variables:
Let R denote the set of real numbers.
We have 𝐑2 = 𝐑 × 𝐑 = {(𝑥, 𝑦): 𝑥 ∈ 𝐑, 𝑦 ∈ 𝐑}.
Let 𝐴 ⊆ R2 .
If 𝑓: 𝐴 → 𝐑, then 𝑓 is called a real valued function of two real variables.
The set 𝐴 is called the domain of this function and the range of this function is some subset of
𝐑.

54 | P a g e

© Department of Distance & Continuing Education, Campus of Open Learning,


School of Open Learning, University of Delhi
Intermediate Mathematical Methods for Economics

Definition. We say that 𝑧 = 𝑓(𝑥, 𝑦) is a real valued function of two independent real variables
𝑥 and 𝑦, if for each pair of values of 𝑥 and 𝑦 of a certain set {(𝑥, 𝑦)} over which the point (𝑥, 𝑦)
ranges, there exists a unique real value of 𝑧.
For example, 𝑧 = 𝑥 2 + 𝑦 2 − 𝑥𝑦, 𝑧 = 𝑥sin 𝑦 + 𝑦cos 𝑥 etc., are real valued functions of two
real independent variables 𝑥 and 𝑦.
6.3.2 Neighbourhood of a Point:
Rectangular neighbourhood of a point.
Let (𝑎, 𝑏) be any point of R2 .
The set 𝑁(𝑎, 𝑏) = {(𝑥, 𝑦): 𝑥 ∈ 𝐑, 𝑦 ∈ 𝐑, 𝑎 − ℎ < 𝑥 < 𝑎 + ℎ, 𝑏 − 𝑘 < 𝑦 < 𝑏 + 𝑘} is called a
rectangular neighbourhood of the point (𝑎, 𝑏), where ℎ and 𝑘 are arbitrarily small positive real
numbers.
In particular,
𝑁(𝑎, 𝑏) = {(𝑥, 𝑦): 𝑥 ∈ 𝐑, 𝑦 ∈ 𝐑, 𝑎 − 𝛿 < 𝑥 < 𝑎 + 𝛿, 𝑏 − 𝛿 < 𝑦 < 𝑏 + 𝛿}
is a square neighbourhood of the point (𝑎, 𝑏), where 𝛿 is an arbitrarily small positive real
number.
A neighbourhood of a point (𝑎, 𝑏) is denoted by 𝑁(𝑎, 𝑏).
We shall often write the word neighbourhood in the abbreviated form as nhd or nbd.
Circular neighbourhood of a point.
The set 𝑁(𝑎, 𝑏) = {(𝑥, 𝑦): 𝑥 ∈ 𝐑, 𝑦 ∈ 𝐑 and √(𝑥 − 𝑎)2 + (𝑦 − 𝑏)2 < 𝛿} is called a circular
neighbourhood of the point (𝑎, 𝑏), where 𝛿 is an arbitrarily small positive real number.
It can be seen that every circular neighbourhood of a point contains a rectangular
neighbourhood and vice versa. But not all points.
Deleted neighbourhood of a point. If from the neighbourhood 𝑁(𝑎, 𝑏) of the point (𝑎, 𝑏), we
delete the point (𝑎, 𝑏), then the remaining set is called a deleted nbd of (𝑎, 𝑏).
6.3.3 Simultaneous and Iterated Limits:
For a function 𝑓(𝑥, 𝑦) of two variables 𝑥 and 𝑦, we can define several kinds of limits. If (𝑎, 𝑏)
is the limiting point of a set of values in two-dimensional space, then we have the limits.
lim 𝑓(𝑥, 𝑦), lim lim 𝑓(𝑥, 𝑦), lim f(x, y)
(𝑥,𝑦)→(𝑎,𝑏) 𝑥→𝑎 𝑦→𝑏 𝑦→𝑏,𝑥→𝑎

55 | P a g e

© Department of Distance & Continuing Education, Campus of Open Learning,


School of Open Learning, University of Delhi
B.A. (Hons.) Economics

The first type of limit is known as 'simultaneous limit' or 'double limit' and the last two types
are known as 'iterated limits' or 'repeated limits'.
The simultaneous limit, lim(𝑥,𝑦)→(𝑎,𝑏) 𝑓(𝑥, 𝑦) is also written as 𝑥 → 𝑎𝑓(𝑥, 𝑦).

The notion of an iterated limit is nothing but a limit of a limit and it can be obtained as in the
case of a function of a single variable. However, the notion of simultaneous limit is quite
different from that of the limit of a function of single variable.
Simultaneous limit.
Definition. We say that the simultaneous limit of 𝑓(𝑥, 𝑦) exists and is equal to 𝐴 as (𝑥, 𝑦) →
(𝑎, 𝑏), if for every 𝜀 > 0, there exists 𝑎𝛿 > 0 such that
|𝑓(𝑥, 𝑦) − 𝐴| < 𝜀
for all values of 𝑥 and 𝑦 in the nbd of (𝑎, 𝑏) defined by
|𝑥 − 𝑎| < 𝛿, |𝑦 − 𝑏| < 𝛿
Non-existence of simultaneous limit.
For the existence of simultaneous limit, not only must we have same limiting value if the
variable point (𝑥, 𝑦) approaches the limiting point (𝑎, 𝑏) through any set of values dense at the
point, but we must also have the same limiting value as the variable point approaches its
limiting position along any curve whatsoever.
Thus, if we can find two methods of approach to the limiting point which give different limiting
values then we can conclude that the simultaneous limit does not exist.
SOLVED EXAMPLES
𝑥𝑦 3
Ex. 1. Show that the simultaneous limit lim(𝑥,𝑦)→(0,0) 𝑥 2 +𝑦 6 does not exist.

Sol. First let (𝑥, 𝑦) approach (0,0) along the line 𝑦 = 𝑥 which is a line through the origin. For
this, we first put 𝑦 = 𝑥 in the function and then allow 𝑥 to approach 0. We thus get
𝑥4 𝑥2
lim = lim =0
𝑥→0 𝑥 2 + 𝑥 6 𝑥→0 1 + 𝑥 4

Again, let (𝑥, 𝑦) approach (0,0) along the curve 𝑥 = 𝑦 3 . For this, we put 𝑥 = 𝑦 3 in the function
and then allow 𝑦 to approach zero. In this case, we get
𝑦6 1
lim 6 6
= .
𝑦→0 𝑦 + 𝑦 2
56 | P a g e

© Department of Distance & Continuing Education, Campus of Open Learning,


School of Open Learning, University of Delhi
Intermediate Mathematical Methods for Economics

Since two methods of approach to the limiting point give different limiting values, the
simultaneous limit does not exist.
Note1. It can be seen that if we approach the origin along any line 𝑦 = 𝑚𝑥, the limit comes
out to be zero. But we cannot conclude that the simultaneous limit exists and is 0 as we have
1
seen that the limit along 𝑥 = 𝑦 3 comes out to be 2.

Note 2. The existence of the simultaneous limit, lim(𝑥,𝑦)→(𝑎,𝑏) 𝑓(𝑥, 𝑦) implies that the single
limits lim𝑥→𝑎 𝑓(𝑥, 𝑏), lim𝑦→𝑏 𝑓(𝑎, 𝑦) also exist. However, it does not follow that the single
limits lim𝑥→𝑎 𝑓(𝑥, 𝑦), lim𝑦→𝑏 𝑓(𝑥, 𝑦) exist for 𝑦 ≠ 𝑏, 𝑥 ≠ 𝑎 respectively.
It is shown by the following example:
1
Ex 2: Show that the simultaneous limit lim(𝑥,𝑦)→(0,0) 𝑦sin exists and is equal to 0 but the
𝑥
1
single limit lim𝑥→0 𝑦1 sin 𝑥 (𝑦1 ≠ 0) does not exist.
1
Sol. First we shall show that the simultaneous limit lim(𝑥,𝑦)→(0,0) 𝑦sin exists. Let 𝜀 > 0 be
𝑥
given. Taking 𝛿 = 𝜀, we see that for all 𝑥, 𝑦 satisfying the inequalities 0 < |𝑥| < 𝛿, 0 < |𝑦| <
𝛿, we have
1 1 1
|𝑦sin − 0| = |𝑦sin | = |𝑦| |sin |
𝑥 𝑥 𝑥
≤ |𝑦|
1
[∵ |sin | ≤ 1]
𝑥
< 𝛿 = 𝜀.
1
∴ lim(𝑥,𝑦)→(0,0) 𝑦sin = 0 i.e., the simultaneous limit exists.
𝑥

But, for any constant value of 𝑦 = 𝑦1 ≠ 0, we get


1 1
lim𝑥→0 𝑦1 sin = 𝑦1 lim𝑥→0 sin 𝑥, which does not exist.
𝑥

Repeated limits or Iterated limits. If a function 𝑓 is defined in some deleted neighbourhood


of (𝑎, 𝑏), then the limit lim𝑦→𝑏 𝑓(𝑥, 𝑦), if it exists, is a function of 𝑥, say 𝜙(𝑥). If then the limit,
lim𝑥→𝑎 𝜙(𝑥) exists and is equal to 𝜆, we write
lim lim 𝑓(𝑥, 𝑦) = 𝜆
𝑥→𝑎 𝑦→𝑏

and say that 𝜆 is a repeated limit of 𝑓 as 𝑦 → 𝑏, 𝑥 → 𝑎.

57 | P a g e

© Department of Distance & Continuing Education, Campus of Open Learning,


School of Open Learning, University of Delhi
B.A. (Hons.) Economics

If we change the order of taking the limits, we get the other repeated limit
lim lim 𝑓(𝑥, 𝑦) = 𝜇 (say)
𝑦→𝑏 𝑥→𝑎

when first 𝑥 → 𝑎 and then 𝑦 → 𝑏.


These two limits may or may not be equal.
Note. In case the simultaneous limit exists, these two repeated limits if they exist are
necessarily equal but the converse is not true. However, if the repeated limits are not equal, the
simultaneous limit cannot exist.
𝑦−𝑥 1+𝑥
Ex. 3. Let 𝑓(𝑥, 𝑦) = 𝑦+𝑥 . 1+𝑦 , (𝑥, 𝑦) ≠ (0,0). Show that the two repeated limits exist at the
origin but are unequal.
1+𝑥
Sol. We have lim𝑥→0 lim𝑦→0 𝑓(𝑥, 𝑦) = lim𝑥→0 ( ) = −1.
1
1
Again, lim𝑦→0 lim𝑥→0 𝑓(𝑥, 𝑦) = lim𝑦→0 (1+𝑦) = 1.
Thus the two repeated limits exist at the origin but are not equal.
Now we give a theorem which is only a sufficient but not a necessary condition for the
interchange of the order of iterated limits.
Theorem 1. Let the simultaneous limit 𝐥𝐢𝐦(𝒙,𝒚)→(𝒂,𝒃) 𝒇(𝒙, 𝒚) exist and be equal to A and let
the limit 𝐥𝐢𝐦𝒙→𝒂 𝒇(𝒙, 𝒚) exist for each constant value of 𝒚 in the nhd of 𝒃 and likewise let
the limit 𝐥𝐢𝐦𝒚→𝒃 𝒇(𝒙, 𝒚) exist for each constant value of 𝒙 in the nhd of 𝒂. Then

𝐥𝐢𝐦 𝐥𝐢𝐦 𝒇(𝒙, 𝒚) = 𝐥𝐢𝐦 𝐥𝐢𝐦 𝒇(𝒙, 𝒚) = 𝑨


𝒙→𝒂 𝒚→𝒃 𝒚→𝒃 𝒙→𝒂

Proof. Since the limit lim𝑥→𝑎 𝑓(𝑥, 𝑦) exists for each value of 𝑦 in the nhd of 𝑏, we get a set of
these limiting values which defines a function of 𝑦, say 𝐹(𝑎, 𝑦). Thus, we can write
lim 𝑓(𝑥, 𝑦) = 𝐹(𝑎, 𝑦),
𝑥→𝑎

where 𝐹(𝑎, 𝑦) may or may not be identical with 𝑓(𝑎, 𝑦).


Let 𝜀 > 0 be given. By virtue of (1), there exists 𝛿1 > 0 such that for each value of 𝑦 in a nhd
of 𝑏, say in the nhd defined by |𝑦 − 𝑏| < 𝛿0 , we get
|𝐹(𝑎, 𝑦) − 𝑓(𝑥, 𝑦)| < 𝜀/2
for all 𝑥 satisfying the inequality |𝑥 − 𝑎| < 𝛿1 .
Again, since the simultaneous limit of 𝑓(𝑥, 𝑦) at (𝑎, 𝑏) is 𝐴, there exists 𝛿2 > 0 such that

58 | P a g e

© Department of Distance & Continuing Education, Campus of Open Learning,


School of Open Learning, University of Delhi
Intermediate Mathematical Methods for Economics

|𝑓(𝑥, 𝑦) − 𝐴| < 𝜀/2


for all 𝑥, 𝑦 satisfying |𝑥 − 𝑎| < 𝛿2 , |𝑦 − 𝑏| < 𝛿2 .
Taking 𝛿 = min(𝛿0 , 𝛿2 ), we get for all 𝑦 satisfying |𝑦 − 𝑏| < 𝛿,
|𝐹(𝑎, 𝑦) − 𝐴| = |𝐹(𝑎, 𝑦) − 𝑓(𝑥, 𝑦) + 𝑓(𝑥, 𝑦) − 𝐴|
≤ |𝐹(𝑎, 𝑦) − 𝑓(𝑥, 𝑦)| + |𝑓(𝑥, 𝑦) − 𝐴|
𝜀 𝜀
< + = 𝜀, by (2) and (3).
2 2
It gives that lim𝑦→𝑏 𝐹(𝑎; 𝑦) = 𝐴

or lim𝑦→𝑏 lim𝑥→𝑎 𝑓(𝑥, 𝑦) = 𝐴, using (1).

Similarly, we can prove that


lim𝑥→𝑎 lim𝑦→𝑏 𝑓(𝑥, 𝑦) = 𝐴.
Thus lim(𝑥,𝑦)→(𝑎,𝑏) 𝑓(𝑥, 𝑦) = lim𝑥→𝑎 lim𝑦→𝑏 𝑓(𝑥, 𝑦) = lim𝑦→𝑏 lim𝑥→𝑎 𝑓(𝑥, 𝑦).

Ex. 4. Give an example to show that the order of iterated limits can be interchanged although
the simultaneous limit does not exist.
𝑥𝑦
Sol. We consider the function 𝑓(𝑥, 𝑦) = 𝑥 2+𝑦 2 for simultancous and iterated limits at (0,0).
First let (𝑥, 𝑦) approach (0,0) along the line 𝑦 = 𝑥. Putting 𝑦 = 𝑥 and then making 𝑥
approach 0, we get
𝑥2 1
lim 2 2
=
𝑥→0 𝑥 + 𝑥 2

Again, let (𝑥, 𝑦) approach (0,0) along 𝑦-axis. Putting 𝑥 = 0, we get


0
lim = 0.
𝑦→0 0 + 𝑦 2

Since two methods of approach give different results, the simultaneous limit does not exist.
For iterated limits, we get
𝑥𝑦 𝑥⋅0
lim lim = lim =0
𝑥→0 𝑦→0 𝑥 2 + 𝑦 2 𝑥→0 𝑥 2 + 0

and

59 | P a g e

© Department of Distance & Continuing Education, Campus of Open Learning,


School of Open Learning, University of Delhi
B.A. (Hons.) Economics

𝑥𝑦 0⋅𝑦
lim lim = lim = 0.
𝑦→0 𝑥→0 𝑥 2 +𝑦 2 𝑦→0 0 + 𝑦 2

Thus, the order of iterated limits can be interchanged.


𝑥 2 −𝑦 2
Ex. 5. Show that lim(𝑥,𝑦)→(0,0) 𝑥 𝑥 2+𝑦 2 = 0.
Sol. Let 𝜀 > 0 be given.
For all (𝑥, 𝑦) ≠ (0,0), we have
(𝑥 2 − 𝑦 2 ) (𝑥 2 − 𝑦 2 ) |𝑥𝑦|
|𝑥𝑦 2 2
− 0| = |𝑥𝑦 2 2
| = |𝑥 2 − 𝑦 2 | ⋅ 2
𝑥 +𝑦 𝑥 +𝑦 𝑥 + 𝑦2

Now for all 𝑥 and 𝑦, we have (|𝑥| − |𝑦|)2 ≥ 0


⇒ |𝑥|2 + |𝑦|2 − 2|𝑥|. |𝑦| ≥ 0
⇒ 𝑥 2 + 𝑦 2 ≥ 2|𝑥𝑦|
|𝑥𝑦| 1
⇒ ≤ , if (𝑥, 𝑦) ≠ (0,0).
𝑥2 + 𝑦2 2
∴ from (1) and (2), for all (𝑥, 𝑦) ≠ (0,0), we have
(𝑥 2 − 𝑦 2 ) 1 1 1
|𝑥𝑦 2 − 0| ≤ |𝑥 2 − 𝑦 2 | ≤ |𝑥 2 + 𝑦 2 | = (𝑥 2 + 𝑦 2 )
𝑥 + 𝑦2 2 2 2
< 𝜀, if 𝑥 2 < 𝜀 and 𝑦 2 < 𝜀 i.e., if |𝑥| < √𝜀 and |𝑦| < √𝜀.

Now if we take 𝛿 = √𝜀, then we see that for any given 𝜀 > 0, there exists

𝑥𝑦(𝑥 2 −𝑦 2 )
𝛿 > 0 such that | − 0| < 𝜀, whenever |𝑥| < 𝛿 and |𝑦| < 𝛿.
𝑥 2 +𝑦 2

𝑥2 − 𝑦2
∴ lim 𝑥𝑦 = 0.
(𝑥,𝑦)→(0,0) 𝑥2 + 𝑦2
Alternative Solution: Take any given 𝜀>0. Putting x=r cos 𝜃, y= r sin 𝜃, we have
𝑥2 − 𝑦2 𝑥2 − 𝑦2
|𝑥𝑦 − 0| = |𝑥𝑦 | = |𝑟 2 sin 𝜃cos 𝜃cos 2𝜃|
𝑥2 + 𝑦2 𝑥2 + 𝑦2

60 | P a g e

© Department of Distance & Continuing Education, Campus of Open Learning,


School of Open Learning, University of Delhi
Intermediate Mathematical Methods for Economics

𝑟2 𝑟2
=| sin 4𝜃| = |sin 4𝜃|
4 4
2
𝑟

4
𝑥2 + 𝑦2
=
4
𝑥2 𝜀 𝑦2 𝜀
< 𝜀, if < and < i.e., if |𝑥| < √2𝜀 and |𝑦| < √2𝜀.
4 2 4 2
Now if we take 𝛿 = √2𝜀, then we see that for any given 𝜀 > 0, there exists 𝛿 > 0 such that
𝑥 2 −𝑦 2
|𝑥𝑦 𝑥 2+𝑦 2 − 0| < 𝜀, whenever |𝑥| < 𝛿 and |𝑦| < 𝛿.

𝑥 2 −𝑦 2
Hence lim(𝑥,𝑦)→(0,0) 𝑥𝑦 𝑥 2 +𝑦 2 = 0.
3𝑥−2𝑦
Ex. 6. Show that lim(𝑥,𝑦)→(0,0) 2𝑥−3𝑦 does not exist.
Sol. When (𝑥, 𝑦) → (0,0) along the st. line 𝑦 = 𝑥, we have
3𝑥 − 2𝑥 𝑥
lim 𝑓(𝑥, 𝑦) = lim = lim = lim − 1 = −1.
(𝑥,𝑦)→(0,0) 𝑥→0 2𝑥 − 3𝑥 𝑥→0 −𝑥 𝑥→0

Again when (𝑥, 𝑦) → (0,0) along the st. line 𝑦 = 0, we have


3𝑥 − 0 3𝑥 3 3
lim 𝑓(𝑥, 𝑦) = lim = lim = lim = .
(𝑥,𝑦)→(0,0) 𝑥→0 2𝑥 − 0 𝑥→0 2𝑥 𝑥→0 2 2
Since the two methods of approach to the limiting point give different limiting values, the
simultaneous limit does not exist.
2𝑥𝑦 2
Ex. 7. Show that does not exist.
(𝑥,𝑦)→(0,0) 𝑥 2 +𝑦 4

Sol. Let (𝑥, 𝑦) approach (0,0) along the curve 𝑥 = 𝑚𝑦 2 . For this, we put 𝑥 = 𝑚𝑦 2 in the
function and then allow 𝑦 to approach zero. Thus, in this case we have
2𝑥𝑦 2 2𝑚𝑦 4 2𝑚 2𝑚
lim = lim = lim =
(𝑥,𝑦)→(0,0) 𝑥 2 + 𝑦 4 𝑦→0 (𝑚 2 + 1)𝑦 4 𝑦→0 1 + 𝑚 2 1 + 𝑚2
which is different for different values of 𝑚.
2⋅1
For example, if 𝑚 = 1, then this limit = 1+12 = 1

61 | P a g e

© Department of Distance & Continuing Education, Campus of Open Learning,


School of Open Learning, University of Delhi
B.A. (Hons.) Economics

2⋅2 4
and if 𝑚 = 2, then this limit = 1+22 = 5.

Since two methods of approach to the limiting point give different limiting values, therefore
the simultaneous limit
2𝑥𝑦 2
lim does not exist.
(𝑥,𝑦)→(0,0) 𝑥 2 + 𝑦 4

Ex. 8. Show that the simultaneous limit exists at the origin, but the repeated limits do not,
where
1 1
𝑥sin + 𝑦sin , 𝑥𝑦 ≠ 0
𝑓(𝑥, 𝑦) = { 𝑦 𝑥
0, 𝑥𝑦 = 0

Sol. Here lim𝑦→0 𝑓(𝑥, 𝑦), lim𝑥→0 𝑓(𝑥, 𝑦) do not exist and therefore both the repeated limits
lim𝑥→0 lim𝑦→0 𝑓(𝑥, 𝑦) and lim𝑦→0 lim𝑥→0 𝑓(𝑥, 𝑦) do not exist.

However, the simultaneous limit lim(𝑥,𝑦)→(0,0) 𝑓(𝑥, 𝑦) exists and is equal to 0 as shown
below.
Take any given 𝑡 > 0.
For all (𝑥, 𝑦) ≠ (0,0) such that 𝑥𝑦 = 0, we have
|𝑓(𝑥, 𝑦) − 0| = |0 − 0| = 0 < 𝜀.
Again, for all (𝑥, 𝑦) ≠ (0,0) sech that 𝑥𝑦 ≠ 0, we have
1 1
|𝑓(𝑥, 𝑦) − 0| = |𝑓(𝑥, 𝑦)| = |𝑥sin + 𝑦sin |
𝑦 𝑥
1 1
≤ |𝑥sin | + |𝑦sin |
𝑦 𝑥
1 1
= |𝑥| |sin | + |𝑦| |sin | | ∵ |𝑎 + 𝑏| ≤ |𝑎| + |𝑏||
𝑦 𝑥
≤ |𝑥| + |𝑦|
≤ 2√𝑥 2 + 𝑦 2
2
𝑥2 2
𝑒2
< 𝑐, if 𝑥 < asd 𝑦 < | ∵ |𝑎𝑏| = |𝑎||𝑏 ∣
8 8
𝜀
Now if we take 𝛿 = 2√2, thea we see that for any given 𝜀 > 0, there exists s > 0 such that

62 | P a g e

© Department of Distance & Continuing Education, Campus of Open Learning,


School of Open Learning, University of Delhi
Intermediate Mathematical Methods for Economics

|𝑓(𝑥, 𝑦) − 0| < 𝜀, whencier |𝑥| < 𝛿 and |𝑦| < 𝛿.

Hence, lim 𝑓(𝑥, 𝑦) = 0


(𝑥,𝑦)→(0,0)
6.3.4 Continuity of Function of Two Variable:
Definition. The function 𝑓(𝑥, 𝑦) is said to be continuous at (𝑎, 𝑏) if for every 𝜀 > 0, there
exists 𝛿 > 0 such that
|𝑥 − 𝑎| < 𝛿, |𝑦 − 𝑏| < 𝛿 ⇒ |𝑓(𝑥, 𝑦) − 𝑓(𝑎, 𝑏)| < 𝜀.
Equivalently, 𝑓(𝑥, 𝑦) is said to be continuous at (𝑎, 𝑏) if the simultaneous limit
lim 𝑓(𝑥, 𝑦)
(𝑥,𝑦)→(𝑎,𝑏)

exists and is equal to its functional value 𝑓(𝑎, 𝑏) at (𝑎, 𝑏). (𝑎, 𝑏)
If 𝑓 is not continuous at (𝑎, 𝑏) ∈ 𝐷 ⊆ 𝑅 2 , then 𝑓 is said to be discontinuous at (a, b).
𝑓 is said to be continuous on the domain 𝐷 if 𝑓 is continuous at each point of D
Note 1. Let 𝐷 ⊆ R2 and 𝑓: 𝐷 → R be a function continuous at (𝑎, 𝑏) ∈ 𝐷. Let 𝑓1 (𝑥) =
𝑓(𝑥, 𝑏). Then 𝑓1 is a function of a single variable 𝑥. 𝑓 is continuous at (𝑎, 𝑏) ⇒ for cach 𝜀 >
0 there exists 𝛿 > 0 such that |𝑓(𝑥, 𝑦) − 𝑓(𝑎, 𝑏)| < 𝜀 for (𝑥, 𝑦) ∈ 𝐷 and |𝑥 − 𝑎| < 𝛿, |𝑦 −
𝑏| < 𝛿 |𝑓(𝑥, 𝑏) − 𝑓(𝑎, 𝑏)| < 𝑒 for (𝑥, 𝑏) ∈ 𝐷 and |𝑥 − 𝑎| < 𝛿
|𝑓1 (𝑥) − 𝑓1 (𝑎)| < 𝜀 for (𝑥, 𝑏) ∈ 𝐷 and |𝑥 − 𝑎| < 𝛿
⇒ 𝑓1 is continuous at 𝑎.
Similarly, we can show that 𝑓2 (𝑦) = 𝑓(𝑎, 𝑦) is continuous at 𝑏.
Hence, if 𝑓(𝑥, 𝑦) is continuous at (𝑎, 𝑏), then
(i) 𝑓(𝑥, 𝑏) is continuous at 𝑥 = 𝑎 and (ii) 𝑓(𝑎, 𝑦) is continuous at 𝑦 = 𝑏.
The converse of the above result is not true. Thus, if 𝑓(𝑥, 𝑏) is continuous at 𝑥 = 𝑎 and
𝑓(𝑎, 𝑦) is continuous at 𝑦 = 𝑏 then 𝑓(𝑥, 𝑦) need not be continuous at (𝑎, 𝑏).
Note 2. For 𝑓(𝑥, 𝑦) to be continuous in both the variables together, it must have the same
limiting value by all possible approaches to the critical point. Thus, the necessary and sufficient
condition is that the function is not only continuous in each direction but the continuity is
uniform for all directions.
In the definition of continuity, if we put 𝑥 = 𝑎 + 𝑟cos 𝜃, 𝑦 = 𝑏 + 𝑟sin 𝜃, we get |𝑓(𝑎 +
𝑟cos 𝜃, 𝑏 + 𝑟sin 𝜃) − 𝑓(𝑎, 𝑏)| < 𝜀,

63 | P a g e

© Department of Distance & Continuing Education, Campus of Open Learning,


School of Open Learning, University of Delhi
B.A. (Hons.) Economics

which must hold for all values of 𝑟 less than some number 𝑟0 which is independent of 𝜃. In
other words, we say that the transformed function must be uniformly continuous in 𝑟 for all
values of |𝜃| ≤ 2𝜋.
SOLVED EXAMPLES
Ex. 1. Examine the continuity at (1,2) of the function 𝑓(𝑥, 𝑦) =
𝑥 2 + 4𝑦 when (𝑥, 𝑦) ≠ (1,2)
{
0 when (𝑥, 𝑦) = (1,2)
Sol. We have lim(𝑥,𝑦)→(1,2) 𝑥 2 + 4𝑦 = 12 + 4 × 2 = 9, so that the limit exists and is equal to
9.
Since 𝑓(1,2) = 0 and lim(𝑥,𝑦)→(1,2) 𝑓(𝑥, 𝑦) = 9,

we have lim(𝑥,𝑦)→(1,2) 𝑓(𝑥, 𝑦) ≠ 𝑓(1,2)

Hence, the function is not continuous at (1,2).


(𝑦 2 −𝑥 2 )𝑦𝑥
Ex. 2. Show that the function 𝑓: 𝐑2 → 𝐑 defined by 𝑓(𝑥, 𝑦) = for (𝑥, 𝑦) ≠ (0,0)
𝑥 2 +𝑦 2
and 𝑓(0,0) = 0 is continuous at the point (0,0).
Sol. First we shall show that lim(𝑥,𝑦)→(0,0) 𝑓(𝑥, 𝑦) = 0.
Let 𝜀 > 0 be given.
For all (𝑥, 𝑦) ≠ (0,0), we have
(𝑦 2 − 𝑥 2 )𝑦𝑥 |𝑥𝑦|
|𝑓(𝑥, 𝑦) − 0| = | 2 2
| = |𝑦 2 − 𝑥 2 | ⋅ 2 .
𝑥 +𝑦 𝑥 + 𝑦2

Now for all 𝑥 and 𝑦, we have (|𝑥| − |𝑦|)2 ≥ 0


⇒ |𝑥|2 + |𝑦|2 − 2|𝑥| ⋅ |𝑦| ≥ 0 ⇒ 𝑥 2 + 𝑦 2 ≥ 2|𝑥𝑦|
|𝑥𝑦| 1
⇒ 2 2
≤ , if (𝑥, 𝑦) ≠ (0,0).
𝑥 +𝑦 2
∴ from (1) and (2), for all (𝑥, 𝑦) ≠ (0,0), we have
1 2 1 1
|𝑓(𝑥, 𝑦) − 0| ≤ |𝑦 − 𝑥 2 | ≤ |𝑦 2 + 𝑥 2 | = (𝑥 2 + 𝑦 2 )
2 2 2
2 2
< 𝜀, if 𝑥 < 𝜀 and 𝑦 < 𝜀 i.e., if |𝑥| < √𝜀 and |𝑦| < √𝜀.

Now if we take 𝛿 = √𝜀, then we see that for any given 𝜀 > 0, there exists 𝛿 > 0 such that

64 | P a g e

© Department of Distance & Continuing Education, Campus of Open Learning,


School of Open Learning, University of Delhi
Intermediate Mathematical Methods for Economics

|𝑓(𝑥, 𝑦) − 0| < 𝜀, whenever |𝑥| < 𝛿 and |𝑦| < 𝛿


∴ lim 𝑓(𝑥, 𝑦) = 0.
(𝑥,𝑦)→(0,0)
But 𝑓(0,0) = 0. [Given]
∴ lim 𝑓(𝑥, 𝑦) = 𝑓(0,0).
(𝑥,𝑦)→(0,0)

Hence 𝑓(𝑥, 𝑦) is continuous at the origin.


Ex. 3. Discuss the continuity of the function
2𝑥𝑦 2
𝑓(𝑥, 𝑦) = , (𝑥, 𝑦) ≠ (0,0) and 𝑓(0,0) = 0
𝑥 3 + 3𝑦 3
with respect to both variables.
Sol. First we observe that lim(𝑥,𝑦)→(0,0) 𝑓(𝑥, 𝑦) does not exist.

When (𝑥, 𝑦) → (0,0) along the st. line 𝑦 = 𝑥, we have


2𝑥 ⋅ 𝑥 2 2𝑥 3 1 1
lim 𝑓(𝑥, 𝑦) = lim 3 3
= lim 3
= lim = .
(𝑥,𝑦)→(0,0) 𝑥→0 𝑥 + 3𝑥 𝑥→0 4𝑥 𝑥→0 2 2
Again when (𝑥, 𝑦) → (0,0) along the st. line 𝑦 = 0, we have
2𝑥 ⋅ 02
lim 𝑓(𝑥, 𝑦) = lim 3 = lim 0 = 0.
(𝑥,𝑦)→(0,0) 𝑥→0 𝑥 + 3 ⋅ 03 𝑥→0

Since the two methods of approach to the limiting point give different limiting values, the
simultaneous limit does not exist. Consequently, the function is not continuous at (0,0) in
(𝑥, 𝑦) together.
However, the function is continuous in 𝑥 alone and in 𝑦 alone at the origin.
Because if we put either variable zero and then let the other variable approach zero, we find
the limiting value zero, which is the value of the function at (0,0).
𝑥𝑦 3
Ex. 4. Show that the function 𝑓(𝑥, 𝑦) = 𝑥 2 +𝑦 6 , (𝑥, 𝑦) ≠ (0,0) and 𝑓(0,0) = 0 is not
continuous at (0,0) in 𝑥, 𝑦 together but that the function is continuous in 𝑥 alone and y alone
at the origin.
Sol. Simultaneous limit of 𝑓(𝑥, 𝑦) at (0,0) does not exist as shown below: and in 𝑦 alone at
the origin.

65 | P a g e

© Department of Distance & Continuing Education, Campus of Open Learning,


School of Open Learning, University of Delhi
B.A. (Hons.) Economics

𝑥⋅𝑚3 𝑥 3
First let (𝑥, 𝑦) approach (0,0) through any line 𝑦 = 𝑚𝑥. We have lim𝑥→0 𝑥 2 +𝑚6𝑥 6 =
𝑚3 𝑥 2 0
lim𝑥→0 1+𝑚6𝑥 4 = 1+0 = 0.
Now let (𝑥, 𝑦) approach (0,0) through the curve 𝑥 = 𝑦 3 .
𝑦 3 ⋅𝑦 3 1
Then lim𝑦→0 𝑦 6+𝑦 6 = 2.

Since the limits obtained by two different approaches are different, the simultaneous limit does
not exist. Consequently, the function is not continuous at (0,0) in (𝑥, 𝑦) together.
The function is however continuous in 𝑥 alone and in 𝑦 alone at the origin. For putting either
variable zero and then letting the other variable approach zero, we get the limiting value zero,
which is the value of 𝑓(0,0).
Ex. 5. Show that the function
𝑥𝑦
, (𝑥, 𝑦) ≠ (0,0)
𝑓(𝑥, 𝑦) = {√(𝑥 2 + 𝑦 2 )
0, (𝑥, 𝑦) = (0,0)
is continuous at the origin in 𝑥 − 𝑦 together.
Sol. First we shall show that lim(𝑥,𝑦)→(0,0) 𝑓(𝑥, 𝑦) = 0.
Let 𝜀 > 0 be given. For all (𝑥, 𝑦) ≠ (0,0), we have
𝑥𝑦 𝑥𝑦 𝑟cos 𝜃𝑟sin 𝜃
|𝑓(𝑥, 𝑦) − 0| = | − 0| = | |=| |
√(𝑥 2 + 𝑦 2 ) √(𝑥 2 + 𝑦 2 ) 𝑟
= 𝑟|cos 𝜃||sin 𝜃|
≤ 𝑟 [∵ |cos 𝜃| ≤ 1, |sin 𝜃| ≤ 1]
= √(𝑥 2 + 𝑦 2 )
𝜀2 𝜀2 𝜀 𝜀
< 𝜀, if 𝑥 2 < and 𝑦 2 < i.e., if |𝑥| < and |𝑦| < .
2 2 √2 √2

𝜀
Now if we take 𝛿 = , then we see that for any given 𝜀 > 0, there exists 𝛿 > 0 such that
√2

|𝑓(𝑥, 𝑦) − 0| < 𝜀, whenever |𝑥| < 𝛿 and |𝑦| < 𝛿.


Hence, lim(𝑥,𝑦)→(0,0) 𝑓(𝑥, 𝑦) = 0.
Since 𝑓(0,0) = 0, therefore lim(𝑥,𝑦)→(0,0) 𝑓(𝑥, 𝑦) = 𝑓(0,0).
Hence, the given function 𝑓 is continuous at (0,0) in 𝑥 − 𝑦 together.

66 | P a g e

© Department of Distance & Continuing Education, Campus of Open Learning,


School of Open Learning, University of Delhi
Intermediate Mathematical Methods for Economics

Important Note. If lim𝑟→0 𝑓(𝑎 + 𝑟cos 𝜃, 𝑏 + 𝑟sin 𝜃) = 𝑓(𝑎, 𝑏) for every value of 𝜃, then it
is not necessary that the function is continuous at (𝑎, 𝑏).

For example, we have seen in Example 4 of $4 that the simultaneous limit of 𝑓(𝑥, 𝑦) =
𝑥𝑦 3
, 𝑥 ≠ 0, 𝑦 ≠ 0 and 𝑓(0,0) = 0 does not exist as (𝑥, 𝑦) → (0,0). Therefore, this function
𝑥 2 +𝑦 6
is discontinuous at the origin. However, if we put 𝑥 = 𝑟cos 𝜃, 𝑦 = 𝑟sin 𝜃, we get
𝑟 4 cos 𝜃sin3 𝜃 2
cos 𝜃sin3 𝜃
lim = lim 𝑟 = 0 = 𝑓(0,0)
𝑟→0 𝑟 2 cos 2 𝜃 + 𝑟 6 sin6 𝜃 𝑟→0 cos 2 𝜃 + 𝑟 4 sin6 𝜃
for each constant value of 𝜃.
6.3.5 Partial Derivatives:
Definition.
Let 𝑧 = 𝑓(𝑥, 𝑦) be a function of two independent variables 𝑥 and y. Then the partial
derivative of 𝑧 with respect to 𝑥 is the ordinary derivative of 𝑧 with respect to 𝑥 when 𝑦 is
regarded as a constant and is denoted by
∂𝑧 ∂𝑓
𝑓𝑥 or ∂𝑥 or ∂𝑥 .
𝑓(𝑎+ℎ,𝑏)−𝑓(𝑎,𝑏)
Thus if limℎ→0 exists, then this limit is called the partial derivative of 𝑓(𝑥, 𝑦)

∂𝑧 ∂𝑓
with respect to 𝑥 at the point (𝑎, 𝑏) and is denoted by 𝑓𝑥 (𝑎, 𝑏) or (∂𝑥) or (∂𝑥 ) .
(𝑎,𝑏) (𝑎,𝑏)

𝑓(𝑎,𝑏+𝑘)−𝑓(𝑎,𝑏)
Similarly, if lim𝑘→0 exists, then this limit is called the partial derivative of
𝑘
𝑓(𝑥, 𝑦) with respect to 𝑦 at (𝑎, 𝑏) and is denoted by
∂𝑧 ∂𝑓
𝑓𝑦 (𝑎, 𝑏) or ( ) or ( ) .
∂𝑦 (𝑎,𝑏) ∂𝑦 (𝑎,𝑏)

Let 𝑓: 𝑋 → 𝐑 and 𝑋 ⊆ 𝐑2 . If the function 𝑓 has partial derivatives at each point of 𝑋 then 𝑓 is
partially differentiable on 𝑋.
Note 1. We have, by definition
𝑓(𝑎 + ℎ, 𝑏) − 𝑓(𝑎, 𝑏) 𝑓(𝑥, 𝑏) − 𝑓(𝑎, 𝑏)
𝑓𝑥 (𝑎, 𝑏) = lim = lim
ℎ→0 ℎ 𝑥→𝑎 𝑥−𝑎
𝑓(𝑎, 𝑏 + 𝑘) − 𝑓(𝑎, 𝑏) 𝑓(𝑎, 𝑦) − 𝑓(𝑎, 𝑏)
𝑓𝑦 (𝑎, 𝑏) = lim = lim .
𝑘→0 𝑘 𝑦→𝑏 𝑦−𝑏

67 | P a g e

© Department of Distance & Continuing Education, Campus of Open Learning,


School of Open Learning, University of Delhi
B.A. (Hons.) Economics

Note 2. In the case of functions of two variables the existence of partial derivatives at a point
need not imply continuity at that point.
SOLVED EXAMPLES
∂𝑓 ∂𝑓
Ex. 1. Find ∂𝑥 , ∂𝑦 at (1,2) if 𝑓(𝑥, 𝑦) = 2𝑥 2 − 𝑥𝑦 + 2𝑦 2 .

∂𝑓 𝑓(1+ℎ,2)−𝑓(1,2)
Sol. We have (∂𝑥 ) = limℎ→0 .
(1,2) ℎ

{2(1 + ℎ)2 − (1 + ℎ) ⋅ 2 + 2 ⋅ 22 } − {2 ⋅ 12 − 1.2 + 2.22 }


= lim
ℎ→0 ℎ
2
2ℎ + 2ℎ
= lim = lim (2ℎ + 2) = 2
ℎ→0 ℎ ℎ→0
∂𝑓 𝑓(1,2 + 𝑘) − 𝑓(1,2)
and ( ) = lim
∂𝑦 (1,2) 𝑘→0 𝑘
{2 − (2 + 𝑘) + 2(2 + 𝑘)2 } − {2 − 2 + 8}
= lim
𝑘→0 𝑘
2𝑘 2 + 7𝑘
= lim = lim (2𝑘 + 7) = 7.
𝑘→0 𝑘 𝑘→0

𝑥 2 −𝑥𝑦
, (𝑥, 𝑦) ≠ (0,0)
Ex. 2. Find 𝑓𝑥 (0,0) and 𝑓𝑦 (0,0) if 𝑓(𝑥, 𝑦) = { 𝑥+𝑦
0, (𝑥, 𝑦) = (0,0).
Sol. We have
ℎ2 −ℎ.0
𝑓(0 + ℎ, 0) − 𝑓(0,0) −0
ℎ+0
𝑓𝑥 (0,0) = lim = lim = lim 1 = 1
ℎ→0 ℎ ℎ→0 ℎ ℎ→0

𝑓(0,0+𝑘)−𝑓(0,0) 0−0
and 𝑓𝑦 (0,0) = lim𝑘→0 = lim𝑘→0 = lim𝑘→0 0 = 0.
𝑘 𝑘

𝑥 3 +𝑦 3
Ex. 3. If 𝑓(𝑥, 𝑦) = , 𝑥 ≠ 𝑦 and 0 if 𝑥 = 𝑦 then show that 𝑓 is discontinuous at the
𝑥−𝑦
origin but the partial derivatives exist at the origin.
Sol. If we let (𝑥, 𝑦) approach (0,0) through the curve 𝑦 = 𝑥 − 𝑚𝑥 3 , we have

68 | P a g e

© Department of Distance & Continuing Education, Campus of Open Learning,


School of Open Learning, University of Delhi
Intermediate Mathematical Methods for Economics

lim 𝑓(𝑥, 𝑦) = lim 𝑓(𝑥, 𝑥 − 𝑚𝑥 3 )


(𝑥,𝑦)→(0,0) (𝑥,𝑦)→(0,0)
3 3 )3
𝑥 + (𝑥 − 𝑚𝑥 𝑥 3 {1 + (1 − 𝑚𝑥 2 )3 }
= lim = lim
𝑥→0 𝑥 − (𝑥 − 𝑚𝑥 3 ) 𝑥→0 𝑚𝑥 3
1
= lim {2 − 𝑚3 𝑥 6 − 3𝑚𝑥 2 + 3𝑚2 𝑥 4 }
𝑥→0 𝑚
2 2
= lim { − 𝑚2 𝑥 6 − 3𝑥 2 + 3𝑚𝑥 4 } = .
𝑥→0 𝑚 𝑚

Hence the simultaneous limit does not exist since it depends upon 𝑚. So the function 𝑓(𝑥, 𝑦)
is not continuous in 𝑥 − 𝑦 at the origin.
ℎ3
𝑓(0+ℎ,0)−𝑓(0,0) −0

But 𝑓𝑥 (0,0) = limℎ→0 = limℎ→0 = limℎ→0 ℎ = 0 and 𝑓𝑦 (0,0) =
ℎ ℎ
𝑘3
𝑓(0,0+𝑘)−𝑓(0,0) −0
−𝑘
lim𝑘→0 = lim𝑘→0 = lim𝑘→0 (−𝑘) = 0.
𝑘 𝑘

Thus, the partial derivatives exist at the origin.


𝑥2𝑦
Ex. 4. Let 𝑓(𝑥, 𝑦) = 𝑥 4 +𝑦 2 for (𝑥, 𝑦) ≠ (0,0) and 𝑓(0,0) = 0. Show that the partial
derivatives 𝑓𝑥 , 𝑓𝑦 exist everywhere in the region −1 ≤ 𝑥 ≤ 1, −1 ≤ 𝑦 ≤ 1, although 𝑓(𝑥, 𝑦)
is discontinuous in 𝑥 − 𝑦 at the origin.
Sol. We have for 𝑥 ≠ 0, 𝑦 ≠ 0
𝑦2 − 𝑥4 2
𝑥4 − 𝑦2
𝑓𝑥 = 2𝑥𝑦 , 𝑓 = 𝑥
(𝑥 4 + 𝑦 2 )2 𝑦 (𝑥 4 + 𝑦 2 )2
For 𝑥 = 0, 𝑦 = 0, we get
𝑓(0 + ℎ, 0) − 𝑓(0,0) 𝑓(ℎ, 0) − 𝑓(0,0)
𝑓𝑥 = lim = lim
ℎ→0 ℎ ℎ→0 ℎ
0−0
= lim = lim 0 = 0
ℎ→0 ℎ ℎ→0

ℎ2 ⋅ 0
[∵ 𝑓(ℎ, 0) = 4 = 0 and 𝑓(0,0) = 0]
ℎ + 02
𝑓(0,0+𝑘)−𝑓(0,0) 𝑓(0,𝑘)−𝑓(0,0)
and 𝑓𝑦 = lim𝑘→0 = lim𝑘→0
𝑘 𝑘

0−0
= lim = lim 0 = 0
𝑘→0 𝑘 𝑘→0

69 | P a g e

© Department of Distance & Continuing Education, Campus of Open Learning,


School of Open Learning, University of Delhi
B.A. (Hons.) Economics

Also, we can show that


𝑓𝑥 (𝑥, 𝑦) = 0 for 𝑥 = 0, 𝑦 ≠ 0;
𝑓𝑥 (𝑥, 𝑦) = 0 for 𝑥 ≠ 0, 𝑦 = 0;
𝑓𝑦 (𝑥, 𝑦) = 0 for 𝑥 = 0, 𝑦 ≠ 0 and
1
𝑓𝑦 (𝑥, 𝑦) = 2 for 𝑥 ≠ 0, 𝑦 = 0.
𝑥
Hence the partial derivatives 𝑓𝑥 , 𝑓𝑦 exist at all points of the given region. However, the function
𝑓(𝑥, 𝑦) is not continuous in 𝑥 − 𝑦 at the origin as the simultaneous limit does not exist. We
1
can see that the limit along the line 𝑦 = 0 is 0 whereas along the curve 𝑦 = 𝑥 2 it is 2

𝑥𝑦 2
Ex. 5. Let 𝑓: 𝐑2 → 𝐑 be a function defined by 𝑓(𝑥, 𝑦) = 𝑥 2+𝑦 4 for (𝑥, 𝑦) ≠ (0,0) and
𝑓(0,0) = 0.
Using definition calculate 𝑓𝑥 (0,0) and 𝑓𝑦 (0,0).
𝑓(0+ℎ,0)−𝑓(0,0)
So1. By definition, 𝑓𝑥 (0,0) = limℎ→0 ℎ

𝑓(ℎ, 0) − 𝑓(0,0) 0−0


= lim = lim = lim 0 = 0.
ℎ→0 ℎ ℎ→0 ℎ ℎ→0
ℎ ⋅ 02
[∵ 𝑓(ℎ, 0) = 2 = 0 and 𝑓(0,0) = 0]
ℎ + 04
𝑓(0,0 + 𝑘) − 𝑓(0,0)
𝑓𝑦 (0,0) = lim
𝑘→0 𝑘
𝑓(0, 𝑘) − 𝑓(0,0) 0−0
Again = lim = lim = lim 0 = 0.
𝑘→0 𝑘 𝑘→0 𝑘 𝑘→0
2
0⋅𝑘
[∵ 𝑓(0, 𝑘) = 2 = 0, 𝑓(0,0) = 0]
0 + 𝑘4
6.3.6 Differentiability:

Definition. Let 𝑧 = 𝑓(𝑥, 𝑦) be a function of two independent variables 𝑥 and 𝑦 and suppose
𝑓(𝑥, 𝑦) possesses a determinate value at a point (𝑎, 𝑏) and at any point (𝑎 + ℎ, 𝑏 + 𝑘) in the
neighbourhood of (𝑎, 𝑏). The function 𝑓(𝑥, 𝑦) is said to be totally differentiable (or simply
differentiable) at (𝑎, 𝑏) if there exist two numbers 𝐴 and 𝐵 independent of ℎ, 𝑘 such that

𝑓(𝑎 + ℎ, 𝑏 + 𝑘) − 𝑓(𝑎, 𝑏) = 𝐴ℎ + 𝐵𝑘 + √(ℎ2 + 𝑘 2 )𝜙(ℎ, 𝑘)


Where

70 | P a g e

© Department of Distance & Continuing Education, Campus of Open Learning,


School of Open Learning, University of Delhi
Intermediate Mathematical Methods for Economics

The part 𝐴ℎ + 𝐵𝑘 is called the derivative of 𝑓(𝑥, 𝑦) at (𝑎, 𝑏).


The distance between the fixed point (𝑎, 𝑏) and the variable point (𝑎 + ℎ, 𝑏 + 𝑘) is
√(ℎ2 + 𝑘 2 ), which may be regarded as a simultaneous increment in the two variables.
Remark: Another Definition of differentiability of function f (x, y) at a point of its domain.
The function 𝑓(𝑥, 𝑦) is 𝑓(𝑎 + ℎ, 𝑏 + 𝑘) − 𝑓(𝑎, 𝑏) = 𝐴ℎ + 𝐵𝑘 + ℎ𝜙(ℎ, 𝑘) + 𝑘𝜓(ℎ, 𝑘). where
𝐴 and 𝐵 are constants independent of ℎ, 𝑘 and 𝜙, 𝜓 are function to zero as ℎ, 𝑘 tend to zero
simultaneously.
According to this definition the concept of differentiability of the function 𝑓(𝑥, 𝑦) at any point
(𝑥, 𝑦) of its domain can be discussed as below.
Let (𝑥, 𝑦), (𝑥 + 𝛿𝑥, 𝑦 + 𝛿𝑦) be two neighbouring points in the domain of the function 𝑓(𝑥, 𝑦).
The change 𝛿𝑓 in the value of the function 𝑓 as the point changes from (𝑥, 𝑦) to (𝑥 + 𝛿𝑥, 𝑦 +
𝛿𝑦) is given by
𝛿𝑓 = 𝑓(𝑥 + 𝛿𝑥, 𝑦 + 𝛿𝑦) − 𝑓(𝑥, 𝑦).
The function 𝑓 is said to be differentiable at (𝑥, 𝑦) if the change 𝛿𝑓 can be expressed in the
form
𝛿𝑓 = 𝐴𝛿𝑥 + 𝐵𝛿𝑦 + 𝛿𝑥𝜙(𝛿𝑥, 𝛿𝑦) + 𝛿𝑦𝜓(𝛿𝑥, 𝛿𝑦)
where 𝐴 and 𝐵 are expressions independent of 𝛿𝑥, 𝛿𝑦 and 𝜙, 𝜓 are functions of 𝛿𝑥, 𝛿𝑦 each
tending to zero as 𝛿𝑥, 𝛿𝑦 tend to zero simultaneously.
Also, 𝐴𝛿𝑥 + 𝐵𝛿𝑦 is then called the differential of 𝑓 at (𝑥, 𝑦) and is denoted by df. Thus, we
have df= 𝐴𝛿𝑥 + 𝐵𝛿𝑦.
From (1), when (𝛿𝑥, 𝛿𝑦) -> (0,0) we get
𝛿𝑓 → 0 ⇒ 𝑓(𝑥 + 𝛿𝑥, 𝑦 + 𝛿𝑦) − 𝑓(𝑥, 𝑦) → 0
⇒ 𝑓(𝑥 + 𝛿𝑥, 𝑦 + 𝛿𝑦) → 𝑓(𝑥, 𝑦) ⇒ 𝑓(𝑥, 𝑦) is continuous at (𝑥, 𝑦).
Hence, every differentiable function is continuous.
The converse is not true i.e., a function 𝑓(𝑥, 𝑦) may be continuous at (𝑎, 𝑏) but may not be
differentiable at (𝑎, 𝑏).
Again, from (1), when 𝛿𝑦 = 0 i.e., when 𝑦 remains constant, we have
𝛿𝑓 = 𝐴𝛿𝑥 + 𝛿𝑥𝜙(𝛿𝑥, 0).
Dividing by 𝛿𝑥 and taking limits of both sides as 𝛿𝑥 → 0, we get
71 | P a g e

© Department of Distance & Continuing Education, Campus of Open Learning,


School of Open Learning, University of Delhi
B.A. (Hons.) Economics

∂𝑓
=𝐴
∂𝑥
∂𝑓
Similarly, ∂𝑦 = 𝐵
Thus the expressions 𝐴 and 𝐵 are respectively the partial derivatives of 𝑓 with respect to 𝑥
and 𝑦.
Hence, a function which is differentiable at a point possesses the first order partial derivatives
at that point.
The converse of this statement is not true i.e., there may exist functions which are continuous
and may even possess partial derivatives at a point but are not differentiable at that point. We
shall later give some examples of such functions.
SOLVED EXAMPLES
Ex. 1. Show that
𝑥2 − 𝑦2
𝑥𝑦 ( ) ; (𝑥, 𝑦) ≠ (0,0)
𝑓(𝑥, 𝑦) = { 𝑥2 + 𝑦2
0 ; (𝑥, 𝑦) = (0,0)
is differentiable at the origin.
ℎ2 −𝑘 2
Sol. We have 𝑓(0 + ℎ, 0 + 𝑘) = 𝑓(ℎ, 𝑘) = ℎ𝑘 (ℎ2+𝑘 2 ).

ℎ𝑘(ℎ2 − 𝑘 2 )
∴ 𝑓(0 + ℎ, 0 + 𝑘) − 𝑓(0,0) = 0 ⋅ ℎ + 0 ⋅ 𝑘 + √(ℎ2 + 𝑘 2 )
(ℎ2 + 𝑘 2 )3/2
ℎ𝑘(ℎ2 −𝑘 2 )
so that 𝐴 = 0, 𝐵 = 0 and 𝜙(ℎ, 𝑘) = (ℎ2+𝑘 2)3/2.
Thus 𝐴 and 𝐵 are independent of ℎ, 𝑘. Now we shall show that 𝜙(ℎ, 𝑘) → 0 as (ℎ, 𝑘) →
(0,0).
Let 𝜀 > 0 be given.
For all (ℎ, 𝑘) ≠ (0,0), we have

ℎ𝑘 ℎ2 − 𝑘 2 ℎ𝑘 |ℎ2 − 𝑘 2 |
|𝜙(ℎ, 𝑘) − 0| = | 2
⋅ 2
| = | | ⋅ 2 2
√(ℎ2 + 𝑘 2 ) ℎ + 𝑘 √(ℎ2 + 𝑘 2 ) ℎ + 𝑘

72 | P a g e

© Department of Distance & Continuing Education, Campus of Open Learning,


School of Open Learning, University of Delhi
Intermediate Mathematical Methods for Economics

ℎ𝑘
≤| |
√ (ℎ2 + 𝑘 2 )
[∵ |ℎ2 − 𝑘 2 | ≤ ℎ2 + 𝑘 2 ]
|ℎ𝑘|
= 2 ⋅ √(ℎ2 + 𝑘 2 )
ℎ + 𝑘2

Now for all ℎ and 𝑘, we have (|ℎ| − |𝑘|)2 ≥ 0


⇒ |ℎ|2 + |𝑘|2 − 2|ℎ|. |𝑘| ≥ 0 ⇒ ℎ2 + 𝑘 2 ≥ 2|ℎ𝑘|
|ℎ𝑘| 1
⇒ ≤ , if (ℎ, 𝑘) ≠ (0,0).
ℎ2 + 𝑘 2 2
∴ from (1) and (2), for all (ℎ, 𝑘) ≠ (0,0), we have
1
|𝜙(ℎ, 𝑘) − 0| ≤ √(ℎ2 + 𝑘 2 )
2
< √(ℎ2 + 𝑘 2 )
𝜀2 𝜀2 𝜀 𝜀
< 𝜀, if ℎ2 < and 𝑘 2 < i.e., if |ℎ| < and |𝑘| < .
2 2 √2 √2
𝜀
Now if we take 𝛿 = , then we see that for any given 𝜀 > 0, there exists 𝛿 > 0 such that
√2
|𝜙(ℎ, 𝑘) − 0| < 𝜀, whenever |ℎ| < 𝛿 and |𝑘| < 𝛿.
∴ lim(ℎ,𝑘)→(0,0) 𝜙(ℎ, 𝑘) = 0.
∴ 𝑓 is differentiable at (0,0) and the derivative of 𝑓 at (0,0) = 𝐴ℎ + 𝐵𝑘 = 0.
Ex. 2. Show that 𝑓(𝑥, 𝑦) = |𝑥| + |𝑦| is not differentiable at the origin.
Sol. We have 𝑓(0 + ℎ, 0 + 𝑘) = |ℎ| + |𝑘|.
|ℎ|+|𝑘|
∴ 𝑓(0 + ℎ, 0 + 𝑘) − 𝑓(0,0) = 0 ⋅ ℎ + 0 ⋅ 𝑘 + √(ℎ2 + 𝑘 2 ) ⋅ { .
√(ℎ2 +𝑘 2 )
|ℎ|+|𝑘|
so that 𝐴 = 0, 𝐵 = 0 and 𝜙(ℎ, 𝑘) = .
√(ℎ2 +𝑘 2 )

Thus 𝐴 and 𝐵 are independent of ℎ, 𝑘. To find the limit of 𝜙(ℎ, 𝑘) let (ℎ, 𝑘) approach (0,0)
along the line 𝑘 = ℎ.
Putting 𝑘 = ℎ and then making ℎ to approach 0, we get
2|ℎ| 2|ℎ|
lim = lim = lim √2 = √2
ℎ→0 √(2ℎ2 ) ℎ→0 √2 ⋅ |ℎ| ℎ→0

73 | P a g e

© Department of Distance & Continuing Education, Campus of Open Learning,


School of Open Learning, University of Delhi
B.A. (Hons.) Economics

Since lim(ℎ,𝑘)→(0,0) 𝜙(ℎ, 𝑘) ≠ 0, 𝑓 is not differentiable at the origin.


Theorem 1. If the function 𝑓(𝑥, 𝑦) is differentiable at the point (𝑎, 𝑏), then it is continuous in
𝑥 - 𝑦 together at (𝑎𝑏).
Proof. Since 𝑓(𝑥, 𝑦) is differentiable at the point (𝑎, 𝑏) so we have a relation

𝑓(𝑎 + ℎ, 𝑏 + 𝑘) − 𝑓(𝑎, 𝑏) = 𝐴ℎ + 𝐵𝑘 + √(ℎ2 + 𝑘 2 )𝜙(ℎ, 𝑘)


where (i) (𝑎 + ℎ, 𝑏 + 𝑘) is a point in the nhd of (𝑎, 𝑏),
(ii) 𝐴, 𝐵 are independent of ℎ, 𝑘 and
(iii) lim(ℎ,𝑘)→(0,0) 𝜙(ℎ, 𝑘) = 0.
Taking limit of both sides of (1) as (ℎ, 𝑘) → (0,0), we get
lim 𝑓(𝑎 + ℎ, 𝑏 + 𝑘) = 𝑓(𝑎, 𝑏)
(ℎ,𝑘)→(0,0)

which shows that 𝑓(𝑥, 𝑦) is continuous in 𝑥 − 𝑦 together at (𝑎𝑏).


Note. The above theorem shows that continuity in the two variables is a necessary
condition for differentiability. However, it is not a sufficient condition as the following
example shows.
Ex. 3. Show that the function defined by
𝑥𝑦
𝑓(𝑥, 𝑦) = for (𝑥, 𝑦) ≠ (0,0) and 𝑓(0,0) = 0
√(𝑥 2 + 𝑦 2 )

is continuous in 𝑥 − 𝑦 at the origin but is not differentiable in 𝑥 − 𝑦 at the origin.


Sol. Continuity of 𝑓(𝑥, 𝑦) in 𝑥 − 𝑦 at the origin.
First we shall show that lim(𝑥,𝑦)→(0,0) 𝑓(𝑥, 𝑦) = 0.
𝜀2 𝜀2 𝜀 𝜀
< 𝜀, if 𝑥 2 < and 𝑦 2 < i.e., if |𝑥| < and |𝑦| < .
2 2 √2 √2
𝜀
Now if we take 𝛿 = , then we see that for any given 𝜀 > 0, there exists 𝛿 > 0 such that
√2

|𝑓(𝑥, 𝑦) − 0| < 𝜀, whenever |𝑥| < 𝛿 and |𝑦| < 𝛿.


Hence, lim(𝑥,𝑦)→(0,0) 𝑓(𝑥, 𝑦) = 0.
Since 𝑓(0,0) = 0, therefore lim(𝑥,𝑦)→(0,0) 𝑓(𝑥, 𝑦) = 𝑓(0,0).

Hence, the given function 𝑓 is continuous at (0,0) in 𝑥-y together: Now we shall show that
ℎ𝑘
𝑓(𝑥, 𝑦) is not differentiable at the origin. We have 𝑓(0 + ℎ, 0 + 𝑘) = .
√(ℎ2 +𝑘 2 )

74 | P a g e

© Department of Distance & Continuing Education, Campus of Open Learning,


School of Open Learning, University of Delhi
Intermediate Mathematical Methods for Economics

ℎ𝑘
∴ 𝑓(0 + ℎ, 0 + 𝑘) − 𝑓(0,0) = 0 ⋅ ℎ + 0 ⋅ 𝑘 + √(ℎ2 + 𝑘 2 ) ⋅ (ℎ2+𝑘 2 ) so that 𝐴 = 0, 𝐵 = 0 and
ℎ𝑘
𝜙(ℎ, 𝑘) = ℎ2 +𝑘 2.
Thus 𝐴 and 𝐵 are independent of ℎ, 𝑘.
ℎ𝑘
Here lim(ℎ,𝑘)→(0,0) 𝜙(ℎ, 𝑘) = lim(ℎ,𝑘)→(0,0) ℎ2 +𝑘 2.
This limit does not exist as shown below.
Taking 𝑘 = 𝑚ℎ, we have
. ℎ𝑘 𝑚ℎ2
lim = lim 2
(ℎ,𝑘)→(0,0) ℎ2 + 𝑘 2 ℎ→0 ℎ + 𝑚 2 ℎ2
𝑚ℎ2 𝑚
= lim 2 = lim
ℎ→0 ℎ (1 + 𝑚 2 ) ℎ→0 1 + 𝑚 2

𝑚
= 1+𝑚2 , which depends upon 𝑚.
0 1 1
If 𝑚 = 0, then lim(ℎ,𝑘)→(0,0) = 1+0 = 0 and if 𝑚 = 1, then lim(ℎ,𝑘)→(0,0) = 1+1 = 2.
Since these two methods of approach to the limiting point give different limiting values,
ℎ𝑘
therefore the simultaneous limit, lim(ℎ,𝑘)→(0,0) ℎ2+𝑘 2 does not exist.
Since lim(ℎ,𝑘)→(0,0) 𝜙(ℎ, 𝑘) ≠ 0, therefore 𝑓 is not differentiable at the origin.
𝑥𝑦
Ex. 4. Let 𝑓(𝑥, 𝑦) = , (𝑥, 𝑦) ≠ (0,0) and 𝑓(0,0) = 0. Prove that 𝑓 has partial
√(𝑥2 +𝑦 2 )
derivatives at (0,0) but is not differentiable at the origin.
Sol. We have
𝑓(0 + ℎ, 0) − 𝑓(0,0)
𝑓𝑥 (0,0) = lim
ℎ→0 ℎ
𝑓(ℎ, 0) − 𝑓(0,0) 0−0
= lim = lim = lim 0 = 0.
ℎ→0 ℎ ℎ→0 ℎ ℎ→0

𝑓(0,0+𝑘)−𝑓(0,0)
Again 𝑓𝑦 (0,0) = lim𝑘→0 𝑘

𝑓(0, 𝑘) − 𝑓(0,0) 0−0


= lim = lim = lim 0 = 0.
𝑘→0 𝑘 𝑘→0 𝑘 𝑘→0

Thus 𝑓 has partial derivatives at (0,0).


To show that 𝑓(𝑥, 𝑦) is not differentiable at the origin, see example 3 above.
Theorem 2. If the function 𝑧 = 𝑓(𝑥, 𝑦) is differentiable at (𝑎, 𝑏), then the partial derivatives
𝑓𝑥 (𝑎, 𝑏) and 𝑓𝑦 (𝑎, 𝑏) both exist and are finite.

75 | P a g e

© Department of Distance & Continuing Education, Campus of Open Learning,


School of Open Learning, University of Delhi
B.A. (Hons.) Economics

Proof. Since 𝑓(𝑥, 𝑦) is differentiable at the point (𝑎, 𝑏) so we have a relation 𝑓(𝑎 + ℎ, 𝑏 +
𝑘) − 𝑓(𝑎, 𝑏) = 𝐴ℎ + 𝐵𝑘 + √(ℎ2 + 𝑘 2 )𝜙(ℎ, 𝑘),
where (i) (𝑎 + ℎ, 𝑏 + 𝑘) is a point in the nhd of (𝑎, 𝑏)
(ii) 𝐴 and 𝐵 are independent of ℎ, 𝑘 and
(iii) lim(ℎ,𝑘)→(0,0) 𝜙(ℎ, 𝑘) = 0.
Putting 𝑘 = 0 and proceeding to the limits as ℎ → 0, we see from (1) that
𝑓(𝑎+ℎ,𝑏)−𝑓(𝑎,𝑏)
limℎ→0 = 𝐴 or 𝑓𝑥 (𝑎, 𝑏) = 𝐴.

∴ 𝑓𝑥 (𝑎, 𝑏) exists and is equal to 𝐴.
Similarly we can show that 𝑓𝑦 (𝑎, 𝑏) exists and is equal to 𝐵.

Note. The converse of the above theorem need not be true i.e., if 𝑓 is continuous and possesses
partial derivatives at a point then 𝑓 need not be differentiable at that point. The following
example illustrates this:
𝑥 3 −𝑦 3
Ex. 5. Let 𝑓(𝑥, 𝑦) = 𝑥 2+𝑦 2 when (𝑥, 𝑦) ≠ (0,0) and 𝑓(0,0) = 0. Show that the function 𝑓 is
continuous and possesses partial derivatives but not differentiable at the origin.
(Osmania 1992)
Sol. Continuity of 𝑓 at (0,0) :
The function 𝑓(𝑥, 𝑦) will be continuous at (0,0) if
lim 𝑓(𝑥, 𝑦) = 𝑓(0,0) = 0
(𝑥,𝑦)→(0,0)

Now we shall show that lim(𝑥,𝑦)→(0,0) 𝑓(𝑥, 𝑦) = 0.


Take any given 𝜀 > 0.
For all (𝑥, 𝑦) ≠ (0,0), we have
𝑥3 − 𝑦3
|𝑓(𝑥, 𝑦) − 0| = | |
𝑥2 + 𝑦2
𝑟 3 (cos 3 𝜃 − sin3 𝜃)
=| | [Putting 𝑥 = 𝑟cos 𝜃 and 𝑦 = 𝑟sin 𝜃]
𝑟2
= 𝑟|cos 3 𝜃 − sin3 𝜃|
≤ 𝑟[|cos 3 𝜃| + |sin3 𝜃|] [∵ |𝑎 − 𝑏| ≤ |𝑎| + |𝑏|]
≤ 2𝑟
= 2√(𝑥 2 + 𝑦 2 ) [∵ cos3 𝜃 ∣≤ 1 and |sin3 𝜃| ≤ 1]
𝜀2 𝜀2 𝜀 𝜀
< 𝜀, if 𝑥 2 < and 𝑦 2 < 𝑖. 𝑒. , if |𝑥| < and |𝑦| < .
8 8 2√2 2√2

76 | P a g e

© Department of Distance & Continuing Education, Campus of Open Learning,


School of Open Learning, University of Delhi
Intermediate Mathematical Methods for Economics

𝜀
Now if we take 𝛿 = 2√2, then we see that for any given 𝜀 > 0, there exists 𝛿 > 0 such that

|𝑓(𝑥, 𝑦) − 0| < 𝜀, whenever |𝑥| < 𝛿 and |𝑦| < 𝛿.


∴ (𝑥, 𝑦) → (0,0)𝑓(𝑥, 𝑦) = 0 = 𝑓(0,0).
Hence 𝑓 is continuous in 𝑥 − 𝑦 together at the origin.
Partial derivatives at (𝟎, 𝟎) :
𝑓(0 + ℎ, 0) − 𝑓(0,0) 1 ℎ3 − 0
𝑓𝑥 (0,0) = lim = lim [ 2 − 0] = 1;
ℎ→0 ℎ ℎ→0 ℎ ℎ + 0

𝑓(0,0 + 𝑘) − 𝑓(0,0) 1 0 − 𝑘3
𝑓𝑦 (0,0) = lim = lim [ − 0] = −1.
𝑘→0 𝑘 𝑘→0 𝑘 0 + 𝑘 2

Thus 𝑓𝑥 (0,0) and 𝑓𝑦 (0,0) exist.


Differentiability at (𝟎, 0) :
ℎ3 −𝑘 3 ℎ2 𝑘−ℎ𝑘 2
We have 𝑓(0 + ℎ, 0 + 𝑘) = ℎ2 +𝑘 2 = ℎ − 𝑘 + .
ℎ2 +𝑘 2

ℎ𝑘(ℎ − 𝑘)
∴ 𝑓(0 + ℎ, 0 + 𝑘) − 𝑓(0,0) = 1 ⋅ ℎ + (−1) ⋅ 𝑘 + √(ℎ2 + 𝑘 2 ) { }
(ℎ2 + 𝑘 2 )3/2
ℎ𝑘(ℎ−𝑘)
so that 𝐴 = 1, 𝐵 = −1 and 𝜙(ℎ, 𝑘) = (ℎ2+𝑘 2 )3/2
Thus 𝐴 and 𝐵 are numbers independent of ℎ, 𝑘. If we put 𝑘 = 𝑚ℎ, then we have
𝑚ℎ2 (ℎ − 𝑚ℎ)
lim 𝜙(ℎ, 𝑘) = lim
(ℎ,𝑘)→(0,0) ℎ→0 (ℎ2 + 𝑚 2 ℎ2 )3/2
ℎ3 𝑚(1 − 𝑚) 𝑚(1 − 𝑚)
= lim 3 2 3/2
= .
ℎ→0 ℎ (1 + 𝑚 ) (1 + 𝑚2 )3/2
This limit does not exist since it depends upon 𝑚 and so is not unique.
∴ lim𝜙(ℎ, 𝑘) ≠ 0 as (ℎ, 𝑘) → (0,0).
It follows that the given function is not differentiable at (0,0).
Self-Assessment questions:
EXERCISE 1
2𝑥𝑦 2
1 Show that lim(𝑥,𝑦)→(0,0) 𝑥 2+𝑦 4 does not exist.

2 Show that lim(𝑥,𝑦)→(0,0) (𝑥 + 𝑦) = 0. 3.

77 | P a g e

© Department of Distance & Continuing Education, Campus of Open Learning,


School of Open Learning, University of Delhi
B.A. (Hons.) Economics

2𝑥𝑦
3 Show that lim𝑥→0 𝑥 2 +𝑦 2 does not exist.

√𝑥 2 𝑦 2 +1−1
4 Show that lim(𝑥,𝑦)→(0,0) = 0.
𝑥 2 +𝑦 2

𝑥 3 +𝑦 3
5 Show that lim(𝑥,𝑦)→(0,0) does not exist.
𝑥−𝑦
𝑥−𝑚𝑥 3 ].
[Hint. Put 𝑦 = 1 .
−𝑥 2
2

𝑥2𝑦
, if 𝑥 4 + 𝑦 2 ≠ 0
6 Let 𝑓(𝑥, 𝑦) = {𝑥 4 +𝑦 2
0, if 𝑥 = 𝑦 = 0.
Show that the straight-line approach gives the limit zero at the origin but the function
possesses no limit at the origin.
𝑥𝑦 2
7 Show that lim(𝑥,𝑦)→(0,0) 𝑥 2+𝑦 2 = 0.

8 Show that the limit, when (𝑥, 𝑦) → (0,0), exists in each case
𝑥𝑦
(i) lim 2 2
√𝑥 +𝑦
𝑥3𝑦3
(ii) lim 𝑥 2 +𝑦 2
𝑥 4 +𝑦 4
(iii) lim 𝑥 2 +𝑦 2
𝑥 3 −𝑦 3
(iv) lim 𝑥 2 +𝑦 2

9 Show that the repeated limits exist at the origin and are equal but the simultaneous
limit does not exist, where
1, if 𝑥𝑦 ≠ 0
𝑓(𝑥, 𝑦) = {
0, if 𝑥𝑦 = 0
10 Show that the limit and the repeated limits exist when (𝑥, 𝑦) → (0,0) :
𝑥2 − 𝑦2
𝑥𝑦 , (𝑥, 𝑦) ≠ (0,0)
𝑓(𝑥, 𝑦) = { 𝑥2 + 𝑦2
0, (𝑥, 𝑦) = (0,0).
11 Show that lim(𝑥,𝑦)→(0,0) 𝑓(𝑥, 𝑦) and lim𝑦→0 lim𝑥→0 𝑓(𝑥, 𝑦) exist, but
lim𝑥→0 lim𝑦→0 𝑓(𝑥, 𝑦) does not exist, where
𝑥−𝑦
12 Show that if 𝑓(𝑥, 𝑦) = 𝑥+𝑦, then the repeated limits exist but the double limit does not
when (𝑥, 𝑦) → (0,0).

78 | P a g e

© Department of Distance & Continuing Education, Campus of Open Learning,


School of Open Learning, University of Delhi
Intermediate Mathematical Methods for Economics

6.4 SUMMARY
The main points which we have covered in these lessons are what is limit, continuity and
differentiability of function of two variable and what is the use.

6.5 GLOSSARY
Motivation: These Problems are very useful in real life, and we can use it in data science,
economics as well as social science.
Attention: Think how the differentiability is useful in real world problems.

6.6 REFERENCES

• Sydsaeter, K., Hammond, P. (2002). Mathematics for economics analysis. Pearson


Education.

• Hoy, M., Livernois, J., McKenna, C., Rees, R., Stengos, T, (2001). Mathematics for
Economics, Prentice-Hall India.

6.7 SUGGESTED READINGS

• A.R Vasishtha, Anurag Sharma, Dr. Vipin Vasishtha, Neenu Agarwal, Dr A.K
Vasishtha, Advanced Calculus,Krishna Publication, 5th Edition.
• A.R Vasishtha, Anurag Sharma, Dr. Vipin Vasishtha, Anil Kumar, Dr A.K Vasishtha,
Analysis, Krishna Publication, 3th Edition.

79 | P a g e

© Department of Distance & Continuing Education, Campus of Open Learning,


School of Open Learning, University of Delhi
B.A. (Hons.) Economics

LESSON-7

SECOND ORDER DERIVATIVES

STRUCTURE

7.1 Learning Objectives


7.2 Introduction
7.3 Second Order Derivatives
7.3.1 Partial Derivatives of Higher Order
7.3.2 Interchange of the Order of Differentiation
7.3.3 Sufficient Conditions for the Equality of Second Order Derivatives
7.3.4 Schwarz’s and Young’s Theorem
7.4 Summary
7.5 Glossary
7.6 References
7.7 Suggested Readings

7.1 LEARNING OBJECTIVES

In differential calculus, the derivative of function of several variables with respect to change in
just one of its variables. Partial derivatives are useful in analysing surfaces for maximum and
minimum points and give rise to partial differential equations. As with ordinary derivatives, a
first partial derivative represents a rate of change or a slope of a tangent line. For a three-
dimensional surface, two first partial derivatives represent the slope in each of two
perpendicular directions. Second, third, and higher partial derivatives give more information
about how the function changes at any point.

7.2 INTRODUCTION

Let 𝑧 = 𝑓(𝑥, 𝑦) be a function of two independent variables 𝑥 and y. Then the partial derivative
of 𝑧 with respect to 𝑥 is the ordinary derivative of 𝑧 with respect to 𝑥 when 𝑦 is regarded as a
constant and is denoted by
∂𝑧 ∂𝑓
𝑓𝑥 or ∂𝑥 or ∂𝑥 .

80 | P a g e

© Department of Distance & Continuing Education, Campus of Open Learning,


School of Open Learning, University of Delhi
Intermediate Mathematical Methods for Economics

𝑓(𝑎+ℎ,𝑏)−𝑓(𝑎,𝑏)
Thus if limℎ→0 exists, then this limit is called the partial derivative of 𝑓(𝑥, 𝑦)

∂𝑧 ∂𝑓
with respect to 𝑥 at the point (𝑎, 𝑏) and is denoted by 𝑓𝑥 (𝑎, 𝑏) or (∂𝑥) or (∂𝑥 ) .
(𝑎,𝑏) (𝑎,𝑏)

𝑓(𝑎,𝑏+𝑘)−𝑓(𝑎,𝑏)
Similarly, if lim𝑘→0 exists, then this limit is called the partial derivative of
𝑘
𝑓(𝑥, 𝑦) with respect to 𝑦 at (𝑎, 𝑏) and is denoted by
∂𝑧 ∂𝑓
𝑓𝑦 (𝑎, 𝑏) or ( ) or ( ) .
∂𝑦 (𝑎,𝑏) ∂𝑦 (𝑎,𝑏)

Let 𝑓: 𝑋 → 𝐑 and 𝑋 ⊆ 𝐑2 . If the function 𝑓 has partial derivatives at each point of 𝑋 then 𝑓 is
partially differentiable on 𝑋.
Note 1. We have, by definition.
𝑓(𝑎 + ℎ, 𝑏) − 𝑓(𝑎, 𝑏) 𝑓(𝑥, 𝑏) − 𝑓(𝑎, 𝑏)
𝑓𝑥 (𝑎, 𝑏) = lim = lim
ℎ→0 ℎ 𝑥→𝑎 𝑥−𝑎
𝑓(𝑎, 𝑏 + 𝑘) − 𝑓(𝑎, 𝑏) 𝑓(𝑎, 𝑦) − 𝑓(𝑎, 𝑏)
𝑓𝑦 (𝑎, 𝑏) = lim = lim .
𝑘→0 𝑘 𝑦→𝑏 𝑦−𝑏
Note 2. In the case of functions of two variables the existence of partial derivatives at a point
need not imply continuity at that point.
SOLVED EXAMPLES
∂𝑓 ∂𝑓
Ex. 1. Find ∂𝑥 , ∂𝑦 at (1,2) if 𝑓(𝑥, 𝑦) = 2𝑥 2 − 𝑥𝑦 + 2𝑦 2 .
∂𝑓 𝑓(𝑖+ℎ,2)−𝑓(1,2)
Sol. We have (∂𝑥 ) = limℎ→0 ℎ
.
(1,2)

{2(1 + ℎ)2 − (1 + ℎ) ⋅ 2 + 2 ⋅ 22 } − {2 ⋅ 12 − 1.2 + 2.22 }


= lim
ℎ→0 ℎ
2ℎ2 + 2ℎ
= lim = lim (2ℎ + 2) = 2
ℎ→0 ℎ ℎ→0
∂𝑓 𝑓(1,2 + 𝑘) − 𝑓(1,2)
and ( ) = lim
∂𝑦 (1,2) 𝑘→0 𝑘
{2 − (2 + 𝑘) + 2(2 + 𝑘)2 } − {2 − 2 + 8}
= lim
𝑘→0 𝑘
2
2𝑘 + 7𝑘
= lim = lim (2𝑘 + 7) = 7.
𝑘→0 𝑘 𝑘→0

81 | P a g e

© Department of Distance & Continuing Education, Campus of Open Learning,


School of Open Learning, University of Delhi
B.A. (Hons.) Economics

𝑥 2 −𝑥𝑦
, (𝑥, 𝑦) ≠ (0,0)
Ex. 2. Find 𝑓𝑥 (0,0) and 𝑓𝑦 (0,0) if 𝑓(𝑥, 𝑦) = { 𝑥+𝑦
0, (𝑥, 𝑦) = (0,0).
Sol. We have
ℎ2 −ℎ.0
𝑓(0 + ℎ, 0) − 𝑓(0,0) −0
ℎ+0
𝑓𝑥 (0,0) = lim = lim = lim 1 = 1
ℎ→0 ℎ ℎ→0 ℎ ℎ→0
𝑓(0,0+𝑘)−𝑓(0,0) 0−0
and 𝑓𝑦 (0,0) = lim𝑘→0 = lim𝑘→0 = lim𝑘→0 0 = 0.
𝑘 𝑘
𝑥 3 +𝑦 3
Ex. 3. If 𝑓(𝑥, 𝑦) = , 𝑥 ≠ 𝑦 and 0 if 𝑥 = 𝑦 then show that 𝑓 is discontinuous at the
𝑥−𝑦
origin but the partial derivatives exist at the origin.
Sol. If we let (𝑥, 𝑦) approach (0,0) through the curve 𝑦 = 𝑥 − 𝑚𝑥 3 , we have
lim 𝑓(𝑥, 𝑦) = lim 𝑓(𝑥, 𝑥 − 𝑚𝑥 3 )
(𝑥,𝑦)→(0,0) (𝑥,𝑦)→(0,0)
3 3 )3
𝑥 + (𝑥 − 𝑚𝑥 𝑥 3 {1 + (1 − 𝑚𝑥 2 )3 }
= lim = lim
𝑥→0 𝑥 − (𝑥 − 𝑚𝑥 3 ) 𝑥→0 𝑚𝑥 3
1
= lim {2 − 𝑚3 𝑥 6 − 3𝑚𝑥 2 + 3𝑚2 𝑥 4 }
𝑥→0 𝑚
2 2
= lim { − 𝑚2 𝑥 6 − 3𝑥 2 + 3𝑚𝑥4 } = .
𝑥→0 𝑚 𝑚

Hence the simultaneous limit does not exist since it depends upon 𝑚. So the function 𝑓(𝑥, 𝑦)
is not continuous in 𝑥 − 𝑦 at the origin.
ℎ3
𝑓(0+ℎ,0)−𝑓(0,0) −0

But 𝑓𝑥 (0,0) = limℎ→0 = limℎ→0 = limℎ→0 ℎ = 0 and 𝑓𝑦 (0,0) =
ℎ ℎ
𝑘3
𝑓(0,0+𝑘)−𝑓(0,0) −0
−𝑘
lim𝑘→0 = lim𝑘→0 = lim𝑘→0 (−𝑘) = 0.
𝑘 𝑘

Thus, the partial derivatives exist at the origin.


𝑥2𝑦
Ex. 4. Let 𝑓(𝑥, 𝑦) = 𝑥 4 +𝑦 2 for (𝑥, 𝑦) ≠ (0,0) and 𝑓(0,0) = 0. Show that the partial
derivatives 𝑓𝑥 , 𝑓𝑦 exist everywhere in the region −1 ≤ 𝑥 ≤ 1, −1 ≤ 𝑦 ≤ 1, although 𝑓(𝑥, 𝑦)
is discontinuous in 𝑥 − 𝑦 at the origin.
Sol. We have for 𝑥 ≠ 0, 𝑦 ≠ 0
𝑦2 − 𝑥4 2
𝑥4 − 𝑦2
𝑓𝑥 = 2𝑥𝑦 4 ,𝑓 = 𝑥
(𝑥 + 𝑦 2 )2 𝑦 (𝑥 4 + 𝑦 2 )2
For 𝑥 = 0, 𝑦 = 0, we get

82 | P a g e

© Department of Distance & Continuing Education, Campus of Open Learning,


School of Open Learning, University of Delhi
Intermediate Mathematical Methods for Economics

𝑓(0 + ℎ, 0) − 𝑓(0,0) 𝑓(ℎ, 0) − 𝑓(0,0)


𝑓𝑥 = lim = lim
ℎ→0 ℎ ℎ→0 ℎ
0−0
= lim = lim 0 = 0
ℎ→0 ℎ ℎ→0

ℎ2 ⋅ 0
[∵ 𝑓(ℎ, 0) = = 0 and 𝑓(0,0) = 0]
ℎ4 + 02
𝑓(0,0+𝑘)−𝑓(0,0) 𝑓(0,𝑘)−𝑓(0,0)
and 𝑓𝑦 = lim𝑘→0 = lim𝑘→0
𝑘 𝑘

0−0
= lim = lim 0 = 0
𝑘→0 𝑘 𝑘→0

Also, we can show that


𝑓𝑥 (𝑥, 𝑦) = 0 for 𝑥 = 0, 𝑦 ≠ 0;
𝑓𝑥 (𝑥, 𝑦) = 0 for 𝑥 ≠ 0, 𝑦 = 0;
𝑓𝑦 (𝑥, 𝑦) = 0 for 𝑥 = 0, 𝑦 ≠ 0 and
1
𝑓𝑦 (𝑥, 𝑦) = 2 for 𝑥 ≠ 0, 𝑦 = 0.
𝑥
Hence the partial derivatives 𝑓𝑥 , 𝑓𝑦 exist at all points of the given region. However, the function
𝑓(𝑥, 𝑦) is not continuous in 𝑥 − 𝑦 at the origin as the simultaneous limit does not exist. We
1
can see that the limit along the line 𝑦 = 0 is 0 whereas along the curve 𝑦 = 𝑥 2 it is 2

𝑥𝑦 2
Ex. 5. Let 𝑓: 𝐑2 → 𝐑 be a function defined by 𝑓(𝑥, 𝑦) = 𝑥 2+𝑦 4 for (𝑥, 𝑦) ≠ (0,0) and
𝑓(0,0) = 0.
Using definition calculate 𝑓𝑥 (0,0) and 𝑓𝑦 (0,0).
𝑓(0+ℎ,0)−𝑓(0,0)
So1. By definition, 𝑓𝑥 (0,0) = limℎ→0 ℎ

𝑓(ℎ, 0) − 𝑓(0,0) 0−0


= lim = lim = lim 0 = 0.
ℎ→0 ℎ ℎ→0 ℎ ℎ→0
ℎ ⋅ 02
[∵ 𝑓(ℎ, 0) = 2 = 0 and 𝑓(0,0) = 0]
ℎ + 04

83 | P a g e

© Department of Distance & Continuing Education, Campus of Open Learning,


School of Open Learning, University of Delhi
B.A. (Hons.) Economics

𝑓(0,0 + 𝑘) − 𝑓(0,0)
𝑓𝑦 (0,0) = lim
𝑘→0 𝑘
𝑓(0, 𝑘) − 𝑓(0,0) 0−0
Again = lim = lim = lim 0 = 0.
𝑘→0 𝑘 𝑘→0 𝑘 𝑘→0
2
0⋅𝑘
[∵ 𝑓(0, 𝑘) = 2 = 0, 𝑓(0,0) = 0]
0 + 𝑘4

7.3 SECOND ORDER DERIVATIVES

Partial Derivatives of Higher Order


Interchange of the Order of Differentiation
Sufficient Conditions for the Equality of Second Order Derivatives
Schwarz’s and Young’s Theorem
Now we shall now, briefly, explain these terms one by one.
7.3.1 Partial Derivative of Higher Order:
∂𝑧
The partial differential coefficient ∂𝑥 of 𝑧 = 𝑓(𝑥, 𝑦) may again be differentiated partially with
respect to 𝑥 and to 𝑦, thus giving the second partial differential coefficients.
∂2 𝑧 ∂ ∂𝑧 ∂2 𝑧 ∂ ∂𝑧
= 𝑓𝑥𝑥 = ( ) and = 𝑓𝑦𝑥 = ( )
∂𝑥 2 ∂𝑥 ∂𝑥 ∂𝑦 ∂𝑥 ∂𝑦 ∂𝑥
∂𝑧
Similarly, from ∂𝑦 may be obtained

∂2 𝑧 ∂ ∂𝑧 ∂2 𝑧 ∂ ∂𝑧
= 𝑓𝑥𝑦 = ( ) and = 𝑓𝑦𝑦 = ( )
∂𝑥 ∂𝑦 ∂𝑥 ∂𝑦 ∂𝑦 2 ∂𝑦 ∂𝑦
If 𝑧 = 𝑓(𝑥, 𝑦) and its partial derivatives are continuous (as is true in all ordinary cases), the
order of differentiation is immaterial, that is,
∂2 𝑧 ∂2 𝑧
=
∂𝑥 ∂𝑦 ∂𝑦 ∂𝑥
We can also think Partial differentiation in this way as defined below
1 The derivative functions 𝑓𝑥 , 𝑓𝑦 are in general functions of 𝑥 and 𝑦 and hence may
themselves have partial derivatives with respect to 𝑥 or 𝑦. Thus, we are led to higher
order partial derivatives. Second order partial derivatives of f are denoted by
84 | P a g e

© Department of Distance & Continuing Education, Campus of Open Learning,


School of Open Learning, University of Delhi
Intermediate Mathematical Methods for Economics

∂ ∂𝑓 ∂2 𝑓
( ) = 2 = 𝑓𝑥𝑥
∂𝑥 ∂𝑥 ∂𝑥
∂ ∂𝑓 ∂2 𝑓
( ) = 2 = 𝑓𝑦𝑦
∂𝑦 ∂𝑦 ∂𝑦
∂ ∂𝑓 ∂2 𝑓
( ) = = 𝑓𝑥𝑦
∂𝑥 ∂𝑦 ∂𝑥 ∂𝑦
∂ ∂𝑓 ∂2 𝑓
( ) = = 𝑓𝑦𝑥
∂𝑦 ∂𝑥 ∂𝑦 ∂𝑥
The second order partial derivatives at a particular point (𝑎, 𝑏) are often denoted by
∂2 𝑓 ∂2 𝑓(𝑎, 𝑏)
( 2) , or 𝑓𝑥𝑥 (𝑎, 𝑏)
∂𝑥 (𝑎,𝑏) ∂𝑥 2
∂2 𝑓 ∂2 𝑓(𝑎, 𝑏)
( ) , or 𝑓𝑥𝑦 (𝑎, 𝑏) and so on.
∂𝑥 ∂𝑦 (𝑎,𝑏) ∂𝑥 ∂𝑦

Thus,
𝑓𝑥 (𝑎 + ℎ, 𝑏) − 𝑓𝑥 (𝑎, 𝑏)
𝑓𝑥𝑥 (𝑎, 𝑏) = lim
ℎ→0 ℎ
𝑓𝑦 (𝑎 + ℎ, 𝑏) − 𝑓𝑦 (𝑎, 𝑏)
𝑓𝑥𝑦 (𝑎, 𝑏) = lim
ℎ→0 ℎ
𝑓𝑥 (𝑎, 𝑏 + 𝑘) − 𝑓𝑥 (𝑎, 𝑏)
𝑓𝑦𝑥 (𝑎, 𝑏) = lim
𝑘→0 𝑘
𝑓𝑦 (𝑎, 𝑏 + 𝑘) − 𝑓𝑦 (𝑎, 𝑏)
𝑓𝑦𝑦 (𝑎, 𝑏) = lim
𝑘→0 𝑘
provided the limits exist.
𝑓𝑥 (𝑎+ℎ,𝑏)−𝑓𝑥 (𝑎,𝑏)
We have 𝑓𝑥𝑥 (𝑎, 𝑏) = limℎ→0 ℎ

1 𝑓(𝑎 + 2ℎ, 𝑏) − 𝑓(𝑎 + ℎ, 𝑏) 𝑓(𝑎 + ℎ, 𝑏) − 𝑓(𝑎, 𝑏)


= lim [lim − lim ]
ℎ→0 ℎ ℎ→0 ℎ ℎ→0 ℎ
𝑓(𝑎 + 2ℎ, 𝑏) − 2𝑓(𝑎 + ℎ, 𝑏) + 𝑓(𝑎, 𝑏)
= lim .
ℎ→0 ℎ2
Similarly,

85 | P a g e

© Department of Distance & Continuing Education, Campus of Open Learning,


School of Open Learning, University of Delhi
B.A. (Hons.) Economics

𝑓(𝑎, 𝑏 + 2𝑘) − 2𝑓(𝑎, 𝑏 + 𝑘) + 𝑓(𝑎, 𝑏)


𝑓𝑦𝑦 (𝑎, 𝑏) = lim
𝑘→0 𝑘2
𝑓(𝑎 + ℎ, 𝑏 + 𝑘) − 𝑓(𝑎 + ℎ, 𝑏) − 𝑓(𝑎, 𝑏 + 𝑘) + 𝑓(𝑎, 𝑏)
𝑓𝑥𝑦 (𝑎, 𝑏) = lim [lim ]
ℎ→0 𝑘→0 ℎ𝑘
𝑓(𝑎 + ℎ, 𝑏 + 𝑘) − 𝑓(𝑎 + ℎ, 𝑏) − 𝑓(𝑎, 𝑏 + 𝑘) + 𝑓(𝑎, 𝑏)
𝑓𝑦𝑥 (𝑎, 𝑏) = lim [lim
𝑘→0 ℎ→0 ℎ𝑘
Note 1. The existence of higher derivatives requires the existence of 𝑛 e corresponding
derivatives of lower order. Thus in order that 𝑓𝑦𝑥 should exist at a formh, it is necessary that
the partial derivative 𝑓𝑥 should exist in the neighbourhood of that point. But it is possible for
the limit defining 𝑓𝑦𝑥 to exist without the partial derivative 𝑓𝑥 existing. In such cases higher
derivatives cannot be said to exist.
Note 2. The existence (and even the continuity) of 𝑓𝑥𝑦 need not imply the existence of 𝑓𝑥 .

Note 3. 𝑓𝑦𝑥 and 𝑓𝑥𝑦 do not always give the same value. However, under certain conditions the
equality 𝑓𝑦𝑥 = 𝑓𝑥𝑦 can hold. We shall examine these conditions later on.

SOLVED EXAMPLES
∂2 𝑢 ∂2 𝑢
Ex. 1. If 𝑢 = 𝑎𝑥 2 + 2ℎ𝑥𝑦 + 𝑏𝑦 2, find ∂𝑦 ∂𝑥 and ∂𝑥 ∂𝑦.

Sol. We have, 𝑢 = 𝑎𝑥 2 + 2ℎ𝑥𝑦 + 𝑏𝑦 2 .


∂𝑢
∴ = 2𝑎𝑥 + 2ℎ𝑦 (treating 𝑦 as constant)
∂𝑥
∂2 𝑢 ∂ ∂𝑢 ∂
Hence = ∂𝑦 (∂𝑥 ) = ∂𝑦 (2𝑎𝑥 + 2ℎ𝑦) = 2ℎ.
∂𝑦 ∂𝑥
(treating 𝑥 as constant)
∂𝑢
Again = 2ℎ𝑥 + 2𝑏𝑦 (treating 𝑥 as constant)
∂𝑦
∂2 𝑢 ∂ ∂𝑢 ∂
∴ = ∂𝑥 (∂𝑦) = ∂𝑥 (2ℎ𝑥 + 2𝑏𝑦) = 2ℎ. (Treating 𝑦 as constant)
∂𝑥 ∂𝑦
∂2 𝑢 ∂2 𝑢
Here we note that ∂𝑥 ∂𝑦 = ∂𝑦 ∂𝑥.

𝑦 ∂𝑢 ∂𝑢
Ex. 2. If 𝑢 = 𝑓 (𝑥 ), show that 𝑥 ∂𝑥 + 𝑦 ∂𝑦 = 0.

𝑦
Sol. We have, 𝑢 = 𝑓 ( ).
𝑥

86 | P a g e

© Department of Distance & Continuing Education, Campus of Open Learning,


School of Open Learning, University of Delhi
Intermediate Mathematical Methods for Economics

∂𝑢 𝑦 𝑦
∴ = {𝑓 ′ ( )} (− 2 ) . (treating 𝑦 as constant)
∂𝑥 𝑥 𝑥
∂𝑢 𝑦 ′ 𝑦
∴ 𝑥 = − 𝑓 ( ).
∂𝑥 𝑥 𝑥
∂𝑢 𝑦 1
Again = {𝑓 ′ ( )} ( ) (treating 𝑥 as constant).
∂𝑦 𝑥 𝑥
∂𝑢 𝑦 ′ 𝑦
∴ 𝑦 = 𝑓 ( ).
∂𝑦 𝑥 𝑥
∂𝑢 ∂𝑢
Adding (1) and (2), we have 𝑥 ∂𝑥 + 𝑦 ∂𝑦 = 0.

Ex. 3. If 𝑢 = log (𝑥 3 + 𝑦 3 + 𝑧 3 − 3𝑥𝑦𝑧), show that

∂ ∂ ∂ 2 9
( + + ) 𝑢=− .
∂𝑥 ∂𝑦 ∂𝑧 (𝑥 + 𝑦 + 𝑧)2
Sol. We have, 𝑢 = log (𝑥 3 + 𝑦 3 + 𝑧 3 − 3𝑥𝑦𝑧).
Differentiating partially with respect to 𝑥, we have
∂𝑢 1
= 3 3 3
(3𝑥 2 − 3𝑦𝑧)
∂𝑥 𝑥 + 𝑦 + 𝑧 − 3𝑥𝑦𝑧
or
∂𝑢 3(𝑥 2 − 𝑦𝑧)
= 3 .
∂𝑥 𝑥 + 𝑦 3 + 𝑧 3 − 3𝑥𝑦𝑧
Similarly, by symmetry, we have
∂𝑢 3(𝑦 2 − 𝑧𝑥)
= 3
∂𝑦 𝑥 + 𝑦 3 + 𝑧 3 − 3𝑥𝑦𝑧
and
∂𝑢 3(𝑧 2 − 𝑥𝑦)
= 3 .
∂𝑧 𝑥 + 𝑦 3 + 𝑧 3 − 3𝑥𝑦𝑧
Adding (1), (2) and (3), we have
∂𝑢 ∂𝑢 ∂𝑢 3(𝑥 2 + 𝑦 2 + 𝑧 2 − 𝑦𝑧 − 𝑧𝑥 − 𝑥𝑦)
+ + =
∂𝑥 ∂𝑦 ∂𝑧 𝑥 3 + 𝑦 3 + 𝑧 3 − 3𝑥𝑦𝑧
3(𝑥 2 + 𝑦 2 + 𝑧 2 − 𝑦𝑧 − 𝑧𝑥 − 𝑥𝑦) 3
= 2 2 2
=
(𝑥 + 𝑦 + 𝑧)(𝑥 + 𝑦 + 𝑧 − 𝑦𝑧 − 𝑧𝑥 − 𝑥𝑦) 𝑥 + 𝑦 + 𝑧

87 | P a g e

© Department of Distance & Continuing Education, Campus of Open Learning,


School of Open Learning, University of Delhi
B.A. (Hons.) Economics

Now

∂ ∂ ∂ 2 ∂ ∂ ∂ ∂𝑢 ∂𝑢 ∂𝑢
( + + ) 𝑢=( + + )( + + )
∂𝑥 ∂𝑦 ∂𝑧 ∂𝑥 ∂𝑦 ∂𝑧 ∂𝑥 ∂𝑦 ∂𝑧
∂ ∂ ∂ 3
=( + + )( )
∂𝑥 ∂𝑦 ∂𝑧 𝑥 + 𝑦 + 𝑧
∂ 1 ∂ 1 ∂ 1
= 3[ ( )+ ( )+ ( )]
∂𝑥 𝑥‾ + 𝑦 + 𝑧 ∂𝑦 𝑥 + 𝑦 + 𝑧 ∂𝑧 𝑥 + 𝑦 + 𝑧
1 1 1 9
= 3 [− − − ] = −
(𝑥 + 𝑦 + 𝑧)2 (𝑥 + 𝑦 + 𝑧)2 (𝑥 + 𝑦 + 𝑧)2 (𝑥 + 𝑦 + 𝑧)2
Ex. 4. If 𝑥 = 𝑟cos 𝜃, 𝑦 = 𝑟sin 𝜃, show that
∂𝑟 ∂𝑥 ∂𝑥 ∂𝜃
= , =𝑟 ,
∂𝑥 ∂𝑟 𝑟 ∂𝜃 ∂𝑥
2 2
∂ 𝜃 ∂ 𝜃
+ = 0.
∂𝑥 2 ∂𝑦 2
∂𝜃
Also find the value of ∂𝑥 .

Sol. We have 𝑥 = 𝑟cos 𝜃.


∂𝑥
∴ = cos 𝜃. (regarding 𝜃 as constant)
∂𝑟

Also we have, 𝑟 2 = 𝑥 2 + 𝑦 2 .
∂𝑟
∴ 2𝑟 ∂𝑥 = 2𝑥
∂𝑟 𝑥 𝑟cos 𝜃
(regarding 𝑦 as constant)
=𝑟= = cos 𝜃.
∂𝑥 𝑟
∂𝑟 ∂𝑥
Thus = .
∂𝑥 ∂𝑟
∂𝑥 1 ∂𝑥
Again = −𝑟sin 𝜃. (regarding 𝑟 as constant)∴ = −sin 𝜃.
∂𝜃 𝑟 ∂𝜃

Finally, we have 𝜃 = tan−1 (𝑦/𝑥).


∂𝜃 𝑦 ∂2 𝜃 2𝑥𝑦
∴ =− 2 2
. ∴ 2
= 2 .
∂𝑥 (𝑥 + 𝑦 ) ∂𝑥 (𝑥 + 𝑦 2 )2
∂𝜃 1 1 𝑥 ∂2 𝜃 −2𝑥𝑦
Also = 2 ⋅ = . ∴ = .
∂𝑦 (1 + 𝑦 ) 𝑥 𝑥 2 + 𝑦 2 ∂𝑦 2 (𝑥 2 + 𝑦 2 )2
𝑥2
∂2 𝜃 ∂2 𝜃
Adding (1) and (2), we get ∂𝑥 2 + ∂𝑦 2 = 0.

88 | P a g e

© Department of Distance & Continuing Education, Campus of Open Learning,


School of Open Learning, University of Delhi
Intermediate Mathematical Methods for Economics

∂ ∂𝑢 ∂ ∂𝑢
Ex. 5. If 𝑢 = (1 − 2𝑥𝑦 + 𝑦 2 )−1/2, prove that ∂𝑥 {(1 − 𝑥 2 ) ∂𝑥 } + ∂𝑦 {𝑦 2 ∂𝑦} = 0.

Sol. Here 𝑢 = (1 − 2𝑥𝑦 + 𝑦 2 )−1/2.


∂𝑢 1
∴ = − (1 − 2𝑥𝑦 + 𝑦 2 )−3/2 (−2𝑦) = 𝑦𝑢3 ,
∂𝑥 2
and
∂𝑢 1
= − (1 − 2𝑥𝑦 + 𝑦 2 )−3/2 ⋅ (−2𝑥 + 2𝑦) = (𝑥 − 𝑦)𝑢3 .
∂𝑦 2
∂ ∂𝑢 ∂
Now ∂𝑥 {(1 − 𝑥 2 ) ∂𝑥 } = ∂𝑥 {(1 − 𝑥 2 ) ⋅ 𝑦𝑢3 }
∂𝑢
= 𝑦(−2𝑥)𝑢3 + 𝑦(1 − 𝑥 2 ) ⋅ 3𝑢2
∂𝑥
= −2𝑥𝑦𝑢3 + 3𝑦(1 − 𝑥 2 )𝑢2 𝑦𝑢3
= −2𝑥𝑦𝑢3 + 3𝑦 2 𝑢5 (1 − 𝑥 2 ).
∂ ∂𝑢 ∂ ∂
Also {𝑦 2 }= {𝑦 2 (𝑥 − 𝑦)𝑢3 } = {(𝑦 2 𝑥 − 𝑦 3 )𝑢3 }
∂𝑦 ∂𝑦 ∂𝑦 ∂𝑦

∂𝑢
= (2𝑥𝑦 − 3𝑦 2 )𝑢3 + (𝑦 2 𝑥 − 𝑦 3 ) ⋅ 3𝑢2
∂𝑦
= (2𝑥𝑦 − 3𝑦 2 )𝑢3 + 𝑦 2 (𝑥 − 𝑦) ⋅ 3𝑢2 ⋅ (𝑥 − 𝑦)𝑢3
= (2𝑥𝑦 − 3𝑦 2 )𝑢3 + 𝑦 2 (𝑥 − 𝑦)2 ⋅ 3𝑢5
= 2𝑥𝑦𝑢3 + 3𝑦 2 𝑢5 [(𝑥 − 𝑦)2 − 𝑢−2 ]
= 2𝑥𝑦𝑢3 + 3𝑦 2 𝑢5 [(𝑥 − 𝑦)2 − (1 − 2𝑥𝑦 + 𝑦 2 )], [∵ 𝑢−2 = 1 − 2𝑥𝑦 + 𝑦 2 ]
= 2𝑥𝑦𝑢3 + 3𝑦 2 𝑢5 [𝑥 2 − 1] = 2𝑥𝑦𝑢3 − 3𝑦 2 𝑢5 (1 − 𝑥 2 ).
Adding (1) and (2), we have
∂ ∂𝑢 ∂ 2 ∂𝑢
{(1 − 𝑥 2 ) } + {𝑦 }=0
∂𝑥 ∂𝑥 ∂𝑦 ∂𝑦
2 /4𝑡 1 ∂ ∂𝜃 ∂𝜃
EX. 6. If 𝜃 = 𝑡 𝑛 𝑒 −𝑟 , what value of 𝑛 will make 𝑟 2 ∂𝑟 (𝑟 2 ∂𝑟 ) = ∂𝑡
∂𝜃 𝑛 −𝑟 2 /4𝑡 2𝑟 𝑟 2
Sol. We have ∂𝑟 = 𝑡 ⋅ 𝑒 ⋅ (− 4𝑡 ) = − 2 𝑡 𝑛−1 𝑒 −𝑟 /4𝑡 .

89 | P a g e

© Department of Distance & Continuing Education, Campus of Open Learning,


School of Open Learning, University of Delhi
B.A. (Hons.) Economics

∂𝜃 1 2
∴ 𝑟2 = − 𝑟 3 𝑡 𝑛−1 𝑒 −𝑟 /4𝑡 .
∂𝑟 2
∂ 2 ∂𝜃 3𝑟 2 𝑛−1 −𝑟2/4𝑡 1 3 𝑛−1 −𝑟 2/4𝑡 2𝑟
∴ (𝑟 )=− 𝑡 𝑒 − 𝑟 𝑡 𝑒 ⋅ (− )
∂𝑟 ∂𝑟 2 2 4𝑡
3 2 𝑛−1 −𝑟2/4𝑡 1 4 𝑛−2 −𝑟 2/4𝑡
=− 𝑟 𝑡 𝑒 + 𝑟 𝑡 𝑒 .
2 4
1 ∂ ∂𝜃 3 2 1 2
∴ 2 (𝑟 2 ) = − 𝑡 𝑛−1 𝑒 −𝑟 /4𝑡 + 𝑟 2 𝑡 𝑛−2 𝑒 −𝑟 /4𝑡 .
𝑟 ∂𝑟 ∂𝑟 2 4

∂𝜃 2 /4𝑡 2 /4𝑡 𝑟2
Also = 𝑛𝑡 𝑛−1 𝑒 −𝑟 + 𝑡 𝑛 𝑒 −𝑟 ⋅ 4𝑡 2
∂𝑡

2 /4𝑡 1 2
= 𝑛𝑡 𝑛−1 𝑒 −𝑟 + 𝑟 2 𝑡 𝑛−2 𝑒 −𝑟 /4𝑡 .
4
1 ∂ ∂𝜃 ∂𝜃
Now 𝑟 2 ∂𝑟 (𝑟 2 ∂𝑟 ) = ∂𝑡

3 2 1 2 2 1 2
⇒ − 𝑡 𝑛−1 𝑒 −𝑟 /4𝑡 + 𝑟 2 𝑡 𝑛−2 𝑒 −𝑟 /4𝑡 = 𝑛𝑡 𝑛−1 𝑒 −𝑟 /4𝑡 + 𝑟 2 𝑡 𝑛−2 𝑒 −𝑟 /4𝑡
2 4 4
3 −𝑛−1 −𝑟 2/4𝑡 2
⇒ − 𝑡 𝑒 = 𝑛𝑡 𝑛−1 𝑒 −𝑟 /4𝑡 , for all possible values of 𝑟 and 𝑡
2
1 1
Ex. 7. Let 𝑓(𝑥, 𝑦) = 𝑥sin + 𝑦sin , (𝑥, 𝑦) ≠ (0,0);
𝑥 𝑦

1 1
𝑓(0, 𝑦) = 𝑦sin , 𝑦 ≠ 0; 𝑓(𝑥, 0) = 𝑥sin , 𝑥 ≠ 0,
𝑦 𝑥
and 𝑓(0,0) = 0. Examine the existence of 𝑓𝑥 and 𝑓𝑦𝑥 at (0,0).
Sol. We have
1
𝑓(0 + ℎ, 0) − 𝑓(0,0) ℎsin ℎ − 0 1
𝑓𝑥 (0,0) = lim = lim = lim sin ,
ℎ→0 ℎ ℎ→0 ℎ ℎ→0 ℎ
which does not exist. So 𝑓𝑥 (0,0) does not exist.
𝑓(ℎ,𝑘)−𝑓(ℎ,0)−𝑓(0,𝑘)+𝑓(0,0)
Now lim𝑘→0 [limℎ→0 { }]
ℎ𝑘
1 1 1 1
ℎsin + 𝑘sin − ℎsin − 𝑘sin +0
ℎ 𝑘 ℎ 𝑘
= lim lim
𝑘→0 ℎ→0 ℎ𝑘
0
= lim lim = 0.
𝑘→0 ℎ→0 ℎ𝑘

90 | P a g e

© Department of Distance & Continuing Education, Campus of Open Learning,


School of Open Learning, University of Delhi
Intermediate Mathematical Methods for Economics

Thus, the limit used for defining the second derivative 𝑓𝑦𝑥 does exist. Despite this fact, the
derivative 𝑓𝑦𝑥 (0,0) cannot be said to exist, since 𝑓𝑥 (0,0) does not exist.
Ex. 8. Let 𝑓(𝑥, 𝑦) = 𝑔(𝑥), where 𝑔 is nowhere differentiable. Show that 𝑓𝑥𝑦 exists and is
continuous and yet 𝑓𝑥 does not exist.
Sol. We have 𝑓(𝑥, 𝑦) = 𝑔(𝑥).
∂𝑓
∴ 𝑓𝑥 = = 𝑔′ (𝑥)
∂𝑥
which does not exist since 𝑔 is nowhere differentiable.
Again 𝑔(𝑥) is a function of 𝑥 only so its partial derivative with respect to 𝑦 is zero.
∂𝑓 ∂
∴ 𝑓𝑦 = = 𝑔(𝑥) = 0
∂𝑦 ∂𝑦

Now 𝑓𝑥𝑦 = ∂𝑥 𝑓𝑦 = 0.
Thus 𝑓𝑥𝑦 exists and has the value 0 at every point. Being a constant function, 𝑓𝑥𝑦 is
continuous.
7.3.2 Interchange of the Order of the Differentiation
∂2 𝑧 ∂2 𝑧
If 𝑧 = 𝑓(𝑥, 𝑦) is a function of two independent variables 𝑥 and 𝑦, then ∂𝑦 ∂𝑥 and ∂𝑥 ∂𝑦 do not
always give the same value.
∂2 𝑧 ∂2 𝑧
However, under certain conditions the equality ∂𝑦 ∂𝑥 = ∂𝑥 ∂𝑦

can hold. These conditions we shall examine. First, we give some examples in which 𝑓𝑥𝑦 may
be different from 𝑓𝑦𝑥 .

SOLVED EXAMPLES
𝑥𝑦(𝑥 2 −𝑦 2 )
Ex. 1. If 𝑓(𝑥, 𝑦) = ; (𝑥, 𝑦) ≠ (0,0) and 𝑓(0,0) = 0 show that 𝑓𝑥𝑦 (0,0) ≠ 𝑓𝑦𝑥 (0,0).
𝑥 2 +𝑦 2
∂ ∂𝑓 ∂ ∂𝑓
Sol. Let us define 𝑓𝑥𝑦 = ∂𝑥 (∂𝑦) and 𝑓𝑦𝑥 = ∂𝑦 (∂𝑥).
𝑓𝑦 (0+ℎ,0)−𝑓𝑦 (0,0)
Thus 𝑓𝑥𝑦 (0,0) = limℎ→0 .

𝑓(ℎ,0+𝑘)−𝑓(ℎ,0)
Now 𝑓𝑦 (0 + ℎ, 0) = 𝑓𝑦 (ℎ, 0) = lim𝑘→0 𝑘

ℎ𝑘(ℎ2 −𝑘 2 )
−0 ℎ(ℎ2 − 𝑘 2 )
ℎ2 +𝑘 2
= lim = lim = ℎ,
𝑘→0 𝑘 𝑘→0 ℎ2 + 𝑘 2

91 | P a g e

© Department of Distance & Continuing Education, Campus of Open Learning,


School of Open Learning, University of Delhi
B.A. (Hons.) Economics

and
𝑓(0,0 + 𝑘) − 𝑓(0,0) 0
𝑓𝑦 (0,0) = lim = lim = lim 0 = 0.
𝑘→0 𝑘 𝑘→0 𝑘 𝑘→0

ℎ−0
∴ 𝑓𝑥𝑦 (0,0) = lim = lim 1 = 1.
ℎ→0 ℎ ℎ→0
𝑓𝑥 (0,0 + 𝑘) − 𝑓𝑥 (0,0)
Again 𝑓𝑦𝑥 (0,0) = lim .
𝑘→0 𝑘
𝑓(0 + ℎ, 𝑘) − 𝑓(0, 𝑘)
But 𝑓𝑥 (0,0 + 𝑘) = 𝑓𝑥 (0, 𝑘) = lim
ℎ→0 ℎ
ℎ𝑘(ℎ2 −𝑘 2 )
−0
𝑘(ℎ2 − 𝑘 2 )
ℎ2 +𝑘 2
= lim = lim = −𝑘
ℎ→0 ℎ ℎ→0 ℎ2 + 𝑘 2
𝑓(ℎ, 0) − 𝑓(0,0)
𝑓𝑥 (0,0) = lim = lim 0 = 0.
ℎ→0 ℎ ℎ→0
−𝑘 − 0
∴ 𝑓𝑦𝑥 (0,0) = lim = lim (−1) = −1.
𝑘→0 𝑘 𝑘→0

Hence 𝑓𝑥𝑦 (0,0) ≠ 𝑓𝑦𝑥 (0,0) in this case.


𝑥𝑦(𝑦 2 −𝑥 2 )
Ex. 2. If 𝑓(𝑥, 𝑦) = when (𝑥, 𝑦) ≠ (0,0) and 𝑓(0,0) = 0, calculate 𝑓𝑥𝑦 (0,0) and
𝑦 2 +𝑥 2
𝑓𝑦𝑥 (0,0). Are they equal ?
∂ ∂𝑓 ∂ ∂𝑓
Sol. Let us define 𝑓𝑥𝑦 = ∂𝑦 (∂𝑥) and 𝑓𝑦𝑥 = ∂𝑥 (∂𝑦).
𝑓𝑥 (0,0+𝑘)−𝑓𝑥 (0,0)
Then 𝑓𝑥𝑦 (0,0) = lim𝑘→0 𝑘
.
𝑓(0+ℎ,𝑘)−𝑓(0,𝑘)
𝑓𝑥 (0,0 + 𝑘) = 𝑓𝑥 (0, 𝑘) = limℎ→0 𝑘(𝑘 2 −ℎ2 )

Now ℎ𝑘(𝑘 2 −ℎ2 )
= limℎ→0 =
𝑓(ℎ,𝑘)−𝑓(0,𝑘) 1 𝑘 2 +ℎ2
= limℎ→0 = limℎ→0 ℎ [ − 0]
ℎ 𝑘 2 +ℎ2
𝑘,
and

92 | P a g e

© Department of Distance & Continuing Education, Campus of Open Learning,


School of Open Learning, University of Delhi
Intermediate Mathematical Methods for Economics

𝑓(0 + ℎ, 0) − 𝑓(0,0) 𝑓(ℎ, 0) − 𝑓(0,0)


𝑓𝑥 (0,0) = lim = lim
ℎ→0 ℎ ℎ→0 ℎ
0−0
= lim = lim 0 = 0.
ℎ→0 ℎ ℎ→0
𝑘−0
∴ 𝑓𝑥𝑦 (0,0) = lim = lim 1 = 1.
𝑘→0 𝑘 𝑘→0
𝑓𝑦 (0 + ℎ, 0) − 𝑓𝑦 (0,0)
Again 𝑓𝑦𝑥 (0,0) = lim .
ℎ→0 ℎ
𝑓(ℎ, 0 + 𝑘) − 𝑓(ℎ, 0)
But 𝑓𝑦 (0 + ℎ, 0) = 𝑓𝑦 (ℎ, 0) = lim
𝑘→0 𝑘
𝑓(ℎ, 𝑘) − 𝑓(ℎ, 0) 1 ℎ𝑘(𝑘 2 − ℎ2 )
= lim = lim [ − 0]
𝑘→0 𝑘 𝑘→0 𝑘 𝑘 2 + ℎ2
ℎ(𝑘 2 − ℎ2 )
= lim = −ℎ
𝑘→0 𝑘 2 + ℎ2

and
𝑓(0,0 + 𝑘) − 𝑓(0,0) 𝑓(0, 𝑘) − 𝑓(0,0)
𝑓𝑦 (0,0) = lim = lim
𝑘→0 𝑘 𝑘→0 𝑘
0−0
= lim = lim 0 = 0.
𝑘→0 𝑘 𝑘→0
−ℎ − 0
∴ 𝑓𝑦𝑥 (0,0) = lim = lim − 1 = −1.
ℎ→0 ℎ ℎ→0

We observe that here 𝑓𝑥𝑦 (0,0) ≠ 𝑓𝑦𝑥 (0,0).

Ex. 3. Examine the equality of 𝑓𝑥𝑦 (0,0) and 𝑓𝑦𝑥 (0,0) for the function

(𝑥 2 + 𝑦 2 )tan−1 (𝑦/𝑥), 𝑥 ≠ 0
𝑓(𝑥, 𝑦) = {
𝜋(𝑦 2 /2) , 𝑥 = 0.
∂ ∂𝑓 ∂ ∂𝑓
So1. Let us define 𝑓𝑥𝑦 = ∂𝑦 (∂𝑥) and 𝑓𝑦𝑥 = ∂𝑥 (∂𝑦).
𝑓𝑥 (0,0+𝑘)−𝑓𝑥 (0,0) 𝑓𝑥 (0,𝑘)−𝑓𝑥 (0,0)
Then 𝑓𝑥𝑦 (0,0) = lim𝑘→0 = lim𝑘→0 .
𝑘 𝑘

93 | P a g e

© Department of Distance & Continuing Education, Campus of Open Learning,


School of Open Learning, University of Delhi
B.A. (Hons.) Economics

𝑓(0 + ℎ, 𝑘) − 𝑓(0, 𝑘) 𝑓(ℎ, 𝑘) − 𝑓(0, 𝑘)


Now 𝑓𝑥 (0, 𝑘) = lim = lim
ℎ→0 ℎ ℎ→0 ℎ
2 2 −1
(ℎ + 𝑘 )tan (𝑘/ℎ) − 𝜋(𝑘 /2) 2
0
= lim [ Form ]
ℎ→0 ℎ 0
2 2 1 𝑘 −1 𝑘
(ℎ + 𝑘 ) ⋅ ⋅ (− ℎ2) + 2ℎtan (ℎ) − 0
1+(𝑘 2 /ℎ2 )
= lim ,
ℎ→0 1
by L' Hospital's rule
𝑘
= lim [−𝑘 + 2ℎtan−1 ( )] = −𝑘
ℎ→0 ℎ
𝑓(0 + ℎ, 0) − 𝑓(0,0) 𝑓(ℎ, 0) − 𝑓(0,0)
𝑓𝑥 (0,0) = lim = lim
ℎ→0 ℎ ℎ→0 ℎ
0−0
= lim = lim 0 = 0.
ℎ→0 ℎ ℎ→0
−𝑘 − 0
∴ 𝑓𝑥𝑦 (0,0) = lim = lim − 1 = −1.
𝑘→0 𝑘 𝑘→0
𝑓𝑦 (0 + ℎ, 0) − 𝑓𝑦 (0,0) 𝑓𝑦 (ℎ, 0) − 𝑓𝑦 (0,0)
Again 𝑓𝑦𝑥 (0,0) = lim = lim .
ℎ→0 ℎ ℎ→0 ℎ
𝑓(ℎ, 0 + 𝑘) − 𝑓(ℎ, 0) 𝑓(ℎ, 𝑘) − 𝑓(ℎ, 0)
But 𝑓𝑦 (ℎ, 0) = lim = lim
𝑘→0 𝑘 𝑘→0 𝑘
(ℎ2 + 𝑘 2 )tan−1 (𝑘/ℎ) − 0 𝑘 2 tan−1 (𝑘/ℎ)
= lim = lim (ℎ + ) ⋅
𝑘→0 𝑘 𝑘→0 ℎ 𝑘/ℎ
−1 −1 2)
tan (𝑘/ℎ) tan 𝑡 1/(1 + 𝑡
= ℎ [∵ lim = lim = lim = 1]
𝑘→0 𝑘/ℎ 𝑡→0 𝑡 𝑡→0 1
𝑓(0,0 + 𝑘) − 𝑓(0,0) 𝑓(0, 𝑘) − 𝑓(0,0)
𝑓𝑦 (0,0) = lim = lim
𝑘→0 𝑘 𝑘→0 𝑘
2
𝜋(𝑘 /2) − 0 𝜋
= lim = lim 𝑘 = 0.
𝑘→0 𝑘 𝑘→0 2

ℎ−0
∴ 𝑓𝑦𝑥 (0,0) = lim = lim 1 = 1.
ℎ→0 ℎ ℎ→0

From (1) and (2), we observe that 𝑓𝑥𝑦 (0,0) ≠ 𝑓𝑦𝑥 (0,0).
7.3.3 Sufficient Conditions for the Equality of Second Order Derivatives:
We have seen that the equality 𝑓𝑥𝑦 = 𝑓𝑦𝑥 does not always hold. We now give two theorems the
object of which is to set out precisely under what conditions it is safe to assume that 𝑓𝑥𝑦 = 𝑓𝑦𝑥
at a point i.e., sufficient conditions for the equality of 𝑓𝑥𝑦 and 𝑓𝑦𝑥

94 | P a g e

© Department of Distance & Continuing Education, Campus of Open Learning,


School of Open Learning, University of Delhi
Intermediate Mathematical Methods for Economics

7.3.4 Schwarz’s and Young Theorem:


Theorem 1. (Schwarz's Theorem).
If 𝒇: 𝑿 → 𝐑, 𝑿 ⊆ 𝐑𝟐 and (𝒂, 𝒃) ∈ 𝑿 is. such that (i) 𝒇𝒙 exists in a neighbourhood 𝑵(𝒂, 𝒃)
of (𝒂, 𝒃) and (ii) 𝒇𝒙𝒚 is continuous at (𝒂, 𝒃), then 𝒇𝒚𝒙 (𝒂, 𝒃) exists and is equal to
𝒇𝒙𝒚 , (𝒂, 𝒃)

Proof. Since 𝑓𝑥𝑦 is continuous at (𝑎, 𝑏), 𝑓𝑦 and 𝑓𝑥𝑦 exist at every point in 𝑁(𝑎, 𝑏).
Let 𝜙(ℎ, 𝑘) = 𝑓(𝑎 + ℎ, 𝑏 + 𝑘) − 𝑓(𝑎 + ℎ, 𝑏) − 𝑓(𝑎, 𝑏 + 𝑘) + 𝑓(𝑎, 𝑏) where (𝑎 + ℎ, 𝑏 +
𝑘) ∈ 𝑁(𝑎, 𝑏). then
If we take 𝑔(𝑦) = 𝑓(𝑎 + ℎ, 𝑦) − 𝑓(𝑎, 𝑦) and
𝑔(𝑏 + 𝑘) = 𝑓(𝑎 + ℎ, 𝑏 + 𝑘) − 𝑓(𝑎, 𝑏 + 𝑘)
𝑔(𝑏) = 𝑓(𝑎 + ℎ, 𝑏) − 𝑓(𝑎, 𝑏).
∴ 𝜙(ℎ, 𝑘) = 𝑔(𝑏 + 𝑘) − 𝑔(𝑏)
Since 𝑓𝑦 exists at every point in 𝑁(𝑎, 𝑏), 𝑓(𝑎 + ℎ, 𝑦) − 𝑓(𝑎, 𝑦) is differentiable at each point
of 𝑁(𝑎, 𝑏) i.e., 𝑔(𝑦) is differentiable in [𝑏, 𝑏 + 𝑘] ⊆ 𝑁(𝑎, 𝑏).
∴ Using mean value theorem for 𝑔(𝑦) in [𝑏, 𝑏 + 𝑘], we get
𝑔(𝑏 + 𝑘) − 𝑔(𝑏) = 𝑘𝑔′ (𝑏 + 𝜃𝑘) where 0 < 𝜃 < 1.
∴ 𝜙(ℎ, 𝑘) = 𝑘[𝑓𝑦 (𝑎 + ℎ, 𝑏 + 𝜃𝑘) − 𝑓𝑦 (𝑎, 𝑏 + 𝜃𝑘)]

[From (2), 𝑔′ (𝑦) = 𝑓𝑦 (𝑎 + ℎ, 𝑦) − 𝑓𝑦 (𝑎, 𝑦) ]

Now 𝑓𝑥𝑦 exists at each point in 𝑁(𝑎, 𝑏) so 𝑓𝑦 (𝑥, 𝑏 + 𝜃𝑘) is differentiable at each point in
𝑁(𝑎, 𝑏) i.e., 𝑓𝑦 (𝑥, 𝑏 + 𝜃𝑘) is differentiable in [𝑎, 𝑎 + ℎ] ⊆ 𝑁(𝑎, 𝑏).
∴ Using mean-value theorem for 𝑓𝑦 (𝑥, 𝑏 + 𝜃𝑘) in [𝑎, 𝑎 + ℎ], we get

𝑓𝑦 (𝑎 + ℎ, 𝑏 + 𝜃𝑘) − 𝑓𝑦 (𝑎, 𝑏 + 𝜃𝑘) = ℎ𝑓𝑥𝑦 (𝑎 + 𝜃1 ℎ, 𝑏 + 𝜃𝑘)


0 < 𝜃1 < 1.
∴ 𝜙(ℎ, 𝑘) = 𝑘[ℎ𝑓𝑥𝑦 (𝑎 + 𝜃1 ℎ, 𝑏 + 𝜃𝑘]; 0 < 𝜃, 𝜃1 < 1
or

𝜙(ℎ, 𝑘)
= 𝑓𝑥𝑦 (𝑎 + 𝜃1 ℎ, 𝑏 + 𝜃𝑘)
ℎ𝑘
1 𝑓(𝑎 + ℎ, 𝑏 + 𝑘) − 𝑓(𝑎, 𝑏 + 𝑘) 𝑓(𝑎 + ℎ, 𝑏) − 𝑓(𝑎, 𝑏)
[ − ]
𝑘 ℎ ℎ
= 𝑓𝑥𝑦 (𝑎 + 𝜃1 ℎ, 𝑏 + 𝜃𝑘).

Taking the limits as ℎ → 0, we get

95 | P a g e

© Department of Distance & Continuing Education, Campus of Open Learning,


School of Open Learning, University of Delhi
B.A. (Hons.) Economics

1
[𝑓 (𝑎, 𝑏 + 𝑘) − 𝑓𝑥 (𝑎, 𝑏)] = lim 𝑓𝑥𝑦 (𝑎 + 𝜃1 ℎ, 𝑏 + 𝜃𝑘)
𝑘 𝑥 ℎ→0
𝑓𝑥 (𝑎, 𝑏 + 𝑘) − 𝑓𝑥 (𝑎, 𝑏)
⇒ lim
𝑘→0 𝑘
= lim [lim 𝑓𝑥𝑦 (𝑎 + 𝜃1 ℎ, 𝑏 + 𝜃𝑘)] = 𝑓𝑥𝑦 (𝑎, 𝑏).
𝑘→0 ℎ→0

Thus 𝑓𝑦𝑥 exists and 𝑓𝑦𝑥 (𝑎, 𝑏) = 𝑓𝑥𝑦 (𝑎, 𝑏).

Similarly, we can prove that if 𝑓𝑦 exists in 𝑁(𝑎, 𝑏) and 𝑓𝑦𝑥 is continuous at (𝑎, 𝑏), then
𝑓𝑥𝑦 (𝑎, 𝑏) exists and is equal to 𝑓𝑦𝑥 (𝑎, 𝑏).

Note. If 𝑓𝑥𝑦 and 𝑓𝑦𝑥 are both continuous at (𝑎, 𝑏), then 𝑓𝑥𝑦 (𝑎, 𝑏) = 𝑓𝑦𝑥 (𝑎, 𝑏).

Theorem 2. (Young's Theorem).


If 𝒇: 𝑿 → 𝐑, 𝑿 ⊆ 𝐑𝟐 and (𝒂, 𝒃) ∈ 𝑿 is such that 𝒇𝒙 , 𝒇𝒚 are differentiable at b. b), then
𝒇𝒙𝒚 (𝒂, 𝒃) = 𝒇𝒚𝒙 (𝒂, 𝒃)

Proof. Since 𝑓𝑥 , 𝑓𝑦 are differentiable at (𝑎, 𝑏) so 𝑓𝑥 , 𝑓𝑦 exist in a neighbourhood Nr (𝑎, 𝑏) of


(𝑎, 𝑏) and 𝑓𝑥𝑥 , 𝑓𝑦𝑥 , 𝑓𝑥𝑦 , 𝑓𝑦𝑦 exist at (𝑎, 𝑏).

Let 𝜙(ℎ, ℎ) = 𝑓(𝑎 + ℎ, 𝑏 + ℎ) − 𝑓(𝑎 + ℎ, 𝑏) − 𝑓(𝑎, 𝑏 + ℎ) + 𝑓(𝑎, 𝑏) and 𝑔(𝑦) = 𝑓(𝑎 +


ℎ, 𝑦) − 𝑓(𝑎, 𝑦).
Then 𝜙(ℎ, ℎ) = 𝑔(𝑏 + ℎ) − 𝑔(𝑏).
Now, using mean value theorem to 𝑔(𝑦) in [𝑏, 𝑏 + ℎ],
𝑔(𝑏 + ℎ) − 𝑔(𝑏) = ℎ𝑔′ (𝑏 + 𝜃ℎ),0 < 𝜃 < 1.
-----(1)
∴ 𝜙(ℎ, ℎ) = ℎ[𝑓𝑦 (𝑎 + ℎ, 𝑏 + 𝜃ℎ) − 𝑓𝑦 (𝑎, 𝑏 + 𝜃ℎ)].

Since 𝑓𝑦 is differentiable at (𝑎, 𝑏), by definition, we have

𝑓𝑦 (𝑎 + ℎ, 𝑏 + 𝜃ℎ) − 𝑓𝑦 (𝑎, 𝑏)
= ℎ𝑓𝑥𝑦 (𝑎, 𝑏) + 𝜃ℎ𝑓𝑦𝑦 (𝑎, 𝑏) + √(ℎ2 + 𝜃 2 ℎ2 )𝜙1 (ℎ, ℎ)
𝑓𝑦 (𝑎, 𝑏 + 𝜃ℎ) − 𝑓𝑦 (𝑎, 𝑏) = 𝜃ℎ𝑓𝑦𝑦 (𝑎, 𝑏) + 𝜃ℎ𝜙2 (ℎ, ℎ)
𝜙1 → 0 and 𝜙2 → 0 as ℎ → 0.
From (1), (2) and (3) we conclude that

96 | P a g e

© Department of Distance & Continuing Education, Campus of Open Learning,


School of Open Learning, University of Delhi
Intermediate Mathematical Methods for Economics

𝜙(ℎ, ℎ)
= 𝑓𝑥𝑦 (𝑎, 𝑏) + √(1 + 𝜃 2 )(ℎ, ℎ) − 𝜃𝜙2 (ℎ, ℎ)
ℎ2
𝜙(ℎ, ℎ)
⇒ lim = 𝑓𝑥𝑦 (𝑎, 𝑏).
ℎ→0 ℎ2
Similarly, if in place of 𝑔(𝑦) we take 𝐹(𝑥) = 𝑓(𝑥, 𝑏 + ℎ) − 𝑓(𝑥, 𝑏)
𝜙(ℎ,ℎ)
we can prove that limℎ→0 = 𝑓𝑦𝑥 (𝑎, 𝑏).
ℎ2
Hence 𝑓𝑥𝑦 (𝑎, 𝑏) = 𝑓𝑦𝑥 (𝑎, 𝑏).
Note. The conditions of above two theorems are only sufficient but not necessary.
BOUNDED FUNCTION.
Let 𝑓: 𝑋 → 𝐑, 𝑋 ⊆ 𝐑2 be a function of two variables. If the set
𝑓(𝑋) = {𝑓(𝑥, 𝑦) ∈ 𝐑: (𝑥, 𝑦) ∈ 𝑋}
is bounded, then we say that 𝑓 is bounded on 𝑋.
Theorem 1. (Mean-value theorem).
If 𝑓: 𝑋 → 𝐑, 𝑋 ⊆ 𝐑2 and (𝑎, 𝑏) ∈ 𝑋 be such that (i) 𝑓𝑦 (𝑎, 𝑏) exists, (ii) 𝑓𝑥 exists throughout 𝑎
nhd 𝑁(𝑎, 𝑏) of (𝑎, 𝑏), then for every point (𝑎 + ℎ, 𝑏 + 𝑘) ∈ 𝑁(𝑎, 𝑏)

𝑓(𝑎 + ℎ, 𝑏 + 𝑘) − 𝑓(𝑎, 𝑏) = ℎ𝑓𝑥 (𝑎 + 𝜃ℎ, 𝑏 + 𝑘) + 𝑘[𝑓𝑦 (𝑎, 𝑏) + 𝜙(𝑘)]

where 0 < 𝜃 < 1 and lim𝑘→0 𝜙(𝑘) = 0.


Proof. We have 𝑓(𝑎 + ℎ, 𝑏 + 𝑘) − 𝑓(𝑎, 𝑏)
= 𝑓(𝑎 + ℎ, 𝑏 + 𝑘) − 𝑓(𝑎, 𝑏 + 𝑘) + 𝑓(𝑎, 𝑏 + 𝑘) − 𝑓(𝑎, 𝑏).
Since (𝑎 + ℎ, 𝑏 + 𝑘) ∈ 𝑁(𝑎, 𝑏) and 𝑓𝑥 exists in 𝑁(𝑎, 𝑏), by Lagrange's mean value theorem
we have
𝑓(𝑎 + ℎ, 𝑏 + 𝑘) − 𝑓(𝑎, 𝑏 + 𝑘) = ℎ𝑓𝑥 (𝑎 + 𝜃ℎ, 𝑏 + 𝑘)
where 0 < 𝜃 < 1.
Also, since 𝑓𝑦 (𝑎, 𝑏) exists, we have
𝑓(𝑎, 𝑏 + 𝑘) − 𝑓(𝑎, 𝑏)
lim = 𝑓𝑦 (𝑎, 𝑏).
𝑘→0 𝑘
∴ 𝑓(𝑎, 𝑏 + 𝑘) − 𝑓(𝑎, 𝑏) = 𝑘𝑓𝑦 (𝑎, 𝑏) + 𝑘𝜙(𝑘)

where 𝜙(𝑘) → 0 as 𝑘 → 0.
Using (2), (3) in (1), we get the required result.
97 | P a g e

© Department of Distance & Continuing Education, Campus of Open Learning,


School of Open Learning, University of Delhi
B.A. (Hons.) Economics

Now we shall prove a sufficient condition for continuity.


Theorem 2. If 𝑓: 𝑋 → 𝐑, 𝑋 ⊆ 𝐑2 , (𝑎, 𝑏) ∈ 𝑋 is such that (i) 𝑓𝑥 exists and is bounded on a
neighbourhood 𝑁(𝑎, 𝑏) of (𝑎, 𝑏) and (ii) 𝑓𝑦 (𝑎, 𝑏) exists, then 𝑓 is continuous at (𝑎, 𝑏).

Proof. Since the conditions of mean-value theorem are satisfied by 𝑓 so for every (𝑎 + ℎ, 𝑏 +
𝑘) ∈ 𝑁(𝑎, 𝑏) we have

𝑓(𝑎 + ℎ, 𝑏 + 𝑘) − 𝑓(𝑎, 𝑏) = ℎ𝑓𝑥 (𝑎 + 𝜃ℎ, 𝑏 + 𝑘) + 𝑘[𝑓𝑦 (𝑎, 𝑏) + 𝜙(𝑘)]

where 0 < 𝜃 < 1 and 𝜙(𝑘) → 0 as 𝑘 → 0.


Since 𝑓𝑥 is bounded on 𝑁(𝑎, 𝑏) and (𝑎 + ℎ, 𝑏 + 𝑘) ∈ 𝑁(𝑎, 𝑏), 𝑓𝑥 (𝑎 + 𝜃ℎ, 𝑏 + 𝑘) is bounded.
∴ lim ℎ𝑓𝑥 (𝑎 + 𝜃ℎ, 𝑏 + 𝑘) = 0.
ℎ→0

Also lim𝑘𝑓𝑦 (𝑎, 𝑏) = 0, lim𝜙(𝑘) = 0 as 𝑘 → 0.


Taking limit of both sides of (1) as (ℎ, 𝑘) → (0,0), we get
lim {𝑓(𝑎 + ℎ, 𝑏 + 𝑘) − 𝑓(𝑎, 𝑏)} = 0.
(ℎ,𝑘)→(0,0)
lim 𝑓(𝑎 + ℎ, 𝑏 + 𝑘) = 𝑓(𝑎, 𝑏).
(ℎ,𝑘)→(0,0)

Hence 𝑓 is continuous at (𝑎, 𝑏).


Theorem 3. (A sufficient condition for differentiability). If 𝒇: 𝑿 → 𝐑, 𝑿 ⊆ 𝐑2 , (𝑎, 𝑏) ∈ 𝑋 is
such that (i) 𝑓𝑥 is contine ous at (𝑎, 𝑏) and (ii) 𝑓𝑦 (𝑎, 𝑏) exists then 𝑓 is differentiable at (𝑎, 𝑏).

Proof. Since 𝑓𝑥 is continuous at (𝑎, 𝑏), 𝑓𝑥 exists in a nhd 𝑁(𝑎, 𝑏) of (𝑎, 𝑏). Also 𝑓𝑦 (𝑎, 𝑏) exists.
Thus the function 𝑓 satisfies the conditions of mean value theorem and so for each (𝑎 + ℎ, 𝑏 +
𝑘) ∈ 𝑁(𝑎, 𝑏), we have

𝑓(𝑎 + ℎ, 𝑏 + 𝑘) − 𝑓(𝑎, 𝑏) = ℎ𝑓𝑥 (𝑎 + 𝜃ℎ, 𝑏 + 𝑘) + 𝑘[𝑓𝑦 (𝑎, 𝑏) + 𝜙1 (𝑘)] … (1)

where 0 < 𝜃 < 1 and 𝜙1 (𝑘) → 0 as 𝑘 → 0.


Again 𝑓𝑥 is continuous at (𝑎, 𝑏) gives
Using (2) in (1), we get

98 | P a g e

© Department of Distance & Continuing Education, Campus of Open Learning,


School of Open Learning, University of Delhi
Intermediate Mathematical Methods for Economics

𝑓(𝑎 + ℎ, 𝑏 + 𝑘) − 𝑓(𝑎, 𝑏) = ℎ{𝑓𝑥 (𝑎, 𝑏) + 𝜙2 (ℎ, 𝑘)} + 𝑘{𝑓𝑦 (𝑎, 𝑏) + 𝜙1 (𝑘)}


= ℎ𝑓𝑥 (𝑎, 𝑏) + 𝑘𝑓𝑦 (𝑎, 𝑏) + {ℎ𝜙2 (ℎ, 𝑘) + 𝑘𝜙1 (𝑘)}
= ℎ𝑓𝑥 (𝑎, 𝑏) + 𝑘𝑓𝑦 (𝑎, 𝑏)
ℎ 𝑘
+ √(ℎ2 + 𝑘 2 ) { 𝜙2 (ℎ, 𝑘) + 𝜙1 (𝑘)}
√(ℎ2 + 𝑘 2 ) √(ℎ2 + 𝑘 2 )
= ℎ𝑓𝑥 (𝑎, 𝑏) + 𝑘𝑓𝑦 (𝑎, 𝑏) + √(ℎ2 + 𝑘 2 )𝜙(ℎ, 𝑘)

where
ℎ 𝑘
𝜙(ℎ, 𝑘) = 𝜙2 (ℎ, 𝑘) + 𝜙1 (𝑘).
√(ℎ2 + 𝑘 2 ) √(ℎ2 + 𝑘 2 )
ℎ 𝑘
Since , are bounded and
√(ℎ2 +𝑘 2 ) √(ℎ2 +𝑘 2 )

lim 𝜙2 (ℎ, 𝑘) = 0, lim 𝜙1 (𝑘) = 0


(ℎ,𝑘)→(0,0) 𝑘→0

so we get lim𝜙(ℎ, 𝑘) = 0 as (ℎ, 𝑘) → (0,0).


Also, the numbers 𝑓𝑥 (𝑎, 𝑏), 𝑓𝑦 (𝑎, 𝑏) are independent of ℎ, 𝑘.

Hence from (3), 𝑓 is differentiable at (𝑎, 𝑏).


Note 1. Similarly we can prove that if (i) 𝑓𝑥 (𝑎, 𝑏) exists and (ii) 𝑓𝑦 is continuous at (𝑎, 𝑏)
then 𝑓 is differentiable at (𝑎, 𝑏).
Note 2. The conditions of the theorem are not necessary for differentiability as the following
example illustrates:
Example. Let 𝑓 be the function defined by
1 1
𝑓(𝑥, 𝑦) = 𝑥 2 sin ( ) + 𝑦 2 sin ( ) ; (𝑥, 𝑦) ≠ (0,0)
𝑥 𝑦
1
𝑓(𝑥, 0) = 𝑥 2 sin ( ) ; 𝑥 ≠ 0
𝑥
1
𝑓(0, 𝑦) = 𝑦 2 sin ( ) ; 𝑦 ≠ 0 and 𝑓(0,0) = 0.
𝑦

99 | P a g e

© Department of Distance & Continuing Education, Campus of Open Learning,


School of Open Learning, University of Delhi
B.A. (Hons.) Economics

1 1 1 1 1
For 𝑥 ≠ 0, 𝑓𝑥 (𝑥, 𝑦) = 2𝑥sin (𝑥) + 𝑥 2 cos (𝑥) ⋅ (− 𝑥 2 ) = 2𝑥sin (𝑥) − cos (𝑥).
1 1
Also 𝑓𝑦 (𝑥, 𝑦) = 2𝑦sin (𝑦) − cos (𝑦) for 𝑦 ≠ 0 and 𝑓𝑥 (0, 𝑦) = 0, 𝑓𝑦 (𝑥, 0) = 0.
1 1
2𝑥sin (𝑥) − cos (𝑥) ; 𝑥 ≠ 0
Thus 𝑓𝑥 (𝑥, 𝑦) = {
0; 𝑥 = 0.
1 1
∴ lim 𝑓𝑥 (𝑥, 𝑦) = lim 2𝑥sin − lim cos ,
𝑥→0 𝑥→0 𝑥 𝑥→0 𝑥
1
which does not exist since lim𝑥→0 cos does not exist. Hence 𝑓𝑥 (𝑥, 𝑦) is not continuous at
𝑥
(0,0).
Similarly, we can show that 𝑓𝑦 (𝑥, 𝑦) is not continuous at (0,0). Now
1 1
𝑓(0 + ℎ, 0 + 𝑘) − 𝑓(0,0) = ℎ2 sin + 𝑘 2 sin
ℎ 𝑘
ℎ2 1 𝑘2 1 so that 𝐴 = 0, 𝐵 = 0 are
= 0. ℎ + 0. 𝑘 + √(ℎ2 + 𝑘2) { sin + sin 𝑘}
√(ℎ2 +𝑘 2 ) ℎ √(ℎ2 +𝑘 2 )
independent of ℎ, 𝑘.
ℎ2 1 𝑘2 1 1 1
Also 𝜙(ℎ, 𝑘) = sin + sin 𝑘. Since limℎ→0 ℎsin = 0, lim𝑘→0 𝑘sin =0
√(ℎ2 +𝑘 2) ℎ √(ℎ2 +𝑘 2 ) ℎ 𝑘
ℎ 𝑘
and , are bounded so lim𝜙(ℎ, 𝑘) = 0 as (ℎ, 𝑘) → (0,0).
√(ℎ2 +𝑘 2 ) √(ℎ2 +𝑘 2 )

∴ 𝑓 is differentiable at (0,0).
SOLVED EXAMPLE
1
Ex. 1. Let 𝑓(𝑥, 𝑦) = 4 (𝑥 2 + 𝑦 2 )log (𝑥 2 + 𝑦 2 ) for (𝑥, 𝑦) ≠ (0,0) and 𝑓(0,0) = 0. Show that
𝑓𝑥𝑦 = 𝑓𝑦𝑥 at all points (𝑥, 𝑦). Also show that neither of the derivatives is continuous in 𝑥 − 𝑦
at the origin.
Sol. We have for (𝑥, 𝑦) ≠ (0,0)
1 1
𝑓𝑥 = 𝑥{1 + log (𝑥 2 + 𝑦 2 )}, 𝑓𝑦 = 𝑦{1 + log (𝑥 2 + 𝑦 2 )}, and hence
2 2
𝑥𝑦
𝑓𝑥𝑦 = 𝑓𝑦𝑥 = 2 .
𝑥 + 𝑦2
Using the definition of partial derivatives, it can be easily shown that for 𝑥 = 0, 𝑦 = 0
𝑓𝑥 = 𝑓𝑦 = 𝑓𝑥𝑦 = 𝑓𝑦𝑥 = 0.

Thus 𝑓𝑥𝑦 = 𝑓𝑦𝑥 at all points (𝑥, 𝑦).


However, 𝑓𝑥𝑦 = 𝑓𝑦𝑥 is not continuous at (0,0).
100 | P a g e

© Department of Distance & Continuing Education, Campus of Open Learning,


School of Open Learning, University of Delhi
Intermediate Mathematical Methods for Economics

𝑥𝑦
We find that the simultaneous limit lim(𝑥,𝑦)→(0,0) 𝑥 2 +𝑦 2 does not exist. For if we let (𝑥, 𝑦)
approach (0,0) through the line 𝑦 = 𝑚𝑥, we get
𝑥𝑦 𝑥, 𝑚𝑥 𝑚
lim = lim 2 =
(𝑥,𝑦)→(0,0) 𝑥 2 +𝑦 2 2
𝑥→0 𝑥 + 𝑚 𝑥 2 1 + 𝑚2
This limit depends upon 𝑚 so it does not exist. It implies that 𝑓𝑥𝑦 = 𝑓𝑦𝑥 is not continuous at
the origin.
EXERCISE 1
2𝑥𝑦
, (𝑥, 𝑦) ≠ (0,0)
1 If 𝑓(𝑥, 𝑦) = {𝑥 2+𝑦 2 show that 𝑓 is discontinuous at the origin but both
0, (𝑥, 𝑦) = (0,0).
the partial derivatives exist at the origin.
𝑥𝑦(𝑥 2 −𝑦 2 )
, (𝑥, 𝑦) ≠ (0,0)
2 If 𝑓(𝑥, 𝑦) = { 𝑥 2 +𝑦 2 show that
0, (𝑥, 𝑦) = (0,0),
𝑓𝑥 (0,0) = 0 and 𝑓𝑦 (0,0) = 0.
𝑥 2 −𝑦 2
3. If 𝑓(𝑥, 𝑦) = 𝑥𝑦 (𝑥 2 +𝑦 2) when 𝑥 2 + 𝑦 2 ≠ 0 and 𝑓(0,0) = 0, show that

𝑓𝑥 (𝑥, 0) = 0 = 𝑓𝑦 (0, 𝑦) and 𝑓𝑥 (0, 𝑦) = −𝑦.


𝑥2𝑦
4. Show that the function 𝑓(𝑥, 𝑦) = 𝑥 4 +𝑦 2 for (𝑥, 𝑦) ≠ (0,0) and 𝑓(0,0) = 0 possesses
partial derivatives at the origin but the function is discontinuous at the origin.

5. If 𝑓(𝑥, 𝑦) = √(|xy|), find 𝑓𝑥 (0,0) and 𝑓𝑦 (0,0).

6 Show that the function


𝑥𝑦 2
, (𝑥, 𝑦) ≠ (0,0)
𝑓(𝑥, 𝑦) = {𝑥 2 + 𝑦 2
0, (𝑥, 𝑦) = (0,0)
is continuous in 𝑥 − 𝑦 together at the origin but not differentiable at (0,0).

7. Prove that the function 𝑓(𝑥, 𝑦) = √|xy| is not (totally) differentiable at (0,0) but that 𝑓𝑥
and 𝑓𝑦 both exist at the origin and have the value 0 .

8. Show that for the function 𝑓(𝑥, 𝑦) = |𝑥 2 − 𝑦 2 |,


𝑓𝑥𝑦 (0,0) = 𝑓𝑦𝑥 (0,0)
101 | P a g e

© Department of Distance & Continuing Education, Campus of Open Learning,


School of Open Learning, University of Delhi
B.A. (Hons.) Economics

9 Show that for the function


𝑥2𝑦2
, (𝑥, 𝑦) ≠ (0,0)
𝑓(𝑥, 𝑦) = {𝑥 2 + 𝑦 2
0, (𝑥, 𝑦) = (0,0)
𝑓𝑥𝑦 (0,0) = 𝑓𝑦𝑥 (0,0), even though the conditions of Schwarz's theorem and Young's theorem
are not satisfied.
10 Show that the function
𝑥𝑦(𝑥 2 − 𝑦 2 )
, (𝑥, 𝑦) ≠ (0,0)
𝑓(𝑥, 𝑦) = { 𝑥 2 + 𝑦 2
0 , (𝑥, 𝑦) = (0,0)
does not satisfy the conditions of Schwarz's theorem and 𝑓𝑥𝑦 (0,0) ≠ 𝑓𝑦𝑥 (0,0).

11. State a set of conditions under which 𝑓𝑥𝑦 = 𝑓𝑦𝑥 where 𝑓 is a function of two variables.
𝑥𝑦(𝑦 2 −𝑥 2 )
If 𝑓(𝑥, 𝑦) = where (𝑥, 𝑦) ≠ (0,0) and 𝑓(0,0) = 0, calculate 𝑓𝑥𝑦 (0,0) and 𝑓𝑦𝑥 (0,0).
𝑦 2 +𝑥 2
Are they equal?
12. Show that the function
𝑥 2 sin (1/𝑥) + 𝑦 2 sin (1/𝑦), 𝑥𝑦 ≠ 0
𝑥 2 sin (1/𝑥) , 𝑥 ≠ 0 and 𝑦 = 0
𝑓(𝑥, 𝑦) =
𝑦 2 sin (1/𝑦) , 𝑥 = 0 and 𝑦 ≠ 0
{ 0 ,𝑥 = 𝑦 = 0
is differentiable at the origin.
13. Show that if 𝑓𝑦 exists in a certain neighbourhood of a point (𝑎, 𝑏) of the domain of definition
of a function 𝑓, and 𝑓𝑦𝑥 is continuous at (𝑎, 𝑏), then 𝑓𝑥𝑦 (𝑎, 𝑏) exists, and is equal to 𝑓𝑦𝑥 (𝑎, 𝑏).

Answers
5. 𝑓𝑥 (0,0) = 0, 𝑓𝑣 (0,0) = 0.
11. No.
Self Assessment Questions
1 Investigate for continuity at (1,2), the function

102 | P a g e

© Department of Distance & Continuing Education, Campus of Open Learning,


School of Open Learning, University of Delhi
Intermediate Mathematical Methods for Economics

𝑥 2 + 2𝑦, (𝑥, 𝑦) ≠ (1,2)


𝑓(𝑥, 𝑦) = {
0 , (𝑥, 𝑦) = (1,2)
2 Show that the function 𝑓 defined as follows has a removable discontinuity at (2,3).
3𝑥𝑦, (𝑥, 𝑦) ≠ (2,3)
𝑓(𝑥, 𝑦) = {
6, (𝑥, 𝑦) = (2,3)
Suitably redefine the function to make it continuous at (2,3).
3. Show that the function 𝑓 defined by
𝑥2 − 𝑦2
, (𝑥, 𝑦) ≠ (0,0)
𝑓(𝑥, 𝑦) = {𝑥 2 + 𝑦 2
0, (𝑥, 𝑦) = (0,0)
is not continuous at the origin.
𝑥 2 −𝑦 2
4. Prove that 𝑓(𝑥, 𝑦) = 𝑥𝑦 𝑥 2 +𝑦 2 when (𝑥, 𝑦) ≠ (0,0) and 𝑓(0,0) = 0 is continuous at the
origin.
5. A function 𝑓 is defined as follows:
2𝑥𝑦
, when (𝑥, 𝑦) ≠ (0,0)
𝑓(𝑥, 𝑦) = {𝑥 2 + 𝑦 2
0, when (𝑥, 𝑦) = (0,0).

Show that this function is continuous at the origin in 𝑥 alone and 𝑦 alone but is discontinuous
at the origin with regard to both the variables 𝑥 and 𝑦.
𝑥𝑦 2
6. Show that the function 𝑓 defined by 𝑓(𝑥, 𝑦) = 𝑥 2 +𝑦 4 , (𝑥, 𝑦) ≠ (0,0) and 𝑓(0,0) = 0 is
discontinuous in 𝑥 − 𝑦 together at the origin.Show also that this function is continuous along
the radius vector 𝜃 = 𝜋/2.
𝑥𝑦
7. Show that the function 𝑓(𝑥, 𝑦) = 𝑥 3 +𝑦 2, when 𝑥 ≠ 0, 𝑦 ≠ 0 and 𝑓(0,0) = 0 is discontinuous
in (x,y) at the origin.
8 Investigate the continuity at (0,0) of the function
𝑥2𝑦
, 𝑥3 + 𝑦3 ≠ 0
𝑓(𝑥, 𝑦) = {𝑥 3 + 𝑦 3
0, 𝑥 3 + 𝑦 3 = 0.
9 Show that the function 𝑓 is continuous at the origin, where
103 | P a g e

© Department of Distance & Continuing Education, Campus of Open Learning,


School of Open Learning, University of Delhi
B.A. (Hons.) Economics

𝑥3 − 𝑦3
, (𝑥, 𝑦) ≠ (0,0)
𝑓(𝑥, 𝑦) = {𝑥 2 + 𝑦 2
0, (𝑥, 𝑦) = (0,0)
10 Show that the following functions are discontinuous at the origin :
1
, (𝑥, 𝑦) ≠ (0,0)
(i) 𝑓(𝑥, 𝑦) = { 𝑥 2 +𝑦 2
0, (𝑥, 𝑦) = (0,0).
𝑥2𝑦2
, (𝑥, 𝑦) ≠ (0,0)
(ii) 𝑓(𝑥, 𝑦) = {𝑥 4 +𝑦 4
0, (𝑥, 𝑦) = (0,0).
𝑥 4 −𝑦 4
; (𝑥, 𝑦) ≠ (0,0)
(iii) 𝑓(𝑥, 𝑦) = {𝑥 4 +𝑦 4
0, (𝑥, 𝑦) = (0,0).
𝑥2
, (𝑥, 𝑦) ≠ (0,0)
(iv) 𝑓(𝑥, 𝑦) = { 𝑥 2 +𝑦 2
0, (𝑥, 𝑦) = (0,0).
11 Show that the following functions are continuous at the origin :
𝑥3𝑦3
, (𝑥, 𝑦) ≠ (0,0)
(i) 𝑓(𝑥, 𝑦) = {𝑥 2 +𝑦 2
0, (𝑥, 𝑦) = (0,0).
Answers
1 Discontinuous.
3𝑥𝑦, (𝑥, 𝑦) ≠ (2,3)
2 𝑓(𝑥, 𝑦) = {
18, (𝑥, 𝑦) = (2,3)
3 Discontinuous.
(OBJECTIVE QUESTIONS)
Fill in the Blanks.
Fill in the blanks "......", so that the following statements are complete and correct.
1 We say that the simultaneous limit of 𝑓(𝑥, 𝑦) exists and is equal to 𝐴 as (𝑥, 𝑦) → (𝑎, 𝑏),
if for every given 𝜀 > 0, there exists a 𝛿 > 0 such that |𝑓(𝑥, 𝑦) − 𝐴| < 𝜀 for all values of
𝑥 and 𝑦 in the nhd of (𝑎, 𝑏) defined by |𝑥 − 𝑎| < 𝛿, …..
𝑦 2 −𝑥 2
2 lim(𝑥,𝑦)→(0,0) 𝑥𝑦 𝑥 2+𝑦 2 = ⋯ ….

3 The function 𝑓(𝑥, 𝑦) is said to be continuous at (𝑎, 𝑏) if


104 | P a g e

© Department of Distance & Continuing Education, Campus of Open Learning,


School of Open Learning, University of Delhi
Intermediate Mathematical Methods for Economics

lim 𝑓(𝑥, 𝑦) = ⋯
(𝑥,𝑦)→(𝑎,𝑏)

Multiple Choice Questions.


Indicate the correct answer for each question by writing the corresponding letter from (a), (b),
(c) and (d).
3𝑥−2𝑦
4. lim(𝑥,𝑦)→(0,0) 2𝑥−3𝑦 =
3
(a) 2
2
(b) 3
(c) 1
(d) does not exist.
2𝑥𝑦 2
5. lim(𝑥,𝑦)→(0,0) 𝑥 2 +𝑦 4 =
(a) 0
(b) does not exist
(c) 1
(d) 2.
True or False.
Write ' 𝑇 ' for true and ' 𝐹 ' for false statement.
𝑥𝑦
6. lim(𝑥,𝑦)→(0,0) 𝑥 2 +𝑦 2 = 0.
7. The function
𝑥2 − 𝑦2
, (𝑥, 𝑦) ≠ (0,0)
𝑓(𝑥, 𝑦) = {𝑥 2 + 𝑦 2
0, (𝑥, 𝑦) = (0,0)
is discontinuous at the origin.
8 The function
is continuous at the origin.
𝑥 2 −𝑦 2
𝑥𝑦 𝑥 2+𝑦 2 , (𝑥, 𝑦) ≠ (0,0)
9 The function 𝑓(𝑥, 𝑦) = { is discontinuous at the origin.
0, (𝑥, 𝑦) = (0,0)
Answer
1 |𝑦 − 𝑏| < 𝛿.
2 0

105 | P a g e

© Department of Distance & Continuing Education, Campus of Open Learning,


School of Open Learning, University of Delhi
B.A. (Hons.) Economics

3 𝑓(𝑎, 𝑏).
4 (d).
5 (b).
6 𝐹.
7 𝑇.
8 𝑇.
9 𝐹.
7.4 SUMMARY

The main points which we have covered in these lessons are what is Higher order Derivatives.

7.5 GLOSSARY

Motivation: These Problems are very useful in real life, and we can use it in data science,
economics as well as social science.
Attention: Think how the best estimator are useful in real world problems.

7.6 REFERENCES

• Sydsaeter, K., Hammond, P. (2002). Mathematics for economics analysis. Pearson


Education.

• Hoy, M., Livernois, J., McKenna, C., Rees, R., Stengos, T, (2001). Mathematics for
Economics, Prentice-Hall India.

7.7 SUGGESTED READINGS

• A.R Vasishtha, Anurag Sharma, Dr. Vipin Vasishtha, Neenu Agarwal, Dr A.K
Vasishtha, Advance Calculus, Krishana Publication, 5th Edition.
• A.R Vasishtha, Anurag Sharma, Dr. Vipin Vasishtha, Anil Kumar, Dr A.K Vasishtha,
Analysis, Krishna Publication, 3rd Edition.

106 | P a g e

© Department of Distance & Continuing Education, Campus of Open Learning,


School of Open Learning, University of Delhi
Intermediate Mathematical Methods for Economics

LESSON-8
IMPLICIT FUNCTIONS THEOREM

STRUCTURE

8.1 Learning Objectives


8.2 Introduction
8.3 Implicit Functions Theorem
8.3.1 Implicit Functions
8.3.2 Implicit Functions Determined by Single Functional Equations
8.4 Summary
8.5 Glossary
8.6 Ref150erences
8.7 Suggested Readings

8.1 LEARNING OBJECTIVES

One of main benefit of Implicit function theory is that when variable x and y are not separated
it means when function cannot be written in the form of y=f(x) then we generally use implicit
function theory to find out the derivatives as well as higher order derivatives.

8.2 INTRODUCTION

we have only talked about functions, which assign to every real number 𝑥 in their domain
exactly one real number 𝑓(𝑥). The graphs of a function 𝑓(𝑥) is the set of all points (𝑥, 𝑦) such
that 𝑦 = 𝑓(𝑥), and we are usually visually the graph of a function as a curve for which every
vertical line crosses that curve at most once. There are other curves that we can draw on the
𝑥𝑦-plane which do not pass the vertical line test. One such curve is the circle of radius 1 centred
at the origin. We can describe this circle with the relation.
𝑥2 + 𝑦2 = 1
that is, the circle of radius 1 centred at the origin is the set of all points (𝑥, 𝑦) such that 𝑥 2 +
𝑦 2 = 1. Now consider one point on this circle, the point (0,1). You may notice that if we
remove some of the circle (for example, the lower half of the circle), the remaining curve is the
graph of a function. This function, for which we will find a formula below, is called an implicit

107 | P a g e

© Department of Distance & Continuing Education, Campus of Open Learning,


School of Open Learning, University of Delhi
B.A. (Hons.) Economics

function, and finding implicit functions and, more importantly, finding the derivatives of
implicit functions is the subject of today's lecture.
In general, we are interested in studying relations in which one function of 𝑥 and 𝑦 is equal to
another function of 𝑥 and 𝑦. A function 𝑓 of 𝑥 and 𝑦 takes each ordered pair (𝑥, 𝑦) and
associates it to some number 𝑓(𝑥, 𝑦). A general way to write down the type of relations in
which we are interested is:
𝑓(𝑥, 𝑦) = 𝑔(𝑥, 𝑦)
The relation 𝑥 2 + 𝑦 2 = 1 which defines the circle of radius 1 centered at the origin is one such
relation: in this case, 𝑓(𝑥, 𝑦) = 𝑥 2 + 𝑦 2 and 𝑔(𝑥, 𝑦) is the constant function 1. Another such
relation is 𝑦 − 1 = 𝑥 2 + 2𝑥. This relation also defines a curve, a parabola. How do we see this?
The natural thing to do is to solve for 𝑦 :
𝑦−1 = 𝑥 2 + 2𝑥
𝑦 = 𝑥 2 + 2𝑥 + 1
Thus, we see that the curve defined by the relation 𝑦 − 1 = 𝑥 2 + 2𝑥 is just the graph of a
quadratic function. The function 𝑦 = 𝑥 2 + 2𝑥 + 1 that we found by solving for 𝑦 is called the
implicit function of the relation 𝑦 − 1 = 𝑥 2 + 2𝑥. In general, any function we get by taking
the relation 𝑓(𝑥, 𝑦) = 𝑔(𝑥, 𝑦) and solving for 𝑦 is called an implicit function for that relation.
What complicates the situation is that a relation may have more than one implicit function.
The standard example of a relation of the form above which has more than one implicit function
is, of course, 𝑥 2 + 𝑦 2 = 1. To see this, let us try to solve for 𝑦 :
𝑥2 + 𝑦2 =1
𝑦2 = 1 − 𝑥2
Once we isolate 𝑦 2 , we discover a problem: in order to get 𝑦 from 𝑦 2 , we need to take the
square root of the right hand side, but we could the positive square root and get one implicit
function, or we could take the negative square root and get another implicit function:
𝑦 = √1 − 𝑥 2 or 𝑦 = −√1 − 𝑥 2
Both implicit functions have domain −1 ≤ 𝑥 ≤ 1 (can you see why?). The graph of the first
implicit function is the non-negative half of the circle, and the graph of the second is the non-
positive half of the circle. Together, their graphs make the entire circle.
In general, a relation has multiple implicit functions if, while solving for 𝑦, we come to a step
in which we must make a choice, like a square root. Another possibility is given by the relation
sin 𝑦 = 𝑥. Aside from the fact that we do not know how to get rid of the sine function like we

108 | P a g e

© Department of Distance & Continuing Education, Campus of Open Learning,


School of Open Learning, University of Delhi
Intermediate Mathematical Methods for Economics

would an exponent, we have the problem that, if −1 ≤ 𝑥 ≤ 1, then there is an infinite number
of numbers 𝑦 such that sin 𝑦 = 𝑥, and each corresponds to an implicit function for this relation.
When a relation has multiple implicit functions, we tend to choose one of those implicit
functions and study it alone instead of looking at all the implicit functions together. One way
to do this is to choose a point (𝑥, 𝑦) which satisfies the original relation (in other words, a point
on the curve defined by the relation), and to take an implicit function ℎ(𝑥) for which 𝑦 = ℎ(𝑥)
(that is, an implicit function for which (𝑥, 𝑦) is on the graph of that function). We call ℎ(𝑥) the
implicit function of the relation at the point (𝑥, 𝑦). For example, we have the relation 𝑥 2 +
𝑦 2 = 1 and the point (0,1). This relation has two implicit functions, and only one of them, 𝑦 =
√1 − 𝑥 2 , has the point (0,1) on its graph. This fits our intuitive idea of an implicit function
from the introduction to this lecture because the graph of this implicit function is the upper half
of the circle. If, however, we took the point (0, −1), then the implicit function of 𝑥 2 + 𝑦 2 = 1
at this point is 𝑦 = −√1 − 𝑥 2
Sometimes a point has more than one implicit function associated with it. For the relation 𝑥 2 +
𝑦 2 = 1, take the point (1,0). Both the implicit function 𝑦 = √1 − 𝑥 2 and the implicit function
𝑦 = −√1 − 𝑥 2 have the point (1,0) on their graphs. In a sense, a point which has more than
one implicit function associated to it is a bad point for the relation. Can you find another bad
point for the relation 𝑥 2 + 𝑦 2 = 1 ? There is a way to identify bad points for a relation, which
we will see when we learn how to differentiate implicit functions.

8.3 IMPLICIT FUNCTION THEOREM

Now we shall now, briefly, explain these terms one by one.


Implicit functions
Implicit Functions Determined by Single Functional Equations
8.3.1 Implicit Functions:
Whenever we come across a functional equation 𝑓(𝑥, 𝑦) = 0, we generally assume that it
determines 𝑦 as a function of 𝑥. But in some cases, such an equation may not define any such
function, or it may define one or more than one such functions.

For example, the equation 𝑥 2 + 𝑦 2 − 16 = 0 determines two functions 𝑦 = √16 − 𝑥 2 and


𝑦 = −√16 − 𝑥 2 , for 𝑥 2 ≤ 16
whereas the equations 𝑥 2 + 𝑦 2 + 7 = 0 and 𝑥 4 + 𝑦 4 + 9 = 0 determine no such function.

109 | P a g e

© Department of Distance & Continuing Education, Campus of Open Learning,


School of Open Learning, University of Delhi
B.A. (Hons.) Economics

Definition. Let 𝑓(𝑥, 𝑦) be a function of two variables 𝑥 and 𝑦 and 𝑦 = 𝜙(𝑥) be a function of
𝑥 such that, for every value of 𝑥, for which 𝜙(𝑥) is defined, 𝑓(𝑥, 𝜙(𝑥)) vanishes identically,
i.e., 𝑦 = 𝜙(𝑥) is a solution of the functional equation 𝑓(𝑥, 𝑦) = 0. Then we say that 𝑦 = 𝜙(𝑥)
is an implicit function defined by the functional equation 𝑓(𝑥, 𝑦) = 0
It is only in elementary cases, such as those given above, that an implicit function determined
by a functional equation 𝑓(𝑥, 𝑦) = 0 may be expressed in the explicit form 𝑦 = 𝜙(𝑥). In the
case of complicated functional equations, the determination of the implicit function in an
explicit form may be too laborious or in some cases it may not even be possible. However, the
difficulty of actual determination of an analytical expression does not rule out the possibility
of the existence of the implicit function or functions, defined by a functional equation. We shall
now consider an existence theorem, known as Implicit function theorem, that mentions
conditions which guarantee the functional.
IMPLICIT FUNCTIONS.
Whenever we come across a functional equation 𝑓(𝑥, 𝑦) = 0, we generally assume that it
determines 𝑦 as a function of 𝑥. But in some cases, such an equation may not define any such
function, or it may define one or more than one such functions.

For example, the equation 𝑥 2 + 𝑦 2 − 16 = 0 determines two functions 𝑦 = √16 − 𝑥 2 and 𝑦 =


−√16 − 𝑥 2 , for 𝑥 2 ≤ 16
whereas the equations 𝑥 2 + 𝑦 2 + 7 = 0 and 𝑥 4 + 𝑦 4 + 9 = 0 determine no such function.
Definition. Let 𝑓(𝑥, 𝑦) be a function of two variables 𝑥 and 𝑦 and 𝑦 = 𝜙(𝑥) be a function of
𝑥 such that, for every value of 𝑥, for which 𝜙(𝑥) is defined, 𝑓(𝑥, 𝜙(𝑥)) vanishes identically,
i.e., 𝑦 = 𝜙(𝑥) is a solution of the functional equation 𝑓(𝑥, 𝑦) = 0. Then we say that 𝑦 = 𝜙(𝑥)
is an implicit function defined by the functional equation 𝑓(𝑥, 𝑦) = 0
It is only in elementary cases, such as those given above, that an implicit function determined
by a functional equation 𝑓(𝑥, 𝑦) = 0 may be expressed in the explicit form 𝑦 = 𝜙(𝑥). In the
case of complicated functional equations, the determination of the implicit function in an
explicit form may be too laborious or in some cases it may not even be possible. However, the
difficulty of actual determination of an analytical expression does not rule out the possibility
of the existence of the implicit function or functions, defined by a functional equation. We shall
now consider an existence theorem, known as Implicit function theorem, that mentions
conditions which guarantee that a functional equation does define an implicit function even
though its actual determination in explicit form may not be possible.

110 | P a g e

© Department of Distance & Continuing Education, Campus of Open Learning,


School of Open Learning, University of Delhi
Intermediate Mathematical Methods for Economics

For many mathematical purposes the real importance lies in the fact that a given functional
equation defines an implicit function rather than in finding an expression for the implicit
function defined by it.
Hence, the implicit function theorem has its own utility in mathematics.
8.3.2 Implicit Functions Determined by a Single Functional Equation:

Implicit function theorem. Let 𝑓(𝑥, 𝑦) be a function of two variables 𝑥 and 𝑦 and let (𝑎, 𝑏)
be a point of its domain such that
(i) 𝑓(𝑎, 𝑏) = 0
(ii) the function 𝑓(𝑥, 𝑦) possesses continuous partial derivatives 𝑓𝑥 and
𝑓𝑦 in a certain neighbourhood of (𝑎, 𝑏) and
(iii) 𝑓𝑦 (𝑎, 𝑏) ≠ 0;
then there exists a rectangle [𝑎 − ℎ, 𝑎 + ℎ; 𝑏 − 𝑘, 𝑏 + 𝑘] about (𝑎, 𝑏) such that for every
value of 𝑥 in the interval [𝑎 − ℎ, 𝑎 + ℎ], the equation 𝑓(𝑥, 𝑦) = 0 determines one and only
one value 𝑦 = 𝜙(𝑥), lying in the inferval [𝑏 − 𝑘, 𝑏 + 𝑘], with the following properties :
(1) 𝑏 = 𝜙(𝑎),
(2) 𝑓[𝑥, 𝜙(𝑥)] = 0, for every 𝑥 in [𝑎 − ℎ, 𝑎 + ℎ], and
(3) 𝜙(𝑥) is derivable and both 𝜙(𝑥) and 𝜙 ′ (𝑥) are continuous in [𝑎 − ℎ, 𝑎 + ℎ].
Proof. It is given that 𝑓𝑦 (𝑎, 𝑏) ≠ 0.
So, either 𝑓𝑦 (𝑎, 𝑏) > 0 or 𝑓𝑦 (𝑎, 𝑏) < 0.
Without any loss of generality we may assume that 𝑓𝑦 (𝑎, 𝑏) > 0, for if 𝑓𝑦 (𝑎, 𝑏) < 0, we
should only have to replace 𝑓(𝑥, 𝑦) by −𝑓(𝑥, 𝑦) and this change would leave the equation
𝑓(𝑥, 𝑦) = 0 unaltered.
Uniqueness and the existence of the implicit function.
Let 𝑓𝑥 , 𝑓𝑦 be continuous in a rectangular neighbourhood.

𝑅1 = [𝑎 − ℎ1 , 𝑎 + ℎ1 ; 𝑏 − 𝑘1 , 𝑏 + 𝑘1 ] of (𝑎, 𝑏)
Since 𝑓𝑥 , 𝑓𝑦 are continuous in 𝑅1 , therefore, 𝑓 is differentiable and hence continuous in 𝑅1 .

Again, since 𝑓𝑦 is continuous at (𝑎, 𝑏) and 𝑓𝑦 (𝑎, 𝑏) > 0, there exists a rectangle.

𝑅2 = [𝑎 − ℎ2 , 𝑎 + ℎ2 ; 𝑏 − 𝑘, 𝑏 + 𝑘], ℎ2 < ℎ1 , 𝑘 < 𝑘1


(i.e., 𝑅2 ⊆ 𝑅1 ) such that for every point (𝑥, 𝑦) of this rectangle 𝑅2 , 𝑓𝑦 (𝑥, 𝑦) > 0.
Now, since 𝑓𝑦 (𝑥, 𝑦) > 0 for all (𝑥, 𝑦) ∈ 𝑅2 , therefore, for all 𝑥 ∈ [𝑎 − ℎ2 , 𝑎 + ℎ2 ]. the
function 𝑓 of 𝑦 strictly increases as 𝑦 increases from 𝑏 − 𝑘 to 𝑏 + 𝑘.
111 | P a g e

© Department of Distance & Continuing Education, Campus of Open Learning,


School of Open Learning, University of Delhi
B.A. (Hons.) Economics

In particular, since 𝑓(𝑎, 𝑏) = 0, we have 𝑓(𝑎, 𝑏 − 𝑘) < 0, 𝑓(𝑎, 𝑏 + 𝑘) > 0.


In view of this and the fact that 𝑓 is continuous, there exists a positive real number ℎ < ℎ2
such that for every 𝑥 in [𝑎 − ℎ, 𝑎 + ℎ], 𝑓(𝑥, 𝑏 − 𝑘) is as close to 𝑓(𝑎, 𝑏 − 𝑘) and 𝑓(𝑥, 𝑏 + 𝑘)
is as close to 𝑓(𝑎, 𝑏 + 𝑘) as we please and, therefore, we have
𝑓(𝑥, 𝑏 − 𝑘) < 0, 𝑓(𝑥, 𝑏 + 𝑘) > 0 for all 𝑥 ∈ [𝑎 − ℎ, 𝑎 + ℎ].
Now for every fixed value of 𝑥 in [𝑎 − ℎ, 𝑎 + ℎ], the continuous function 𝑓 of 𝑦 strictly
increases from a negative to a positive value as 𝑦 increases from 𝑏 − 𝑘 to 𝑏 + 𝑘 and therefore
there exists one and only one value of 𝑦 for which the function 𝑓 vanishes.
Thus, for each 𝑥 in [𝑎 − ℎ, 𝑎 + ℎ] there is a uniquely determined value of 𝑦 in [𝑏 − 𝑘, 𝑏 + 𝑘]
for which 𝑓(𝑥, 𝑦) = 0; this 𝑦 is a function of 𝑥, say 𝜙(𝑥) such that the properties (1) and (2)
are true.
Hence, there exists a rectangular neighbourhood
𝑅3 = [𝑎 − ℎ, 𝑎 + ℎ; 𝑏 − 𝑘, 𝑏 + 𝑘] of (𝑎, 𝑏)
such that for each 𝑥 belonging to [𝑎 − ℎ, 𝑎 + ℎ] there exists a unique value of 𝑦 = 𝜙(𝑥)
belonging to [𝑏 − 𝑘, 𝑏 + 𝑘] such that 𝑏 = 𝜙(𝑎) and 𝑓(𝑥, 𝜙(𝑥)) = 0 for all 𝑥 in [𝑎 − ℎ, 𝑎 + ℎ]
This completes the proof of the existence and the uniqueness of the implicit function 𝜙.
Continuity and derivability of the implicit function.
Let (𝑥, 𝑦), (𝑥 + 𝛿𝑥, 𝑦 + 𝛿𝑦) be two points in
𝑅3 = [𝑎 − ℎ, 𝑎 + ℎ; 𝑏 − 𝑘, 𝑏 + 𝑘] such that
𝑦 = 𝜙(𝑥), 𝑦 + 𝛿𝑦 = 𝜙(𝑥 + 𝛿𝑥)
and 𝑓(𝑥, 𝑦) = 0, 𝑓(𝑥 + 𝛿𝑥, 𝑦 + 𝛿𝑦) = 0.
Since 𝑓 is differentiable in 𝑅1 and 𝑅3 ⊆ 𝑅1, therefore 𝑓 is also differentiable in 𝑅3
∴ 𝑓(𝑥 + 𝛿𝑥, 𝑦 + 𝛿𝑦) − 𝑓(𝑥, 𝑦) = 𝛿𝑥𝑓𝑥 + 𝛿𝑦𝑓𝑦 + 𝛿𝑥𝜓1 + 𝛿𝑦𝜓2 ,

where 𝜓1 , 𝜓2 are functions of 𝛿𝑥 and 𝛿𝑦, each tending to zero as (𝛿𝑥, 𝛿𝑦) → 0.
But 𝑓(𝑥 + 𝛿𝑥, 𝑦 + 𝛿𝑦) − 𝑓(𝑥, 𝑦) = 0 − 0 = 0.
∴ in 𝑅3 , 𝛿𝑥𝑓𝑥 + 𝛿𝑦𝑓𝑦 + 𝛿𝑥𝜓1 + 𝛿𝑦𝜓2 = 0

or 𝛿𝑦𝑓𝑦 = −𝛿𝑥𝑓𝑥 − 𝛿𝑥𝜓1 − 𝛿𝑦𝜓2 . ---(1)

Since 𝑓𝑦 > 0 in 𝑅2 and 𝑅3 ⊆ 𝑅2 , therefore 𝑓𝑦 ≠ 0 in 𝑅3 . So, dividing both sides of (1) by


𝛿𝑥𝑓𝑦 , we have

112 | P a g e

© Department of Distance & Continuing Education, Campus of Open Learning,


School of Open Learning, University of Delhi
Intermediate Mathematical Methods for Economics

𝛿𝑦 𝑓𝑥 𝜓1 𝛿𝑦 𝜓2
=− − − .
𝛿𝑥 𝑓𝑦 𝑓𝑦 𝛿𝑥 𝑓𝑦

Proceeding to limits as (𝛿𝑥, 𝛿𝑦) → (0,0), we get


𝑑𝑦 𝑓𝑥
𝜙 ′ (𝑥) = =− .
𝑑𝑥 𝑓𝑦

Thus 𝜙(𝑥) is derivable and continuous in 𝑅3 . Also 𝜙 ′ (𝑥), being a quotient of two continuous
functions, is itself continuous in 𝑅3 .
Note 1. The function 𝑦 = 𝜙(𝑥) is said to be the unique solution of 𝑓(𝑥, 𝑦) = 0 near (𝑎, 𝑏) or
the unique implicit function determined by 𝑓(𝑥, 𝑦) = 0 near (𝑎, 𝑏).
Note 2. It should be clearly understood that the theorem just proved is essentially of a local
character. It states that if 𝑓 is a function of two variables satisfying certain assumptions in
respect of continuity and derivability in a neighbourhood of a point (𝑎, 𝑏) and if 𝑓(𝑎, 𝑏) = 0,
then there exists a rectangular neighbourhood [𝑎 − ℎ, 𝑎 + ℎ; 𝑏 − 𝑘, 𝑏 + 𝑘] of (𝑎, 𝑏) such that
for each 𝑥 belonging to [𝑎 − ℎ, 𝑎 + ℎ] there exists a unique 𝑦 belonging to [𝑏 − 𝑘, 𝑏 + 𝑘] such
that 𝑓(𝑥, 𝑦) = 0. Thus, 𝑓(𝑥, 𝑦) = 0 determines a unique implicit function 𝑦 = 𝜙(𝑥) in [𝑎 −
ℎ, 𝑎 + ℎ], 𝑦 lying in [𝑏 − 𝑘, 𝑏 + 𝑘] and this function 𝜙 of 𝑥 is derivable. The implicit function
𝑦 = 𝜙(𝑥) is a unique solution of 𝑓(𝑥, 𝑦) = 0 in a certain neighbourhood [𝑎 − ℎ, 𝑎 + ℎ; 𝑏 −
𝑘, 𝑏 + 𝑘] of (𝑎, 𝑏). The functional equation 𝑓(𝑥, 𝑦) = 0 may have a different solution if a
different neighbourhood of (𝑎, 𝑏) is considered.
SOLVED EXAMPLES
Ex. 1. Examine the following equation for the existence of a unique implicit function near the
point indicated and verify your assertion by direct calculation. Find also the first derivative of
the solution:
𝑓(𝑥, y) = 𝑥 2 + 𝑦 2 − 1 = 0, point (0,1).
Sol. Let 𝑓(𝑥, 𝑦) = 𝑥 2 + 𝑦 2 − 1.
Then 𝑓𝑥 = 2𝑥, 𝑓𝑦 = 2𝑦.
We have 𝑓(0,1) = 02 + 12 − 1 = 0
𝑓𝑦 (0,1) = 2,1 = 2 ≠ 0

Obviously 𝑓𝑥 and 𝑓𝑦 are continuous functions in the whole 𝑥 − 𝑦 plane and so also in some
neighbourhood of (0,1).

113 | P a g e

© Department of Distance & Continuing Education, Campus of Open Learning,


School of Open Learning, University of Delhi
B.A. (Hons.) Economics

Thus 𝑓(𝑥, 𝑦) satisfies all the conditions of implicit funçtion theorem in some neighbourhood
of (0,1). Hence, by implicit function theorem, there exists a rectangular neighbourhood [0 −
ℎ, 0 + ℎ; 1 − 𝑘, 1 + 𝑘] of (0,1) such that for each 𝑥 belonging to [0 − ℎ, 0 + ℎ] there exists a
unique 𝑦 belonging to [1 − 𝑘, 1 + 𝑘] such that 𝑓(𝑥, 𝑦) = 0. Thus, 𝑓(𝑥, 𝑦) = 0 determines a
unique implicit function 𝑦 = 𝜙(𝑥) in [0 − ℎ, 0 + ℎ], 𝑦 lying in [1 − 𝑘, 1 + 𝑘] and this function
𝜙 of 𝑥 is derivable.
Solving the equation 𝑥 2 + 𝑦 2 − 1 = 0 as a quadratic in 𝑦, we get

𝑦 2 = 1 − 𝑥 2 or 𝑦 = ±√1 − 𝑥 2

Obviously (0,1). satisfies the equation, 𝑦 = √1 − 𝑥 2 .


Hence, 𝑦 = √1 − 𝑥 2 is the unique implicit function 𝑦 = 𝜙(𝑥) determined by the equation
𝑓(𝑥, 𝑦) = 0 in some neighbourhood of (0,1), where |𝑥| ≤ 1, 𝑦 > 0.
𝑑𝑦 𝑓 2𝑥
We have (𝑑𝑥 ) = − (𝑓𝑥 ) = − (2𝑦) = 0.
(0,1) 𝑦 (0,1) (0,1)

Remark. Here, 𝑦 = −√𝑥 2 − 1 is the unique implicit function determined by the equation
𝑓(𝑥, 𝑦) = 0 in some neighbourhcod of (0, −1), where |𝑥| ≤ 1, 𝑦 < 0.
Ex. 2. Show that the following equation determines a unique solution near the point indicated;
find also the first derivative of the solution:
1
𝑥𝑦sin 𝑥 + cos 𝑦 = 0, (0, 𝜋) .
2
Sol. Let 𝑓(𝑥, 𝑦) = 𝑥𝑦sin 𝑥 + cos 𝑦.
Then 𝑓𝑥 = 𝑦sin 𝑥 + 𝑥𝑦cos 𝑥, 𝑓𝑦 = 𝑥sin 𝑥 − sin 𝑦.
1 1 1 1
We. have 𝑓 (0, 2 𝜋) = 0 + cos 2 𝜋 = 0, 𝑓𝑦 (0, 2 𝜋) = 0 − sin 2 𝜋 = −1 ≠ 0.

Obviously, the partial derivatives 𝑓𝑥 and 𝑓𝑦 are continuous functions in a neighbourhood of


1
(0, 2 𝜋).

Thus 𝑓(𝑥, 𝑦) satisfies all the conditions of implicit function theorem in a neighbourhood of
1
(0, 2 𝜋). Hence, by implicit function theorem, the equation 𝑓(𝑥, 𝑦) = 0 determines a unique
1
implicit function, say 𝑦 = 𝜙(𝑥), in a nèighbourhood of (0, 2 𝜋)
𝑑𝑦 𝑓 𝑦sin 𝑥+𝑥𝑦cos 𝑥
We have 𝑑𝑥 = 𝜙 ′ (𝑥) = − 𝑓𝑥 = − .
𝑦 𝑥sin 𝑥−sin 𝑦

114 | P a g e

© Department of Distance & Continuing Education, Campus of Open Learning,


School of Open Learning, University of Delhi
Intermediate Mathematical Methods for Economics

𝑑𝑦 0+0 0
∴ (𝑑𝑥 ) 1 =− 1 = − −1 = 0.
(0, 𝜋) 0−sin 𝜋
2 2

Ex. 3. Examine the following equation for the existence of a unique implicit function near the
point indicated and verify your assertion by direct calculation:
𝑦 2 − 𝑦𝑥 2 − 2𝑥 5 = 0, point (1, −1)
Sol. Let 𝑓(𝑥, 𝑦) = 𝑦 2 − 𝑦𝑥 2 − 2𝑥 5 .
Then 𝑓𝑥 = −2𝑥𝑦 − 10𝑥 4 , 𝑓𝑦 = 2𝑦 − 𝑥 2 .
We have
𝑓𝑥 (1, −1) = (−1)2 − (−1) ⋅ 12 − 2 ⋅ 15 = 0
𝑓𝑦 (1, −1) = 2 ⋅ (−1) − 12 = −3 ≠ 0

Obviously, the partial derivatives 𝑓𝑥 (𝑥, 𝑦) = −2𝑥𝑦 − 10𝑥 4 and 𝑓𝑦 (𝑥, 𝑦) = 2𝑦 − 𝑥 2 are
continuous in a neighbourhood of (1, −1).
Hence, by implicit function theorem, the equation 𝑓(𝑥, 𝑦) = 0 determines a unique implicit
function 𝑦 = 𝜙(𝑥) in a neighbourhood of (1, −1).
Solving the equation 𝑦 2 − 𝑦𝑥 2 − 2𝑥 5 = 0 as a quadratic in 𝑦, we get

𝑥 2 ± √𝑥 4 + 8𝑥 5 𝑥 2 1
𝑦= = [1 ± √1 + 8𝑥], 𝑥 ≥ −
2 2 8
𝑥2 1
Of the above two possible solutions, 𝑦 = [1 − √1 + 8𝑥], 𝑥 ≥ − 8 is the unique solution of
2
𝑓(𝑥, 𝑦) = 0 in a neighbourhood of (1, −1), since, −1 = 𝑦(1).

Ex. 4. If 𝑢 = 𝑥 4 𝑦 5 , where 𝑥 = 𝑡 2 and 𝑦 = 𝑡 3 , find 𝑑𝑢/𝑑𝑡. Verify by substitution.


We have
𝑑𝑢 ∂𝑢 𝑑𝑥 ∂𝑢 𝑑𝑦
= ⋅ + ⋅
𝑑𝑡 ∂𝑥 𝑑𝑡 ∂𝑦 𝑑𝑡
= 4𝑥 3 𝑦 5 ⋅ 2𝑡 + 5𝑥 4 𝑦 4 ⋅ 3𝑡 2
As a verification we notice that this value of 𝑑𝑢/𝑑𝑡
= 4(𝑡 2 )3 (𝑡 3 )5 ⋅ 2𝑡 + 5(𝑡 2 )4 (𝑡 3 )4 ⋅ 3𝑡 2 = 23𝑡 22 .
Also, substituting the values of 𝑥 and 𝑦 in 𝑢, we have
𝑢 = (𝑡 2 )4 (𝑡 3 )5 = 𝑡 23 .
115 | P a g e

© Department of Distance & Continuing Education, Campus of Open Learning,


School of Open Learning, University of Delhi
B.A. (Hons.) Economics

Therefore 𝑑𝑢/𝑑𝑡 = 23𝑡 22 , as before.


Ex. 5. If 𝑥 3 + 3𝑥 2 𝑦 + 6𝑥𝑦 2 + 𝑦 3 = 1, find 𝑑𝑦/𝑑𝑥.
Let 𝑓(𝑥, 𝑦) = 𝑥 3 + 3𝑥 2 𝑦 + 6𝑥𝑦 2 + 𝑦 3 − 1
Then ∂𝑓/ ∂𝑥 = 3𝑥 2 + 6𝑥𝑦 + 6𝑦 2 , ∂𝑓/ ∂𝑦 = 3𝑥 2 + 12𝑥𝑦 + 3𝑦 2
𝑑𝑦 ∂𝑓 ∂𝑓 𝑥 2 +2𝑥𝑦+2𝑦 2
Hence = − ∂𝑥 / ∂𝑦 = − .
𝑑𝑥 𝑥 2 +4𝑥𝑦+𝑦 2
Ex. 6. If 𝑢 = 𝑥 2 − 𝑦 2 + sin 𝑦𝑧 where 𝑦 = 𝑒 𝑥 and 𝑧 = log 𝑥, find 𝑑𝑢/𝑑𝑥.
𝑑𝑢 ∂𝑢 ∂𝑢 𝑑𝑦 ∂𝑢 𝑑𝑧
We have 𝑑𝑥 = ∂𝑥 + ∂𝑦 , 𝑑𝑥 + ∂𝑧 , 𝑑𝑥

= 2𝑥 + (−2𝑦 + 𝑧cos 𝑦𝑧)𝑒 𝑥 + (𝑦cos 𝑦𝑧)/𝑥


It is easy to verify that we shall get the same value of 𝑑𝑢/𝑑𝑥 if we first convert 𝑢 into a
function of 𝑥 alone by substituting in it the values of 𝑦 and 𝑧, and then differentiate.
Ex. 7. Find 𝑑2 𝑦/𝑑𝑥 2 if 𝑎𝑥 2 + 2ℎ𝑥𝑦 + 𝑏𝑦 2 = 1.
Here 𝑝 = 2(𝑎𝑥 + ℎ𝑦), 𝑞 = 2(ℎ𝑥 + 𝑏𝑦),
𝑟 = 2𝑎, 𝑠 = 2ℎ, 𝑡 = 2𝑏.
Hence
𝑑2𝑦 (ℎ𝑥 + 𝑏𝑦)2 𝑎 − 2(𝑎𝑥 + ℎ𝑦)(ℎ𝑥 + 𝑏𝑦)ℎ + (𝑎𝑥 + ℎ𝑦)2 𝑏
= − .
𝑑𝑥 2 (ℎ𝑥 + 𝑏𝑦)3
∂𝑢 ∂𝑢 ∂𝑢
Ex. 8. If 𝑢 = 𝑓(𝑥 − 𝑦, 𝑦 − 𝑧, 𝑧 − 𝑥), prove that + + = 0.
∂𝑥 ∂𝑦 ∂𝑧
Let 𝑋 = 𝑥 − 𝑦, 𝑌 = 𝑦 − 𝑧, 𝑍 = 𝑧 − 𝑥, then we have,
∂𝑋 ∂𝑋 ∂𝑋
= 1, = −1, =0
∂𝑥 ∂𝑦 ∂𝑧
∂𝑌 ∂𝑌 ∂𝑌
= 0, = 1, = −1
∂𝑥 ∂𝑦 ∂𝑧
∂𝑍 ∂𝑍 ∂𝑍
= −1, = 0, =1
∂𝑥 ∂𝑦 ∂𝑧
Since 𝑢 = 𝑓(𝑋, 𝑌, 𝑍)

116 | P a g e

© Department of Distance & Continuing Education, Campus of Open Learning,


School of Open Learning, University of Delhi
Intermediate Mathematical Methods for Economics

∂𝑢 ∂𝑢 ∂𝑋 ∂𝑢 ∂𝑌 ∂𝑢 ∂𝑍
= + +
∂𝑥 ∂𝑋 ∂𝑥 ∂𝑌 ∂𝑥 ∂𝑍 ∂𝑥
∂𝑢 ∂𝑢 ∂𝑢 ∂𝑢 ∂𝑢
= (1) + (0) + (−1) = −
∂𝑋 ∂𝑌 ∂𝑍 ∂𝑋 ∂𝑍
∂𝑢 ∂𝑢 ∂𝑋 ∂𝑢 ∂𝑌 ∂𝑢 ∂𝑍
= + +
∂𝑦 ∂𝑋 ∂𝑦 ∂𝑌 ∂𝑦 ∂𝑍 ∂𝑦
∂𝑢 ∂𝑢 ∂𝑢 ∂𝑢 ∂𝑢
= (−1) + (1) + (0) = − +
∂𝑋 ∂𝑌 ∂𝑍 ∂𝑋 ∂𝑌
∂𝑢 ∂𝑢 ∂𝑋 ∂𝑢 ∂𝑌 ∂𝑢 ∂𝑍
= + +
∂𝑧 ∂𝑋 ∂𝑧 ∂𝑌 ∂𝑧 ∂𝑍 ∂𝑧
∂𝑢 ∂𝑢 ∂𝑢 ∂𝑢 ∂𝑢
= (0) + (−1) + (1) = − +
∂𝑋 ∂𝑌 ∂𝑍 ∂𝑌 ∂𝑍
∂𝑢 ∂𝑢 ∂𝑢
Adding (1), (2) and (3), we get ∂𝑥 + ∂𝑦 + ∂𝑧 = 0.

Show that the following equations determine unique solutions near the points indicated;
find also the first derivatives of the solution:
1 𝑥 3 + 𝑦 3 − 3𝑥𝑦 + 𝑦 = 0, point (0,0).
1
2 𝑦 3 cos 𝑥 + 𝑦 2 sin2 𝑥 = 7, point (3 𝜋, 2).

3 2𝑥𝑦 − log 𝑥𝑦 = 2, point (1,1).


Examine the following equations for the existence of a unique implicit function near the point
indicated and verify your assertion by direct calculations:
4. 𝑦 2 − 𝑦𝑥 2 − 2𝑥 5 = 0, near (0,0).
5. 𝑦 2 + 2𝑥 2 𝑦 + 𝑥 5 = 0, near (1, −1).
6. 𝑦 2 + 𝑦𝑥 3 + 𝑥 2 = 0, near (0,0).
7. 𝑦 4 + 𝑥 2 𝑦 2 − 2𝑥 5 = 0, near (1,1).
8. 𝑥 2 + 𝑥𝑦 + 𝑦 2 − 1 = 0, near (1,0).
Answers:
1 0.
2√3
2 .
9

3 -1.

117 | P a g e

© Department of Distance & Continuing Education, Campus of Open Learning,


School of Open Learning, University of Delhi
B.A. (Hons.) Economics

4 𝜙(𝑥) not unique near (0,0).


5 𝜙(𝑥) not unique near (1, −1).
6 𝜙(𝑥) does not exist near (0,0).
7 𝜙(𝑥) unique near (1,1).
8 𝜙(𝑥) unique near (1,0)

(OBJECTIVE QUESTIONS)
Fill In the Blanks.
Fill in the blanks "......", so that the following statements are complete and correct.
1. The partial derivative 𝑓𝑥 (𝑎, 𝑏) of a function 𝑓(𝑥, 𝑦) at the point (𝑎, 𝑏) is given by
𝑓(𝑎 + ℎ, 𝑏) − ⋯
𝑓𝑥 (𝑎, 𝑏) = lim
ℎ→0 ℎ
provided the limit exists.
2. If a function 𝑓(𝑥, 𝑦) is differentiable at (𝑎, 𝑏), then it is … in 𝑥 − 𝑦 together at (𝑎, 𝑏)
3. If 𝑓: 𝑋 → 𝐑, 𝑋 ⊆ 𝐑2 and (𝑎, 𝑏) ∈ 𝑋 is such that 𝑓𝑥 , 𝑓𝑦 are differentiable at (𝑎, 𝑏), then
𝑓𝑥𝑦 (𝑎, 𝑏) = ⋯.

Multiple Choice Questions.


Indicate the correct answer for each question by writing the corresponding letter from (a), (b),
(c) and (d).
𝑥𝑦
, (𝑥, 𝑦) ≠ (0,0)
4. Let 𝑓(𝑥, 𝑦) = { 2+𝑦 2)
√(𝑥
0 , (𝑥; 𝑦) = 0.
Then
(a) 𝑓(𝑥, 𝑦) is discontinuous at (0,0)
(b) 𝑓(𝑥, 𝑦) is continuous at (0,0)
(c) 𝑓𝑥 (0,0) = 1
(d) 𝑓𝑦 (0,0) = 1.
𝑥𝑦 2
, (𝑥, 𝑦) ≠ (0,0)
5 Let 𝑓(𝑥, 𝑦) = {𝑥 2 +𝑦 4
0, (𝑥, 𝑦) = (0,0)
Then
(a) 𝑓(𝑥, 𝑦) is not defined at the origin.

118 | P a g e

© Department of Distance & Continuing Education, Campus of Open Learning,


School of Open Learning, University of Delhi
Intermediate Mathematical Methods for Economics

(b) 𝑓𝑥 (0,0) = 0
(c) 𝑓𝑥 (0,0) = 1
(d) 𝑓𝑦 (0,0) does not exist.
True or False.
Write ' 𝑻 ' for true and ' 𝑭 ' for false statement.
6 If a function 𝑓(𝑥, 𝑦) is continuous at a point (𝑎, 𝑏), it must also be differentiable at (𝑎, 𝑏).
7 If a function 𝑓(𝑥, 𝑦) possesses both the partial derivatives 𝑓𝑥 (𝑎, 𝑏) and 𝑓𝑦 (𝑎, 𝑏), it must
be differentiable at (𝑎, 𝑏).
8 If a function 𝑓(𝑥, 𝑦) is differentiable at (𝑎, 𝑏), it must be continuous at (𝑎, 𝑏).
9 For every function 𝑓(𝑥, 𝑦) defined at (𝑎, 𝑏), we have 𝑓𝑥𝑦 (𝑎, 𝑏) = 𝑓𝑦𝑥 (𝑎, 𝑏).
10 If 𝑓𝑥𝑦 and 𝑓𝑦𝑥 are both continuous at (𝑎, 𝑏), then 𝑓𝑥𝑦 (𝑎, 𝑏) = 𝑓𝑦𝑥 (𝑎, 𝑏).
11 If 𝑓: 𝑋 → 𝐑, 𝑋 ⊆ 𝐑2 and (𝑎, 𝑏) ∈ 𝑋 is such that (i) 𝑓𝑥 exists in 𝑁(𝑎, 𝑏) and
(ii) 𝑓𝑥𝑦 is continuous at (𝑎, 𝑏), then 𝑓𝑥𝑦 (𝑎, 𝑏) = 𝑓𝑦𝑥 (𝑎, 𝑏).
12 If 𝑓: 𝑋 → 𝐑, 𝑋 ⊆ 𝐑2 and (𝑎, 𝑏) ∈ 𝑋 is such that 𝑓𝑥 , 𝑓𝑦 are differentiable at (𝑎, 𝑏), then
𝑓𝑥𝑦 (𝑎, 𝑏) may or may not be equal to 𝑓𝑦𝑥 (𝑎, 𝑏).
13 The conditions of Young’s theorem are necessary for 𝑓𝑥𝑦 = 𝑓𝑦𝑥 .
14 If a function 𝑓(𝑥, 𝑦) is differentiable at (𝑎, 𝑏), then the partial derivatives 𝑓𝑥 (𝑎, 𝑏) and
𝑓𝑦 (𝑎, 𝑏) both exist at (𝑎, 𝑏).
15 If a function 𝑓(𝑥, 𝑦) is discontinuous at (𝑎, 𝑏), then both the partial derivatives 𝑓𝑥 (𝑎, 𝑏)
and 𝑓𝑦 (𝑎, 𝑏) do not exist.
16 The function
𝑥𝑦
, (𝑥, 𝑦) ≠ (0,0)
𝑓(𝑥, 𝑦) = {√(𝑥 2 + 𝑦 2 )
0 , (𝑥, 𝑦) = (0,0)
is differentiable at the origin.
Answers To Objective Questions
1 𝑓(𝑎, 𝑏).
2 continuous.
3 𝑓𝑦𝑥 (𝑎, 𝑏).
119 | P a g e

© Department of Distance & Continuing Education, Campus of Open Learning,


School of Open Learning, University of Delhi
B.A. (Hons.) Economics

4 (b).
5 (b).
6 𝐹.
7 𝐹.
8 𝑇.
9 𝐹.
10 𝑇.
11 𝑇.
12 𝐹.
13 𝐹.
14. 𝑇.
14 𝐹.
15 𝐹.
SELF ASSESSMENT QUESTIONS
1 Find 𝑑𝑦/𝑑𝑥 if
(i) 𝑎𝑥 2 + 2ℎ𝑥𝑦 + 𝑏𝑦 2 = 1.
(ii) 𝑦 𝑥 + 𝑥 𝑦 = 𝑐.
2 If 𝑢 = 𝑥log 𝑥𝑦, where 𝑥 3 + 𝑦 3 + 3𝑥𝑦 = 1, find 𝑑𝑢/𝑑𝑥.
3 Find 𝑑𝑢/𝑑𝑥 if 𝑢 = sin (𝑥 2 + 𝑦 2 ), where 𝑎2 𝑥 + 𝑏 2 𝑦 2 = 𝑐 2 .
𝑑2 𝑦 2𝑎2 𝑥 2
4 II 𝑦 3 − 3𝑎𝑥 2 + 𝑥 3 = 0, then prove that 𝑑𝑥 2 + = 0.
𝑦5

5 If 𝑓(𝑥, 𝑦) = 0, 𝜙(𝑦, 𝑧) = 0, show that


∂𝑓 ∂𝜙 𝑑𝑧 ∂𝑓 ∂𝜙
⋅ ⋅ = ⋅
∂𝑦 ∂𝑧 𝑑𝑥 ∂𝑥 ∂𝑦
∂𝑓
6. If the curves 𝑓(𝑥, 𝑦) = 0, 𝜙(𝑦, 𝑧) = 0 touch, show that at the point of contact ⋅
∂𝑥
∂𝜙 ∂𝑓 ∂𝜙
− ∂𝑦 ⋅ ∂𝑥 = 0
∂𝑦
[HINT: At the point of contact, the value of 𝑑𝑦/𝑑𝑥 for the common tangent will be
the same for both curves.]

120 | P a g e

© Department of Distance & Continuing Education, Campus of Open Learning,


School of Open Learning, University of Delhi
Intermediate Mathematical Methods for Economics

∂𝑧 ∂𝑧
7. If 𝑧 = 𝑓(𝑢, 𝑣), 𝑢 = 𝑥 2 − 2𝑥𝑦 − 𝑦 2 , 𝑣 = 𝑦, show that (𝑥 + 𝑦) ∂𝑥 + (𝑥 − 𝑦) ∂𝑦 = 0 is
transformed into ∂𝑧/ ∂𝑣 = 0.
∂𝑧
8 If 𝑧 is a function of 𝑥 and 𝑦, and 𝑥 = 𝑒 ′′ + 𝑒 −𝑦 , 𝑦 = 𝑒 −𝑦 − 𝑒 ′ , then prove that ∂𝑢 −
∂𝑧 ∂𝑧 ∂𝑧
= 𝑥 ∂𝑥 − 𝑦 ∂𝑦.
∂𝑣

∂2 𝑢 ∂2 𝑢 ∂2 𝑢 ∂2 𝑢
9 Prove that ∂𝑥 2 + ∂𝑦 2 = + ∂𝜂2 , where 𝑥 = 𝜉cos 𝛼 − 𝜂sin 𝛼, 𝑦 = 𝜉sin 𝛼 + 𝜂cos 𝛼.
∂𝜉 2

∂2 𝑉
10 If 𝑢 = 𝑒 𝑥 sec 𝑦, 𝑣 = 𝑒 𝑥 tan 𝑦 and 𝑉 is a function of 𝑢, 𝑣, show that cos 𝑦 ( −
∂𝑥 ∂𝑦
∂𝑉 ∂2 𝑉 ∂2 𝑉 ∂2 𝑉
) = 𝑢𝑣 (∂𝑢2 + ∂𝑣2 ) + (𝑢2 + 𝑣 2 ) ∂𝑢 ∂𝑣.
∂𝑦

∂2 𝑣
11 If 𝑥 + 𝑦 = 2𝑒 𝜃 cos 𝜙 and 𝑥 − 𝑦 = 2𝑖𝑒 𝜃 sin 𝜙, where 𝑖 = √−1, Show that ∂𝜃2 +
∂2 𝑣 ∂2 𝑣
= 4𝑥𝑦 ∂𝑥 ∂𝑦.
∂𝜙2

ANSWERS
12 (i) −(𝑎𝑥 + ℎ𝑦)/(ℎ𝑥 + 𝑏𝑦).
(ii) −(𝑦 𝑥 log 𝑦 + 𝑦𝑥 𝑦−1 )/(𝑥𝑦 𝑥−1 + 𝑥 𝑦 log 𝑥).
13 1 + log 𝑥𝑦 − 𝑥(𝑥 2 + 𝑦)/𝑦(𝑥 + 𝑦 2 ).
14 2𝑥{cos (𝑥 2 + 𝑦 2 }(1 − 𝑎2 /𝑏 2 ).

8.4 SUMMARY
The main points which we have covered in these lessons are what is Implicit function and how
to find the derivatives of Implicit functions.

8.5 GLOSSARY

Motivation: These Problems are very useful in real life, and we can use it in data science,
economics as well as social science.
Attention: Think how Implicit functions are useful in real world problems.

8.6 REFERENCES

• Sydsaeter, K., Hammond, P. (2002). Mathematics for economics analysis. Pearson


Education.

121 | P a g e

© Department of Distance & Continuing Education, Campus of Open Learning,


School of Open Learning, University of Delhi
B.A. (Hons.) Economics

• Hoy, M., Livernois, J., McKenna, C., Rees, R., Stengos, T, (2001). Mathematics for
Economics, Prentice-Hall India.

8.7 SUGGESTED READINGS

• A.R Vasishtha, Anurag Sharma, Dr. Vipin Vasishtha, Neenu Agarwal, Dr A.K
Vasishtha, Advanced Calculus,Krishna Publication, 5th Edition.
• A.R Vasishtha, Anurag Sharma, Dr. Vipin Vasishtha, Anil Kumar, Dr A.K Vasishtha,
Analysis,Krishna Publication, 3th Edition.

122 | P a g e

© Department of Distance & Continuing Education, Campus of Open Learning,


School of Open Learning, University of Delhi
Intermediate Mathematical Methods for Economics

LESSON 9

HOMOGENEOUS FUNCTIONS

STRUCTURE

9.1 Learning Objectives


9.2 Introduction
9.3 Homogeneous Functions
9.3.1 Homogeneous Functions
9.3.2 Euler Theorem on Homogeneous Functions
9.3.3 Total Differential Coefficient
9.3.4 First Differential Coefficient of Implicit Function
9.4 Summary
9.5 Glossary
9.6 References
9.7 Suggested Readings

9.1 LEARNING OBJECTIVES

In this chapter we will discuss what is homogeneous function and its application in differential
calculus and also we will discuss what is Euler equation for homogeneous function and what
is its application in differential calculus as well as differential equations.

9.2 INTRODUCTION

Homogeneous functions are very useful, it is useful when we want to find out partial derivatives
of order one and higher one by using Euler theorem also in differential equation homogeneous
functions are useful , it makes problem to solve easily.

9.3 HOMOGENEOUS FUNCTIONS


Homogeneous Functions
Euler Theorem on Homogeneous Functions
Total Differential Coefficient

123 | P a g e

© Department of Distance & Continuing Education, Campus of Open Learning,


School of Open Learning, University of Delhi
B.A. (Hons.) Economics

First Differential Coefficient of Implicit Function


Now we shall now, briefly, explain these terms one by one.
9.3.1 Homogeneous Functions:
An expression in 𝑥 and 𝑦 in which every term is of the same degree is called a homogeneous
function of 𝑥 and 𝑦. Consider the function defined by
𝑓(𝑥, 𝑦) = 𝑎0 𝑥 𝑛 + 𝑎1 𝑥 𝑛−1 𝑦 + 𝑎2 𝑥 𝑛−2 𝑦 2 + ⋯ … + 𝑎𝑛−1 𝑥𝑦 𝑛−1 + 𝑎𝑛 𝑦 𝑛 .
In this function every term is of degree 𝑛. Therefore, it is a homogeneous function of 𝑥 and 𝑦
of degree 𝑛. Moreover, (1) may be written as
𝑦 𝑦 2 𝑦 𝑛
𝑓(𝑥, 𝑦) = 𝑥 𝑛 {𝑎0 + 𝑎1 ( ) + 𝑎2 ( ) + ⋯ + 𝑎𝑛 ( ) }
𝑥 𝑥 𝑥
or 𝑓(𝑥, 𝑦) = 𝑥 𝑛 𝐹(𝑦/𝑥),
where 𝐹(𝑦/𝑥) is some function of 𝑦/𝑥. Thus, a homogeneous function of 𝑥 and 𝑦 of degree 𝑛
may be put in the form 𝑥 𝑛 𝐹(𝑦/𝑥). Therefore, we give the general definition of a homogeneous
function as follows:
𝑥 𝑛 𝐹(𝑦/𝑥) is called a homogeneous function of 𝑥 and 𝑦 of degree 𝑛, whatever the function 𝐹
may be. Similarly, 𝑦 𝑛 𝐹(𝑥/𝑦) is also a homogeneous function of 𝑥 and 𝑦 of degree 𝑛.
Thus 𝑥 3 sin (𝑦/𝑥) is a homogeneous function of 𝑥 and 𝑦 of degree 3. Similarly, 𝑦 2 cos (𝑥/𝑦)
is a homogeneous function of 𝑥 and 𝑦 of degree 2.

In general, if the function 𝑓(𝑥1 , 𝑥2 , … , 𝑥𝑝 ) of the 𝑝 variables 𝑥1 , 𝑥2 , … , 𝑥𝑝 can be put in the


form
𝑥1 𝑥2 𝑥𝑝
𝑥𝑟𝑛 𝐹 ( , ,…, ),
𝑥𝑟 𝑥𝑟 𝑥𝑟

then 𝑓(𝑥1 , 𝑥2 , … , 𝑥𝑝 ) is called a homogeneous function of 𝑥1 , 𝑥2 , … , 𝑥𝑝 of degree 𝑛.

9.3.2 Euler Theorem on Homogeneous Functions


∂𝑢 ∂𝑢
If 𝑢 be a homogeneous function of 𝑥 and 𝑦 of degree 𝑛, then 𝑥 +𝑦 = 𝑛𝑢
∂𝑥 ∂𝑦

Proof. Since 𝑢 is a homogeneous function of 𝑥 and 𝑦 of degree 𝑛, therefore 𝑢 may be put in


the form

124 | P a g e

© Department of Distance & Continuing Education, Campus of Open Learning,


School of Open Learning, University of Delhi
Intermediate Mathematical Methods for Economics

𝑢 = 𝑥 𝑛 𝐹(𝑦/𝑥).
Differentiating (1) partially with respect to 𝑥, we get
∂𝑢 𝑦 𝑦 𝑦
= 𝑛𝑥 𝑛−1 𝐹 ( ) + 𝑥 𝑛 {𝐹 ′ ( )} (− 2 ) .
∂𝑥 𝑥 𝑥 𝑥
∂𝑢 𝑦 𝑦
∴ 𝑥 = 𝑛𝑥 𝑛 𝐹 ( ) − 𝑦𝑥 𝑛−1 𝐹 ′ ( ) .
∂𝑥 𝑥 𝑥
Again differentiating (1) partially with respect to 𝑦, we get
∂𝑢 𝑦 1
= 𝑥 𝑛 {𝐹 ′ ( )} ( ) .
∂𝑦 𝑥 𝑥
∂𝑢 𝑦
∴ 𝑦 = 𝑦𝑥 𝑛−1 𝐹 ′ ( ) .
∂𝑦 𝑥
Adding (2) and (3), we get
∂𝑢 ∂𝑢 𝑦
𝑥 +𝑦 = 𝑛𝑥 𝑛 𝐹 ( ) = 𝑛𝑢.
∂𝑥 ∂𝑦 𝑥
∂𝑢 ∂𝑢
Hence 𝑥 ∂𝑥 + 𝑦 ∂𝑦 = 𝑛𝑢.
This proves the theorem.
In general, if 𝑢 be a homogeneous function of 𝑥1 , 𝑥2 , … , 𝑥𝑚 of degree 𝑛, then
∂𝑢 ∂𝑢 ∂𝑢
(×) 𝑥1 ∂𝑥 + 𝑥2 ∂𝑥 + ⋯ + 𝑥𝑚 ∂𝑥 = 𝑛𝑢.
1 2 𝑚
The proof is similar to that of two variables.
Corollary. If 𝑢 be a a homogeneous functior of 𝑥 and 𝑦 of degree 𝑛, then
∂2 𝑢 ∂2 𝑢 ∂𝑢
(i) 𝑥 ∂𝑥 2 + 𝑦 ∂𝑥 ∂𝑦 = (𝑛 − 1) ∂𝑥 .

∂2 𝑢 ∂2 𝑢 ∂𝑢
(ii) 𝑦 ∂𝑥 ∂𝑦 + ∂𝑦 2 = (𝑛 − 1) ∂𝑦.

∂2 𝑢 ∂2 𝑢 ∂2 𝑢
(iii) 𝑥 ∂𝑥 2 + 2𝑥𝑦 ∂𝑥 ∂𝑦 + 𝑦 2 ∂𝑦 2 = 𝑛(𝑛 − 1)𝑢.
Since 𝑢 is a a homogeneous function of degree 𝑛, then by Euler's theorem, we have
∂𝑢 ∂𝑢
𝑥 +𝑦 = 𝑚𝑢.
∂𝑥 ∂𝑦
(i) Differentiating both sides of (1) partially with respect to 𝑥, we get

125 | P a g e

© Department of Distance & Continuing Education, Campus of Open Learning,


School of Open Learning, University of Delhi
B.A. (Hons.) Economics

∂2 𝑢 ∂𝑢 ∂2 𝑢 ∂𝑢
𝑥 2+ +𝑦 =𝑛 ,
∂𝑥 ∂𝑥 ∂𝑥 ∂𝑦 ∂𝑥
i.e.,
∂2 𝑢 ∂2 𝑢 ∂𝑢
𝑥 + 𝑦 = (𝑛 − 1) .
∂𝑥 2 ∂𝑥 ∂𝑦 ∂𝑥
(ii) Differentiating both sides of (1) partially with respect to 𝑦, we get
∂2 𝑢 ∂𝑢 ∂2 𝑢 ∂𝑢
𝑥 + +𝑦 2 =𝑛 ,
∂𝑦 ∂𝑥 ∂𝑦 ∂𝑦 ∂𝑦
i.e.,
(iii) Multiplying (2) by 𝑥 and (3) by 𝑦, and then adding the resulting equations, we have
∂2 𝑢 ∂2 𝑢 ∂2 𝑢 ∂𝑢 ∂𝑢
𝑥 + 2𝑥𝑦 +𝑦 2 = 𝑛(𝑛 − 1) (𝑥 +𝑦 )
∂𝑥 2 ∂𝑥 ∂𝑦 ∂𝑦 2 ∂𝑥 ∂𝑦
= (𝑛 − 1)𝑛𝑢 = 𝑛(𝑛 − 1)𝑢, using (1).
SOLVED EXAMPLES
(𝑥 1/4 +𝑦 1/4 )
Ex. 1. Verify Euler's theorem for the function 𝑢 = (𝑥 1/5+𝑦 1/5).

Sol. Here we see that 𝑢 is a homogeneous function of 𝑥 and 𝑦 of degree 1/4 − 1/5 i.e., 1/20.
Therefore, in order to verify Euler's theorem we are to showing that
∂𝑢 ∂𝑢 1
𝑥 +𝑦 = 𝑢.
∂𝑥 ∂𝑦 20

We have, log 𝑢 = log (𝑥 1/4 + 𝑦1/4 ) − log (𝑥1/5 + 𝑦 1/5 ).


Differentiating (1) partially with respect to 𝑥, we have.
1 ∂𝑢 1 1 −3/4 1 1 −4/5
= 1/4 ⋅ ( 𝑥 ) − ( 𝑥 ).
𝑢 ∂𝑥 𝑥 + 𝑦1/4 4 𝑥1/5 + 𝑦1/5 5
∂𝑢 1 𝑥 −3/4 1 𝑥 −4/5
∴ = 𝑢 [ 1/4 − ].
∂𝑥 4 𝑥 + 𝑦 1/4 5 𝑥1/5 + 𝑦 1/5
∂𝑢 1 𝑥1/4 1 𝑥1/5
∴ 𝑥 = 𝑢 [ 1/4 − ].
∂𝑥 4 𝑥 + 𝑦1/4 5 𝑥1/5 + 𝑦1/5

Again differentiating (1) partially with respect to 𝑦, we get

126 | P a g e

© Department of Distance & Continuing Education, Campus of Open Learning,


School of Open Learning, University of Delhi
Intermediate Mathematical Methods for Economics

1 ∂𝑢 1 𝑦 −3/4 1 𝑦 −4/5
= [ 1/4 − ].
𝑢 ∂𝑦 4 𝑥 + 𝑦 1/4 5 𝑥1/5 + 𝑦 1/5
∂𝑢 1 𝑦1/4 1 𝑦 1/5
∴ 𝑦 = 𝑢 [ 1/4 − ].
∂𝑦 4 𝑥 + 𝑦 1/4 5 𝑥1/5 + 𝑦1/5
Adding (2) and (3), we get
∂𝑢 ∂𝑢 1 𝑥1/4 + 𝑦 1/4 1 𝑥1/5 + 𝑦1/5 1 1 1
𝑥 +𝑦 = 𝑢 [ 1/4 − ] = 𝑢 [ − ] = 𝑢.
∂𝑥 ∂𝑦 4 𝑥 + 𝑦 1/4 5 𝑥1/5 + 𝑦1/5 4 5 20

This verifies Euler's theorem.


𝑥 2 +𝑦 2 ∂𝑢 ∂𝑢
Ex. 2. If 𝑢 = sin−1 { }, show that 𝑥 + 𝑦 ∂𝑦 = tan 𝑢.
𝑥+𝑦 ∂𝑥

𝑥 2 +𝑦 2
Sol. We have, sin 𝑢 = .
𝑥+𝑦
𝑥 2 +𝑦 2
Let 𝑣 = . Then 𝑣 is a homogeneous function of 𝑥 and 𝑦 of degree 1.
𝑥+𝑦
∂𝑣 ∂𝑣
Therefore by Euler's theorem, we have 𝑥 ∂𝑥 + 𝑦 ∂𝑦 = 𝑣.
Now 𝑣 = sin 𝑢.
∂𝑣 ∂𝑢 ∂𝑣 ∂𝑢
∴ = cos 𝑢 ∂𝑥 and = cos 𝑢 ∂𝑦.
∂𝑥 ∂𝑦
Putting these values in (1), we get
∂𝑢 ∂𝑢
𝑥cos 𝑢 + 𝑦cos 𝑢 =𝑣
∂𝑥 ∂𝑦
∂𝑢 ∂𝑢 𝑣 sin 𝑢 𝑥2 + 𝑦2
(𝑥 +𝑦 )= = [∵ 𝑣 = sin 𝑢 = ]
∂𝑥 ∂𝑦 cos 𝑢 cos 𝑢 𝑥+𝑦
= tan 𝑢.
Ex. 3. Verify Euler's theorem when
𝑓(𝑥, 𝑦, 𝑧) = 3𝑥 2 𝑦𝑧 + 5𝑥𝑦 2 𝑧 + 4𝑧 4 .
∂𝑓
Here = 6𝑥𝑦𝑧 + 5𝑦 2 𝑧,
∂𝑥
∂𝑓
Therefore 𝑥 ∂𝑥 = 6𝑥 2 𝑦𝑧 + 5𝑥𝑦 2 𝑧
∂𝑓
Similarly 𝑦 ∂𝑦 = 3𝑥 2 𝑦𝑧 + 10𝑥𝑦 2 𝑧.
and

127 | P a g e

© Department of Distance & Continuing Education, Campus of Open Learning,


School of Open Learning, University of Delhi
B.A. (Hons.) Economics

∂𝑓
𝑧 = 3𝑥 2 𝑦 + 5𝑥𝑦 2 𝑧 + 16𝑧 4 ,
∂𝑧
∂𝑓 ∂𝑓 ∂𝑓
𝑥 ∂𝑥 + 𝑦 ∂𝑦 + 𝑧 ∂𝑧 = 12𝑥 2 𝑦𝑧 + 20𝑥𝑦 2 𝑧 + 16𝑧 4
Hence
= 4𝑓(𝑥, 𝑦, 𝑧),
which verifies Euler's theorem for a function of degree 4 in this case.
𝑥 2 +𝑦 2 ∂𝑢 ∂𝑢
Ex. 4. If 𝑢 = log { }, prove that 𝑥 + 𝑦 ∂𝑦 = 1.
𝑥+𝑦 ∂𝑥

𝑥 2 +𝑦 2
Here 𝑢 = log { } = log 𝑧
𝑥+𝑦
𝑥 2 +𝑦 2 1+(𝑦/𝑥)2
where 𝑧 = 𝑒 4 = =𝑥
𝑥+𝑦 1+(𝑦/𝑥)

i.e. 𝑧 is a homogeneous function of 𝑥, 𝑦 of degree 1.


∂𝑧 ∂𝑧
Hence by Euler's theorem, we have 𝑥 ∂𝑥 + 𝑦 ∂𝑦 = 𝑧.

∂𝑧 ∂𝑢 ∂𝑧 ∂𝑢
Then ∂𝑥 = 𝑒 𝑢 ∂𝑥 and ∂𝑦 = 𝑒 𝑢 ∂𝑦
∂𝑢 ∂𝑢 ∂𝑢 ∂𝑢
or 𝑒 𝑢′ 𝑥 ∂𝑥 + 𝑒 𝑥 𝑦 ∂𝑦 = 𝑒 𝑢 [from (1) and (2)] or 𝑥 ∂𝑥 + 𝑦 ∂𝑦 = 1

𝑥 3 +𝑦 3
Ex. 5. If 𝑢 = tan−1 , 𝑥 ≠ 𝑦, show that.
𝑥−𝑦

∂𝑢 ∂𝑢
𝑥 +𝑦 = sin 2𝑢
∂𝑥 ∂𝑦
∂2 𝑢 ∂2 𝑢 ∂2 𝑢
𝑥 2 ∂𝑥 2 + 2𝑥𝑦 ∂𝑥 ∂𝑦 + 𝑦 2 ∂𝑦 2 = (1 − 4sin2 𝑢)sin 2𝑢.

Here
where
𝑥3 + 𝑦3
𝑢 = tan−1 = tan−1 𝑧
𝑥−𝑦
𝑥3 + 𝑦3 1 + (𝑦/𝑥)3
𝑧 = tan 𝑢 = = 𝑥2
𝑥−𝑦 1 − (𝑦/𝑥)
which is a homogeneous function of 𝑥, 𝑦 of degree 2 .

128 | P a g e

© Department of Distance & Continuing Education, Campus of Open Learning,


School of Open Learning, University of Delhi
Intermediate Mathematical Methods for Economics

∂𝑧 ∂𝑧 ∂𝑧 ∂𝑢 ∂𝑧
Hence by Euler's theorem, we have 𝑥 ∂𝑥 + 𝑦 ∂𝑦 = 2𝑧 Since ∂𝑥 = sec 2 𝑢 ∂𝑥 and ∂𝑦 =
∂𝑢 ∂𝑢 ∂𝑢 ∂𝑢 ∂𝑢
sec 2 𝑢 ∂𝑦 [from (1)] ∴ 𝑥sec 2 𝑢 ∂𝑦 + 𝑦sec 2 = 2tan 𝑢 [from (2) & (3)] or 𝑥 ∂𝑥 + 𝑦 ∂𝑦 =
∂𝑦
2sin 𝑢cos 𝑢 = sin 2𝑢
∂2 𝑢 ∂𝑢 ∂2 𝑢
Differentiating (4) partially w.r.t. 𝑥 and w.r.t. to 𝑦, we get 𝑥 ∂𝑥 2 + ∂𝑥 + 𝑦 ∂𝑥 ∂𝑦 = 2cos 2𝑢 ⋅
∂𝑢 ∂2 𝑢 ∂𝑢 ∂2 𝑢 ∂𝑢
and 𝑥 ∂𝑥 ∂𝑦 + ∂𝑦 + 𝑦 ∂𝑦 2 = 2cos 2𝑢 ⋅ ∂𝑦
∂𝑥
Multiplying the first equation by 𝑥, the second by 𝑦, and adding, we get
∂2 𝑢
2
∂2 𝑢 2
∂2 𝑢 ∂𝑢 ∂𝑢 ∂𝑢 ∂𝑢
𝑥 + 2𝑥𝑦 + 𝑦 + 𝑥 + 𝑦 = 2cos 2𝑢 (𝑥 + 𝑦 )
∂𝑥 2 ∂𝑥 ∂𝑦 ∂𝑦 2 ∂𝑥 ∂𝑦 ∂𝑥 ∂𝑦
2
∂2 𝑢 ∂2 𝑢 2
∂2 𝑢 ∂𝑢 ∂𝑢
or 𝑥 2
+ 2𝑥𝑦 +𝑦 2
= (2cos 2𝑢 − 1) (𝑥 +𝑦 )
∂𝑥 ∂𝑥 ∂𝑦 ∂𝑦 ∂𝑥 ∂𝑦
2
= (1 − 4sin 𝑢)sin 2𝑢. [using (4)]

Ex. 4. If 𝑢 = 𝑥𝜙(𝑦/𝑥) + 𝜓(𝑦/𝑥), prove that


∂2 𝑢 ∂2 𝑢 ∂2 𝑢
𝑥2 + 2𝑥𝑦 + 𝑦 2
=0
∂𝑥 2 ∂𝑥 ∂𝑦 ∂𝑦 2
Let 𝑢1 = 𝑥𝜙(𝑦/𝑥) and 𝑢2 = 𝜓(𝑦/𝑥) so that 𝑢 = 𝑢1 + 𝑢2 .
Here 𝑢1 is a homogeneous function of degree 1 and 𝑢2 is of degree zero, so that by Euler's
theorem,
∂𝑢1 ∂𝑢1 ∂𝑢2 ∂𝑢2
𝑥 +𝑦 = 𝑢1 and 𝑥 +𝑦 =0
∂𝑥 ∂𝑦 ∂𝑥 ∂𝑦

Now
∂𝑢 ∂𝑢 ∂ ∂
𝑥 +𝑦 = 𝑥 (𝑢1 + 𝑢2 ) + 𝑦 (𝑢1 + 𝑢2 )
∂𝑥 ∂𝑦 ∂𝑥 ∂𝑦
∂𝑢1 ∂𝑢1 ∂𝑢2 ∂𝑢2
= (𝑥 +𝑦 ) + (𝑥 +𝑦 ) = 𝑢1
∂𝑥 ∂𝑦 ∂𝑥 ∂𝑦
[ from (1)]
or

129 | P a g e

© Department of Distance & Continuing Education, Campus of Open Learning,


School of Open Learning, University of Delhi
B.A. (Hons.) Economics

∂𝑢 ∂𝑢
𝑥 +𝑦 = 𝑢1
∂𝑥 ∂𝑦
Differentiating (2) partially with respect to first 𝑥 and then 𝑦, we have
∂2 𝑢 ∂𝑢 ∂2 𝑢 ∂𝑢1
𝑥 2
+ + 𝑦 =
∂𝑥 ∂𝑥 ∂𝑥 ∂𝑦 ∂𝑥
and
∂2 𝑢 ∂𝑢 ∂2 𝑢 ∂𝑢1
𝑥 + +𝑦 2 =
∂𝑦 ∂𝑥 ∂𝑦 ∂𝑦 ∂𝑦
Multiplying (3) by 𝑥 and (4) by 𝑦, adding and using (1) and (2), we get
∂2 𝑢
2
∂2 𝑢 2
∂2 𝑢
𝑥 + 2𝑥𝑦 +𝑦 + 𝑢1 = 𝑢1 .
∂𝑥 2 ∂𝑥 ∂𝑦 ∂𝑦 2
∂2 𝑢 ∂2 𝑢 ∂2 𝑢
𝑥 2 2 + 2𝑥𝑦 + 𝑦 2 2 = 0.
∂𝑥 ∂𝑥 ∂𝑦 ∂𝑦
9.3.3 Total Differential Coefficient:

If 𝑢 = 𝑓(𝑥, 𝑦) where 𝑥 = 𝜙1 (𝑡) and 𝑦 = 𝜙2 (𝑡), then 𝑥 and 𝑦 are not independent variables.
Substituting the values of 𝑥 and 𝑦 in 𝑢, we can express 𝑢 as a function of the single variable 𝑡
and we can find the ordinary differential coefficient 𝑑𝑢/𝑑𝑡.

To distinguish 𝑑𝑢/𝑑𝑡 from the partial differential coefficients ∂𝑢/ ∂𝑥 and ∂𝑢/ ∂𝑦, we shall
call 𝑑𝑢/𝑑𝑡 as the total differential coefficient. We shall now obtain a formula which will enable
us to find 𝑑𝑢/𝑑𝑡 without first expressing 𝑢 in terms of 𝑡 only.

Suppose 𝛿𝑥, 𝛿𝑦 and 𝛿𝑢 are the increments in 𝑥, 𝑦 and 𝑢 respectively corresponding to an


increment 𝛿𝑡 in 𝑡.

Then 𝑢 + 𝛿𝑢 = 𝑓(𝑥 + 𝛿𝑥, 𝑦 + 𝛿𝑦).

∴ 𝛿𝑢 = 𝑓(𝑥 + 𝛿𝑥, 𝑦 + 𝛿𝑦) − 𝑓(𝑥, 𝑦).

130 | P a g e

© Department of Distance & Continuing Education, Campus of Open Learning,


School of Open Learning, University of Delhi
Intermediate Mathematical Methods for Economics

𝛿𝑢 𝑓(𝑥 + 𝛿𝑥, 𝑦 + 𝛿𝑦) − 𝑓(𝑥, 𝑦)


∴ =
𝛿𝑡 𝛿𝑡
{𝑓(𝑥 + 𝛿𝑥, 𝑦 + 𝛿𝑦) − 𝑓(𝑥, 𝑦 + 𝛿𝑦)} + {𝑓(𝑥, 𝑦 + 𝛿𝑦) − 𝑓(𝑥, 𝑦)}
=
𝛿𝑡
𝑓(𝑥 + 𝛿𝑥, 𝑦 + 𝛿𝑦) − 𝑓(𝑥, 𝑦 + 𝛿𝑦) 𝑓(𝑥, 𝑦 + 𝛿𝑦) − 𝑓(𝑥, 𝑦)
= +
𝛿𝑡 𝛿𝑡
𝑓(𝑥 + 𝛿𝑥, 𝑦 + 𝛿𝑦) − 𝑓(𝑥, 𝑦 + 𝛿𝑦) 𝛿𝑥 𝑓(𝑥, 𝑦 + 𝛿𝑦) − 𝑓(𝑥, 𝑦) 𝛿𝑦
= ⋅ + ⋅ .
𝛿𝑥 𝛿𝑡 𝛿𝑦 𝛿𝑡
𝑑𝑢 𝛿𝑢
= lim
𝑑𝑡 𝛿𝑡→0 𝛿𝑡
𝑓(𝑥 + 𝛿𝑥, 𝑦 + 𝛿𝑦) − 𝑓(𝑥, 𝑦 + 𝛿𝑦) 𝛿𝑥
= lim ⋅
𝛿𝑡→0 𝛿𝑥 𝛿𝑡
𝑓(𝑥, 𝑦 + 𝛿𝑦) − 𝑓(𝑥, 𝑦) 𝛿𝑦
lim ⋅ .
𝑥𝑡 𝛿𝑦 𝛿𝑡
Now 𝛿𝑥 and 𝛿𝑦 also tend to zero when 𝛿𝑡 tends to zero.
So we have
𝑓(𝑥 + 𝛿𝑥, 𝑦 + 𝛿𝑦) − 𝑓(𝑥, 𝑦 + 𝛿𝑦) ∂𝑓 ∂𝑢
lim = = ,
𝛿𝑥→0 𝛿𝑥 ∂𝑥 ∂𝑥
because while 𝑥 becomes 𝑥 + 𝛿𝑥, 𝑦 + 𝛿𝑦 remains unchanged.
𝑓(𝑥,𝑦+𝛿𝑦)−𝑓(𝑥,𝑦) ∂𝑓 ∂𝑢
Similarly, lim𝑦→0 = ∂𝑦 = ∂𝑦.
𝛿𝑦
𝛿𝑥 𝑑𝑥 𝛿𝑦 𝑑𝑦
Also lim𝑡→0 = and lim𝛿𝑡→0 = .
𝛿𝑡 𝑑𝑡 𝛿𝑡 𝑑𝑡
𝑑𝑢 ∂𝑢 𝑑𝑥 ∂𝑢 𝑑𝑦
Therefore (1) gives = ∂𝑥 ⋅ 𝑑𝑡 + ∂𝑦 ⋅ 𝑑𝑡 .
𝑑𝑡
Similarly, if 𝑢 = 𝑓(𝑥1 , 𝑥2 , … , 𝑥𝑚 ) and 𝑥1 , 𝑥2 , … , 𝑥𝑚 are all functions of 𝑡, we can
prove that
𝑑𝑢 ∂𝑢 𝑑𝑥1 ∂𝑢 𝑑𝑥2 ∂𝑢 𝑑𝑥𝑚
= ⋅ + ⋅ + ⋯+ ⋅
𝑑𝑡 ∂𝑥1 𝑑𝑡 ∂𝑥2 𝑑𝑡 ∂𝑥𝑚 𝑑𝑡
9.3.4 First Differential Coefficient of Implicit Functions:

Suppose 𝑦 = 𝑓(𝑥, 𝑦), where 𝑦 = 𝜙(𝑥)., Then supposing 𝑡 to be the same as 𝑥 in the formula,
we get
𝑑𝑢 ∂𝑢 𝑑𝑥 ∂𝑢 𝑑𝑦 𝑑𝑢 ∂𝑢 ∂𝑢 𝑑𝑦
= ⋅ + ⋅ or = + ⋅
𝑑𝑥 ∂𝑥 𝑑𝑥 ∂𝑦 𝑑𝑥 𝑑𝑥 ∂𝑥 ∂𝑦 𝑑𝑥

131 | P a g e

© Department of Distance & Continuing Education, Campus of Open Learning,


School of Open Learning, University of Delhi
B.A. (Hons.) Economics

Now suppose we are given an implicit relation between 𝑥 and 𝑦 of the form 𝑢 ≡ 𝑓(𝑥, 𝑦) = 𝑐,
whêre 𝑐 is a constant and 𝑦 is a function of 𝑥.

• Then, we have 𝑑𝑢/𝑑𝑥 = 0


∂𝑢 ∂𝑢 𝑑𝑦 𝑑𝑦 ∂𝑢/ ∂𝑥 𝑑𝑦 ∂𝑓/ ∂𝑥
+ = 0 or =− or =− .
∂𝑥 ∂𝑦 𝑑𝑥 𝑑𝑥 ∂𝑢/ ∂𝑦 𝑑𝑥 ∂𝑓/ ∂𝑦
SOLVED EXAMPLES
Ex. 1. If (tan 𝑥)𝑦 + (𝑦)cot 𝑥 = 𝑎, find the value of 𝑑𝑦/𝑑𝑥.
Sol. Let 𝑓(𝑥, 𝑦) ≡ (tan 𝑥)𝑦 + (𝑦)cot 𝑥 = 𝑎.
𝑑𝑦 ∂𝑓/∂𝑥
Then, we have 𝑑𝑥 = − ∂𝑓/∂𝑦.

∂𝑓 ∂𝑓
Now = 𝑦(tan 𝑥)𝑦−1 sec 2 𝑥 + 𝑦 cot 𝑥 ⋅ log 𝑦 ⋅ (−cosec 2 𝑥) and =
∂𝑥 ∂𝑦
(tan 𝑥) log tan 𝑥 + (cot 𝑥) ⋅ 𝑦 cot 𝑥−1.
𝑦

Therefore (1) gives


𝑑𝑦 𝑦(tan 𝑥)𝑦−1 ⋅ sec 2 𝑥 − 𝑦 cot 𝑥 ⋅ log 𝑦 ⋅ cosec 2 𝑥
=− .
𝑑𝑥 (tan 𝑥)𝑦 log tan 𝑥 + cot 𝑥 ⋅ 𝑦 cot 𝑥−1
Ex. 2. If 𝑢 = 𝑥 2 𝑦, where 𝑥 2 + 𝑥𝑦 + 𝑦 2 = 1, find 𝑑𝑢/𝑑𝑥.
𝑑𝑢 ∂𝑢 ∂𝑢 𝑑𝑦
Sol. We have, 𝑑𝑥 = ∂𝑥 + ∂𝑦 ⋅ 𝑑𝑥 .

∂𝑢 ∂𝑢
Now = 2𝑥𝑦 and ∂𝑦 = 𝑥 2 .
∂𝑥

Let 𝑓(𝑥, 𝑦) ≡ 𝑥 2 + 𝑥𝑦 + 𝑦 2 = 1.
𝑑𝑦 ∂𝑓/∂𝑥 2𝑥+𝑦
Then = − ∂𝑓/∂𝑦 = − 𝑥+2𝑦.
𝑑𝑥

So, putting the values in (1), we get


𝑑𝑢 2
2𝑥 + 𝑦 𝑥 2 (2𝑥 + 𝑦)
= 2𝑥𝑦 + 𝑥 ⋅ (− ) = 2𝑥𝑦 − .
𝑑𝑥 𝑥 + 2𝑦 𝑥 + 2𝑦

132 | P a g e

© Department of Distance & Continuing Education, Campus of Open Learning,


School of Open Learning, University of Delhi
Intermediate Mathematical Methods for Economics

LAGRANGE'S NECESSARY AND SUFFICIENT CONDITIONS FOR THE MAXIMA


OR MINIMA OF A FUNCTION OF THREE INDEPENDENT VARIABLES.
Necessary Conditions. Let 𝑓(𝑥, 𝑦, 𝑧) be a function of three independent variables 𝑥, 𝑦 and 𝑧.
Then as derived in §2, for 𝑓(𝑥, 𝑦, 𝑧) to be a maximum or a minimum at any point (𝑎, 𝑏, 𝑐), it
∂𝑓 ∂𝑓 ∂𝑓
is necessary that ∂𝑥 = 0, ∂𝑦 = 0 and ∂𝑧 = 0 at that point.

Hence the points where the value of the function 𝑓(𝑥, 𝑦, 𝑧) is stationary (i.e., may be a
maximum or a minimum) are obtained by solving the simultaneous equations.
∂𝑓 ∂𝑓 ∂𝑓
= 0, = 0, =0
∂𝑥 ∂𝑦 ∂𝑧
Sufficient Conditions. Before deriving the sufficient conditions for the existence of a
maximum or a minimum of a function of three independent variables, we obtain the following
two algebraic lemmas regarding the signs of quadratic expressions.
Lemma 1. Let 𝐼2 = 𝑎𝑥 2 + 2ℎ𝑥𝑦 + 𝑏𝑦 2 be a quadratic expression in two variables 𝑥 and 𝑦.
We can write
1 2 2
𝐼2 = [𝑎 𝑥 + 2𝑎ℎ𝑥𝑦 + 𝑎𝑏𝑦 2 ], if 𝑎 ≠ 0
𝑎
1
= [(𝑎𝑥 + ℎ𝑦)2 + (𝑎𝑏 − ℎ2 )𝑦 2 ].
𝑎
The expression within the square brackets will be positive if 𝑎𝑏 − ℎ2 is positive and in that
case the sign of the expression 𝐼2 will be the same as that of 𝑎.
In case 𝑎𝑏 − ℎ2 is not positive, we can say nothing about the sign of the expression within the
square brackets and hence nothing about the sign of the given quadratic expression 𝐼2 .
Lemma 2. In three variables 𝑥, 𝑦 and 𝑧,
𝐼3 ≡ 𝑎𝑥 2 + 𝑏𝑦 2 + 𝑐𝑧 2 + 2𝑓𝑦𝑧 + 2𝑔𝑧𝑥 + 2ℎ𝑥𝑦
1
= [𝑎2 𝑥 2 + 𝑎𝑏𝑦 2 + 𝑎𝑐𝑧 2 + 2𝑓𝑎𝑦𝑧 + 2𝑔𝑎𝑧𝑥 + 2ℎ𝑎𝑥𝑦], if 𝑎 ≠ 0
𝑎
1
= [𝑎2 𝑥 2 + 2𝑎𝑥(𝑔𝑧 + ℎ𝑦) + 𝑎𝑏𝑦 2 + 𝑎𝑐𝑧 2 + 2𝑓𝑎𝑦𝑧]
𝑎
1
= [(𝑎𝑥 + ℎ𝑦 + 𝑔𝑧)2 + 𝑎𝑏𝑦 2 + 𝑎𝑧 2 + 2𝑓𝑎𝑦𝑧 − (𝑔𝑧 + ℎ𝑦)2 ]
𝑎
1
= [(𝑎𝑥 + ℎ𝑦 + 𝑔𝑧)2 + (𝑎𝑏 − ℎ2 )𝑦 2 + 2𝑦𝑧(𝑓𝑎 − 𝑔ℎ) + (𝑎𝑐 − 𝑔2 )𝑧 2 ].
𝑎

133 | P a g e

© Department of Distance & Continuing Education, Campus of Open Learning,


School of Open Learning, University of Delhi
B.A. (Hons.) Economics

Now 𝐼3 will be of the same sign as 𝑎 provided the expression within the square brackets is
positive which will of course be so if
𝑎𝑏 − ℎ2 and {(𝑎𝑏 − ℎ2 )(𝑎𝑐 − 𝑔2 ) − (𝑓𝑎 − 𝑔ℎ)2 }
are both positive i.e., if
𝑎𝑏 − ℎ2 and 𝑎(𝑎𝑏𝑐 + 2𝑓𝑔ℎ − 𝑎𝑓 2 − 𝑏𝑔2 − 𝑐ℎ2 )
are both positive.
Thus 𝐼3 will be positive if
𝑎 ℎ 𝑔
𝑎 ℎ ℎ 𝑏 𝑓|
𝑎, | |,|
ℎ 𝑏
𝑔 𝑓 𝑐

be all positive and will be negative if these three expressions are alternately negative and
positive.
Now we are in a position to derive Lagrange's sufficient conditions for the existence of a
maximum or a minimum of a function of three independent variables at a stationary point.
Let a set of the values of 𝑥, 𝑦, 𝑧 obtained by solving the equations
∂𝑓 ∂𝑓 ∂𝑓
= = = 0 be 𝑎, 𝑏, 𝑐.
∂𝑥 ∂𝑦 ∂𝑧
Let the values of the six second order partial derivatives
∂2 𝑓 ∂2 𝑓 ∂2 𝑓 ∂2 𝑓 ∂2 𝑓 ∂2 𝑓
, , , , and
∂𝑥 2 ∂𝑦 2 ∂𝑧 2 ∂𝑦 ∂𝑧 ∂𝑧 ∂𝑥 ∂𝑥 ∂𝑦
at the point (𝑎, 𝑏, 𝑐) be denoted by 𝐴, 𝐵, 𝐶, 𝐹, 𝐺 and 𝐻 respectively.
Then, we have
𝑓(𝑎 + ℎ, 𝑏 + 𝑘, 𝑐 + 𝑙) − 𝑓(𝑎, 𝑏, 𝑐)
1
= (𝐴ℎ2 + 𝐵𝑘 2 + 𝐶𝑙 2 + 2𝐹𝑘𝑙 + 2𝐺𝑙ℎ + 2𝐻ℎ𝑘) + 𝑅3 ,
2!
where 𝑅3 consists of terms of third and higher orders of small quantities ℎ, 𝑘 and 𝑙. By taking
ℎ, 𝑘 and 𝑙 sufficiently small, the second-degree terms in ℎ, 𝑘 and 𝑙 can be made to govern the
sign of the right hand side and therefore of the left hand side of (1). If this group of terms forms
an expression of invariable sign for all such values

134 | P a g e

© Department of Distance & Continuing Education, Campus of Open Learning,


School of Open Learning, University of Delhi
Intermediate Mathematical Methods for Economics

of ℎ, 𝑘 and 𝑙, we shall have a maximum or a minimum value of 𝑓(𝑥, 𝑦, 𝑧) at (𝑎, 𝑏, 𝑐)


I-42 according as that sign is negative or positive.
Hence by our lemma 2, if the expressions
𝐴 𝐻 𝐺
𝐴 𝐻
𝐴, | | , |𝐻 𝐵 𝐹|
𝐻 𝐵
𝐺 𝐹 𝐶
be all positive, we shall have a minimum of 𝑓(𝑥, 𝑦, 𝑧) at (𝑎, 𝑏, 𝑐) and if these expressions be
alternately negative and positive, we shall have a maximum of 𝑓(𝑥, 𝑦, 𝑧) at (𝑎, 𝑏, 𝑐), whilst if
these conditions are not satisfied, we shall in general have neither a maximum nor a minimum
of 𝑓(𝑥, 𝑦, 𝑧) at (𝑎, 𝑏, 𝑐).
WORKING RULE FOR FINDING THE MAXIMA AND MINIMA OF A FUNCTION
OF THREE INDEPENDENT VARIABLES.
Suppose 𝑓(𝑥, 𝑦, 𝑧) is a given function of three independent variables 𝑥, 𝑦 and 𝑧. Find
∂𝑓/ ∂𝑥, ∂𝑓/ ∂𝑦 and ∂𝑓/ ∂𝑧 and solve the simultaneous equations ∂𝑓/ ∂𝑥 = 0, ∂𝑓/ ∂𝑦 = 0 and
∂𝑓/ ∂𝑧 = 0. All the triads (𝑎, 𝑏, 𝑐) of the values of 𝑥, 𝑦 and 𝑧 obtained on solving these
equations will give the stationary values of 𝑓(𝑥, 𝑦, 𝑧) i.e., will give the points at which the
function 𝑓(𝑥, 𝑦, 𝑧) may be a maximum or a minimum.
To discuss the maximum or minimum of 𝑓(𝑥, 𝑦, 𝑧) at any point (𝑎, 𝑏, 𝑐) obtained on solving
the equations ∂𝑓/ ∂𝑥 = 0, ∂𝑓/ ∂𝑦 = 0 and ∂𝑓/ ∂𝑧 = 0, we find the values at this point of the
six partial derivatives of second order of 𝑓(𝑥, 𝑦, 𝑧) symbolically denoted as follows :
∂2 𝑓 ∂2 𝑓 ∂2 𝑓 ∂2 𝑓 ∂2 𝑓 ∂2 𝑓
𝐴 = 2 ,𝐵 = 2,𝐶 = 2 ,𝐹 = ,𝐺 = and 𝐻 = .
∂𝑥 ∂𝑦 ∂𝑧 ∂𝑦 ∂𝑧 ∂𝑧 ∂𝑥 ∂𝑥 ∂𝑦
If the expressions
𝐴 𝐻 𝐺
𝐴 𝐻
𝐴, | | , |𝐻 𝐵 𝐹|
𝐻 𝐵
𝐺 𝐹 𝐶
be all positive, we shall have a minimum of 𝑓(𝑥, 𝑦, 𝑧) at (𝑎, 𝑏, 𝑐) and if these expressions be
alternately negative and positive, we shall have a maximum of 𝑓(𝑥, 𝑦, 𝑧) a (𝑎, 𝑏, 𝑐), whilst if
these conditions are not satisfied, we shall in general have neither a maximum nor a minimum
of 𝑓(𝑥, 𝑦, 𝑧) at (𝑎, 𝑏, 𝑐).
SOLVED EXAMPLES
Ex. 1. Discuss the maximum or minimum values of 𝑢 where
135 | P a g e

© Department of Distance & Continuing Education, Campus of Open Learning,


School of Open Learning, University of Delhi
B.A. (Hons.) Economics

𝑢 = 𝑥 2 + 𝑦 2 + 𝑧 2 + 𝑥 − 2𝑧 − 𝑥𝑦
Sol. For a maximum or a minimum of 𝑢, we must have
∂𝑢
= 2𝑥 − 𝑦 + 1 = 0
∂𝑥
∂𝑢
= −𝑥 + 2𝑦 = 0
∂𝑦
∂𝑢
= 2𝑧 − 2 = 0
∂𝑧
These equations give 𝑥 = −2/3, 𝑦 = −1/3, 𝑧 = 1.
∴ (−2/3, −1/3,1) is the only point at which 𝑢 is statior.ary i.e., at which 𝑢 may have a
maximum or a minimum.
∂2 𝑢 ∂2 𝑢 ∂2 𝑢 ∂2 𝑢 ∂2 𝑢 ∂2 𝑢
Now = 2, ∂𝑦 2 = 2, ∂𝑧 2 = 2, ∂𝑦 ∂𝑧 = 0, ∂𝑧 ∂𝑥 = 0 and = −1.
∂𝑥 2 ∂𝑥 ∂𝑦

If 𝐴, 𝐵, 𝐶, 𝐹, 𝐺 and 𝐻 denote the respective values of these six partial derivatives of second
order at the point (−2/3, −1/3,1), then
𝐴 = 2, 𝐵 = 2, 𝐶 = 2, 𝐹 = 0, 𝐺 = 0, 𝐻 = −1.
Now we have
and
𝐴 𝐻 2 −1
𝐴 = 2, | |=| |=3
𝐻 𝐵 −1 2
𝐴 𝐻 𝐺 2 −1 0
|𝐻 𝐵 𝐹 | = |−1 2 0| = 6.
𝐺 𝐹 𝐶 0 0 2
Since these three expressions are all positive, we have a minimum of 𝑢 when 𝑥 = −2/3, 𝑦 =
−1/3, 𝑧 = 1.
Ex. 2. Show that the point such that the sum of the squares of its distances from 𝑛 given points
shall be minimum, is the centre of the mean position of the given points.

Sol. Let the 𝑛 given points be (𝑎1 , 𝑏1 , 𝑐1 ), (𝑎2 , 𝑏2 , 𝑐2 ), … … , (𝑎𝑛 , 𝑏𝑛 , 𝑐𝑛 ) and let (𝑥, 𝑦, 𝑧) be the
coordinates of the required point.
If 𝑢 denotes the sum of the squares of the distances of (𝑥, 𝑦, 𝑧) from the 𝑛 given points, then.

136 | P a g e

© Department of Distance & Continuing Education, Campus of Open Learning,


School of Open Learning, University of Delhi
Intermediate Mathematical Methods for Economics

𝑢 = Σ[(𝑥 − 𝑎1 )2 + (𝑦 − 𝑏1 )2 + (𝑧 − 𝑐1 )2 ]
= Σ(𝑥 − 𝑎1 )2 + Σ(𝑦 − 𝑏1 )2 + Σ(𝑧 − 𝑐1 )2 .
For a maximum or a minimum of 𝑢, we must have
∂𝑢
= 2Σ(𝑥 − 𝑎1 ) = 2𝑛𝑥 − 2Σ𝑎1 = 0
∂𝑥
∂𝑢
= 2Σ(𝑦 − 𝑏1 ) = 2𝑛𝑦 − 2Σ𝑏1 = 0
∂𝑦
∂𝑢
= 2Σ(𝑧 − 𝑐1 ) = 2𝑛𝑧 − 2Σ𝑐1 = 0
∂𝑧
Solving these equations, we get
Now
Σ𝑎1 Σ𝑏1 Σ𝑐1
𝑥= ,𝑦 = ,𝑧 = .
𝑛 𝑛 𝑛
∂2 𝑢 ∂2 𝑢 ∂2 𝑢
𝐴 = 2 = 2𝑛, 𝐵 = 2 = 2𝑛, 𝐶 = 2 = 2𝑛,
∂𝑥 ∂𝑦 ∂𝑧
2 2
∂ 𝑢 ∂ 𝑢 ∂2 𝑢
𝐹= = 0, 𝐺 = = 0, 𝐻 = = 0.
∂𝑦 ∂𝑧 ∂𝑧 ∂𝑥 ∂𝑥 ∂𝑦
𝐴 𝐻 2𝑛 0
We have 𝐴 = 2𝑛, | |=| | = 4𝑛2 ,
𝐻 𝐵 0 2𝑛
and
𝐴 𝐻 𝐺 2𝑛 0 0
|𝐻 𝐵 𝐹| = | 0 2𝑛 0 | = 8𝑛3
𝐺 𝐹 𝐶 0 0 2𝑛
Since these three expressions are all positive, 𝑢 is minimum when.
Σ𝑎1 Σ𝑏1 Σ𝑐1
𝑥= ,𝑦 = and 𝑧 =
𝑛 𝑛 𝑛
i.e., 𝑢 is minimum when the point (𝑥, 𝑦, 𝑧) is the centre of the mean position of the 𝑛 given
points.
EX3. Find the maximum value of 𝑢 where
𝑥𝑦𝑧
𝑢 = (𝑎+𝑥)(𝑥+𝑦)(𝑦+𝑧)(𝑧+𝑏).

Sol. We have
137 | P a g e

© Department of Distance & Continuing Education, Campus of Open Learning,


School of Open Learning, University of Delhi
B.A. (Hons.) Economics

log 𝑢 = log 𝑥 + log 𝑦 + log 𝑧 − log (𝑎 + 𝑥) − log (𝑥 + 𝑦) − log (𝑦 + 𝑧)


−log (𝑧 + 𝑏)

1 ∂𝑢 1 1 1 𝑎𝑦 − 𝑥 2
∴ ⋅ = − − =
𝑢 ∂𝑥 𝑥 𝑎 + 𝑥 𝑥 + 𝑦 𝑥(𝑎 + 𝑥)(𝑥 + 𝑦)
∂𝑢 (𝑎𝑦 − 𝑥 2 )𝑢
= .
∂𝑥 𝑥(𝑎 + 𝑥)(𝑥 + 𝑦)
∂𝑢 (𝑥𝑧−𝑦 2 )𝑢
Similarly, ∂𝑦 = 𝑦(𝑥+𝑦)(𝑦+𝑧)

∂𝑢 (𝑏𝑦 − 𝑧 2 )𝑢
=
∂𝑧 𝑧(𝑦 + 𝑧)(𝑧 + 𝑏)
Now for a maximum or a minimum of 𝑢, we must have
∂𝑢
= 0 i.e., 𝑎𝑦 − 𝑥 2 = 0
∂𝑥
∂𝑢
= 0 i.e., 𝑥𝑧 − 𝑦 2 = 0
∂𝑦
∂𝑢
and = 0 i.e., 𝑏𝑦 − 𝑧 2 = 0.
∂𝑧

From the above equations, it follows that 𝑎, 𝑥, 𝑦, 𝑧 and 𝑏 are in geometrical progression. Let 𝑟
be the common ratio of this geometrical progression. Then
𝑎𝑟 4 = 𝑏 or 𝑟 = (𝑏/𝑎)1/4 .
Also 𝑥 = 𝑎𝑟, 𝑦 = 𝑎𝑟 2 , 𝑧 = 𝑎𝑟 3 .
Substituting these values, we get
𝑎𝑟 ⋅ 𝑎𝑟 2 ⋅ 𝑎𝑟 3
𝑢 =
𝑎(1 + 𝑟) ⋅ 𝑎𝑟(1 + 𝑟) ⋅ 𝑎𝑟 2 (1 + 𝑟) ⋅ 𝑎𝑟 3 (1 + 𝑟)
1 1 1
= 4
= 1/4 4
= 1/4 .
𝑎(1 + 𝑟) 𝑎[1 + (𝑏/𝑎) ] (𝑎 + 𝑏1/4 )4
This gives a stationary value of 𝑢. To decide whether this value of 𝑢 is a maximum or a
minimum we proceed to find the second order partial derivatives of
We have

138 | P a g e

© Department of Distance & Continuing Education, Campus of Open Learning,


School of Open Learning, University of Delhi
Intermediate Mathematical Methods for Economics

∂2 𝑢 −2𝑥𝑢 2)
∂ 𝑢
= + (𝑎𝑦 − 𝑥 [ ]
∂𝑥 2 𝑥(𝑎 + 𝑥)(𝑥 + 𝑦) ∂𝑥 𝑥(𝑎 + 𝑥)(𝑥 + 𝑦)
∴ when 𝑥 = 𝑎𝑟, 𝑦 = 𝑎𝑟 2 , 𝑧 = 𝑎𝑟 3 , we have
∂2 𝑢 −2⋅𝑎𝑟⋅𝑢 −2𝑢
𝐴 = ∂𝑥 2 = 𝑎𝑟⋅𝑎(1+𝑟)⋅𝑎𝑟(1+𝑟) = 𝑎2 𝑟(1+𝑟)2 , which is negative. Hence the above stationary value
of 𝑢 is a maximum.
1
Ans. Maximum value of 𝑢 = 4 .
(𝑎1/4 +𝑏 1/4 )

Note. In the complicated problems in order to find whether a stationary value of 𝑢 is a


maximum or a minimum, it is sufficient to find the value of a second partial differential
coefficient of 𝑢 with respect to any of the independent variables. The value of 𝑢 will be
maximum or minimum according as the value of this second partial derivative at the stationary
point under consideration is -ive or +ive.
Self-Assessment questions:
1 State Euler's theorem on homogeneous functions.
2 Verify Euler's theorem in the following cases:
(i) 𝑢 = 𝑎𝑥 2 + 2ℎ𝑥𝑦 + 𝑏𝑦 2.
𝑥(𝑥 3 −𝑦 3 )
(ii) 𝑢 = 𝑥 3 +𝑦 3

(iii) 𝑢 = 𝑎𝑥𝑦 + 𝑏𝑦𝑧 + 𝑐𝑧𝑥.


(iv) 𝑢 = 𝑥 𝑛 sin (𝑦/𝑥).
(Meerut 1984)
(v) 𝑢 = 𝑥 𝑛 log (𝑦/𝑥).
(vi) 𝑢 = 1/√(𝑥 2 + 𝑦 2 ).
𝑥 3 +𝑦 3 ∂𝑢 ∂𝑢
3. If 𝑢 = tan−1 ( ), prove that 𝑥 ∂𝑥 + 𝑦 ∂𝑦 = sin 2𝑢.
𝑥−𝑦
√ 𝑥− 𝑦 ∂𝑢 ∂𝑢
4. If 𝑢 = sin−1 𝑥+√𝑦, show that 𝑥 ∂𝑥 + 𝑦 ∂𝑦 = 0.
√ √
−1 𝑥+𝑦 ∂𝑢 ∂𝑢 1
5. If 𝑢 = sin , show that 𝑥 ∂𝑥 + 𝑦 ∂𝑦 = 2 tan 𝑢.
√𝑥+√ 𝑦
𝑥 4 +𝑦 4 ∂𝑢 ∂𝑢
6. If 𝑢 = log , show that 𝑥 ∂𝑥 + 𝑦 ∂𝑦 = 3.
𝑥+𝑦
𝑥 3 +𝑦 3 ∂𝑢 ∂𝑢
7. If 𝑢 = log , show that 𝑥 ∂𝑥 + 𝑦 ∂𝑦 = 2.
𝑥+𝑦
8. Use Euler's theorem on homogeneous functions to show that if 𝑢 = tan−1 (𝑦/𝑥), then

139 | P a g e

© Department of Distance & Continuing Education, Campus of Open Learning,


School of Open Learning, University of Delhi
B.A. (Hons.) Economics

∂𝑢 ∂𝑢
𝑥 ∂𝑥 + 𝑦 ∂𝑦 = 0.
9. If 𝑢 be a homogeneous function of 𝑥 and 𝑦 of degree 𝑛, show that
∂2 𝑢 ∂2 𝑢 ∂𝑢
(i) 𝑥 ∂𝑥 2 + 𝑦 ∂𝑥 ∂𝑦 = (𝑛 − 1) ∂𝑥 .
∂2 𝑢 ∂2 𝑢 ∂𝑢
(ii) 𝑥 ∂𝑥 ∂𝑦 + 𝑦 ∂𝑦 2 = (𝑛 − 1) ∂𝑦.
∂2 𝑢 ∂2 𝑢 ∂2 𝑢
(iii) 𝑥 2 ∂𝑥 2 + 2𝑥𝑦 ∂𝑥 ∂𝑦 + 𝑦 2 ∂𝑦 2 = 𝑛(𝑛 − 1)𝑢.

𝑥𝑦 ∂2 𝑢 ∂2 𝑢 ∂2 𝑢
10 If 𝑢 = , show that 𝑥 2 2
+ 2𝑥𝑦 + 𝑦2 = 0.
𝑥+𝑦 ∂𝑥 ∂𝑥 ∂𝑦 ∂𝑦 2

∂2 𝑢 ∂2 𝑢 ∂2 𝑢
11 If 𝑢 = 𝑥𝜙(𝑦/𝑥) + 𝜓(𝑦/𝑥), prove that 𝑥 2 ∂𝑥 2 + 2𝑥𝑦 ∂𝑥 ∂𝑦 + 𝑦 2 ∂𝑦 2 = 0.

Self-Assessment Questions
1 Find 𝑑𝑦/𝑑𝑥 in the following:
(i) 𝑥 𝑦 + 𝑦 𝑥 = 𝑎𝑏 .
(ii) 𝑎𝑥 2 + 2ℎ𝑥𝑦 + 𝑏𝑦 2 = 1.
𝑑𝑦 √(1−𝑦 2 )
2 If √(1 − 𝑥 2 ) + √(1 − 𝑦 2 ) = 𝑎(𝑥 − 𝑦), prove that 𝑑𝑥 = .
√(1−𝑥 2 )

3 Find 𝑑𝑢/𝑑𝑥 if 𝑢 = sin (𝑥 2 + 𝑦 2 ), where 𝑎2 𝑥 2 + 𝑏 2 𝑦 2 = 𝑐 2 .


4 If 𝑢 = 𝑥 4 𝑦 5 , where 𝑥 = 𝑡 2 and 𝑦 = 𝑡 3 , find 𝑑𝑢/𝑑𝑡.
∂𝑓 ∂𝜙 𝑑𝑧 ∂𝑓 ∂𝜙
If 𝑓(𝑥, 𝑦) = 0, 𝜙(𝑦, 𝑧) = 0, show that ∂𝑦 ⋅ ∂𝑧 ⋅ 𝑑𝑥 = ∂𝑥 ⋅ ∂𝑦 .
(Meerut 1983)

5 If 𝑢 = √(𝑥 2 + 𝑦 2 )and 𝑥 3 + 𝑦 3 + 3𝑎𝑥𝑦 = 5𝑎2 , find the value of 𝑑𝑢/𝑑𝑥 when 𝑥 =


𝑎, 𝑦 = 𝑎.
Answers
{𝑦𝑥 𝑦−1 +𝑦 𝑥 log 𝑦}
1 (i) − {𝑥 𝑦 log 𝑥+𝑥𝑦 𝑥−1 },
(𝑎𝑥+ℎ𝑦)
(ii) − (ℎ𝑥+𝑏𝑦).

𝑎2
2 2𝑥{cos (𝑥 2 + 𝑦 2 )} (1 − 𝑏2 ).

6. 0.
3 23𝑡 22 .

140 | P a g e

© Department of Distance & Continuing Education, Campus of Open Learning,


School of Open Learning, University of Delhi
Intermediate Mathematical Methods for Economics

(OBJECTIVE QUESTIONS)
Fill In the Blanks.
Fill in the blanks "......", so that the following statements are complete and correct.
∂2 𝑢 ∂2 𝑢
1. If 𝑢 = 𝑒 𝑚𝑦 cos 𝑚𝑥, then ∂𝑥 2 + ∂𝑦 2 = ⋯ ….

𝑥 2 +𝑦 2 ∂𝑢
2. If 𝑢 = tan−1 , then ∂𝑥 =
𝑥+𝑦

3. An expression in which every term is of the same degree is called a function.


𝑑𝑢
If 𝑢 = 𝑓(𝑥, 𝑦), where 𝑥 = 𝜙(𝑡) and 𝑦 = 𝜓(𝑡), then =
𝑑𝑡

4. If 𝑢(𝑥, 𝑦) is a homogeneous function of 𝑥 and 𝑦 of degree 𝑛, then


∂ ∂
𝑥 (𝑢𝑥 ) + 𝑦 (𝑢𝑥 ) = ⋯ ….
∂𝑥 ∂𝑦
∂𝑢
where 𝑢𝑥 = .
∂𝑥
∂2 𝑢 𝑎2 ∂2 𝑢
5. If 𝑢 = 𝑓(𝑦 + 𝑎𝑥) + 𝜙(𝑦 − 𝑎𝑥), then ∂𝑥 2 − =
∂𝑦 2

∂𝑢 ∂𝑢
6. If 𝑢 = 𝑓(𝑦/𝑥), then, 𝑥 ∂𝑥 + 𝑦 ∂𝑦 = ⋯ ….

Multiple Choice Questions.


Indicate the correct answer for each question by writing the corresponding letter from (a), (b),
(c) and (d).
∂2 𝑧 ∂2 𝑧
8. If 𝑧 = tan (𝑦 + 𝑎𝑥) + (𝑦 − 𝑎𝑥)3/2, then (∂𝑥 2 ) − 𝑎2 (∂𝑦 2 ) is equal to
(a) 0
(b) (𝑦 − 𝑎𝑥)
(c) 1
(d) sec (𝑦 + 𝑎𝑥)
∂𝑢 ∂𝑢
9 If 𝑢 is a homogeneous function of 𝑥 and 𝑦 of degree 𝑛, then 𝑥 ∂𝑥 + 𝑦 ∂𝑦 is equal to
(a) 𝑛
(b) 𝑛𝑢
𝑛
(c) 𝑢
(d) 𝑢

141 | P a g e

© Department of Distance & Continuing Education, Campus of Open Learning,


School of Open Learning, University of Delhi
B.A. (Hons.) Economics

1 ∂𝑢 ∂𝑢 ∂𝑢
10 If 𝑢 = √(𝑥 2 + 𝑦 2 + 𝑧 2 ), then 𝑥 ∂𝑥 + 𝑦 ∂𝑦 + 𝑧 ∂𝑧 is equal to
(a) 𝑢
(b) −𝑢
(c) 𝑢2
(d) 0.
𝑑𝑦
11 If 𝑥 and 𝑦 are connected by an equation of the form 𝑓(𝑥, 𝑦) = 0, then 𝑑𝑥 is
∂𝑓/∂𝑥
(a) ∂𝑓/∂𝑦
∂𝑓/∂𝑥
(b) (−1)𝑛
∂𝑓/∂𝑦
∂𝑓/∂𝑦
(c) ∂𝑓/∂𝑥
∂𝑓/∂𝑥
(d) − ∂𝑓/∂𝑦

∂𝜃
12 If 𝑥 = rcos 𝜃 , 𝑦 = rsin 𝜃, then the value of ∂𝑥 is
sin 𝜃
(a) − 𝑟
𝑟
(b) sin 𝜃
sin 𝜃
(c) 𝑟
1
(d) − 𝑟sin 𝜃

13 If 𝑓(𝑥, 𝑦) be a homogeneous function of 𝑥 and 𝑦 of degree 𝑛, then


∂𝑓 ∂𝑓
(a) 𝑥 ∂𝑥 + 𝑦 ∂𝑦 = 𝑓
∂𝑓 ∂𝑓
(b) 𝑦 ∂𝑥 + 𝑥 ∂𝑦 = 𝑓
∂𝑓 ∂𝑓
(c) 𝑦 ∂𝑥 + 𝑥 ∂𝑦 = 𝑛𝑓
∂𝑓 ∂𝑓
(d) 𝑥 ∂𝑥 + 𝑦 ∂𝑦 = 𝑛𝑓
True or False.
Write ' 𝑇 ' for true and ' 𝐹 ' for false statement.
14 If 𝑢 = 𝑓(𝑥, 𝑦) and its partial derivatives are continuous, then the order of differentiation
is immaterial:
∂𝑢 ∂𝑢
15 If 𝑢 is a homogeneous function of 𝑥 and 𝑦 of degree 𝑛, then and are also
∂𝑥 ∂𝑦
homogeneous functions of 𝑥 and 𝑦 each being of degree 𝑛.

142 | P a g e

© Department of Distance & Continuing Education, Campus of Open Learning,


School of Open Learning, University of Delhi
Intermediate Mathematical Methods for Economics

∂𝑓 ∂𝑓 ∂2 𝑓 ∂2 𝑓
16 If 𝑓(𝑥, 𝑦) = 0 be an implicit function of 𝑥 and 𝑦 and 𝑝 = ∂𝑥 , 𝑞 = ∂𝑦 , 𝑟 = ∂𝑥 2 , 𝑠 = ∂𝑥 ∂𝑦
∂2 𝑓 𝑑2 𝑦
and 𝑡 = ∂𝑦 2, then 𝑑𝑥 2 = −(𝑞 2 𝑟 − 2𝑝𝑞𝑠 + 𝑝2 𝑡)/𝑞 3

Answers
1 0.
(𝑥 2 +2𝑥𝑦−𝑦 2 )
2 {(𝑥+𝑦)2 +(𝑥2 +𝑦 2 )2 }
.

3 homogeneous.
∂𝑢 𝑑𝑥 ∂𝑢 𝑑𝑦
4 ⋅ 𝑑𝑡 + ∂𝑦 ⋅ 𝑑𝑡 .
∂𝑥

5 (𝑛 − 1)𝑢𝑥 .
6 (a).
7 (b).
8 (b).
9 (d).
10 (a).
11 (d).
12 𝑇.
13 𝐹.
14 𝑇.

Self-Assessment Questions
1 Verify Euler's theorem in the following cases:
(i) 𝑓(𝑥, 𝑦) = 𝑎𝑥 2 + 2ℎ𝑥𝑦 + 𝑏𝑦 2.
(ii) 𝑓(𝑥, 𝑦, 𝑧) = 𝑎𝑦𝑧 + 𝑏𝑥𝑧 + 𝑐𝑥
2 Verify Euler's Theorem for the function
(i) 𝑢 = (𝑥1/4 + 𝑦 1/4 )/(𝑥1/5 + 𝑦 1/5 ).

(ii) 𝑧 = sin 𝑢 where 𝑢 = sin−1 (√𝑥 2 + 𝑦 2 )/(𝑥 + 𝑦).

143 | P a g e

© Department of Distance & Continuing Education, Campus of Open Learning,


School of Open Learning, University of Delhi
B.A. (Hons.) Economics

𝑥 4 +𝑦 4 ∂𝑢 ∂𝑢
3 If 𝑢 = log , show that 𝑥 ∂𝑥 + 𝑦 ∂𝑦 = 3.
𝑥+𝑦

4 If 𝑢 = 𝑥 3 + 𝑦 3 + 𝑧 3 + 3𝑥𝑦𝑧, show that


∂𝑢 ∂𝑢 ∂𝑢
𝑥 +𝑦 +𝑧 = 3𝑢.
∂𝑥 ∂𝑦 ∂𝑧
∂𝑢 ∂𝑢 1
5 If 𝑢 = cos−1 (𝑥 + 𝑦)/(√𝑥 + √𝑦), prove that 𝑥 ∂𝑥 + 𝑦 ∂𝑦 + 2 cot 𝑢 = 0

𝑥 −2 +𝑦 2 ∂𝑢 ∂𝑢
6 If 𝑢 = cos−1 , prove that 𝑥 ∂𝑥 + 𝑦 ∂𝑦 = −cot 𝑢.
𝑥+𝑦

√𝑥−√𝑦 ∂𝑢 ∂𝑢
7 If 𝑢 = sin−1 , show that 𝑥 ∂𝑥 + 𝑦 ∂𝑦 = 0.
√𝑥+√𝑦
𝑥+𝑦 ∂𝑢 ∂𝑢 1
8 If 𝑢 = sin−1 , prove that 𝑥 ∂𝑥 + 𝑦 ∂𝑦 = 2 tan 𝑢.
√𝑥+√ 𝑦

𝑥 2 +𝑦 2 ∂𝑢 ∂𝑢
9 If 𝑢 = sin−1 , show that 𝑥 ∂𝑥 + 𝑦 ∂𝑦 = tan 𝑢.
𝑥+𝑦

𝑥 3 +𝑦 3 ∂𝑧 ∂𝑧
10 If 𝑧 = sec −1 , show that 𝑥 ∂𝑥 + 𝑦 ∂𝑦 = 2cot 𝑧.
𝑥+𝑦

∂2 𝑢 ∂2 𝑢
11 If 𝑢 = sin−1 {(𝑥1/3 + 𝑦1/3 )/(𝑥1/2 + 𝑦1/2 )}, prove that 𝑥 2 ∂𝑥 2 + 2𝑥𝑦 ∂𝑥 ∂𝑦 +
∂2 𝑢 1
𝑦 2 ∂𝑦 2 = 144 tan 𝑢(13 + tan2 𝑢).

𝑥 2 +𝑦 2
12 If 𝑢 = tan−1 , prove that
𝑥+𝑦
∂𝑢 ∂𝑢 1
(i) 𝑥 ∂𝑥 + 𝑦 ∂𝑦 = 2 sin 𝑢.
∂2 𝑢 ∂2 𝑢 ∂2 𝑢
(ii) 𝑥 2 ∂𝑥 2 + 2𝑥𝑦 ∂𝑥 ∂𝑦 + 𝑦 2 ∂𝑦 2 = (1 − 4sin2 𝑢)sin 2𝑢.

𝑥2𝑦2
13 If 𝑢 = 𝑥 2 +𝑦 2, show that

∂2 𝑢 ∂2 𝑢 ∂2 𝑢
𝑥2 + 2𝑥𝑦 + 𝑦 2
= 2𝑢
∂𝑥 2 ∂𝑥 ∂𝑦 ∂𝑦 2
1 Find the maximum and minimum values of
𝑢 = 𝑎2 𝑥 2 + 𝑏 2 𝑦 2 + 𝑐 2 𝑧 2
where 𝑥 2 + 𝑦 2 + 𝑧 2 = 1 and 𝑙𝑥 + 𝑚𝑦 + 𝑛𝑧 = 0.

144 | P a g e

© Department of Distance & Continuing Education, Campus of Open Learning,


School of Open Learning, University of Delhi
Intermediate Mathematical Methods for Economics

2 Show that the maximum and minimum values of 𝑢 = 𝑥 2 + 𝑦 2 + 𝑧 2 subject to the


conditions.
𝑝𝑥 + 𝑞𝑦 + 𝑟𝑧 = 0 and 𝑥 2 /𝑎2 + 𝑦 2 /𝑏 2 + 𝑧 2 /𝑐 2 = 1
are given by the equation.
𝑎2 𝑝2 𝑏2𝑞2 𝑐2𝑟 2
+ + =0
𝑢 − 𝑎2 𝑢 − 𝑏 2 𝑢 − 𝑐 2
3 Find the maximum and minimum values of 𝑢2 when while
𝑢2 = 𝑎2 𝑥 2 + 𝑏 2 𝑦 2 + 𝑐 2 𝑧 2
𝑥 2 + 𝑦 2 + 𝑧 2 = 1 and 𝑙𝑥 + 𝑚𝑦 + 𝑛𝑧 = 0.
4 Find the maximum value of 𝑥 𝑚 𝑦 𝑛 𝑧𝑝 subject to the condition 𝑥 + 𝑦 + 𝑧 = 𝑎.
5 Find the minimum value of 𝑥 2 + 𝑦 2 + 𝑧 2 , given that 𝑎𝑥 + 𝑏𝑦 + 𝑐𝑧 = 𝑝.
6 Find the minimum value of 𝑥 + 𝑦 + 𝑧, subject to the condition
𝑎 𝑏 𝑐
+ + =1
𝑥 𝑦 𝑧
7 Find the maximum or minimum value of 𝑥 𝑝 𝑦 𝑞 𝑧 𝑟 subject to the condition 𝑎𝑥 + 𝑏𝑦 +
𝑐𝑧 = 𝑝 + 𝑞 + 𝑟.
8 Find the maximum or minimum value of 𝑥 𝑝 𝑦 𝑞 𝑧 𝑟 subject to the condition
𝑎 𝑏 𝑐
9 + 𝑦 + 𝑧 = 1 Find the minimum value of 𝑥 4 + 𝑦 4 + 𝑧 4 , where 𝑥𝑦𝑧 = 𝑐 3 .
𝑥

10 Divide a number 𝑎 into three parts such that their product will be maximum.
11 In a plane triangle 𝐴𝐵𝐶, find the maximum value of
𝑢˙ = cos 𝐴cos 𝐵cos 𝐶.
12 Find a plane triangle 𝐴𝐵𝐶 such that 𝑢 = sin𝑚 𝐴sin𝑛 𝐵sin𝑃 𝐶 has maximum value.
13 Show that if the perimeter of a triangle is constant, its area is a maximum when it is
equilateral.
14 Find the triangle of maximum area inscribed in a circle.
15 Prove that the rectangular solid of maximum volume which can be inscribed in a sphere is
a cube.
16 Given 𝑢 = 5𝑥𝑦𝑧/(𝑥 + 2𝑦 + 4𝑧). Find the values of 𝑥, 𝑦, 𝑧 for which 𝑢 is maximum,
subject to the condition 𝑥𝑦𝑧 = 8.
145 | P a g e

© Department of Distance & Continuing Education, Campus of Open Learning,


School of Open Learning, University of Delhi
B.A. (Hons.) Economics

17 Show that the maximum and minimum of the radii vectors of the sections of the surface
𝑥2 𝑦2 𝑧2
(𝑥 2 + 𝑦 2 + 𝑧 2 )2 = + +
𝑎2 𝑏2 𝑐 2
by the plane 𝜆𝑥 + 𝜇𝑦 + 𝜈𝑧 = 0
𝑎2 𝜆2 𝑏2𝜇2 𝑐 2𝑣2
are given by the equation 1−𝑎2𝑟 2 + 1−𝑏2𝑟 2 + 1−𝑐 2𝑟 2 = 0.

18 If two variables 𝑥 and 𝑦 are connected by the relation 𝑎𝑥 2 + 𝑏𝑦 2 = 𝑎𝑏, show that the
maximum and minimum values of the function 𝑢 = 𝑥 2 + 𝑦 2 + 𝑥𝑦 will be the roots of the
equation
4(𝑢 − 𝑎)(𝑢 − 𝑏) = 𝑎𝑏.
19 Find the maximum or minimum value of 𝑥 2 + 𝑦 2 + 𝑧 2 , subject to the conditions𝑙𝑥 +
𝑚𝑦 + 𝑛𝑧 = 1 𝑙 ′ 𝑥 + 𝑚′ 𝑦 + 𝑛′ 𝑧 = 1.
20 Show that the maximum and minimum values of
𝑢 = 𝑎𝑥 2 + 𝑏𝑦 2 + 𝑐𝑧 2 + 2ℎ𝑥𝑦 + 2𝑔𝑧𝑥 + 2𝑓𝑦𝑧
subject to the conditions 𝑙𝑥 + 𝑚𝑦 + 𝑛𝑧 = 0 and 𝑥 2 + 𝑦 2 + 𝑧 2 = 1 are given by the
equation
𝑎−𝑢 ℎ 𝑔 𝑙
ℎ 𝑏−𝑢 𝑓 𝑚
| | = 0.
𝑔 𝑓 𝑐−𝑢 𝑛
𝑙 𝑚 𝑛 0
21 Find the maximum value of 𝑢, when 𝑢 = 𝑥 2 𝑦 3 𝑧 4 and 2𝑥 + 3𝑦 + 4𝑧 = 𝑎.
22 Find the points where 𝑢 = 𝑎𝑥 𝑝 + 𝑏𝑦 𝑞 + 𝑐𝑧 𝑟 has extreme values subject to the condition
𝑥 𝑙 + 𝑦 𝑚 + 𝑧 𝑛 = 𝑘.
23 Determine the maximum value of 𝑂𝑃, 𝑂 being the origin of coordinates where 𝑃
describes the curve
𝑥 2 + 𝑦 2 + 2𝑧 2 = 5, 𝑥 + 2𝑦 + 𝑧 = 5.
24 Prove that if 𝑥 + 𝑦 + 𝑧 = 1, 𝑎𝑦𝑧 + 𝑏𝑧𝑥 + 𝑐𝑥𝑦 has an extreme value equal to
𝑎𝑏𝑐
2𝑏𝑐 + 2𝑐𝑎 + 2𝑎𝑏 − 𝑎2 − 𝑏 2 − 𝑐 2
Prove also that if 𝑎, 𝑏, 𝑐 are all positive and 𝑐 lies between 𝑎 + 𝑏 − 2√(ab) and

146 | P a g e

© Department of Distance & Continuing Education, Campus of Open Learning,


School of Open Learning, University of Delhi
Intermediate Mathematical Methods for Economics

𝑎 + 𝑏 + 2√(ab) this value is true maximum and that if 𝑎, 𝑏, 𝑐 are all negative and 𝑐 lies
between 𝑎 + 𝑏 ± 2, it is true minimum.
25. Find the maxima and minima of 𝑥 2 + 𝑦 2 subject to the condition
𝑎𝑥 2 + 2ℎ𝑥𝑦 + 𝑏𝑦 2 = 1
Answers
1. The maximum or minimum values of 𝑢 are the roots of the equation
𝑅2 𝑚2 𝑛2
+ + = 0.
𝑢 − 𝑎2 𝑢 − 𝑏 2 𝑢 − 𝑐 2
2. The required values are the roots of the equation
𝑙2 𝑚2 𝑛2
+ + = 0.
𝑢2 − 𝑎2 𝑢2 − 𝑏 2 𝑢2 − 𝑐 2
3. 𝑝2 /(𝑎2 + 𝑏 2 + 𝑐 2 ).
4. A minimum when
𝑥 𝑦 𝑧
= = = √(a) + √(b) + √(c)
√(a) √(b) √(c)

and the minimum value = (√(a) + √(b) + √(c))2 .


5. A maximum value is (𝑝/𝑎)𝑝 (𝑞/𝑏)𝑞 (𝑟/𝑐)𝑟 .
6. A minimum when
𝑝𝑥 𝑞𝑦 𝑟𝑧
= = = 𝑝 + 𝑞 + 𝑟.
𝑎 𝑏 𝑐
7. A minimum when 𝑥 = 𝑦 = 𝑧 = 𝑐 and the minimum value = 3𝑐 4 .
8. 𝑎/3, 𝑎/3, 𝑎/3.
𝜋 1
9. 𝑢 is maximum when 𝐴 = 𝐵 = 𝐶 = and the maximum value of 𝑢 = 8.
3

10. A maximum when


tan 𝐴 tan 𝐵 tan 𝐶
= = .
𝑚 𝑛 𝑝
11. An equilateral triangle.
12. 𝑥 = 4, 𝑦 = 2, 𝑧 = 1.
147 | P a g e

© Department of Distance & Continuing Education, Campus of Open Learning,


School of Open Learning, University of Delhi
B.A. (Hons.) Economics

13. The minimum value Σ(𝑙 − 𝑙 ′ )2 /Σ(𝑚𝑛′ − 𝑚′ 𝑛)2.


14. (𝑎/9)9 .
15. The values of 𝑥, 𝑦, 𝑧 are given by
𝑥 𝑝−𝑙 𝑦 𝑞−𝑚 𝑧 𝑟−𝑛
= = .
𝑙/𝑝𝑎 𝑚/𝑞𝑏 𝑛/𝑟𝑐

16. √(5)

(OBJECTIVE QUESTIONS)
Fill In the Blanks.
Fill in the blanks "......" so that the following statements are complete and correct.
1 Let 𝑓(𝑥, 𝑦, 𝑧) be a function of three independent variables 𝑥, 𝑦 and 𝑧. The necessary
conditions for the existence of a maximum or a minimum of 𝑓(𝑥, 𝑦, 𝑧) at 𝑥 = 𝑎, 𝑦 = 𝑏 and
𝑧 = 𝑐 are.
∂𝑓 ∂𝑓 ∂𝑓
= 0, = 0 and = ⋯ at 𝑥 = 𝑎, 𝑦 = 𝑏, 𝑧 = 𝑐.
∂𝑥 ∂𝑦 ∂𝑧
2 For a maximum or a minimum of 𝑢 = 𝑥 2 + 𝑦 2 + 𝑧 2 + 𝑥 − 2𝑧 − 𝑥𝑦, we must have 2𝑥 −
𝑦 + 1 = 0, …, and 2𝑧 − 2 = 0.
Multiple Choice Questions
Indicate the correct answer for each question by writing the corresponding letter from (a), (b),
(c) and (d).
3 The function 𝑢 = sin 𝑥sin 𝑦sin 𝑧, where 𝑥, 𝑦, 𝑧 are the angles of a triangle, is stationary
at the point
𝜋 𝜋 𝜋
(a) 𝑥 = 2 , 𝑦 = 4 , 𝑧 = 4
𝜋
(b) 𝑥 = 𝑦 = 𝑧 = 3
𝜋 𝜋 𝜋
(c) 𝑥 = 4 , 𝑦 = 2 , 𝑧 = 4
𝜋 𝜋
(d) 𝑥 = 0, 𝑦 = 2 , 𝑧 = 2

4. The maximum and minimum values of 𝑢 = 𝑎2 𝑥 2 + 𝑏 2 𝑦 2 + 𝑐 2 𝑧 2 where 𝑥 2 + 𝑦 2 + 𝑧 2 =


1 and
𝐿𝑥 + 𝑚𝑦 + 𝑛𝑧 = 0 are the roots of the equation

148 | P a g e

© Department of Distance & Continuing Education, Campus of Open Learning,


School of Open Learning, University of Delhi
Intermediate Mathematical Methods for Economics

𝑙2 𝑚2 𝑛2
(a) 𝑢−𝑎2 + 𝑢−𝑏2 + 𝑢−𝑐 2 = 0
𝑙 𝑚 𝑛
(b) 𝑢−𝑎2 + 𝑢−𝑏2 + 𝑢−𝑐 2 = 0

𝑙2 𝑚2 𝑛2
(c) 𝑢−𝑎 + 𝑢−𝑏 + 𝑢−𝑐 = 0
1 𝑚 𝑛
(d) 𝑢−𝑎 + 𝑢−𝑏 + 𝑢−𝑐 = 0

True or False.
Write ' 𝑻 ' for the and ' 𝑭 ' for false statement.
5. The function 𝑢 = 𝑥 2 + 𝑦 2 + 𝑧 2 + 𝑥 − 2𝑧 − 𝑥𝑦 has a maximum at the point 𝑥 =
2 1
− 3 , 𝑦 = − 3 , 𝑧 = 1.

6. The volume of the greatest rectangular parallelopiped that can be inscribed in the
ellipsoid
𝑥2 𝑦2 𝑧2 8𝑎𝑏𝑐
2
+ 2 + 2 = 1 is
𝑎 𝑏 𝑐 3√3
7. The maximum or minimum values of 𝑢 = 𝑥 2 + 𝑦 2 + 𝑧 2 subject to the conditions
𝑎𝑥 2 + 𝑏𝑦 2 + 𝑐𝑧 2 = 1 and 𝑙𝑥 + 𝑚𝑦 + 𝑛𝑧 = 0
𝑙2 𝑚2 𝑛2
are the roots of the equation 𝑎𝑢−1 + 𝑏𝑢−1 + 𝑐𝑢−1 = 0.
𝑎 𝑏 𝑐
8. The minimum value of 𝑥 + 𝑦 + 𝑧, subject to the condition 𝑥 + 𝑦 + 𝑧 = 1, is (𝑎 + 𝑏 +
𝑐)2
Answer
1 0.
2 −𝑥 + 2𝑦 = 0.
3 (a).
4 (b).
5 𝐹.
6 𝑇.
7 𝑇.
8 𝐹.

149 | P a g e

© Department of Distance & Continuing Education, Campus of Open Learning,


School of Open Learning, University of Delhi
B.A. (Hons.) Economics

9.4 SUMMARY

The main points which we have covered in these lessons are what is maxima and minima of
function of one and more variable which is highly useful for economics and Statistics.

9.5 GLOSSARY

Motivation: These Problems are very useful in real life, and we can use it in data science,
economics as well as social science.
Attention: Think how Maxima and Minima are useful in real world problems.

9.6 REFERENCES

• Sydsaeter, K., Hammond, P. (2002). Mathematics for economics analysis. Pearson


Education.

• Hoy, M., Livernois, J., McKenna, C., Rees, R., Stengos, T, (2001). Mathematics for
Economics, Prentice-Hall India.

9.7 SUGGESTED READINGS

• A.R Vasishtha, Anurag Sharma, Dr. Vipin Vasishtha, Neenu Agarwal, Dr A.K
Vasishtha, Advanced Calculus, Krishna Publication, 5th Edition.
• A.R Vasishtha, Anurag Sharma, Dr. Vipin Vasishtha, Anil Kumar, Dr A.K Vasishtha,
Analysis, Krishna Publication, 3rd Edition.

150 | P a g e

© Department of Distance & Continuing Education, Campus of Open Learning,


School of Open Learning, University of Delhi
Intermediate Mathematical Methods for Economics

LESSON 10
CONVEX SETS AND THEIR PROPERTIES

STRUCTURE
10.1 Learning Objectives
10.2 Introduction
10.3 Convex Sets and Their Properties
10.3.1 Vector Space
10.3.2 Convex Combination
10.3.3 Convex Set
10.3.4 Extreme Points of a Convex Set
10.3.5 Convex Hull and Convex Polyhedron
10.4 Summary
10.5 Glossary
10.6 References
10.7 Suggested Readings
10.1 LEARNING OBJECTIVES

one of the main objectives of convex set and convex function is to understand the nature of
function as well as nature of set in n dimension. It is widely use in linear programming problem
and economics, management, and statistics.
10.2 INTRODUCTION

1 Point Sets. Point sets are sets whose elements are points or vectors in 𝐸 𝑛 or 𝑅 𝑛 (n-
dimensional Euclidean space).
For example.
(i) A linear equation n two variables 𝑥1 , 𝑥2 , i.e., 𝑎1 𝑥1 + 𝑎2 𝑥2 = 𝑏 represents a line in
two dimensions. This line may be considered as a set of those points (𝑥1 , 𝑥2 ) which
satisfy 𝑎1 𝑥1 + 𝑎2 𝑥2 = 𝑏. This set of points can be written as
𝑆1 = {(𝑥1 , 𝑥2 ): 𝑎1 𝑥1 + 𝑎2 𝑥2 = 𝑏}
(ii) Consider the set of points lying inside a circle of unit radius with centre at the
origin, in two-dimensional space (𝐸 2 ). Obviously, the points (𝑥1 , 𝑥2 ) can be
151 | P a g e

© Department of Distance & Continuing Education, Campus of Open Learning,


School of Open Learning, University of Delhi
B.A. (Hons.) Economics

shown.
This set of points can be written as
𝑆2 = {(𝑥1 , 𝑥2 ): 𝑥12 + 𝑥22 < 1}
These sets may contain either a finite or infinite number of elements. Usually, however, they
will contain an infinite number of elements. Further, we shall always assume that there is at
least one element in a set unless otherwise stated.
2 Hypersphere. A hypersphere in 𝐸 𝑛 with centre at a and radius 𝜀 > 0 is defined to be
the set of points.
𝑋 = {𝑥: |𝑥 − 𝑎| = 𝜀}
i.e., the equation of a hypersphere in 𝐸 𝑛 is
(𝑥1 − 𝑎1 )2 + (𝑥2 − 𝑎2 )2 + ⋯ + (𝑥𝑛 − 𝑎𝑛 )2 = 𝜀 2
where a = (𝑎1, 𝑎2 , … , 𝑎𝑛 ), 𝐱 = (𝑥1 , 𝑥2 , … , 𝑥𝑛 )
which represents a circle in 𝐸 2 and sphere in 𝐸 3 .
The set of points inside the hypersphere is the set.
𝑋 = {𝑥: |𝑥 − 𝑎| < 𝜀}.
3 An e-neighbourhood. An 𝜀-neighbourhood about the point a is defined as the set of
points lying inside the hypersphere with centre at a and radius 𝜀 > 0. i.e., the 𝜀-
neighbourhood about the point a is the set of points.
𝑋 = {𝐱: |𝐱 − 𝐚| < 𝜀}
4 An Interior Point. A point a is an interior point of the set 𝑆 if there exists an E-
neighbourhood about a which contains only points of the set 𝑆.
An interior point of 𝑆 must be an element of 𝑆.
5 Boundary Point. A point a is a boundary point of the set 𝑆 if every E-neighbourhood
about a ( 𝜀 > 0 may be, however, small) contains points which are in the set and the
points which are not in the set.
A boundary point of 𝑆 does not have to be an element of 𝑆.
6 An Open Set. A set 𝑆 is said to be an open set if it contains only the interior points.
7 A Closed Set. A set 𝑆 is said to be a closed set if it contains all its boundary points.
8 Lines. In 𝐸 𝑛 , the line through the two points x1 and x2 , x1 ≠ x2 is defined to be the
set of points.

152 | P a g e

© Department of Distance & Continuing Education, Campus of Open Learning,


School of Open Learning, University of Delhi
Intermediate Mathematical Methods for Economics

𝑋 = {𝑥: 𝑥 = 𝜆𝑥1 + (1 − 𝜆)𝑥2 , for all real 𝜆}


9 Line Segments. In 𝐸 𝑛 , the line segment joining two points 𝑥1 and 𝑥2 is defined to be
the set of points.
𝑋 = {𝐱: 𝐱 = 𝜆𝐱1 + (1 − 𝜆)𝐱 2 , 0 ≤ 𝜆 ≤ 1}
Note that the restriction 0 ≤ 𝜆 ≤ 1 restricts the point 𝑥 to lie within the line joining the points
𝑥1 and 𝑥2 . Line segment of 𝑥1 , 𝑥2 is also denoted by [𝑥1 : 𝑥2 ].
10 Hyperplane. A hyperplane is defined as the set of points satisfying 𝑐1 𝑥1 + 𝑐2 𝑥2 +
⋯ + 𝑐𝑛 𝑥𝑛 = 𝑧 (not all 𝑐𝑖 = 0)
or
𝐜𝐱 = 𝑧
for prescribed values of 𝑐1 , 𝑐2 , … , 𝑐𝑛 and 𝑧.
For optimum value of 𝑧 this hyperplane is called optimal hyperplane.
𝐜
The vector 𝐜 is called a vector normal to the hyperplane and ± |𝐜| are called unit normals.
It can be easily seen that hyperplanes are closed sets.
A hyperplane divides the whole space 𝐸 𝑛 into three mutually disjoint sets given by
𝑋1 = {𝐱: 𝐜𝐱 > 𝑧}
𝑋2 = {𝐱: 𝐜𝐱 = 𝑧}
𝑋3 = {𝐱: 𝐜𝐱 < 𝑧}.
The sets 𝑋1 and 𝑋3 are called open half spaces.
The sets {𝐱: 𝐜𝐱 ≤ 𝑧} and {𝐱: 𝐜𝐱 ≥ 𝑧} are called closed half spaces.
Note. The objective function of a L.P.P. represents a hyperplane and each constraint (sign ≤
, ≥) is a closed half space produced by the hyperplane given by the constraint by taking ⇔ sign
in place of ≤ and ≥.
Paralle1 Hyperplanes. Two hyperplanes 𝐜1 𝐱 = 𝑧1 and 𝐜2 𝐱 = 𝑧2 are said to be parallel if
they have the same unit normals i.e., if 𝐜1 = 𝜆𝐜2 for some 𝜆, 𝜆 being non-zero.

10.3 CONVEX SETS AND THEIR PROPERTIES

10.3.1 Vector Space:


Now we shall study another important algebraic structure known is vector space. Before giving
the definition of a vector space we shall make a distinction between internal and external
compositions.

153 | P a g e

© Department of Distance & Continuing Education, Campus of Open Learning,


School of Open Learning, University of Delhi
B.A. (Hons.) Economics

Let 𝐴 be any set. If 𝑎 ∗ 𝑏 ∈ 𝐴 ∀𝑎, 𝑏 ∈ 𝐴, and 𝑎 ∗ 𝑏 is unique then + is said to be an internal


composition in the set 𝐴. Here 𝑎 and 𝑏 are both elements of the set 𝐴.
Let 𝑉 and 𝐹 be any two sets. If 𝑎 ∘ 𝛼 ∈ 𝑉 for all 𝑎 ∈ 𝐹 and for all 𝛼 ∈ 𝑉 and 𝑎 ∘ 𝛼 is unique,
then 𝑜 is said to be an external composition in 𝑉 over 𝐹. Here 𝑎 is an element of the set 𝐹 and
𝛼 is an element of the set 𝑉 and the resulting element 𝑎 ∘ 𝛼 is an element of the set 𝑉.
Vector Space
Let (𝐹, ++) be a field. The elements of 𝐹 will be called scalars. Let 𝑉 be a non-empry ser
whose elements will be called vectors. Then 𝑉 is a vector space over the field. if
1 There is defined an internal composition in 𝑉 called addition of vectors and denoted by
'+'. Also, for this composition 𝑉 is an abelian group.
2 There is an external composition in 𝑉 over 𝐹 called scalar multiplication and denoted
multiplicatively ie, 𝑎𝛼 ∈ 𝑉 for all 𝑎 ∈ 𝐹 and for all 𝛼 ∈ 𝑉. In other words, 𝑉 is cloged
with respect to scalar multiplication.
3 The two compositions i.e., scalar multiplication and addition of vectors satisfy the
following postulates :
(i) 𝑎(𝛼 + 𝛽) = 𝑎𝛼 + 𝑎𝛽∀𝑎 ∈ 𝐹 and ∀𝛼, 𝛽 ∈ 𝑉.
(ii) (𝑎 + 𝑏)𝛼 = 𝑎𝛼 + 𝑏𝛼∀𝑎, 𝑏 ∈ 𝐹 and ∀𝛼 ∈ 𝑉.
(iii) (𝑎𝑏)𝛼 = 𝑎(𝑏𝛼)∀𝑎, 𝑏 ∈ 𝐹 and ∀𝛼 ∈ 𝑉.
(in) 1𝛼 = 𝛼∀𝛼 ∈ 𝑉 and 1 is the unity element of the field F
{ When 𝑉 is a vector space over the field 𝐹, we shall
4 When 𝑉 is a vector space over the field 𝐹, we shall say that 𝑉(𝐹) is a vector space. If
theld 𝐹 is unge. If 𝐹 is the field 𝐑 of real numbers, 𝑉 is called a real vector space.
Similarly if 𝐹 is 𝑄 of 𝐹 is 𝐶, we speak of rational vector spaces or complex vector
spaces.
In the above definition of a vector space 𝑉 over the field 𝐹, we have denoted the addition of
vectors by the symbol ' + '. This symbol also denotes the addition composition of the field 𝐹,
i.e., addition of scalars. There should be no confusion about the two compositions though we
have used the same symbol to denote each of them 𝛼, 𝛽 ∈ 𝑉, then 𝛼 + 𝛽 represents addition in
𝑉 i.e., addition of vectors. If 𝑎, 𝑏 ∈ 𝐹 then
𝑎 + 𝑏 represents addition of sealars L.e., addition in the field 𝐹. Similarly, there should be no
confusion in multiplication of scalars i.e., multiplication of the elements of 𝐹 and in scalar
multiplication i.e., multiplication of an element of 𝑉 by an element of 𝐹. If 𝑎, 𝑏 ∈ 𝐹, then 𝑎𝑏
represents multiplication of 𝐹 and 𝑎𝑏 ∈ 𝐹, If 𝑎 ∈ 𝐹, and 𝛼 ∈ 𝑉, then 𝑎𝛼 represents scalar
multiplication and 𝑎𝛼 ∈ 𝑉. Since 1 ∈ 𝐹 and 𝛼 ∈ 𝑉, therefore 1𝛼 represents scalar
multiplication. Again 𝑎𝛼 ∈ 𝑉, 𝑎𝛽 ∈ 𝑉, therefore 𝑎𝛼 + 𝑎𝛽 represents addition of vectors and
154 | P a g e

© Department of Distance & Continuing Education, Campus of Open Learning,


School of Open Learning, University of Delhi
Intermediate Mathematical Methods for Economics

thus 𝑎𝛼 + 𝑎𝛽 is an element of 𝑉. Further 𝑎 ∈ 𝐹 and 𝛼 + 𝛽 ∈ 𝑉, therefore 𝑎(𝛼 + 𝛽) represents


scalar multiplication and we have 𝑎(𝛼 + 𝛽) ∈ 𝑉
Note 1. For 𝑉 to be an abelian group with respect to addition of vectors, we must have the
following conditions satisfied
(i) 𝛼 + 𝛽 ∈ 𝑉 for all 𝛼, 𝛽 ∈ 𝑉.
(ii) 𝛼 + 𝛽 = 𝛽 + 𝛼 for all 𝛼, 𝛽 ∈ 𝑉.
(iii) 𝛼 + (𝛽 + 𝛾) = (𝛼 + 𝛽) + 𝛾 for all 𝛼, 𝛽, 𝛾 ∈ 𝑉.
(iv) There exists an element 0 ∈ 𝑉 such that 0 + 𝛼 = 𝛼∀𝛼 ∈ 𝑉.
The element 𝜃 ∈ 𝑉 will be called the zero vector. It is the additive identity in 𝑉.
(v) To every vector 𝛼 ∈ 𝑉 there exists a vector −𝛼 ∈ 𝑉 such that −𝛼 + 𝛼 = 0. Thus, each
vector should possess additive inverse. The vector −𝛼 is called the negative of the vector
𝛼.
Note 2. Since (𝑉, +) is an abelian group therefore all the properties of an abelian group will
hold in 𝑉. A few of them are as follows:
(i) 𝛼+𝛽 =𝛼+𝛾 ⇒𝛽 =𝛾
(left cancellation law)
(ii) 𝛽+𝛼 =𝛾+𝛼 ⇒𝛽 =𝛾
(iii) 𝛼 + 𝛽 = 0 ⇒ 𝛼 = −𝛽 and 𝛽 = −𝛼.
(iv) −(𝛼 + 𝛽) = −𝛼 − 𝛽 where by 𝛼 − 𝛽 we mean 𝛼 + (−𝛽).
(v) The additive identity 0 will be unique.
(vi) The additive inverse of each vector will be unique.
(vii) If 𝛼 + 𝛽 = 𝛾, then 𝛼 + 𝛽 − 𝛾 = 0.
Note 3. There should also be no confusion about the use of the word vector. Here by vector,
we do not mean the vector quantity which we have defined in vector algebra as a directed line
segment. Here we shall call the elements of the set 𝑉 as vectors.
Note 4. In a vector space we shall be dealing with two types of zero elements, one is the zero
vector and the other is the zero element of the field 𝐹 i.e., the 0 scalar. To distinguish between
the two, we shall use the zero letter in bold type to represent the zero vector. However, the
students may use the same symbol 0 to denote the zero vector as well as the zero scalar. There
will be no confusion in this use. The use of 0 will itself tell whether it stands for zero vector or
for zero scalar.

155 | P a g e

© Department of Distance & Continuing Education, Campus of Open Learning,


School of Open Learning, University of Delhi
B.A. (Hons.) Economics

Note 5. We shall usually use the lower-case Greek letters 𝛼, 𝛽, 𝛾 ete. to denote vectors i.e., the
elements of 𝑉 and the lower-case Latin letters, 𝑎, 𝑏, 𝑐 etc. to denote the scalars i.e., the elements
of the field 𝐹.
Ex. 1. A field 𝐾 can be regarded as a vector space over any subfield 𝐹 of 𝐾.
Sol. Here 𝐾 is the set of vectors. Addition of veetors is the addition composition in the field
𝐾. Since 𝐾 is a field, therefore (𝐾, +) is an abelian group. Further the
elements of the subfield 𝐹 constitute the set of senlars. The composition of scalar multiplication
is the multiplication composition in the field 𝐾, 𝐾 is a field, therefore 𝑎𝛼 ∈ 𝐾∀𝑎 ∈ 𝐹 and ∀𝛼 ∈
𝐾 because both 𝑎 and 𝛼 are elements of 𝐾. If 𝐼 is the unity element of 𝐾, then 1 is also the
unity element of the subfield 𝐹. We make the following observations.
(i) 𝑎(𝛼 + 𝛽) = 𝑎𝛼 + 𝑎𝛽 ∀𝑎 ∈ 𝐹 and ∀𝛼, 𝛽 ∈ 𝐾. This result follows from the left distributive
law in 𝐾.
(ii) (𝑎 + 𝑏)𝛼 = 𝑎𝛼 + 𝑏𝛼∀𝑎, 𝑏 ∈ 𝐹 and ∀𝛼 ∈ 𝐾. This result is a consequence of the right
distributive law in 𝐾.
(iii) (𝑎𝑏)𝛼 = 𝑎(𝑏𝛼)∀𝑎, 𝑏 ∈ 𝐹 and ∀𝛼 ∈ 𝐾. This result is a consequence of associativity of
multiplication in 𝐾.
(iv) 1𝛼 = 𝛼∀𝛼 ∈ 𝐾 and 1 is the unity element of the subfield 𝐹. Since 1 is also the unity
element of the field 𝐾, therefore 1𝛼 = 𝛼 ∀∈ 𝐾.
Hence 𝐾(𝐹) is a vector space.
Note 1. If 𝐹 is any field, then 𝐹 itself is a vector space over the field 𝐹.
Note 2. If C is the field of complex numbers and R is the field of real numbers, then 𝐂 is a
vector space over 𝐑 because 𝐑 is a subfield of 𝐂. But 𝐑 is not a vector space over 𝐂. Here 𝐑 is
not closed with respect to scalar multiplication. For example, 2 ∈ 𝐑 and 3 + 4𝑖 ∈ 𝐂 and (3 +
4𝑖)2 ∉ 𝐑.
Ex. 2. Let 𝑉 be the set of all pairs (𝑥, 𝑦) of real numbers and let 𝐹 be the field of real numbers.
Define
(𝑥, 𝑦) + (𝑥1 , 𝑦1 ) = (3𝑦 + 3𝑦1 , −𝑥 − 𝑥1 )
𝑐(𝑥, 𝑦) = (3𝑐𝑦2 − 𝑐𝑥).
Verify that 𝑉, with these operations, is not a vector space over the field of real numbers.
Sol. Take 𝛼 = (1,2), 𝛽 = (2,3), 𝛾 = (1,3).
Then (𝛼 + 𝛽) + 𝛾 = {(1,2) + (2,3)} + (1,3)

156 | P a g e

© Department of Distance & Continuing Education, Campus of Open Learning,


School of Open Learning, University of Delhi
Intermediate Mathematical Methods for Economics

= (15, −3) + (1,3) = (0, −16)


and
𝛼 + (𝛽 + 𝛾) = (1,2) + {(2,3) + (1,3)}
= (1,2) + (18, −3) = (−3, −19)
Thus, we conclude that in general (𝛼 + 𝛽) + 𝛾 ≠ 𝛼 + (𝛽 + 𝛾). Hence 𝑉 is not a vector space
over the field of real numbers.
10.3.2 Convex Combination:

A convex combination of a finite number of points x1 , x2 , … , xn is defined as a point.


x = 𝜆1 x1 + 𝜆2 x2 + ⋯ + 𝜆𝑛 xn
where 𝜆𝑖 is real and ≥ 0, ∀𝑖 and ∑𝑛𝑖=1 𝜆𝑖 = 1.
The convex linear combination of two points x1 and x2 is given by
𝐱 = 𝜆1 𝑥1 + 𝜆2 𝑥2 , s.t. 𝜆1, 𝜆2 ≥ 0, 𝜆1 + 𝜆2 = 1.
It can also be written as
𝐱 = 𝜆x1 + (1 − 𝜆)x2 , 0 ≤ 𝜆 ≤ 1.
This shows that the line segment of the two points 𝑥1 and 𝑥2 is nothing but the set of all possible
convex combinations of the two points 𝑥1 and 𝑥2 .
10.3.3 Convex Set:
A set of points is said to be convex if for any two points in the set, the line segment joining
these two points is also in the set. In other words, a set is convex if the convex combination of
any wo points in the set, is also in the set.
It can be easily seen that the convex combination of any number of points in the convex set
also belongs to the set.
By convention a set of one point is always convex.

157 | P a g e

© Department of Distance & Continuing Education, Campus of Open Learning,


School of Open Learning, University of Delhi
B.A. (Hons.) Economics

10.3.4 Extreme Point of a Convex Set:

A point x in a convex set 𝐶 is called an extreme point if x cannot be expressed as a convex


combination of any two distinct points x1 and x2 in 𝐶.
In other words, a point x in a convex set 𝐶 is an extreme point of 𝐶 if it does not lie on the line
segment of any two points, different from 𝐱 in the set.
Mathematically, a point 𝐱 is an extreme point of a convex set if there do not exist other points
𝑥1 , 𝑥2 (𝑥1 ≠ 𝑥2 ) in the set such that
x = 𝜆x1 + (1 − 𝜆)x2 , 0 < 𝜆 < 1.
For example : The set 𝐶 = {(𝑥1 , 𝑥2 ): 𝑥1 2 + 𝑥2 2 ≤ 1} is convex. Every point on the
circumference is an extreme point. Thus, a convex set may also have infinite number of extreme
points.
The polygons which are convex sets have the extreme points as their vertices.
Obviously, an extreme point is a boundary point of the set.
It is important to note that all boundary points of a convex set are not necessarily extreme
points.
A point of 𝐶 which is not an extreme point, is referred as an internal point of C.
10.3.5 Convex Hull and Convex Polyhedron:

The convex hull 𝑪(𝑿) of any given set of points 𝑋 is the set of all convex combinations of sets
of points from 𝑋.
In other words, the intersection of all convex sets, containing 𝑋 ⊂ 𝐸 𝑛 , is called the convex hull
of 𝑋 and is denoted by < X >. Thus, the convex hull of a set 𝑋 ⊂ 𝐸 𝑛 , is the smallest convex
set containing 𝑋.
Ex. If 𝑋 is the set of eight vertices of a cube, then the convex hull 𝐶(𝑋) is the whole cube.
The set of all convex combinations of finite number of points is called the convex polyhedron
generated by these points.
Alternatively, if the set 𝑋 consists of a finite number of points, the convex hull of 𝑋 is called a
convex polyhedron with vertices at these points.
Example. The set of the area of a triangle is a convex polyhedron of the set of its vertices.

158 | P a g e

© Department of Distance & Continuing Education, Campus of Open Learning,


School of Open Learning, University of Delhi
Intermediate Mathematical Methods for Economics

SOLVED EXAMPLES
Ex. 1. A hyperplane is given by equation 3𝑥1 + 2𝑥2 + 4𝑥3 + 7𝑥4 = 8. Find in which half
spaces do the following poines (−6,1,7,2) and (1,2, −4,1) lie.
Sol. The given equation of the hyperplane is
3𝑥1 + 2𝑥2 + 4𝑥3 + 7𝑥4 = 8
Substituting (−6,1,7,2) in the L.H.S., we get
L.H.S, = 3(−6) + 2.1 + 4.7 + 7,2 = 26 > 8 = R.H.S.
∴ the point (−6,1,7,2) lies in the opea balf space,
3𝑥1 + 2𝑥2 + 4𝑥3 + 7𝑥4 > 8
Again substituting (T, 2, −4,1) in the L.H.S, we get
L.H.S. = 3 ⋅ 1 + 2,2 + 4(−4) + 7.1 = −2 < 8 = R.H.S.
∴ the poiat (1,2, −4,1) lies in the open half space
3𝑥1 + 2𝑥2 + 4𝑥3 + 7𝑥4 < 8
Ex. 2. Sketch the caner polygon panned by the following points in a two dimensional Euclidean
space. Which of these points are vertices? Express the other as the convex linear combination
of the vertices.
1 1
(0,0), (0,1), (1,0), ( , )
2 4
Sol. The convex combinations of the points (0,0), (1,0); (0,0), (0,1) and (1,0), (0,1) give the
lise segments 𝑂𝐴, 𝑂𝐵 and 𝐴𝐵 respectively. Thus, the convex combination of points (0,0), (1,0)
and (0,1) is the interiur of the triangle 𝑂𝐴𝐵.
1 1
The points 𝑂(0,0), 𝐴(1,0) and 𝐵(0,1) are the vertices and the poiat 𝐶 (2 , 4) is the interior point
of the conves polygon spanticd by the given poists.
𝟏 𝟏
To express (𝟐 , 𝟒) as the linear combination of (𝟎, 𝟎), (𝟎, 𝟏), (𝟏, 𝟎).
𝟏 𝟏
Let(𝟐 , 𝟒) = 𝜆1 (0,0) + 𝜆2 (0,1) + 𝜆3 (1,0)

where 𝜆1 + 𝜆2 + 𝜆3 = 1 and 𝜆1 , 𝜆2 , 𝜆3 ≥ 0

159 | P a g e

© Department of Distance & Continuing Education, Campus of Open Learning,


School of Open Learning, University of Delhi
B.A. (Hons.) Economics

1 1 1 1
∴ ( , ) = (𝜆3 , 𝜆2 ) which gives 𝜆2 = + 𝜆3 =
2 4 4 2
1 1 1
∴ 𝜆1 = 1 − 𝜆2 − 𝜆3 = 1 − = =
4 2 4
1 1 1 1 1
Thus (2 , 4) = 4 (0,0) + 4 (0,1) + 2 (1,0).
Ex. 3. Show that 𝐶 = ((𝑥1 , 𝑥2 ): 2𝑥1 + 3𝑥2 = 7) ⊂ 𝑅 2 is a convex tet.

Sol. Let u, v ∈ 𝐶i , where u = (𝑤1 , w2 ), v = (𝑣1 , 𝑣2 ).


Thes 2𝑤1 + 3𝑤2 = 7 and 2𝑣1 + 3𝑣2 = 7.
If 𝑤 = (𝑤1 , 𝑤2 ) is a point on the line segment joining the points 𝑢 and 𝑣, then
w = 𝜆u + (1 − 𝜆)𝑣, , 0 ≤ 𝜆 ≤ 1
⇒ (𝑤1 , 𝑤2 ) = 𝜆(𝑢1 , 𝑖2 ) + (1 − 𝜆)(𝑣1 , 𝑣2 )
= (𝜆𝑢1 + (1 − 𝜆)𝑣1 , 𝜆𝑢2 + (1 − 𝜆)𝑣2 )
⇒ 𝑤1 = 𝜆𝑤1 + (1 − 𝜆)𝑣1 and 𝑤2 = 𝜆𝑎2 + (1 − 𝜆)𝑣2 .
Now 2𝑚1 + 3𝑚2 = 2[𝜆𝑚1 + (1 − 𝜆)𝑣1 ] + 3[𝜆𝑤2 + (1 − 𝜆)𝑣2 ]
= 𝜆[2𝑢1 + 3𝑢2 ] + (1 − 𝜆)[2𝑣1 + 3𝑣2 ]
= 𝜆. 7 + (1 − 𝜆),7 = 7, using (1)
∴ 𝑤 = (𝑤1 , 𝑤2 ) ∈ 𝐶.

Hence the set 𝐶 is a convex set.

Ex. 4. Show that 𝑆 = ((𝑥1 , 𝑥2 , 𝑥3 ): 2𝑥1 − 𝑥2 + 𝑥3 ≤ 4) ⊂ 𝑅 3 , is a comver set.

Sol. Let 𝑥 = (𝑥1 , 𝑥2 , 𝑥3 ) and 𝑦 = (𝑦1 , 𝑦2 , 𝑦3 ) be any two points of 5. Then we


2𝑥1 − 𝑥2 + 𝑥3 ≤ 4 and 2𝑦1 − 𝑦2 + 𝑦3 ≤ 4
If 𝑤 = (𝑤1 , 𝑤2 , 𝑤3 ) is a point on the line segment joining the points 𝑥 and 𝑦, then
𝑤 = 𝜆𝑥 + (1 − 𝜆)𝑦, 0 ≤ 𝜆 ≤ 1 (𝑤1 , 𝑤2 , 𝑤3 ) = 𝜆(𝑥1 , 𝑥2 , 𝑥3 ) + (1 − 𝜆)(𝑦1 , )2 , 𝑦3 )
Now 𝑤1 = 𝜆𝑥1 + (1 − 𝜆)𝑦1 , 𝑤2 = 𝜆𝑥2 + (1 − 𝜆)𝑦2 , 𝑤1 = 𝜆𝑥3 + (1 − 𝜆)𝑦3 .
2𝑤1 − 𝑤2 + 𝑤3 = 𝜆(2𝑥1 − 𝑥2 + 𝑥3 ) + (1 − 𝜆)(2𝑦1 − 𝑦2 + 𝑦3 )
≤ 4𝜆 + 4(1 − 𝜆). using (1)
=4
∴ 𝑤 = (𝑤1 , 𝑤2 , 𝑤3 ) ∈ 𝑆.
Hence the set 𝑆 is a convex set.
Ex. 5. Examine convexity to the set
(i) 𝐴 = {(𝑥1 , 𝑥2 ) ∈ 𝑅 2 : 4𝑥1 + 3𝑥2 ≤ 6, 𝑥1 + 𝑥2 ≥ 1}
160 | P a g e

© Department of Distance & Continuing Education, Campus of Open Learning,


School of Open Learning, University of Delhi
Intermediate Mathematical Methods for Economics

(iii) 𝐴 = {(𝑥1 , 𝑥2 ): 𝑥1 ≥ 2, 𝑥1 ≤ 3}.


Sol. (i) Given, 𝐴 = {(𝑥1 , 𝑥2 ) ∈ 𝑅 2 : 4𝑥1 + 3𝑥2 ≤ 6, 𝑥1 + 𝑥2 ≥ 1}
If 𝑢 = (𝑥1 , 𝑥2 ) ∈ 𝐴 then 4𝑥1 + 3𝑥2 ≤ 6 and 𝑥1 + 𝑥2 ≥ 1
and if v = (𝑦1 , 𝑦2 ) ∈ 𝐴 then 4𝑦1 + 3𝑦2 ≤ 6 and 𝑦1 + 𝑦2 ≥ 1.
If 𝑤 = (𝑧1 , 𝑧2 ) is a point on the line segment joining points 𝑢 and 𝑣s then
𝑤 = 𝜆𝐮 + (1 − 𝜆)𝑣 0 ≤ 𝜆 ≤ 1
⇒ (𝑧1 , 𝑧2 ) = 𝜆(𝑥1 , 𝑥2 ) + (1 − 𝜆)(𝑦1 , 𝑦2 )
= (𝜆𝑥1 + (1 − 𝜆)𝑦1 , 𝜆𝑥2 + (1 − 𝜆)𝑦2 )
⇒ 𝑧1 = 𝜆𝑥1 + (1 − 𝜆)𝑦1 and 𝑧2 = 𝜆𝑥2 + (1 − 𝜆)𝑦2 .
Now 4𝑧1 + 3𝑧2 = 𝜆(4𝑥1 + 3𝑥2 ) + (1 − 𝜆)(4𝑦1 + 3𝑦2 ) ≤ 𝜆. 6 + (1 − 𝜆) ⋅ 6
[ 0 ≤ 𝜆 ≤ 1,0 ≤ 1 − 𝜆 ≤ 1,4𝑥1 + 3𝑥2 ≤ 6 and 4𝑦1 + 3𝑦2 ≤ 6]
i.e. 4𝑧1 + 3𝑧2 ≤ 6
and 𝑧1 + 𝑧2 = 𝜆(𝑥1 + 𝜆2 ) + (1 − 𝜆)(𝑦1 + 𝑦2 ) ≥ 𝜆. 1 + (1 − 𝜆).1
ie. 𝑧1 + 𝑧2 ≥ 1 [∵ 0 ≤ 𝜆 ≤ 1,0 ≤ 1 − 𝜆 ≤ 1, 𝑥1 + 𝑥2 ≥ 1, 𝑦1 + 𝛾2 ≥ 1]
∴ 𝑤 = (𝑧1 , 𝑧2 ) ∈ 𝐴.

Hence the set 𝑆 is a convex set.


(ii) Hint. Procced similarly
𝐵 is a convex set.
Alter. Obviously 𝑆 is the intersection of two half spaces, viz.

𝐻1 = ((𝑥1 , 𝑥2 ): 4𝑥1 + 3𝑥2 ≤ 6)

and 𝐻2 = {(𝑥1 , 𝑥2 ): 𝑥1 + 𝑥2 ≥ 1}, in (i) part


and 𝐻1 = {(𝑥1 , 𝑥2 ): 𝑥1 ≥ 2} and 𝐻2 = {(𝑥1 , 𝑥2 ): 𝑥1 ≤ 3} is (ii) part.
Since H1 and H2 are convex sets so S = H1 ∩ H2 is also convex set.
Ex. 6. For any points x, y ∈ 𝐾 𝑛 , show that the line segment |x: y| is at convex set.
Sol. Let 𝑢, 𝑣 ∈ [𝑥: 𝑦]. Then
and 𝑢 = 𝜆1 𝑥 + (1 − 𝜆1 )𝑦, 0 ≤ 𝜆1 ≤ 1
𝑣 = 𝜆2 𝑥 + (1 − 𝜆2 )𝑦, 0 ≤ 𝜆2 ≤ 1
Now let 𝑤 be a point on the line segment joining the points 𝑢 and 𝑣.
Then

161 | P a g e

© Department of Distance & Continuing Education, Campus of Open Learning,


School of Open Learning, University of Delhi
B.A. (Hons.) Economics

𝑤 = 𝜆𝑢 + (1 − 𝜆)𝑣1 0 ≤ 𝜆 ≤ 1
= 𝜆[𝜆1 𝑥 + (1 − 𝜆1 )𝑦] + (1 − 𝜆)[𝜆2 𝑥 + (1 − 𝜆2 )𝑦]
= [𝜆𝜆1 + (1 − 𝜆)𝜆2 ]𝑥 + [𝜆(1 − 𝜆1 ) + (1 − 𝜆)(1 − 𝜆2 )]𝑦.
If we take 𝜇 = 𝜆𝜆1 + (1 − 𝜆)𝜆2 , then
1 − 𝜇 = 1 − 𝜆𝜆1 − (1 − 𝜆)𝜆2 = 𝜆 + (1 − 𝜆) − 𝜆𝜆1 − (1 − 𝜆)𝜆2
= 𝜆(1 − 𝜆1 ) + (1 − 𝜆)(1 − 𝜆2 ).
Since, 0 ≤ 𝜆1 ≤ 1,0 ≤ 𝜆2 ≤ 1 ⇒ 0 ≤ 𝜆𝜆1 + (1 − 𝜆)𝜆2 ≤ 1,
𝑤 = 𝜇𝑥 + (1 − 𝜇)𝑦, 0 ≤ 𝜇 ≤ 1
∴ 𝑤 ∈ [𝑥: 𝑦]
Hence the set [𝑥: 𝑦] is a convex set.
Ex. 7. Let 𝐴 be an 𝑚 × 𝑛 matrix and 𝐛 an 𝑚-vector, then show that {𝐱 ∈ 𝑅 𝑛 : 𝐴𝐱 ≤ 𝐛} is a
convex set.

Sol. Let 𝐴 = [𝑎𝑖𝑗 ]𝑚×𝑛 , 𝐱 = (𝑥1 , 𝑥2 , … , 𝑥𝑛 ) and 𝐛 = (𝑏1 , 𝑏2 , … , 𝑏𝑚 ), then the set 𝑆 =
{x ∈ 𝑅 𝑛 : 𝐴x ≤ b} can be written in m-inequalities :
𝑎11 𝑥1 + 𝑎12 𝑥2 + ⋯ + 𝑎1𝑛 𝑥𝑛 ≤ 𝑏1
𝑎21 𝑥1 + 𝑎22 𝑥2 + ⋯ + 𝑎2𝑛 𝑥𝑛 ≤ 𝑏2
… … … … …
𝑎𝑚1 𝑥1 + 𝑎𝑚2 𝑥2 + ⋯ + 𝑎𝑚𝑛 𝑥𝑛 ≤ 𝑏𝑚 .
Thus, the set 𝑆 is the intersection of 𝑚 half spaces.
𝐻𝑖 = {(𝑥1 , 𝑥2 , … , 𝑥𝑛 ): 𝑎𝑖1 𝑥1 + 𝑎𝑖2 𝑥2 + ⋯ + 𝑎𝑖𝑛 𝑥𝑛 ≤ 𝑏𝑖 , 𝑖 = 1,2, … , 𝑚}.
It follows that 𝑆 = ⋂𝑚
𝑖=1 𝐻𝑖 is convex as eaci half space is convex.
Ex. 8. Show that the set 𝑆 = {𝐱: 𝐱 = (𝑥1 , 𝑥2 , 𝑥3 ), 𝑥12 + 𝑥22 + 𝑥32 ≤ 1} is a convex set.
Sol. Let 𝐱, 𝐲 ∈ 𝑆, where 𝐱 = (𝑥1 , 𝑥2 , 𝑥3 ), 𝐲 = (𝑦1 , 𝑦2 , 𝑦3 ).
Then, by the given condition, we have.
𝑥12 + 𝑥22 + 𝑥32 ≤ 1
𝑦12 + 𝑦22 + 𝑦32 ≤ 1
If 𝐳 = (𝑧1 , 𝑧2 , 𝑧3 ) is a point on the line segment joining the points 𝐱 and 𝐲, then.

162 | P a g e

© Department of Distance & Continuing Education, Campus of Open Learning,


School of Open Learning, University of Delhi
Intermediate Mathematical Methods for Economics

𝐳 = 𝜆𝐱 + (1 − 𝜆)𝐲, 0 ≤ 𝜆 ≤ 1
⇒ (𝑧1 , 𝑧2 , 𝑧3 ) = 𝜆(𝑥1 , 𝑥2 , 𝑥3 ) + (1 − 𝜆)(𝑦1 , 𝑦2 , 𝑦3 )
⇒ 𝑧1 = 𝜆𝑥1 + (1 − 𝜆)𝑦1 , 𝑧2 = 𝜆𝑥2 + (1 − 𝜆)𝑦2 , 𝑧3 = 𝜆𝑥3 + (1 − 𝜆)𝑦3 .
Now 𝑧12 + 𝑧22 + 𝑧32 = [𝜆𝑥1 + (1 − 𝜆)𝑦1 ]2 + [𝜆𝑥2 + (1 − 𝜆)𝑦2 ]2 + [𝜆𝑥3 + (1 − 𝜆)𝑦3 ]2
= 𝜆2 (𝑥12 + 𝑥22 + 𝑥32 ) + (1 − 𝜆)2 (𝑦12 + 𝑦22 + 𝑦32 )
+2𝜆(1 − 𝜆)(𝑥1 𝑦1 + 𝑥2 𝑦2 + 𝑥3 𝑦3 )
≤ 𝜆2 ⋅ 1 + (1 − 𝜆)2 ⋅ 1 + 2𝜆(1 − 𝜆)(𝑥1 𝑦1 + 𝑥2 𝑦2 + 𝑥3 𝑦3 )

using (1) and (2)


By Lagrange's identity, we have
(𝑥12 + 𝑥22 + 𝑥32 )(𝑦12 + 𝑦22 + 𝑦32 ) − (𝑥1 𝑦1 + 𝑥2 𝑦2 + 𝑥3 𝑦3 )2 ≡ Σ(𝑥1 𝑦2 − 𝑥2 𝑦1 )2 ≥ 0

∴ 𝑥1 𝑦1 + 𝑥2 𝑦2 + 𝑥3 𝑦3 ≤ √(𝑥12 + 𝑥22 + 𝑥32 )√(𝑦12 + 𝑦22 + 𝑦32 ) ≤ 1

Thus 𝑧12 + 𝑧22 + 𝑧32 ≤ 𝜆2 + (1 − 𝜆)2 + 2𝜆. (1 − 𝜆) = 1.


using (1) and (2)
∴ 𝑧 = (𝑧1 , 𝑧2 , 𝑧3 ) ∈ 𝑆.
Hence the set 𝑆 is a convex set.
Ex. 9. Express any point 𝐰 inside a triangle as a convex combination of the vertices (extreme
points) 𝐱1 , 𝐱 2 , 𝐱 3 of the triangle.
Sol. Let 𝐴𝐵𝐶 be the triangle with vertices 𝐱1 , 𝐱 2 , 𝐱 3 respectively and 𝑃 be any point 𝑤 inside
the triangle. Join the points 𝐴 and 𝑃 and extend this line to meet the base 𝐵𝐶 at 𝐷(𝐮). Since 𝐷
is a point on the line segment 𝐵𝐶, so u can be written as a convex combination of 𝑥2 and 𝑥3 .
Thus 𝐮 = 𝜆1 𝐱 2 + (1 − 𝜆1 )𝐱 3 , 0 ≤ 𝜆1 ≤ 1.
(1)
Again 𝑃 is a point on the line segment
𝐴𝐷, therefore.
𝑤 = 𝜆2 𝐱1 + (1 − 𝜆2 )𝐮, 0 ≤ 𝜆2 ≤ 1
= 𝜆2 𝐱1 + (1 − 𝜆2 )[𝜆1 𝐱 2 + (1 − 𝜆1 )𝐱 3 ], using (1)
= 𝜆2 𝐱1 + 𝜆1 (1 − 𝜆2 )𝐱 2 + (1 − 𝜆1 )(1 − 𝜆2 )𝐱 3
or
𝑤 = 𝜇1 𝑥1 + 𝜇2 𝑥2 + 𝜇3 𝑥3
where

163 | P a g e

© Department of Distance & Continuing Education, Campus of Open Learning,


School of Open Learning, University of Delhi
B.A. (Hons.) Economics

Each of the 𝜇1 , 𝜇2 , 𝜇3 lies between 0 and 1 i.e., 0 ≤ 𝜇𝑖 ≤ 1, 𝑖 = 1,2,3 as 0 ≤ 𝜆1 ≤ 1 and 0 ≤


𝜆2 ≤ 1.
Also
𝜇1 + 𝜇2 + 𝜇3 = 𝜆2 + 𝜆1 (1 − 𝜆2 ) + (1 − 𝜆1 )(1 − 𝜆2 )
= 𝜆2 + 𝜆1 − 𝜆1 𝜆2 + 1 − 𝜆1 − 𝜆2 + 𝜆1 𝜆2 = 1
⇒ 𝜇1 𝐱1 + 𝜇2 𝐱 2 + 𝜇3 𝐱 3 is a convex combination of the points 𝐱1 , 𝐱 2 , 𝐱 3 .
Hence (2) is the required convex combination for the point 𝑤.
Ex. 10. Find the extreme points, if any, of the following sets:
(i) 𝑆 = {(𝑥, 𝑦): 𝑥 2 + 𝑦 2 ≤ 25}
(ii) {(𝑥, 𝑦): |𝑥| ≤ 1, |𝑦| ≤ 1}
Sol. (i) Draw the region of the set 𝑆. It represents the boundary and interior of the circle with
centre at (0,0) and radius 5.
Every point on its circumference is an extreme point.
(ii) We have |𝑥| ≤ 1 ⇒ −1 ≤ 𝑥 ≤ 1 and |𝑦| ≤ 1 ⇒ −1 ≤ 𝑦 ≤ 1.
Thus the set 𝑆 represents the region of square bounded by the lines 𝑥 = 1, 𝑥 = −1, 𝑦 =
1, 𝑦 = −1. The extreme points of this convex set are the vertices (1,1), (−1,1), (−1, −1) and
(1, −1).
Ex. 11. Is the union of two convex sets necessarily a convex set?
Sol. Ans. No. The union of two convex sets may or may not be a convex set.
For example. Consider 𝑆1 = {(𝑥, 𝑦): 𝑥 ≥ 2} and 𝑇1 = {(𝑥, 𝑦): 𝑥 ≥ 3}
Then 𝑆1 ∪ 𝑇1 = {(𝑥, 𝑦): 𝑥 ≥ 2}
Obviously 𝑆1 ∪ 𝑇1 is a convex set.
Again consider 𝑆2 = {(𝑥, 𝑦): 𝑥 ≥ 2} and 𝑇2 = {(𝑥, 𝑦): 𝑦 ≥ 1}.
Then 𝑆2 ∪ 𝑇2 = {(𝑥, 𝑦): 𝑥 ≥ 2 or 𝑦 ≥ 1}.
Now the points 𝐴(9/4,1/2) and 𝐵(1/2,5/4) are the points of 𝑆2 ∪ 𝑇2 but their mid-point
𝑃(11/8,7/8) is not the point of the set 𝑆2 ∪ 𝑇2 as 11/8 < 2 and 7/8 < 1.
Thus 𝑆2 ∪ 𝑇2 is not a convex set.
Hence the union of two convex sets is not necessarily a convex set.
Ex. 12. Find all the basic feasible solutions for the equations.

164 | P a g e

© Department of Distance & Continuing Education, Campus of Open Learning,


School of Open Learning, University of Delhi
Intermediate Mathematical Methods for Economics

2𝑥1 + 6𝑥2 + 2𝑥3 + 𝑥4 =3


6𝑥1 + 4𝑥2 + 4𝑥3 + 6𝑥4 =2
𝑥𝑖 ≥0
and determine the associated general convex combination of the extreme point solutions.
Sol. In matrix form the given system of equations can be written as
where
𝐴𝐱 = 𝐛
𝐴 = (𝛼1 , 𝛼2 , 𝛼3 , 𝛼4 )
𝑥1
2 6 2 1 𝑥2 3
𝛼1 = [ ] , 𝛼2 = [ ] , 𝛼3 = [ ] , 𝛼4 = [ ] , 𝐱 = [𝑥 ] , 𝐛 = [ ] .
6 4 4 6 3 2
𝑥4
This problem can have at most 4 𝐶2 = 6 basic solutions. Now the six sets of two vectors are
2 6 2 2
𝐵1 = [𝛼1 , 𝛼2 ] = [ ] , 𝐵2 = [𝛼1 , 𝛼3 ] = [ ]
6 4 6 4
2 1 6 2
𝐵3 = [𝛼1 , 𝛼4 ] = [ ] , 𝐵4 = [𝛼2 , 𝛼3 ] = [ ]
6 6 4 4
6 1 2 1
𝐵5 = [𝛼2 , 𝛼4 ] = [ ] , 𝐵6 = [𝛼3 , 𝛼4 ] = [ ].
4 6 4 6
Here |𝐵1 | = −28, |𝐵2 | = −4, |𝐵3 | = 6, |𝐵4 | = 16, |𝐵5 | = 32, |𝐵6 | = 8.
Since none of these is zero, therefore all these sets are L.I.
Hence all the six basic solutions exist.
If 𝐱 𝐵𝑖 , 𝑖 = 1,2, … ,6 are the vectors of the basic variables associated to the sets 𝐵𝑖 , 𝑖 =
1,2, … ,6 respectively, then
𝑥1 1 4 −6 3 0
[𝑥 ] = x𝐵1 = 𝐵1−1 𝐛 = − [ ][ ] = [ ]
2 28 −6 2 2 1/2
𝑥1 1 4 −2 3 −2
[𝑥 ] = x𝐵2 = 𝐵2−1 𝐛 = − [ ][ ] = [ ]
3 4 −6 2 2 7/2
𝑥1 1 6 −1 3 8/3
[𝑥 ] = x𝐵3 = 𝐵3−1 𝐛 = [ ][ ] = [ ]
4 6 −6 2 2 −7/3
𝑥2 1 4 −2 3 1/2
[𝑥 ] = x𝐵4 = 𝐵4−1 𝐛 = [ ][ ] = [ ]
3 16 −4 6 2 0
𝑥2 1 6 −1 3 1/2
[𝑥 ] = 𝑥𝐵5 = 𝐵5−1 𝐛 = [ ][ ] = [ ]
4 32 −4 6 2 0
𝑥3 1 6 −1 3 2
[𝑥 ] = 𝐱 𝐵6 = 𝐵6−1 𝐛 = [ ] [ ] = [ ].
4 8 −4 2 2 −1
165 | P a g e

© Department of Distance & Continuing Education, Campus of Open Learning,


School of Open Learning, University of Delhi
B.A. (Hons.) Economics

Thus it is obvious that out of these only three basic solutions are B.F.S. (in which variables are
non-negative). But the B.F.S.'s correspond to the extreme points. Hence the only three extreme
point solutions are given by
1 1 1
𝐱1 = (0, , 0,0) , x2 = (0, , 0,0) , 𝐱 3 = (0, , 0,0) .
2 2 2
Here 𝑥1 = 𝑥2 = 𝑥3 . Hence there is unique extreme point solution.
Note. To find the basic solution 𝑥𝐵1 we can also proceed as follows. Putting 𝑥3 = 0, 𝑥4 = 0 in
the given eqns., we get 2𝑥1 + 6𝑥2 = 3 and 6𝑥1 + 4𝑥2 = 2 solving 𝑥1 = 0, 𝑥2 = 1/2, etc.
Solve the following Questions
1 Which of the following sets are convex ?
(i) 𝐴 = {(𝑥1 , 𝑥2 ): 𝑥1 𝑥2 ≤ 1, 𝑥1 ≥ 0, 𝑥2 ≥ 0}
(ii) 𝐴 = {(𝑥1 , 𝑥2 ): 𝑥22 − 3 ≥ −𝑥12 , 𝑥1 ≥ 0, 𝑥2 ≥ 0}.
2 Prove that the set {(𝑥1 , 𝑥2 ): 𝑥12 + 𝑥22 ≤ 4} is a convex set.
3 Define a convex set. Show that the set 𝑆 = {(𝑥1 , 𝑥2 ): 3𝑥12 + 2𝑥22 ≤ 6} is convex.
4 Given two planes 𝑎1 𝑥 + 𝑏1 𝑦 + 𝑐1 𝑧 + 𝑑1 = 0, 𝑎2 𝑥 + 𝑏2 𝑦 + 𝑐2 𝑧 + 𝑑2 = 0 in 𝑅 3 ,
prove that their intersection is a convex set but their union is not.
5 Show that 𝑆 = {(𝑥1 , 𝑥2 , 𝑥3 ): 2𝑥1 − 𝑥2 + 𝑥3 ≤ 4, 𝑥1 + 2𝑥2 − 𝑥3 ≤ 1} is a convex set.
6 Examine convexity of the following sets :
𝑥12 𝑥22
(i) 𝑆 = {(𝑥1 , 𝑥2 ): + ≤ 1}
4 9

(ii) 𝑆 = {(𝑥1 , 𝑥2 ): 𝑥12 + 𝑥22 ≤ 1, 𝑥1 + 𝑥2 ≥ 1}


7 If 𝑆1 and 𝑆2 be two non-empty disjoint convex sets and 𝑆 be a set such that if 𝐱1 ∈ 𝑆1
and x2 ∈ 𝑆2 then x1 − x2 ∈ 𝑆. Show that 𝑆 is also a convex set and does not contain
the origin.
8 Show that the set of all the internal points of a convex set 𝑆 is a convex set.
9 Determine whether the vector [7,0] is a convex combination of the vectors [6,
3], [9, −6], [1,2], [1, −1].
10 Give an example of a convex set whose every boundary point is an extreme point.
11 Determine the convex hull of the following sets:
(i) 𝐴 = {(𝑥1 , 𝑥2 ): 𝑥12 + 𝑥22 = 1}
166 | P a g e

© Department of Distance & Continuing Education, Campus of Open Learning,


School of Open Learning, University of Delhi
Intermediate Mathematical Methods for Economics

(ii) 𝐴 = {𝑥1 , 𝑥2 }.
1
12 If x1 , x2 ∈ 𝑆 ⇒ 2 (x1 + x2 ) ∈ 𝑆 then the set 𝑆 is convex or not.

13 Can there be any convex set without any extreme point ? Prove that an extreme point
of a convex set is a boundary point of the set.
14 Is the set 𝑆 = {𝐱: 𝐱 ∈ 𝐸 𝑚 , |𝑥| = 1} convex?
15 Find the extreme points of the polygonal convex set 𝑥 determmed by the system.
2𝑥1 + 𝑥2 + 9 ≥ 0, −𝑥1 + 3𝑥2 + 6 ≥ 0, 𝑥1 + 𝑥2 ≤ 0, 𝑥1 + 2𝑥2 − 3 ≤ 0
16 𝐴 and 𝐵 are two convex sets in 𝑅 𝑛 , 𝐶 is a set in 𝑅 𝑛 defined as
𝐶 = {𝐳 ∈ 𝑅 𝑛 : 𝐳 = 𝐱 + 𝐲, 𝐱 ∈ 𝐴, 𝐲 ∈ 𝐁}
Examine convexity of 𝐶.
3
17. Express (2,1), (0, 2), if possible, as a convex combination of (1,1) and (−1,2).

18. What is meant by convex combination of vectors? Prove that the set of all convex
combinations of linearly independent vectors is a convex set.
19. Prove that the objective function of a L.P.P. assumes minimum value at an extreme point
of the convex set 𝑋 generated by the set of all feasible solutions.
20. Prove that the collection of feasible solutions of a L.P.P. constitutes a convex set, whose
extreme points correspond to B.F. solutions.
21. If a set of 𝑘 ≤ 𝑚 vectors 𝐏1 , 𝐏2 , … , P𝑘 ∈ 𝐴 can be found that are linearly independent and
such that 𝑥1 𝐏1 + 𝑥2 𝐏2 + ⋯ + 𝑥𝑘 𝐏𝑘 = 𝐛, and all 𝑥𝑖 ≥ 0, then show that the point 𝐱 =
(𝑥1 , 𝑥2 , … , 𝑥𝑘 , 0,0 … ,0) is an extreme point of the convex set of feasible solutions of
𝐴𝐱 = 𝐛.
22. Explain the procedure of generating extreme point solutions (Analytical method) to a
linear programming problem pointing out the assumptions made, if any.
23. Find an optimal solution of the following L.P.P. without using the simplex algorithm:
Max. 𝑍 = 2𝑥1 + 3𝑥2 + 4𝑥3 + 7𝑥4
such that 2𝑥1 + 3𝑥2 − 𝑥3 + 4𝑥4 = 8
𝑥1 − 2𝑥2 + 6𝑥3 − 7𝑥4 = −3, 𝑥𝑖 ≥ 0, 𝑖 = 1,2,3,4.

167 | P a g e

© Department of Distance & Continuing Education, Campus of Open Learning,


School of Open Learning, University of Delhi
B.A. (Hons.) Economics

Answers
1 (i) No.
(ii) No.
2 (i) convex
(ii) convex.
3 Yes.
4 The set {𝐱: |𝐱| ≤ 1} in 𝑅 2 .
5 (i) {(𝑥1 , 𝑥2 ): 𝑥12 + 𝑥22 ≤ 1}
(ii) Line segment [𝑥1 : 𝑥2 ].
6 Convex.
7 Yes.
8 No.
3 3
9 (2 , − 2) , (−3, −3), (−7,5), (−3,3).

10 Convex.
3
11 (2,1) cannot be written as a convex combination of (1,1) and (−1,2); (0, 2) =
1 1
(1,1) + 2 (−1,2).
2
44 45 491
12 𝑥1 = 0, 𝑥2 = 0, 𝑥3 = 17 , 𝑥4 = 17 ; max. 𝑍 = .
17

(OBJECTIVE QUESTIONS)
Fill In the Blanks.
Fill in the blanks "......" so that the following statements are complete and correct.
1 The convex linear combination of two points 𝑥1 and 𝑥2 is given by
𝑥 = 𝜆1 𝑥1 + 𝜆2 𝑥2 , s.t. 𝜆1 , 𝜆2 ≥ 0, 𝜆1 + 𝜆2 = ⋯ … ...
2 A hyperplane is a ...... set.
3 Set of all feasible solutions of a L.P.P. is a ...... set.
4 Any point on the line segment joining two points in 𝑅 𝑛 can be expressed as a convex
combination of ..... points.

168 | P a g e

© Department of Distance & Continuing Education, Campus of Open Learning,


School of Open Learning, University of Delhi
Intermediate Mathematical Methods for Economics

5 The polygons which are convex sets have the extreme points as their .......
6 The optimal solution of L.P. problem occurs at an ...... point.
7 Every extreme point of a convex set is a ...... point of the set.
Multiple Choice Questions.
Indicate the correct answer for each question by writing the corresponding letter from
(a), (b), (c) and (d).
1 1
8 In a two-dimensional Euclidean space the points (0,0), (0,1), (1,0), ( , ) span the
2 4
convex polygon. Then the vertices of the polygon are
(a) (0,0), (1,0), (0,1)
1 1
(b) (0,0), (1,0), ( , )
2 4
1 1
(c) (0,0), (0,1), (2 , 4)
(d) none of these.
9 The maximum number of extreme points for a L.P.P. max. 𝑍 = 𝐜𝐱 subject to 𝐴𝐱 =
𝐛, 𝐱 ≥ 0, where 𝐴 is 𝑚 × 𝑛 matrix, is equal to
𝑚!
(𝑎)
𝑛! (𝑛 − 𝑚)!
𝑛!
(b) 𝑚!(𝑛−𝑚)!
(c) 𝑚
(d) 𝑛.
10. A rectangle with sides 𝑎1 and 𝑎2 (𝑎1 ≠ 𝑎2 ) is placed with one corner at the origin
and two of its sides along the axes. The interior of the rectangle plus its edges form a
(a) convex set
(b) non-convex set
(c) polyhedron convex set
(d) none of these
11. Which of the following sets in 𝐸 2 is not a convex set:
(a) {(𝑥1 , 𝑥2 ): 𝑥12 + 𝑥22 ≤ 1}
(b) {(𝑥1 , 𝑥2 ): 𝑥12 + 𝑥22 ≤ 4}
(c) {(𝑥1 , 𝑥2 ): 𝑥12 + 𝑥22 ≥ 1, 𝑥12 + 𝑥22 ≤ 4}
(d) {(𝑥1 , 𝑥2 ): 𝑥1 ≥ 0}.

169 | P a g e

© Department of Distance & Continuing Education, Campus of Open Learning,


School of Open Learning, University of Delhi
B.A. (Hons.) Economics

12. The convex hull of the set of all the points on the boundary of the circle is the
(a) interior of the circle
(b) whole circle
(c) boundary of the circle
(d) none of these.
13 Consider the triangle with vertices (0,0), (2,0), (1,1). The point (⋅ 3, .2) as a convex
combination of these vertices is
(a) . 75(0,0) + .05(2,0) + .2(1,1)
(b) ⋅ 25(0,0) + .50(2,0) + .25(1,1)
(c) . 30(0,0) + .60(2,0) +⋅ 10(1,1)
(d) none of these.
14 The extreme points of the set {(𝑥, 𝑦): |𝑥| ≤ 1, |𝑦| ≤ 1} are
(a) (1,1)(1, −1)
(b) (1,1), (1, −1), (−1,1)
(c) (1,1), (1, −1), (−1,1), (−1, −1)
(d) (1,1), (−1, −1).
True or False.
Write ' 𝑻 ' for true and ' 𝑭 ' for false statement.
15 A vertex is a boundary point but all boundary points are not vertices.
16 The intersection of the arbitrary family of convex sets in 𝑅 𝑛 is not necessarily convex.
17 Every convex set in 𝑅 𝑛 is a polygon also.
18 A line passing through two distinct points x1 and x2 is the set of all the points x such
that x = 𝜆x1 + (1 − 𝜆)x2 , 𝜆 ∈ [0,1].
19 The number of edges that can emanate from any given extreme point of the convex
set of F.S. is two.
20 The set of all feasible solutions (if not empty) of a L.P.P. is a convex set.
21 If a L.P.P. has two feasible solutions, then it has an infinite number of feasible
solutions.
22 The basic feasible solutions of a L.P.P. are infinite in number.
23 The extreme points of the convex set of feasible solutions to a L.P.P. are finite in
number.
170 | P a g e

© Department of Distance & Continuing Education, Campus of Open Learning,


School of Open Learning, University of Delhi
Intermediate Mathematical Methods for Economics

24 Every B.F.S. to a L.P.P. is not an extreme point of the convex set of feasible
solutions.
25 If the convex set of feasible solutions to a L.P.P. is a convex polyhedron, the at least
one B.F.S. is optimal.

Answers to Objective Questions


1 1.
2 convex.
3 convex.
4 two.
5 vertices.
6 extreme.
7 boundary.
8 (b).
9 (c).
10 (b).
11 (a).
12 (c).
13 𝐹.
14 𝑇.
15 T.
16 𝐹.
17 𝐹.
18 𝑇.
19 𝐹.
20 𝐹.

10.4 SUMMARY

The main points which we have covered in this lessons are what is Convex Set and its properties
and different type of examples to check whether a given set is convex set or not.

171 | P a g e

© Department of Distance & Continuing Education, Campus of Open Learning,


School of Open Learning, University of Delhi
B.A. (Hons.) Economics

10.5 GLOSSARY

Motivation: These Problems are very useful in economics, statistics and management
problems.
Attention: Think how the Convex Set theory is useful in real world problems.

10.6 REFERENCES

• Sydsaeter, K., Hammond, P. (2002). Mathematics for economics analysis. Pearson


Education.

• Hoy, M., Livernois, J., McKenna, C., Rees, R., Stengos, T, (2001). Mathematics for
Economics, Prentice-Hall India.

10.7 SUGGESTED READINGS

• A.R Vasishtha, Dr. R. K Gupta, Dr. Rajesh Kumar, Dr. Rajiv Atri, Dr Hemlata
Vasishtha, Linear programming, Krishna Publication, 6th Edition.
• Hamdy A. Taha, Operation Research An Introduction, Pearson Publication, 6th Edition

172 | P a g e

© Department of Distance & Continuing Education, Campus of Open Learning,


School of Open Learning, University of Delhi
Intermediate Mathematical Methods for Economics

LESSON 11
CONVEX AND QUASICONVEX FUNCTIONS

STRUCTURE

11.1 Learning Objectives


11.2 Introduction
11.3 Convex and Quasiconvex Functions
11.3.1 Convex Functions
11.3.2 Quasiconvex Functions and Properties
11.4 Summary
11.5 Glossary
11.6 References
11.7 Suggested Readings

11.1 LEARNING OBJECTIVES

The main objective to learn convex and quasiconvex function is to analyse the function more
deeply. We can check convexity as well as quasiconvexity of function of more than two
variable also.

11.2 INTRODUCTION

we studied about stationary points and the definition of relative and global optimum. The
necessary and sufficient conditions required for a relative optimum in functions of one variable
and its extension to functions of two variables was also studied. In this lecture, determination
of the convexity and concavity of functions is discussed.
The analyst must determine whether the objective functions and constraint equations are
convex or concave. In real-world problems, if the objective function or the constraints are not
convex or concave, the problem is usually mathematically intractable.

11.3 CONVEX AND QUASICONVEX FUNCTION

11.3.1 Convex Function:


A function 𝑓(𝑥) is said to be strictly convex at x if for any two other distinct points 𝐱1 and x2

173 | P a g e

© Department of Distance & Continuing Education, Campus of Open Learning,


School of Open Learning, University of Delhi
B.A. (Hons.) Economics

𝑓{𝜆𝐱1 + (1 − 𝜆)𝐱 2 } < 𝜆𝑓(x1 ) + (1 − 𝜆)𝑓(x2 ), where 0 < 𝜆 < 1.


On the other hand, a function 𝑓(𝑥) is strictly concave if −𝑓(𝑥) is strictly convex.
Testing for convexity of a single variable function
A function is convex if its slope is non decreasing or ∂2 𝑓/ ∂𝑥 2 ≥ 0. It is strictly convex if its
slope is continually increasing or ∂2 𝑓/ ∂𝑥 2 > 0 throughout the function.
Properties of convex functions
A convex function 𝑓, defined on some convex open interval 𝐶, is continuous on 𝐶 and
differentiable at all or at most, countable many points. If 𝐶 is closed, then 𝑓 may fail to be
continuous at the end points of 𝐶.
A continuous function on an interval 𝐶 is convex if and only if.
𝑎+𝑏 𝑓(𝑎) + 𝑓(𝑏)
𝑓( )≤
2 2
for all 𝑎 and 𝑏 in 𝐶.
A differentiable function of one variable is convex on an interval if and only if its derivative is
monotonically non-decreasing on that interval.
A continuously differentiable function of one variable is convex on an interval if and only if
the function lies above all its tangents: 𝑓(𝑏) ≥ 𝑓(𝑎) + 𝑓 ′ (𝑎)(𝑏 − 𝑎) for all a and b in the
interval.
A twice differentiable function of one variable is convex on an interval if and only if its second
derivative is non-negative in that interval; this gives a practical test for convexity. If its second
derivative is positive then it is strictly convex, but the converse does not hold, as shown by
𝑓(𝑥) = 𝑥 4
More generally, a continuous, twice differentiable function of several variables is convex on a
convex set if and only if its Hessian matrix is positive semi definite on the interior of the convex
set.
If two functions 𝑓 and 𝑔 are convex, then so is any weighted combination 𝑎𝑓 + 𝑏𝑔 with
nonnegative coefficients 𝑎 and 𝑏. Likewise, if 𝑓 and 𝑔 are convex, then the function max{𝑓𝑔}
is convex.
A strictly convex function will have only one minimum which is also the global minimum.

174 | P a g e

© Department of Distance & Continuing Education, Campus of Open Learning,


School of Open Learning, University of Delhi
Intermediate Mathematical Methods for Economics

Examples
• The second derivative of 𝑥 2 is 2, it follows that 𝑥 2 is a convex function of 𝑥.
• The absolute value function |𝑥| is convex, even though it does not have a derivative at
𝑥 = 0.
• The function 𝑓 with domain [0,1] defined by 𝑓(0) = 𝑓(1) = 1, 𝑓(𝑥) = 0 for 0 < 𝑥 <
1 is convex; it is continuous on the open interval (0,1), but not continuous at 0 and 1.
• Every linear transformation is convex but not strictly convex, since if 𝑓 is linear, then
𝑓(𝑎 + 𝑏) = 𝑓(𝑎) + 𝑓(𝑏). This implies that the identity map (i.e., 𝑓(𝑥) = 𝑥) is convex
but not strictly convex. The fact holds good if we replace "convex" by "concave".
• An affine function (𝑓(𝑥) = 𝑎𝑥 + 𝑏) is simultaneously convex and concave.
11.3.2 Quasiconvex Functions and Properties:

A function is called quasi concave if and only if there is an 𝑥0 such that for all 𝑥 < 𝑥0 , 𝑓(𝑥) is
non-decreasing while for all 𝑥 > 𝑥0 it is non-increasing. 𝑥0 can also be ±∞, making the
function non-decreasing (non-increasing) for all 𝑥. The opposite of quasiconcave is
quasiconvex.
Example 1
Consider the example in lecture notes 1 for a function of two variables. Locate the stationary
points of 𝑓(𝑥) = 12𝑥 5 − 45𝑥 4 + 40𝑥 3 + 5 and find out if the function is convex, concave or
neither at the points of optima based on the testing rules discussed above.
Solution.
𝑓 ′ (𝑥) = 60𝑥 4 − 180𝑥 3 + 120𝑥 2 = 0
⇒ 𝑥 4 − 3𝑥 3 + 2𝑥 2 = 0
or 𝑥 = 0,1,2
Consider the point 𝑥 = 𝑥 ∗ = 0
𝑓 ′′ (𝑥 ∗ ) = 240(𝑥 ∗ )3 − 540(𝑥 ∗ )2 + 240𝑥 ∗ = 0 at 𝑥 ∗ = 0
𝑓 ′′′ (𝑥 ∗ ) = 720(𝑥 ∗ )2 − 1080𝑥 ∗ + 240 = 240 at 𝑥 ∗ = 0
Since the third derivative is non-zero 𝑥 = 𝑥 ∗ = 0 is neither a point of maximum or minimum
but it is a point of inflection. Hence the function is neither convex nor concave at this point.
Consider 𝑥 = 𝑥 ∗ = 1
𝑓 ′′ (𝑥 ∗ ) = 240(𝑥 ∗ )3 − 540(𝑥 ∗ )2 + 240𝑥 ∗ = −60 at 𝑥 ∗ = 1
175 | P a g e

© Department of Distance & Continuing Education, Campus of Open Learning,


School of Open Learning, University of Delhi
B.A. (Hons.) Economics

Since the second derivative is negative, the point 𝑥 = 𝑥 ∗ = 1 is a point of local maxima with
a maximum value of 𝑓(𝑥) = 12 − 45 + 40 + 5 = 12. At this point the function is concave
since ∂2 𝑓/ ∂𝑥 2 < 0
Consider 𝑥 = 𝑥 ∗ = 2
𝑓 ′′ (𝑥 ∗ ) = 240(𝑥 ∗ )3 − 540(𝑥 ∗ )2 + 240𝑥 ∗ = 240 at 𝑥 ∗ = 2
Since the second derivative is positive, the point 𝑥 = 𝑥 ∗ = 2 is a point of local minima with a
minimum value of 𝑓(𝑥) = −11. At this point the function is convex since ∂2 𝑓/ ∂𝑥 2 > 0.
To determine convexity or concavity of a function of multiple variables, the eigenvalues of its
Hessian matrix are examined and the following rules apply.
(a) If all eigenvalues of the Hessian are positive the function is strictly convex.
(b) If all eigenvalues of the Hessian are negative the function is strictly concave.
(c) If some eigenvalues are positive and some are negative, or if some are zero, the function is
neither strictly concave nor strictly convex.
Example 2
Consider the example in lecture notes 1 for a function of two variables. Locate the stationary
points of 𝑓(𝐗) and find out if the function is convex, concave or neither at the points of optima
based on the rules discussed in this lecture.
𝑓(𝐗) = 2𝑥13 /3 − 2𝑥1 𝑥2 − 5𝑥1 + 2𝑥22 + 4𝑥2 + 5
Solution
∂𝑓 ∗
(X )
∂𝑥1 2𝑥 2 − 2𝑥2 − 5 0
Δ𝑥 𝑓 = =[ 1 ]=[ ]
∂𝑓 ∗ −2𝑥1 + 4𝑥2 + 4 0
(X )
[∂𝑥2 ]
Solving the above the two stationary points are
X1 = [−1, −3/2]
and
X2 = [3/2, −1/4]
The Hessian of 𝑓(𝐗) is

176 | P a g e

© Department of Distance & Continuing Education, Campus of Open Learning,


School of Open Learning, University of Delhi
Intermediate Mathematical Methods for Economics

∂2 𝑓 ∂2 𝑓 ∂2 𝑓 ∂2 𝑓
= 4𝑥1 ; = 4; = = −2
∂𝑥1 2 ∂𝑥2 2 ∂𝑥1 ∂𝑥2 ∂𝑥2 ∂𝑥1
4𝑥 −2
𝐇 =[ 1 ]
−2 4
𝜆 − 4𝑥1 2
|𝜆𝐈 − 𝐇| = | |
2 𝜆−4
At 𝑋1
𝜆+4 2
|𝜆𝐈 − 𝐇| = | | = (𝜆 + 4)(𝜆 − 4) − 4 = 0
2 𝜆−4
𝜆2 − 16 − 4 = 0
𝜆2 = 12
𝜆1 = +√12 𝜆2 = −√12
Since one eigen value is positive and one negative, X1 is neither a relative maximum nor a
relative minimum. Hence at X1 the function is neither convex nor concave.
At X2 = [3/2, −1/4]
𝜆−6 2
|𝜆𝐈 − 𝐇| = | | = (𝜆 − 6)(𝜆 − 4) − 4 = 0
2 𝜆−4
𝜆2 − 10𝜆 + 20 = 0
𝜆1 = 5 + √5 𝜆2 = 5 − √5
Since both the eigen values are positive, X2 is a local minimum, and the function is convex at
this point as both the eigen values are positive.
SOME IMPORTANT THEOREMS.
Theorem 1. A hyperplane is a convex set.
Proof. Consider the hyperplane
𝑋 = {𝐱: 𝐜𝐱 = 𝑧}.
Let 𝑥1 and 𝑥2 be any two points in the hyperplane 𝑋.
∴ 𝐜𝐱 𝟏 = 𝑧 and 𝐜𝐱 𝟐 = 𝑧.
If 𝑥3 = 𝜆𝑥1 + (1 − 𝜆)𝑥2 , 0 ≤ 𝜆 ≤ 1
then
𝑐x3 = 𝜆𝑐x1 + (1 − 𝜆)𝑐x2 ,
= 𝜆𝑧 + (1 − 𝜆)𝑧 = 𝑧
177 | P a g e

© Department of Distance & Continuing Education, Campus of Open Learning,


School of Open Learning, University of Delhi
B.A. (Hons.) Economics

which implies that


x3 = 𝜆x1 + (1 − 𝜆)x2 is also a point in the hyperplane 𝑋.
Hence by definition, the hyperplane 𝑋 is a convex set.
Theorem 2. The closed half spaces 𝑯𝟏 = {𝐱: 𝐞𝐱 ≥ 𝒛} and 𝑯𝟐 = {𝐱: 𝐞𝐱 ≤ 𝒛} are convex
sets.
Proof. Let 𝑥1 and 𝑥2 be any two points of 𝐻1 . Then
𝐜x1 ≥ 𝑧, ex2 ≥ 𝑧
If 0 ≤ 𝜆 ≤ 1, then 𝑐[𝜆𝐱1 + (1 − 𝜆)𝐱 2 ] = 𝜆𝐜𝐱1 + (1 − 𝜆)𝐜𝐱 2
≥ 𝜆𝑧 + (1 − 𝜆)𝑧 = 𝑧
Hence x1 , x2 ∈ 𝐻1 and 0 ≤ 𝜆 ≤ 1 ⇒ 𝜆x1 + (1 − 𝜆)x2 ∈ 𝐻1.
So H1 is a convex set.
Similarly, if 𝐱1 , 𝐱 2 ∈ 𝐻2 , 0 ≤ 𝜆 ≤ 1, then replacing the inequality sign ≥ by ≤ in above, we
get 𝜆𝑥1 + (1 − 𝜆)𝑥2 ∈ 𝐻2 .
So H2 is also a convex set.
Corollary. The open half spaces: {𝐱: 𝐜𝐱 > 𝑧} and {𝐱: 𝐜𝐱 < 𝑧} are convex sets.
Theorem 3 (a). The intersection of two convex sets is also a convex set.
Proof. Consider two convex sets 𝑋1 and 𝑋2. Let 𝑋3 be the intersection of sets 𝑋1 and 𝑋2, i.e.,
𝑋3 = 𝑋1 ∩ 𝑋2.
Now 𝐱1 ∈ 𝑋1 ∩ 𝑋2 ⇒ x1 ∈ 𝑋1
and x1 ∈ 𝑋2 𝐱 2 ∈ 𝑋1 ∩ 𝑋2 ⇒ 𝐱 2 ∈ 𝑋1 and 𝐱 2 ∈ 𝑋2.
Since 𝑋1 and 𝑋2 are convex sets,
∴ 𝑥1 , 𝑥2 ∈ 𝑋1 ⇒ 𝜆𝑥1 + (1 − 𝜆)𝑥2 ∈ 𝑋1 0 ≤ 𝜆 ≤ 1
and 𝑥1 , 𝑥2 ∈ 𝑋2 ⇒ 𝜆𝑥1 + (1 − 𝜆)𝑥2 ∈ 𝑋2 0 ≤ 𝜆 ≤ 1
Thus 𝜆𝑥1 + (1 − 𝜆)𝑥2 ∈ 𝑋1 and 𝜆𝑥1 + (1 − 𝜆)𝑥2 ∈ 𝑋2
⇒ 𝜆𝑥1 + (1 − 𝜆)𝐱 2 ∈ 𝑋1 ∩ 𝑋2 0 ≤ 𝜆 ≤ 1.
Hence by definition, 𝑋3 = 𝑋1 ∩ 𝑋2 is a convex set.
(b) Intersection of any finite number of convex sets is also a convex set.
Proof. Let 𝑋1 , 𝑋2 , … , 𝑋𝑛 be 𝑛 convex sets and

178 | P a g e

© Department of Distance & Continuing Education, Campus of Open Learning,


School of Open Learning, University of Delhi
Intermediate Mathematical Methods for Economics

𝑋 = 𝑋1 ∩ 𝑋2 ∩ … ∩ 𝑋𝑛
Now x1 ∈ 𝑋1 ∩ 𝑋2 ∩ … ∩ 𝑋𝑛 ⇒ x1 ∈ 𝑋𝑖 , ∀𝑖 = 1,2, … , 𝑛
and x2 ∈ 𝑋1 ∩ 𝑋2 ∩ … ∩ 𝑋𝑛 ⇒ x2 ∈ 𝑋𝑖 , ∀𝑖 = 1,2, … , 𝑛.
Since 𝑋𝑖 is convex set for 𝑖 = 1,2, … , 𝑛
∴ x1 , x2 ∈ 𝑋𝑖 ⇒ 𝜆x1 + (1 − 𝜆)x2 ∈ 𝑋𝑖 , ∀𝑖 = 1,2, … , 𝑛
𝜆𝑥1 + (1 − 𝜆)𝑥2 ∈ 𝑋1 ∩ 𝑋2 ∩ … ∩ 𝑋𝑛 where 0 ≤ 𝜆 ≤ 1

0 ≤ 𝜆 ≤ 1.
i.e., x1 ∈ 𝑋1 ∩ 𝑋2 ∩ … ∩ 𝑋𝑛 and x2 ∈ 𝑋1 ∩ 𝑋2 ∩ … ∩ 𝑋𝑛
⇒ 𝜆𝑥1 + (1 − 𝜆)𝑥2 ∈ 𝑋1 ∩ 𝑋2 ∩ … ∩ 𝑋𝑛 , 0 ≤ 𝜆 ≤ 1

Hence by definition 𝑋1 ∩ 𝑋2 ∩ … ∩ 𝑋𝑛 is a convex set.


(c) Arbitrary intersection of convex sets is also a convex set.
Theorem 4. The set of all convex combinations of a finite number of points 𝐱 𝟏 , 𝐱 𝟐 , … , 𝐱 𝒏
is a convex set.
Proof: Let X be the set of all convex combination of finite number of points.
i.e., 𝑋 = {𝐱: 𝐱 = ∑𝑛𝑖=1 𝜆𝑖 𝐱 i , ∑𝑛𝑖=1 𝜆𝑖 = 1, 𝜆𝑖 ≥ 0}.
Let 𝐮, 𝐯 ∈ 𝑋
𝑛 𝑛

∴ 𝐮 = ∑ 𝑎𝑖 𝐱 𝐢 , ∑ 𝑎𝑖 = 1, 𝑎𝑖 ≥ 0
𝑖=1 𝑖=1

and
𝑛 𝑛

𝐯 = ∑ 𝑏𝑖 𝐱 𝐢 , ∑ 𝑏𝑖 = 1, 𝑏𝑖 ≥ 0
𝑖=1 𝑖=1

Consider 𝐰 = 𝜆𝐮 + (1 − 𝜆)𝐯, 0 ≤ 𝜆 ≤ 1
𝑛 𝑛

∴ 𝐰 = 𝜆 ∑ 𝑎𝑖 𝐱 𝐢 + (1 − 𝜆) ∑ 𝑏𝑖 xi
𝑖=1 𝑖=1
𝑛

= ∑ {𝜆𝑎𝑖 + (1 − 𝜆)𝑏𝑖 }𝐱 i
𝑖=1
𝑛

= ∑ 𝑐𝑖 𝐱 𝐢 where 𝑐𝑖 = 𝜆𝑎𝑖 + (1 − 𝜆)𝑏𝑖


𝑖=1

179 | P a g e

© Department of Distance & Continuing Education, Campus of Open Learning,


School of Open Learning, University of Delhi
B.A. (Hons.) Economics

Now ∑𝑛𝑖=1 𝑐𝑖 = ∑𝑛𝑖=1 {𝜆𝑎𝑖 + (1 − 𝜆)𝑏𝑖 }


𝑛 𝑛

= 𝜆 ∑ 𝑎𝑖 + (1 − 𝜆) ∑ 𝑏𝑖 = 𝜆 ⋅ 1 + (1 − 𝜆),1 = 1.
𝑖=1 𝑖=1

Also 𝑐𝑖 = 𝜆𝑎𝑖 + (1 − 𝜆)𝑏𝑖 ≥ 0, ∀𝑖.


Hence w = ∑𝑛𝑖=1 𝑐𝑖 𝐱 𝑖 is a convex combination of x1 , x2 , … , x𝑛 i.e., 𝐰 = 𝜆𝐮 + (1 − 𝜆)𝐯 ∈
𝑋, 0 ≤ 𝜆 ≤ 1.
Hence by definition 𝑋 is a convex set.
Theorem 5. Let 𝑺 and 𝑻 be two convex sets in 𝑬𝒏 , then for any scalars 𝜶, 𝜷, 𝜶𝑺 + 𝜷𝑻 is
also convex.
Proof. Let 𝐱, 𝐲 be two points of 𝛼𝑆 + 𝛽𝑇.
Then 𝐱 = 𝛼u1 + 𝛽𝐯1 and 𝐲 = 𝛼u2 + 𝛽𝐯2 –(1)

where u1 , u2 ∈ 𝑆 and 𝐯1 , 𝐯2 ∈ 𝑇.
For any scalar 𝜆, 0 ≤ 𝜆 ≤ 1, we have
𝜆𝐱 + (1 − 𝜆)𝐲 = 𝜆(𝛼𝐮1 + 𝛽𝐯1 ) + (1 − 𝜆)(𝛼𝐮2 + 𝛽𝐯2 )
= 𝛼[𝜆𝐮1 + (1 − 𝜆)𝐮2 ] + 𝛽[𝜆𝐯1 + (1 − 𝜆)𝐯2 ] − −(2)
But 𝑆 and 𝑇 are convex sets,
u1 , u2 ∈ 𝑆 ⇒ 𝜆u1 + (1 − 𝜆)u2 ∈ 𝑆, 0 ≤ 𝜆 ≤ 1 − −(3)
v1 , v2 ∈ 𝑇 ⇒ 𝜆v1 + (1 − 𝜆)v2 ∈ 𝑇, 0 ≤ 𝜆 ≤ 1. − − (4)
And
Now from (2), (3) and (4) we have
𝜆𝑥 + (1 − 𝜆)𝑦 ∈ 𝛼𝑆 + 𝛽𝑇, 0 ≤ 𝜆 ≤ 1.
Thus 𝐱, 𝐲 ∈ 𝛼𝑆 + 𝛽𝑇 ⇒ [𝐱: 𝐲] ⊂ 𝛼𝑆 + 𝛽𝑇.
Hence 𝛼𝑆 + 𝛽𝑇 is a convex set.
Corollary. If 𝑆 and 𝑇 be two convex sets in 𝐸 𝑛 , then 𝑆 + 𝑇 and 𝑆 − 𝑇 are also convex sets.
Theorem 6. A set 𝑪 is convex iff every convex linear combination of points in 𝑪, also
belongs to 𝑪.
Proof. Let every convex linear combination of points in 𝐶 belong to 𝐶. Then, in particular
convex linear combination of every two points in 𝐶 also belongs to 𝐶.

180 | P a g e

© Department of Distance & Continuing Education, Campus of Open Learning,


School of Open Learning, University of Delhi
Intermediate Mathematical Methods for Economics

Hence 𝐶 is a convex set.


Conversely let 𝐶 be a convex set. Then to prove that convex linear combination of any number
of points in 𝐶 also belongs to 𝐶.
We shall use the induction principle.
Since 𝐶 is convex, the convex linear combination of two points in 𝐶 belongs to C. Thus, the
result is true for 𝑛 = 2.
Now suppose the convex linear combination of any 𝑛 points in 𝐶, belongs to
C.
Let 𝐱1 , 𝐱 2 , … , 𝐱 𝑛 be any 𝑛 points in 𝐶. Then by assumption, 𝜆1 𝐱1 + 𝜆2 𝐱 2 + ⋯ + 𝜆𝑛 𝐱 𝑛 ∈ 𝐶
where 𝜆𝑖 ≥ 0, Σ𝜆𝑖 = 1.
Let 𝐱 = 𝜇1 𝐱1 + 𝜇2 𝐱 2 + ⋯ + 𝜇𝑛 𝐱 𝑛 + 𝜇𝑛+1 𝐱 𝑛+1
i.e., 𝐱 is a convex linear combination of (𝑛 + 1) points of 𝐶, where 𝜇𝑖 ≥ 0 and ∑𝑛+1
𝑖=1 𝜇𝑖 = 1.

Now we shall show that 𝐱 ∈ 𝐶.


Now we shall show that 𝐱 ∈ 𝐶.
If 𝜇𝑛+1 = 0 then 𝐱 becomes a convex linear combination of 𝐱1 , 𝐱 2 , … , 𝐱 𝑛 which by assumption,
belongs to 𝐶 and hence the result holds in this case. Also, if 𝜇𝑛+1 = 1, then the result is trivially
true.
Let 𝜇𝑛+1 be neither 0 nor 1. Then
(𝜇1 + 𝜇2 + ⋯ + 𝜇𝑛 )
𝐱 = (𝜇 𝐱 + 𝜇2 𝐱 2 + ⋯ + 𝜇𝑛 𝐱 𝑛 ) + 𝜇𝑛+1 𝐱 𝑛+1
(𝜇1 + 𝜇2 + ⋯ + 𝜇𝑛 ) 1 1
= (∑𝑛𝑖=1 𝜇𝑖 )(∑𝑛𝑖=1 𝑎𝑖 𝐱 𝑖 ) + 𝜇𝑛+1 𝐱 𝑛+1 ,
𝜇𝑖
where 𝑎𝑖 = , 𝑖 = 1,2, … , 𝑛.
𝜇1 + 𝜇2 + ⋯ + 𝜇𝑛
As each 𝜇𝑖 ≥ 0, we have 𝑎𝑖 ≥ 0
and
𝑛 𝑛
𝜇𝑖 Σ𝜇𝑖
∑ 𝑎𝑖 = ∑ ( )= =1
𝜇1 + 𝜇2 + ⋯ + 𝜇𝑛 Σ𝜇𝑖
𝑖=1 𝑖=1

Thus ∑𝑛𝑖=1 𝑎𝑖 𝐱 𝑖 = 𝐲 (say) is also a convex linear combination of 𝐱1 , 𝐱 2 , … , 𝐱 𝑛 . So, y belongs to


𝐶, by assumption.

181 | P a g e

© Department of Distance & Continuing Education, Campus of Open Learning,


School of Open Learning, University of Delhi
B.A. (Hons.) Economics

Now x = (𝜇1 + 𝜇2 + ⋯ + 𝜇𝑛 )y + 𝜇𝑛+1 𝐱 𝑛+1


Bat (𝜇1 + 𝜇2 + ⋯ + 𝜇𝑛 ) ≥ 0, 𝜇𝑛+1 ≥ 0 and (𝜇1 + 𝜇2 + ⋯ + 𝜇𝑛 ) + 𝜇𝑛+1 = 1.
It follows that 𝐱 is a convex combination of two points 𝐲 and 𝐱 𝑛+1 of 𝐶. So 𝐱 belongs to 𝐶.
Hence convex linear combination of (𝑛 + 1) points of 𝐶 also belongs to 𝐶. So, by induction
principle the result is true.
Theorem 7. The set of all feasible solutions (if not empty) of a L.P.P. is a convex set.
Proof. Let 𝑋 be the set of all feasible solutions of a L.P.P.
𝐴x = 𝐛, 𝐱 ≥ 0.
Case I. If the set 𝑋 has only one element, then 𝑋 is convex set. Hence the theorem is true in
this case.
Case II. If the set 𝑋 has at least two elements.
Let x1 and x2 be any two distinct elements in 𝑋.
and 𝐴𝑥2 = 𝑏, 𝑥2 ≥ 0.
If 𝑥3 = 𝜆𝑥1 + (1 − 𝜆)𝑥2 , 0 ≤ 𝜆 ≤ 1
then 𝐴𝑥3 = 𝐴𝜆𝑥1 + (1 − 𝜆)𝐴𝑥2
= 𝜆𝐛 + (1 − 𝜆)𝐛 = 𝐛.
Also, since 𝑥1 ≥ 0, 𝑥2 ≥ 0, 𝜆 ≥ 0,1 − 𝜆 ≥ 0, as 0 ≤ 𝜆 ≤ 1
∴ 𝐱 3 = 𝜆𝐱1 + (1 − 𝜆)x2 ≥ 0
i.e., 𝑥3 satisfies (1). Thus 𝑥3 = 𝜆𝑥1 + (1 − 𝜆)𝑥2 is also a F.S. and so belongs to set 𝑋.
But 𝑥3 is a convex combination of any two distinct points, 𝑥1 and 𝑥2 in 𝑋.
Hence by definition the set 𝑋 is a convex set.
Note. Since the convex combinations of two points are infinite in number so from the above
theorem, we conclude that:
If a given L.P.P. has two feasible solutions, then it has infinite number of feasible solutions.

182 | P a g e

© Department of Distance & Continuing Education, Campus of Open Learning,


School of Open Learning, University of Delhi
Intermediate Mathematical Methods for Economics

Theorem 8. Every basic feasible solution of the system 𝑨𝐱 = 𝐛, 𝐱 ≥ 𝟎 is an extreme point


of the convex set of feasible solutions and conversely.
Proof. To prove that every B.F.S. is an extreme point of the convex set of all feasible
solutions.
Let 𝐱 be a B.F.S. of 𝐴𝐱 = 𝐛 which is a 𝑛-component vector containing both zero (non-basic)
and non-zero (basic) variables. Let 𝐱 𝐵 and 𝐵 be the vector of 𝑚 basic variables and the matrix
of vectors associated to basic variables in the B.F.S. 𝐱 respectively, then.
𝐱 = [𝐱 𝐵 , 0]
where 0 is a null vector of (𝑛 − 𝑚) components, and 𝐴𝐱 = 𝐛 ⇒ 𝐵 ⋅ 𝐱 𝐵 = 𝐛.
Now we must prove that 𝐱 is an extreme point.
We shall prove this by using contradiction.
Suppose that 𝐱 is not an extreme point. If 𝑋 is the convex set of all feasible solutions of 𝐴𝑥 =
𝑏, then 𝑥 ∈ 𝑋.
If 𝐱 is not an extreme point then there exist two distinct points 𝑥1 and 𝑥2 in 𝜒 such that
𝑥 = 𝜆𝑥1 + (1 − 𝜆)𝑥2 , 0 < 𝜆 < 1 ------(3)
But 𝑥1 and 𝑥2 can be expressed as
𝑥1 = [𝑢1 , 𝑣1 ] and 𝑥2 = [𝑢2 , 𝑣2 ] ---(4)
where 𝑢1 and 𝑢2 are vectors of 𝑚 components of 𝑥1 and 𝑥2 respectively and 𝑣1 , 𝑣2 are (𝑛 −
𝑚) component vectors.
Substituting the values of 𝐱 and 𝐱1 , 𝐱 2 from (1) and (4) in (3), we have
[𝐱 𝐵 , 0] = 𝜆[𝐮1 , 𝐯1 ] + (1 − 𝜆)[𝐮2 , 𝐯2 ], 0 < 𝜆 < 1
= [𝜆𝐮1 + (1 − 𝜆)𝐮2 , 𝜆𝐯1 + (1 − 𝜆)𝐯2 ]
∴ 0 = 𝜆𝐯1 + (1 − 𝜆)𝐯2 , 0 < 𝜆 < 1
Now 1 > 𝜆 > 0,1 − 𝜆 > 0, and the components of 𝑣1 and 𝑣2 are ≥ 0.
The relation (5) can only be satisfied when 𝑣1 = 0 and 𝑣2 = 0
∴ x1 = [u1 , 0], x2 = [u2 , 0].
Since x1 and x2 are in 𝑋, therefore from (2) we have.
𝐴x1 = 𝐵𝐮1 = 𝐛 and 𝐴x2 = 𝐵u2 = 𝐛
183 | P a g e

© Department of Distance & Continuing Education, Campus of Open Learning,


School of Open Learning, University of Delhi
B.A. (Hons.) Economics

i.e.,
which gives 𝐱 𝐵 = 𝐮1 = 𝐮2
∴ x = x1 = x2
which is contradiction to the assumption that x1 ≠ x2
i.e., 𝐱 cannot be expressed as a convex combination of any two distinct points in the set of all
feasible solutions. Hence 𝐱 must be an extreme point.
Converse. To prove that every extreme point of the convex set of feasible solutions is a
B.F.S.
Let 𝐱 = [𝑥1 , 𝑥2 , … , 𝑥𝑛 ] be an extreme point. Now in order to prove that 𝐱 is a B.F.S., we shall
prove that the vectors associated with the positive elements of 𝐱 are L.I.
Suppose that 𝑘-components (variables) in x are non-zero and (𝑛 − 𝑘) components are zero.
We can assume these components as the first 𝑘 components of 𝐱.
𝑘

∴ ∑ 𝑥𝑖 𝛼𝑖 = 𝐛, 𝑥𝑖 > 0, 𝑖 = 1,2, … , 𝑘,
𝑖=1

where 𝛼𝑖 is the column vector in 𝐴 associated to the 𝑖 th variable in 𝐱.


If possible, let the column vectors 𝛼1 , 𝛼2 , … , 𝛼𝑘 of matrix 𝐴 be 𝐿.D. Then there exist some
scalars 𝜆𝑖 (𝑖 = 1,2, … , 𝑘) with at least one of them non-zero, s.t.,
𝑘

∑ 𝜆𝑖 𝛼𝑖 = 0
𝑖=1

From (6) and (7), for some arbitrary 𝛿 > 0, we have


𝑘 𝑘

∑ 𝑥𝑖 𝛼𝑖 ± 𝛿 ∑ 𝜆𝑖 𝛼𝑖 = 𝐛
𝑖=1 𝑖=1
𝑘

∑ (𝑥𝑖 ± 𝛿𝜆𝑖 )𝛼𝑖 = 𝐛


𝑖=1

from which it is obvious that the two points


satisfy

184 | P a g e

© Department of Distance & Continuing Education, Campus of Open Learning,


School of Open Learning, University of Delhi
Intermediate Mathematical Methods for Economics

𝐱1∗ = [𝑥1 + 𝛿𝜆1 , 𝑥2 + 𝛿𝜆2 , … , 𝑥𝑘 + 𝛿𝜆𝑘 , 0,0, … … . ,0]


𝐱 2∗ = [𝑥1 − 𝛿𝜆1 , 𝑥2 − 𝛿𝜆2 , … , 𝑥𝑘 − 𝛿𝜆𝑘 , 0,0, … … . . ,0]
𝐴𝐱 = 𝐛.
Also, since 𝑥𝑖 > 0, therefore, taking 𝛿 s.t.
𝑥𝑖
0 < 𝛿 < Min𝑖 { } , 𝜆 ≠ 0, 𝑖 = 1,2, … , 𝑘.
|𝜆𝑖 | 𝑖
We conclude that first 𝑘 components of x1 ∗ and x2 ∗ are always positive. But the remaining
components of x1 ∗ and x2 ∗ are zero, which follows that x1 ∗ and x2 ∗ are feasible solutions
different from 𝐱.
Now 𝐱1∗ + 𝐱 2∗ = 2[𝑥1 , 𝑥2 , … , 𝑥𝑘 , 0,0, … ,0]
or
1 ∗ 1 ∗
𝐱 + 𝐱 = [𝑥1 , 𝑥2 , … , 𝑥𝑘 , 0,0, … ,0] = 𝐱
2 1 2 2
or
1
𝐱 = 𝜆𝐱1∗ + (1 − 𝜆)𝐱 2∗ where 𝜆 =
2
i.e., 𝐱 can be expressed as a convex combination of two distinct feasible solutions 𝐱1∗ and 𝐱 2 ∗ .
But this is a contradiction as 𝐱 is an extreme point. Hence the vectors 𝛼1 , 𝛼2 , … , 𝛼𝑘 are L.I.
Further, we know that at most 𝑚 vectors of 𝐸 𝑚 can be L.I. So 𝛼1 , 𝛼2 , … , 𝛼𝑘 cannot be more
than 𝑚 and hence the extreme point x will have atmost 𝑚 non-zero variables, i.e., atleast (𝑛 −
𝑚) variables will be zero.
Thus 𝐱 is a B.F.S. Hence, every extreme point of the convex set of feasible solutions is a B.F.S.
Cor. 1. The extreme points of the convex set of feasible solutions are finite in number.
From the above theorem, we conclude that there is only one extreme point for a given B.F.S.
and vice versa. That is there is one-to-one correspondence between the extreme points and the
B.F. solutions in the absence of degeneracy. Also, in case of degeneracy corresponding to an
extreme point with the number of non-zero variables less than 𝑚, we can form more than one
degenerate B.F.S. Hence the number of extreme points of the feasible region is finite and it
cannot exceed the number of its B.F. solutions.
Cor. 2. An extreme point can have at most 𝑚-positive 𝑥𝑖 's where 𝑚 is the number of
constraints.

185 | P a g e

© Department of Distance & Continuing Education, Campus of Open Learning,


School of Open Learning, University of Delhi
B.A. (Hons.) Economics

Cor. 3. In an extreme point, vectors associated to the positive 𝑥𝑖 's are L.I. Theorem 9. If the
convex set of the feasible solutions of 𝐴𝐱 = 𝐛, 𝐱 ≥ 0 is a convex polyhedron, then atleast one
of the extreme points gives an optimal solution.
Proof. In the Cor. 1 of last theorem, we have proved that the extreme points of the convex set
of feasible solutions of 𝐴𝐱 = 𝐛, 𝐱 ≥ 𝟎 are finite in number.
Let 𝑥1 , 𝑥2 , … , 𝑥𝑘 be the extreme points of the set 𝑋 of all the feasible solutions of 𝐴𝐱 = 𝐛, 𝐱 ≥
0. Let 𝑍 be the objective function which is to be maximized be given by
𝑍 = 𝐜𝐱
If 𝐱 ∗ ∈ 𝑋 is the optimal solution, then
Max, 𝑍 = 𝐜𝐱 ∗ .
Now if 𝐱 ∗ is an extreme point, then the theorem is proved.
Now if x ∗ is not an extreme point in 𝑋, then since 𝑋 is convex polyhedron, therefore 𝐱 ∗ can
be expressed as a convex combination of the extreme points of 𝑋,
i.e.,
𝑘

𝐱∗ = 𝜆1 𝐱1 + 𝜆2 𝐱 2 + ⋯ + 𝜆𝑘 𝐱 𝑘 = ∑ 𝜆𝑖 ⋅ 𝐱 𝑖 , 𝜆𝑖 ≥ 0 and Σ𝜆𝑖 = 1
𝑖=1
∴ 𝑍∗ = 𝐜𝐱𝐱 ∗ = 𝐜(𝜆1 𝐱1 + 𝜆2 𝐱 2 + ⋯ + 𝜆𝑘 𝐱 𝑘 )
= (𝜆1 𝑐𝐱1 + 𝜆2 𝑐𝐱 2 + ⋯ + 𝜆𝑘 𝑐x𝑘 )
If maximum of 𝐜𝐱 𝐢 is 𝐜𝐱 𝑝 , then

𝑍 ∗ ≤ (𝜆1 + 𝜆2 + ⋯ + 𝜆𝑘 ) ⋅ 𝐜𝐱 𝑝

or 𝑍 ∗ ≤ 𝐜𝐱 𝑝 .

But 𝑍 ∗ is the maximum value of 𝑍. Therefore,


𝑍 ∗ = 𝐜𝐱 𝑝 or 𝐜𝐱𝐱 ∗ = 𝐜𝐱 𝑝

i.e., 𝐱 ∗ = x𝑝 (one of the extreme points).

Hence the optimal solution is attained at the extreme point.


This proves the theorem.

186 | P a g e

© Department of Distance & Continuing Education, Campus of Open Learning,


School of Open Learning, University of Delhi
Intermediate Mathematical Methods for Economics

Theorem 10. If the objective function of a L.P.P. assumes its optimal value at more than
one extreme point, then every convex combination of these extreme points gives the
optimal value of the objective function.
Proof. Let us consider the L.P.P. as follows:
Max. 𝑍 = 𝐜𝐱
s.t. 𝐴𝐱 = 𝐛, 𝐱 ≥ 0.
Let 𝑥1 , 𝑥2 , … , 𝑥𝑘 be the extreme points of the feasible region. If the objective function 𝑍
assumes its optimal value 𝑍 ∗ at the extreme points, 𝐱1 , 𝐱 2 , … , 𝐱 𝑝 , (𝑝 ≤ 𝑘) then.

𝑍 ∗ = 𝐜𝑥1 + 𝐜𝐱 𝟐 + ⋯ + 𝐜𝐱 𝐩

If 𝑥0 is the convex combination of the extreme points 𝑥1 , 𝑥2 , … , 𝑥𝑝 , then.


𝑝

x0 = 𝜆1 x1 + 𝜆2 x2 + ⋯ + 𝜆𝑝 x𝑝 , 𝜆𝑖 ≥ 0, ∑ 𝜆𝑖 = 1
𝑖=1

cx0 = c[𝜆1 𝐱1 + 𝜆2 𝐱 2 + ⋯ + 𝜆𝑝 𝐱 𝑝 ]
= 𝜆1 𝑐𝑥1 + 𝜆2 𝑐𝑥2 + ⋯ + 𝜆𝑝 𝑐𝑥𝑝
= 𝜆1 𝑍 ∗ + 𝜆2 𝑍 ∗ + ⋯ + 𝜆𝑝 𝑍 ∗
= (𝜆1 + 𝜆2 + ⋯ + 𝜆𝑝 )𝑍 ∗ = 𝑍 ∗

[∵ ∑ 𝜆𝑖 = 1]
𝑖=1

Hence the optimal value 𝑍 ∗ is alse attained at 𝑥0 which is the convex comhination of the
extreme poigts at which optimal value occurs, Hence the theorem.
11.4 SUMMARY
The main points which we have covered in these lessons are what is Convex and
Quasiconvexity functions with its properties.
11.5 GLOSSARY
Motivation: These Problems are very useful in real life, and we can use it in data science,
economics as well as social science.

187 | P a g e

© Department of Distance & Continuing Education, Campus of Open Learning,


School of Open Learning, University of Delhi
B.A. (Hons.) Economics

Attention: Think how the best Convexity and Quasiconvexity are useful in real world
problems.
11.6 REFERENCES

• Sydsaeter, K., Hammond, P. (2002). Mathematics for economics analysis. Pearson


Education.

• Hoy, M., Livernois, J., McKenna, C., Rees, R., Stengos, T, (2001). Mathematics for
Economics, Prentice-Hall India.

11.7 SUGGESTED READINGS

• A.R Vasishtha, Dr. R. K Gupta, Dr. Rajesh Kumar, Dr. Rajiv Atri, Dr Hemlata
Vasishtha, Linear programming, Krishna Publication, 6th Edition.
• Hamdy A. Taha, Operation Research an Introduction, Pearson Publication, 6th Edition
• D. Nagesh Kumar, Optimization Methods: Optimization using Calculus-Convexity and
Concavity, NPTEL, IISC Bangalore.

188 | P a g e

© Department of Distance & Continuing Education, Campus of Open Learning,


School of Open Learning, University of Delhi
Intermediate Mathematical Methods for Economics

LESSON-12
UNCONSTRAINED OPTIMIZATION

STRUCTURE

12.1 Learning Objectives


12.2 Introduction
12.3 Unconstrained Optimization
12.3.1 Maxima and Minima of Two Variable
12.3.2 Maxima and Minima of Several Independent Variable
12.3.3 Lagrange Methods of Undetermined Multipliers
12.4 Summary
12.5 Glossary
12.6 References
12.7 Suggested Readings

12.1 LEARNING OBJECTIVES

In this chapter our main objective is to understand maxima and minima of function of two and
more than two variable which is highly use in optimization.

12.2 INTRODUCTION

Let 𝑓(𝑥, 𝑦) be any function of two independent variables 𝑥 and 𝑦 supposed to be continuous
for all values of these variables in the neighbourhood of their values 𝑎 and 𝑏 respectively. Then
𝑓(𝑎, 𝑏) is said to be a maximum or 𝑎 minimum value of 𝑓(𝑥, 𝑦) according as 𝑓(𝑎 + ℎ, 𝑏 + 𝑘)
is less or greater than 𝑓(𝑎, 𝑏) for all sufficiently small independent values of ℎ and 𝑘, positive
or negative, provided both are not equal to zero.
NECESSARY CONDITIONS FOR THE EXISTENCE OF A MAXIIMUM OR A
MINIMUM OF 𝒇(𝒙, 𝒚) AT 𝒙 = 𝒂, 𝒚 = 𝒃.
From the definition it is obvious that we shall have a maximum or a minimum of 𝑓(𝑥, 𝑦) at
𝑥 = 𝑎, 𝑦 = 𝑏 if the expression 𝑓(𝑎 + ℎ, 𝑏 + 𝑘) − 𝑓(𝑎, 𝑏) is of invariable sign for all
sufficiently small independent values of ℎ and 𝑘 provided both of them are not equal to zero.

189 | P a g e

© Department of Distance & Continuing Education, Campus of Open Learning,


School of Open Learning, University of Delhi
B.A. (Hons.) Economics

If the sign of 𝑓(𝑎 + ℎ, 𝑏 + 𝑘) − 𝑓(𝑎, 𝑏) is negative, we shall have a maximum of 𝑓(𝑥, 𝑦) at


𝑥 = 𝑎, 𝑦 = 𝑏. If it is positive, 𝑓(𝑥, 𝑦) has a minimum at 𝑥 ≠ 𝑎, 𝑦 = 𝑏.
By Taylor's theorem for two variables, we have
∂𝑓 ∂𝑓 𝑥 = 𝑎,
𝑓(𝑎 + ℎ, 𝑏 + 𝑘) = 𝑓(𝑎, 𝑏) + (ℎ
+ 𝑘 )𝑦 = 𝑏
∂𝑥 ∂𝑦
2 2 2
1 ∂ 𝑓 ∂ 𝑓 ∂ 𝑓
+ + (ℎ2 2 + 2ℎ𝑘 + 𝑘2 2) +⋯
2! ∂𝑥 ∂𝑥 ∂𝑦 ∂𝑦 𝑥=𝑎,

∂𝑓 𝑥 = 𝑎 ∂𝑓 𝑥 = 𝑎
∴ 𝑓(𝑎 + ℎ, 𝑏 + 𝑘) − 𝑓(𝑎, 𝑏) = ℎ ( ) 𝑦 = 𝑏 𝑦 (∂𝑦) 𝑦 = 𝑏
∂𝑥

+terms of the second and higher orders in ℎ and 𝑘.


By taking ℎ and 𝑘, sufficiently small, the first-degree terms in ℎ and 𝑘 can be made to govern
the sign of the right hand side and therefore of the left hand side of (1).
Thus, the sign of [𝑓(𝑎 + ℎ, 𝑏 + 𝑘) − 𝑓(𝑎, 𝑏)]
∂𝑓
ℎ (∂𝑥 ) = 𝑎
𝑥
= the sign of 𝑦 = 𝑏 ∂𝑓
+𝑘 (∂𝑦) = 𝑎
𝑥
[ 𝑦 = 𝑏]
∂𝑓
Taking 𝑘 = 0, we find that if (∂𝑥 ) ≠ 0, the right-hand side of (2) changes sign when ℎ
𝑥=𝑎,𝑦=𝑏
changes sign. Therefore 𝑓(𝑥, 𝑦) cannot have a maximum or minimum at
∂𝑓
x=a, y=b if (∂𝑥) ≠0
𝑥=𝑎,𝑦=𝑏

Similarly taking ℎ = 0, we can see that 𝑓(𝑥, 𝑦) cannot have a maximum or a minimum at
∂𝑓
𝑥 = 𝑎, 𝑦 = 𝑏 if (∂𝑦) ≠ 0.
𝑥=𝑎 𝑦=𝑏

Thus, a set of necessary conditions that 𝑓(𝑥, 𝑦) should have a maximum or a minimum at
𝑥 = 𝑎, 𝑦 = 𝑏 is that
∂𝑓 ∂𝑓
( ) = 0 and ( ) = 0.
∂𝑥 𝑥=𝑎 ∂𝑦 𝑥=𝑎
𝑦=𝑏𝑦=𝑏

190 | P a g e

© Department of Distance & Continuing Education, Campus of Open Learning,


School of Open Learning, University of Delhi
Intermediate Mathematical Methods for Economics

The above conditions are necessary but not sufficient for the existence of maxima or minima.
STATIONARY, EXTREME AND SADDLE POINTS.
A point (𝑎, 𝑏) is called 𝑎 stationary point, if the first order partial derivatives of the function
𝑓(𝑥, 𝑦) vanish at that point. A stationary point which is either a maximum or a minimum is
called an extreme point and the value of the function at the point is called an extreme value. A
stationary point is not necessarily an extreme point. Thus, a stationary point may be a maximum
or a minimum or neither of these two. To decide whether a point is really an extreme point, a
further investigation is necessary.
A stationary point which is neither a maximum nor a minimum is called a saddle point.

12.3 UNCONSTRAINED OPTIMIZATION


12.3.1 Maxima and Minima of Two variable:
SUFFICIENT CONDITION FOR MAXIMA OR MINIMA.
∂2 𝑓 ∂2 𝑓 ∂2 𝑓
Let 𝑟 = (𝑎𝑥 2 ) , 𝑠 = (∂𝑥 ∂𝑦) and 𝑡 = (∂𝑦 2 ) 𝑦 = 𝑏 𝑦 = 𝑏 𝑦 = 𝑏.
𝑥=𝑎 𝑥=𝑎 𝑥=𝑎
∂𝑓 ∂𝑓
If (∂𝑥 ) = 0 and (∂𝑦) = 0, i.e. if the necessary conditions for the 𝑦 = 𝑏 𝑦=𝑏
𝑥=𝑎 𝑥=𝑎
existence of maxima or minima are satisfied, we have
1
𝑓(𝑎 + ℎ, 𝑏 + 𝑘) − 𝑓(𝑎, 𝑏) = (𝑟ℎ2 + 2𝑠ℎ𝑘 + 𝑡𝑘 2 ) + 𝑅3
2!
where 𝑅3 consists of terms of third and higher orders in ℎ and 𝑘.
For sufficiently small values of ℎ and 𝑘, the sign of
1
(𝑟ℎ2 + 2𝑠ℎ𝑘 + 𝑡𝑘 2 ) + 𝑅3
2
is the same as that of 𝑟ℎ2 + 2𝑠ℎ𝑘 + 𝑡𝑘 2 .
Now the following three different cases arise:
Case I. 𝒓𝒕 − 𝒔𝟐 > 𝟎. In this case obviously neither 𝑟 nor 𝑡 can be zero. Therefore, we write.
1 2 2
𝑟ℎ2 + 2𝑠ℎ𝑘 + 𝑡𝑘 2 = [𝑟 ℎ + 2𝑠𝑟ℎ𝑘 + 𝑟𝑡𝑘 2 ]
𝑟
1
= [(𝑟ℎ + 𝑠𝑘)2 + (𝑟𝑡 − 𝑠 2 )𝑘 2 ].
𝑟

191 | P a g e

© Department of Distance & Continuing Education, Campus of Open Learning,


School of Open Learning, University of Delhi
B.A. (Hons.) Economics

Since 𝑟𝑡 − 𝑠 2 is positive, therefore (𝑟ℎ + 𝑠𝑘)2 + (𝑟𝑡 − 𝑠 2 )𝑘 2 is positive for all values of ℎ
and 𝑘 except when 𝑟ℎ + 𝑠𝑘 = 0, 𝑘 = 0 i.e., when ℎ = 0, 𝑘 = 0 which is obviously not
possible.
Thus in this case the expression 𝑟ℎ2 + 2𝑠ℎ𝑘 + 𝑡𝑘 2 will have the same sign for all values of ℎ
and 𝑘. This sign is determined by the sign of 𝑟.
Thus 𝑓(𝑥, 𝑦) will have a maximum or a minimum at 𝑥 = 𝑎, 𝑦 = 𝑏 if 𝑟𝑡 > 𝑠 2 . Further 𝑓(𝑥, 𝑦)
is a maximum or a minimum according as 𝑟 is negative or positive.
Case II. 𝒓𝒕 − 𝒔𝟐 < 𝟎. In this case if 𝑟 ≠ 0, we can write
1
𝑟ℎ2 + 2𝑠ℎ𝑘 + 𝑡𝑘 2 = [(𝑟ℎ + 𝑠𝑘)2 + (𝑟𝑡 − 𝑠 2 )𝑘 2 ].
𝑟
If 𝑘 = 0, ℎ ≠ 0, the sign of this expression will be the same as that of 𝑟. But if 𝑘 ≠ 0, 𝑟ℎ +
𝑠𝑘 = 0, the sign of this expression will be opposite to that of 𝑟 since 𝑟𝑡 − 𝑠 2 is negative. Thus,
in this case the expression 𝑟ℎ2 + 2𝑠ℎ𝑘 + 𝑡𝑘 2 is not of invariable sign.
A similar argument can be given if 𝑡 ≠ 0.
In case 𝑟 = 0 as well as 𝑡 = 0, we have
𝑟ℎ2 + 2𝑠ℎ𝑘 + 𝑡𝑘 2 = 2𝑠ℎ𝑘
which obviously does not keep the same sign for all values of ℎ and 𝑘.
Thus 𝑓(𝑥, 𝑦) will have neither a maximum nor a minimum at 𝑥 = 𝑎, 𝑦 = 𝑏, if 𝑟𝑡 < 𝑠 2
Case IIL. 𝒓𝒕 − 𝒔𝟐 = 𝟎. If 𝑟 ≠ 0, we can write
1
𝑟ℎ2 + 2𝑠ℎ𝑘 + 𝑡𝑘 2 = [(𝑟ℎ + 𝑠𝑘)2 + (𝑟𝑡 − 𝑠 2 )𝑘 2 ]
𝑟
1
= (𝑟ℎ + 𝑠𝑘)2 .
𝑟
[∵ 𝑟𝑡 − 𝑠 2 = 0]
This expression becomes zero when 𝑟ℎ + 𝑠𝑘 = 0. Therefore, the nature of the sign of
𝑓(𝑎 + ℎ, 𝑏 + 𝑘) − 𝑓(𝑎, 𝑏)
depends upon the consideration of 𝑅3 . The case is, therefore, doubtful.
In case 𝑟 = 0, we must have 𝑠 = 0, because of the condition 𝑟𝑡 − 𝑠 2 = 0.
∴ 𝑟ℎ2 + 2𝑠ℎ𝑘 + 𝑡𝑘 2 = 𝑡𝑘 2 ,
192 | P a g e

© Department of Distance & Continuing Education, Campus of Open Learning,


School of Open Learning, University of Delhi
Intermediate Mathematical Methods for Economics

which is zero when 𝑘 = 0 whatever ℎ may be. The case is again doubtful.
Thus, if 𝑟𝑡 − 𝑠 2 = 0, the case is doubtful and further investigation is needed determine whether
𝑓(𝑥, 𝑦) is a maximum or a minimum at 𝑥 =a, y=b or not.
$5. WORKING RULE FOR MAXIMA
Suppose f (x, y) is a given function of 𝑥 and 𝑦. Find ∂𝑓/ ∂𝑥 and ∂𝑓/ ∂𝑦 and solve the Suppose
𝑓(𝑥, 𝑦) is a given function simultaneous equation ∂𝑓/ ∂𝑥 = 0 and ∂𝑓/ ∂𝑦 = 0. In order to
solve these equations, we may either eliminate one of the variables, or factorise the equations.
In the later case each factor of the first equation must be solved in conjunction with each factor
of the second equation. Suppose solving these equations we get the pairs of values of 𝑥 and 𝑦
as (𝑎1 , 𝑏1 ), (𝑎2 , 𝑏2 ) etc. Then all these pairs of roots will give stationary values of 𝑓(𝑥, 𝑦).
To discuss the maximum or minimum at 𝑥 = 𝑎1 , 𝑦 = 𝑏1 , we should find
∂2 𝑢 ∂2 𝑢 ∂2 𝑢
𝑟=( ) , 𝑠 = ( ) , 𝑡 = ( )
∂𝑥 2 𝑥=𝑎 ∂𝑥 ∂𝑦 𝑥=𝑎 ∂𝑦 2 𝑥=𝑎
1 1 1
𝑦 = 𝑏1 𝑦 = 𝑏1 𝑦 = 𝑏1 .

Then calculate 𝑟𝑡 − 𝑠 2
If 𝑟𝑡 − 𝑠 2 > 0 and 𝑟 is negative, 𝑓(𝑥, 𝑦) is maximum at 𝑥 = 𝑎1 , 𝑦 = 𝑏1 .
If 𝑟𝑡 − 𝑠 2 > 0 and 𝑟 is positive, 𝑓(𝑥, 𝑦) is minimum at 𝑥 = 𝑎1 , 𝑦 = 𝑏1.
If 𝑟𝑡 − 𝑠 2 < 0, 𝑓(𝑥, 𝑦) is neither maximum nor minimum at 𝑥 = 𝑎1 , 𝑦 = 𝑏1 .
In this case the function 𝑧 = 𝑓(𝑥, 𝑦) is stationary at 𝑥 = 𝑎, 𝑦 = 𝑏 but the stationary value
𝑓(𝑎, 𝑏). is neither maximum nor minimum. Hence, (𝑥 = 𝑎, 𝑦 = 𝑏, 𝑧 = (𝑎, 𝑏)) is a saddle
point of the surface 𝑧 = 𝑓(𝑥, 𝑦).
If 𝑟𝑡 − 𝑠 2 = 0, the case is doubtful and further investigation will be required to decide it. We
shall leave this case.
SOLVED EXAMPLES
Ex. 1. Discuss the maximum or minimum values of 𝑢 where.
𝑢 = 2𝑎2 𝑥𝑦 − 3𝑎𝑥 2 𝑦 − 𝑎𝑦 3 + 𝑥 3 𝑦 + 𝑥𝑦 3
Sol. We have ∂𝑢/ ∂𝑥 = 2𝑎2 𝑦 − 6𝑎𝑥𝑦 + 3𝑥 2 𝑦 + 𝑦 3 , and ∂𝑢/ ∂𝑦 = 2𝑎2 𝑥 − 3𝑎𝑥 2 −
3𝑎𝑦 2 + 𝑥˙ 3 + 3𝑥𝑦 2 .

193 | P a g e

© Department of Distance & Continuing Education, Campus of Open Learning,


School of Open Learning, University of Delhi
B.A. (Hons.) Economics

Also 𝑟 = ∂2 𝑢/ ∂𝑥 2 = 6𝑎𝑦 + 6𝑥𝑦,


𝑠 = ∂2 𝑢/ ∂𝑥 ∂𝑦 = 2𝑎2 − 6𝑎𝑥 + 3𝑥 2 + 3𝑦 2 ,
and
𝑡 = ∂2 𝑢/ ∂𝑦 2 = −6𝑎𝑦 + 6𝑥𝑦.
For a maximum or minimum of 𝑢, we have ∂𝑢/ ∂𝑥 = 0 and ∂𝑢/ ∂𝑦 = 0
Thus, we have
and
𝑦(2𝑎2 − 6𝑎𝑥 + 3𝑥 2 + 𝑦 2 ) = 0
}
−3𝑎𝑥 2 − 3𝑎𝑦 2 + 𝑥 3 + 3𝑥𝑦 2 = 0
Therefore, we have to consider the pairs of equations, viz.,
and
2𝑎2 𝑥 − 3𝑎𝑥 2 − 3𝑎𝑦 2 + 𝑥 3 + 3𝑥𝑦 2 = 0
2𝑎2 − 6𝑎𝑥 + 3𝑥 2 + 𝑦 2 = 0}
2𝑎2 𝑥 − 3𝑎𝑥 2 − 3𝑎𝑦 2 + 𝑥 3 + 3𝑥𝑦 2 = 0
Putting 𝑦 = 0 in the second equation of the pair (1), we get
2𝑎2 𝑥 − 3𝑎𝑥 2 + 𝑥 3 = 0 i.e., 𝑥(𝑥 2 − 3𝑎𝑥 + 2𝑎2 ) = 0
𝑥(𝑥 − 𝑎)(𝑥 − 2𝑎) = 0 i.e., 𝑥 = 0, 𝑥 = 𝑎, 𝑥 = 2𝑎.
Thus, the pair (1) gives the following values of 𝑥 and 𝑦 :
𝑥 = 0, 𝑦 = 0; 𝑥 = 𝑎, 𝑦 = 0; 𝑥 = 2𝑎, 𝑦 = 0.
Multiplying the first equation of the pair (2) by 𝑥 and subtracting it from the second equation
of the pair, we get
3𝑎𝑥 2 − 3𝑎𝑦 2 − 2𝑥 3 + 2𝑥𝑦 2 = 0 or (𝑥 2 − 𝑦 2 )(3𝑎 − 2𝑥) = 0.
3
∴ 𝑥 = 𝑎 and 𝑥 = ±𝑦.
2
3 1
When 𝑥 = 2 𝑎, the first equation of the pair (2) gives 𝑦 = ± 2 𝑎.
1
When 𝑥 = 𝑦, we have 2𝑎2 − 6𝑎𝑦 + 4𝑦 2 = 0 i.e., 𝑦 = 𝑎, 2 𝑎.
1
Also when 𝑥 = −𝑦, we have 2𝑎2 + 6𝑎𝑦 + 4𝑦 2 = 0 i.e., 𝑦 = −𝑎, − 𝑎.
2
Thus in all we get the following pairs of values of 𝑥 and 𝑦 which make the function 𝑢
stationary :

194 | P a g e

© Department of Distance & Continuing Education, Campus of Open Learning,


School of Open Learning, University of Delhi
Intermediate Mathematical Methods for Economics

3 1 3 1
(0,0), (𝑎, 0), (2𝑎, 0), ( 𝑎, 𝑎) , ( 𝑎, − 𝑎)
2 2 2 2
1 1 1 1
(𝑎, 𝑎), ( 𝑎, 𝑎) , (𝑎, −𝑎), ( 𝑎, − 𝑎) .
2 2 2 2
For (𝟎, 𝟎),
𝑟 = 0, 𝑠 = 2𝑎2 , 𝑡 = 0 so that 𝑟𝑡 − 𝑠 2 is negative.
Therefore, we have neither a maximum nor a minimum of 𝑢 at (0,0).
Similarly, we can show that 𝑢 has neither a maximum nor a minimum at
(𝑎, 0), (2𝑎, 0), (𝑎, 𝑎), (𝑎, −𝑎).
For (𝟑𝒂/𝟐, 𝒂/𝟐),
3 1 3
𝑟 = 2 𝑎2 , 𝑠 = 2 𝑎2 , 𝑡 = 2 𝑎2 so that 𝑟𝑡 − 𝑠 2 is positive. Since 𝑟 is positive, therefore 𝑢 has
minimum at this point.
1 1
Similarly, we can show that 𝑢 has a maximum at (2 𝑎, − 2 𝑎).

For (𝟑𝒂/𝟐, −𝒂/𝟐),


3 1 3
𝑟 = − 2 𝑎2 , 𝑠 = − 2 𝑎2 , 𝑡 = − 2 𝑎2 so that 𝑟𝑡 − 𝑠 2 is positive. Since 𝑟 is negative, therefore 𝑢
has a maximum at this point.
Similarly, we can show that 𝑢 has a maximum at (𝑎/2, 𝑎/2).
Ex. 2 (a). Find the extreme values of 𝑥𝑦(𝑎 − 𝑥 − 𝑦).
Sol, Let 𝑢 = 𝑥𝑦(𝑎 − 𝑥 − 𝑦).
∂𝑢
Then = 𝑎𝑦 − 2𝑥𝑦 − 𝑦 2
∂𝑥

And
Also
∂𝑢
= 𝑎𝑥 − 𝑥 2 − 2𝑥𝑦.
∂𝑦
∂2 𝑢 ∂2 𝑢
𝑟 = 2 = −2𝑦, 𝑠 = = 𝑎 − 2𝑥 − 2𝑦
∂𝑥 ∂𝑦 ∂𝑥
and

195 | P a g e

© Department of Distance & Continuing Education, Campus of Open Learning,


School of Open Learning, University of Delhi
B.A. (Hons.) Economics

For a maximum or minimum of 𝑢, we have


∂𝑢/ ∂𝑥 = 0 and ∂𝑢/ ∂𝑦 = 0.
Thus, we have
𝑎𝑦 − 2𝑥𝑦 − 𝑦 2 = 0
}
𝑎𝑥 − 𝑥 2 − 2𝑥𝑦 = 0
These equations can be written as
𝑦(𝑎 − 2𝑥 − 𝑦) = 0, 𝑥(𝑎 − 𝑥 − 2𝑦) = 0
so that we have to consider the four pairs of equations, viz.,
𝑦 = 0, 𝑥 = 0; 𝑎 − 2𝑥 − 𝑦 = 0, 𝑥 = 0; 𝑦 = 0, 𝑎 − 𝑥 − 2𝑦 = 0
𝑎 − 2𝑥 − 𝑦 = 0, 𝑎 − 𝑥 − 2𝑦 = 0
Solving these, we get the following pairs of values of 𝑥 and 𝑦 which make the function
stationary:
1 1
(0,0), (0, 𝑎), (𝑎, 0), ( 𝑎, 𝑎)
3 3
For (𝟎, 𝟎),
𝑟 = 0, 𝑠 = 𝑎, 𝑡 = 0 so that 𝑟𝑡 − 𝑠 2 is negative.
Therefore, we have neither a maximum nor a minimum of 𝑢 at (0,0).
For (𝟎, 𝒂),
𝑟 = −2𝑎, 𝑠 = −𝑎, 𝑡 = 0 so that 𝑟𝑡 − 𝑠 2 is negative.
Therefore 𝑢 has not an extreme value at (0, 𝑎).
Similarly, we may show that 𝑢 has not an extreme value at (𝑎, 0).
𝟏 𝟏
For (𝟑 𝒂, 𝟑 𝒂),
2 1 2
𝑟 = − 3 𝑎, 𝑠 = − 3 𝑎, 𝑡 = − 3 𝑎 so that 𝑟𝑡 − 𝑠 2 is positive.
1 1
Therefore 𝑢 has an extreme value at (3 𝑎, 3 𝑎) and it will be a maximum or minimum according
as, 𝑟 is negative or positive i.e., according as, 𝑎 is positive or negative.
∴ the extreme value of 𝑢 is (1/27)𝑎3.

196 | P a g e

© Department of Distance & Continuing Education, Campus of Open Learning,


School of Open Learning, University of Delhi
Intermediate Mathematical Methods for Economics

Ex. 3. Show that the distance 𝑙 of any point (𝑥, 𝑦, 𝑧) on the plane 2𝑥 + 3𝑦 − 𝑧 = 12 from the
origin is given by

𝑙 = √ [𝑥 2 + 𝑦 2 + (2𝑥 + 3𝑦 − 12)2 ]
Hence find the point on the plane that is nearest to the origin.

Sol. If 𝑙 is the distance from (0,0,0) of any point (𝑥, 𝑦, 𝑧), then 𝑙 = √ (𝑥 2 + 𝑦 2 + 𝑧 2 ). If the
point (𝑥, 𝑦, 𝑧) lies on the plane 2𝑥 + 3𝑦 − 𝑧 = 12, then 𝑙 = √ [𝑥 2 + 𝑦 2 + (2𝑥 + 3𝑦 −
12)2 ].
[∵ 𝑧 = 2𝑥 + 3𝑦 − 12, from the equation of the plane].
∴ 𝑙2 = 𝑥 2 + 𝑦 2 + (2𝑥 + 3𝑦 − 12)2
= 5𝑥 2 + 10𝑦 2 + 12𝑥𝑦 − 48𝑥 − 72𝑦 + 144 = 𝑢, say.
Now 𝑙 is maximum or minimum according as 𝑙 2 i.e., 𝑢 is maximum or minimum. For a
maximum or minimum of 𝑢, we have
∂𝑢
= 10𝑥 + 12𝑦 − 48 = 0
∂𝑥
∂𝑢
= 20𝑦 + 12𝑥 − 72 = 0
∂𝑦
Solving these equations, we get
𝑥 = 12/7, and 𝑦 = 18/7.
∂2 𝑢 ∂2 𝑢
Also 𝑟 = ∂𝑥 2 = 10, 𝑠 = ∂𝑥 ∂𝑦 = 12,

∂2 𝑢
and 𝑡 = ∂𝑦 2 = 20.

∴ 𝑟𝑡 − 𝑠 2 = 10 × 20 − 122 = + ive. Since 𝑟𝑡 − 𝑠 2 > 0 and 𝑟 > 0, therefore 𝑢 is minimum


and hence 𝑙 is minimum when 𝑥 = 12/7 and 𝑦 = 18/7. Putting these values of 𝑥 and 𝑦 in the
equation of the plane, we get
𝑧 = 2 ⋅ (12/7) + 3 ⋅ (18/7) − 12 = −6/7
Therefore, the required point is (12/7,18/7, −6/7).
Ex. 4. Locate the stationary points of 𝑥 4 + 𝑦 4 − 2𝑥 2 + 4𝑥𝑦 − 2𝑦 2 and determine their
nature.

197 | P a g e

© Department of Distance & Continuing Education, Campus of Open Learning,


School of Open Learning, University of Delhi
B.A. (Hons.) Economics

Sol. Let 𝑢 = 𝑥 4 + 𝑦 4 − 2𝑥 2 + 4𝑥𝑦 − 2𝑦 2 .


∂𝑢
Then = 4𝑥 3 − 4𝑥 + 4𝑦
∂𝑥
∂𝑢
and = 4𝑦 3 + 4𝑥 − 4𝑦
∂𝑦
The stationary points are given by
∂𝑢
= 0 i.e., 4𝑥 3 − 4𝑥 + 4𝑦 = 0
∂𝑥
∂𝑢
= 0 i.e., 4𝑦 3 + 4𝑥 − 4𝑦 = 0
∂𝑦
Now we shall find the points (𝑥, 𝑦) satisfying the simultaneous equations (1) and
(2).
Adding (1) and (2), we get
4𝑥 3 + 4𝑦 3 = 0 i.e., 𝑥 3 + 𝑦 3 = 0
i.e., (𝑥 + 𝑦)(𝑥 2 − 𝑥𝑦 + 𝑦 2 ) = 0.
∴ either 𝑥 + 𝑦 = 0
or 𝑥 2 − 𝑥𝑦 + 𝑦 2 = 0.
First we solve the simultaneous equations (1) and (3).
From (3), we have 𝑦 = −𝑥.
Putting 𝑦 = −𝑥 in (1), we get
4𝑥 3 − 8𝑥 = 0 i.e., 𝑥 3 − 2𝑥 = 0 i.e., 𝑥(𝑥 2 − 2) = 0

i.e., 𝑥 = 0, √2 or −√2.

The corresponding values of 𝑦 are 𝑦 = 0, −√2, √2.

Thus, the points (0,0), (√2, −√2) and (−√2, √2) satisfy (1) and (2).
If we solve the equations (1) and (4), we get (0,0) as the only real solution, Hence the
function 𝑢 is stationary at the points

(0,0), (√2, −√2), (−√2, √2).


∂2 𝑢 ∂2 𝑢 ∂2 𝑢
We have 𝑟 = ∂𝑥 2 = 12𝑥 2 − 4, 𝑠 = ∂𝑥 ∂𝑦 = 4, 𝑡 = ∂𝑦 2 = 12𝑦 2 − 4

At (𝟎, 𝟎), 𝑟 = −4, 𝑠 = 4, 𝑡 = −4, so that 𝑟𝑡 − 𝑠 2 = 16 − 16 = 0.

198 | P a g e

© Department of Distance & Continuing Education, Campus of Open Learning,


School of Open Learning, University of Delhi
Intermediate Mathematical Methods for Economics

Thus, at the point (0,0), the case is doubtful and further investigation is needed.

. At (√2, −√2), 𝑟 = 20, 𝑠 = 4, 𝑡 = 20, so that 𝑟𝑡 − 𝑠 2 = 400 − 16 = + ive.


Therefore 𝑢 has an extreme value at this point.
Since 𝑟 is positive, therefore 𝑢 has a minimum at this point.

At (−√2, √2), 𝑟 = 20, 𝑠 = 4, 𝑡 = 20, so that 𝑟𝑡 − 𝑠 2 is positive. Since 𝑟 is positilite therefore


𝑢 has a minimum at this point also.
Note. We may tackle the doubtful case at the point (0,0) by the following consideration:
We have 𝑢 = 𝑥 4 + 𝑦 4 − 2(𝑥 − 𝑦)2 .
At the point (0,0), we have 𝑢 = 0.
At the points in the neighbourhood of the point (0,0) where 𝑥 ≠ 𝑦, the value of 𝑢 is
approximately given by
𝑢 = −2(𝑥 − 𝑦)2
[Neglecting the terms 𝑥 4 + 𝑦 4 because the numerical values of 𝑥 and 𝑦 are small..
Thus, at such points 𝑢 is -ive.
Again, at the points in the neighbourhood of the point (0,0), where 𝑥 = 𝑦, me have 𝑢 = 𝑥 4 +
𝑦 4 which is positive.
Thus, in the neighbourhood of the point (0,0), there are points at which 𝑢 take values less than
its value at the point (0,0) and there are points at which 𝑢 takies values greater than its value
at the point (0,0). Hence 𝑢 cannot have a maximum of a minimum value at the point (0,0).
Ex. 5. Find the minimum value of 𝑥 2 + 𝑦 2 + 𝑧 2 when 𝑎𝑥 + 𝑏𝑦 + 𝑐𝑧 = 𝑝.
Sol. Let 𝑢 = 𝑥 2 + 𝑦 2 + 𝑧 2 .
(Here 𝑢 is a function of three variables 𝑥, 𝑦 and 𝑧. But we can eliminate one variable with the
help of the given relation, viz.,
𝑎𝑥 + 𝑏𝑦 + 𝑐𝑧 = 𝑝.
𝑝−𝑎𝑥−𝑏𝑦
From this relation, we have 𝑧 = .
𝑐
Putting this value of 𝑧 in the value of 𝑢, we get.

2
(𝑝 − 𝑎𝑥 − 𝑏𝑦)2
2
𝑢 =𝑥 +𝑦 +
𝑐2
199 | P a g e

© Department of Distance & Continuing Education, Campus of Open Learning,


School of Open Learning, University of Delhi
B.A. (Hons.) Economics

where 𝑢 has been expressed as a function of two independent variables 𝑥 and 𝑦.


∂𝑢 2𝑎
We have ∂𝑥 = 2𝑥 − 𝑐 2 (𝑝 − 𝑎𝑥 − 𝑏𝑦)

and
∂𝑢 2𝑏
= 2𝑦 − 2 (𝑝 − 𝑎𝑥 − 𝑏𝑦)
∂𝑦 𝑐
∂𝑢 ∂𝑢
Solving ∂𝑥 = 0 and ∂𝑦 = 0, we get

𝑎𝑝 𝑏𝑝
𝑥= , and 𝑦 = 2 .
𝑎2 2
+𝑏 +𝑐 2 𝑎 + 𝑏2 + 𝑐 2
∂2 𝑢 2𝑎2 ∂2 𝑢 2𝑎𝑏
Again, we get 𝑟 = − ∂𝑥 2 = 2 + , 𝑠 = ∂𝑥⋅∂𝑦 = ,
𝑐2 𝑐2

and
∂2 𝑢 2𝑏 2
𝑡= = 2 + .
∂𝑦 2 𝑐2
2
𝑎2 𝑏2 4𝑎2 𝑏 2 𝑎2 𝑏 2
∴ 𝑟𝑡 − 𝑠 = 4 (1 + 2 ) (1 + 2 ) − = 4 (1 + 2 + 2 ) .
𝑐 𝑐 𝑐4 𝑐 𝑐

Since 𝑟𝑡 − 𝑠 2 is positive and 𝑟 is also positive, therefore 𝑢 is minimum for the values of 𝑥
and 𝑦 found above.
𝑝2
The minimum value of 𝑢, therefore, is (𝑎2+𝑏2+𝑐 2).

12.3.2 Maxima and Minima of Several Independent Variable:

Let 𝑓(𝑥, 𝑦, 𝑧, … ) be any function of several independent variables 𝑥, 𝑦, 𝑧, … supposed to be


continuous for all values of these variables in the neighbourhood of their values 𝑎, 𝑏, 𝑐, …
respectively. Then 𝑓(𝑎, 𝑏, 𝑐, … ) is said to be a maximum or a minimum value of 𝑓(𝑥, 𝑦, 𝑧, … )
according as 𝑓(𝑎 + ℎ, 𝑏 + 𝑘, 𝑐 + 𝑡, … ) is less or greater than 𝑓(𝑎, 𝑏, 𝑐, … ) for all sufficiently
small independent values of ℎ, 𝑘, 𝑡1 .. positive or negative, provided they are not all zero.
NECESSARY CONDITIONS FOR THE EXISTENCE OF MAXIMA OR MINIMA.
From the definition it is obvious that we shall have a maximum or a minimum of 𝑓(𝑥, 𝑦, 𝑧, … )
for those values of 𝑥, 𝑦, 𝑧, … for which the expression 𝑓(𝑥 + ℎ, 𝑦 + 𝑘, 𝑧 + 𝑙, … ) − 𝑓(𝑥, 𝑦, 𝑧, … )
is of invariable sign for all sufficiently small independent values of ℎ, 𝑘, 𝑙, … provided they are

200 | P a g e

© Department of Distance & Continuing Education, Campus of Open Learning,


School of Open Learning, University of Delhi
Intermediate Mathematical Methods for Economics

not all equal to zero. There will be a maximum or a minimum according as this sign is negative
or positive.
Expanding by Taylor's theorem for several variables, we have
𝑓(𝑥 + ℎ, 𝑦 + 𝑘, 𝑧 + 𝑙, … )
1 ∂ ∂ ∂
= [1 + (ℎ +𝑘 +𝑙 +⋯)
1! ∂𝑥 ∂𝑦 ∂𝑧
2
1 ∂ ∂ ∂
+ (ℎ +𝑘 + 𝑙 + ⋯ ) + ⋯ ] 𝑓(𝑥, 𝑦, 𝑧, … ).
2! ∂𝑥 ∂𝑦 ∂𝑧
∴ 𝑓(𝑥 + ℎ, 𝑦 + 𝑘, 𝑧 + 𝑙, … ) − 𝑓(𝑥, 𝑦, 𝑧, … )
∂𝑓 ∂𝑓 ∂𝑓
= (ℎ +𝑘 +𝑙 +⋯)+
∂𝑥 ∂𝑦 ∂𝑧
terms of the second and higher orders in ℎ, 𝑘, 𝑙, ….
Now by taking ℎ, 𝑘, 𝑙, … sufficiently small, the first-degree terms in ℎ, 𝑘, 𝑙, … can be made to
∂𝑓
govern the sign of the right hand side and therefore of the left hand side of (1). But if ℎ ∂𝑥 +
∂𝑓 ∂𝑓
𝑘 ∂𝑦 + 𝑙 ∂𝑧 + ⋯, is not equal to zero, the sign of this expression will change by changing the
sign of each of ℎ, 𝑘, 𝑙, …,. Hence as a necessary condition for the occurrence of a maximum or
a minimum of 𝑓(𝑥, 𝑦, 𝑧, … ), we must have
∂𝑓 ∂𝑓 ∂𝑓
ℎ +𝑘 +𝑙 +⋯=0
∂𝑥 ∂𝑦 ∂𝑧
Since (2) is true whatever be the values of ℎ, 𝑘, 𝑙, … independent of each other, we must have
as a necessary consequence
∂𝑓 ∂𝑓 ∂𝑓
= 0, = 0, = 0, …
∂𝑥 ∂𝑦 ∂𝑧
If there are 𝑛 independent vairables, we have then obtained 𝑛 simultaneous equations to give
us the values 𝑎, 𝑏, 𝑐, … of the 𝑛 variables 𝑥, 𝑦, 𝑧, … for which 𝑓(𝑥, 𝑦, 𝑧, … ) may have a maximum
or a minimum value.
∂𝑓 ∂𝑓 ∂𝑓
The conditions = 0, ∂𝑦 = 0, ∂𝑧 = 0, … are necessary but not sufficient for the existence of
∂𝑥
maxima and minima.
A point (𝑎1 , 𝑎2 , … , 𝑎𝑛 ) is called derivatives of the function 𝑓(𝑥1 , 𝑥2 , … , 𝑥𝑛 ) of the function
𝑓(𝑥1 , 𝑥2 , … , 𝑥𝑛 ) is said to be stationary at that point. A stationary point which is either a
maximum or a minimum is called an extreme point and the value of the function at that point
201 | P a g e

© Department of Distance & Continuing Education, Campus of Open Learning,


School of Open Learning, University of Delhi
B.A. (Hons.) Economics

is called an extreme value. A stationary point is not necessarily an extreme point. Thus, a
stationary value may be a maximum or a minimum or neither of these two. To decide whether
a stationary point is really an extreme point, a further investigation is required.
LAGRANGE'S NECESSARY AND SUFFICIENT CONDITIONS FOR THE MAXIMA
OR MINIMA OF A FUNCTION OF THREE INDEPENDENT VARIABLES.
Necessary Conditions. Let 𝑓(𝑥, 𝑦, 𝑧) be a function of three independent variables 𝑥, 𝑦 and 𝑧.
Then as derived in §2, for 𝑓(𝑥, 𝑦, 𝑧) to be a maximum or a minimum at any point (𝑎, 𝑏, 𝑐), it
∂𝑓 ∂𝑓 ∂𝑓
is necessary that ∂𝑥 = 0, ∂𝑦 = 0 and ∂𝑧 = 0 at that point.

Hence the points where the value of the function 𝑓(𝑥, 𝑦, 𝑧) is stationary (i.e., may be a
maximum or a minimum) are obtained by solving the simultaneous equations
∂𝑓 ∂𝑓 ∂𝑓
= 0, = 0, =0
∂𝑥 ∂𝑦 ∂𝑧
Sufficient Conditions. Before deriving the sufficient conditions for the existence of a
maximum or a minimum of a function of three independent variables, we obtain the following
two algebraic lemmas regarding the signs of quadratic expressions.
Lemma 1. Let 𝐼2 = 𝑎𝑥 2 + 2ℎ𝑥𝑦 + 𝑏𝑦 2 be a quadratic expression in two variables 𝑥 and 𝑦.
We can write
1 2 2
𝐼2 = [𝑎 𝑥 + 2𝑎ℎ𝑥𝑦 + 𝑎𝑏𝑦 2 ], if 𝑎 ≠ 0
𝑎
1
= [(𝑎𝑥 + ℎ𝑦)2 + (𝑎𝑏 − ℎ2 )𝑦 2 ].
𝑎
The expression within the square brackets will be positive if 𝑎𝑏 − ℎ2 is positive and in that
case the sign of the expression 𝐼2 will be the same as that of 𝑎.
In case 𝑎𝑏 − ℎ2 is not positive, we can say nothing about the sign of the expression within the
square brackets and hence nothing about the sign of the given quadratic expression 𝐼2 .
Lemma 2. In three variables 𝑥, 𝑦 and 𝑧,

202 | P a g e

© Department of Distance & Continuing Education, Campus of Open Learning,


School of Open Learning, University of Delhi
Intermediate Mathematical Methods for Economics

𝐼3 ≡ 𝑎𝑥 2 + 𝑏𝑦 2 + 𝑐𝑧 2 + 2𝑓𝑦𝑧 + 2𝑔𝑧𝑥 + 2ℎ𝑥𝑦


1
= [𝑎2 𝑥 2 + 𝑎𝑏𝑦 2 + 𝑎𝑐𝑧 2 + 2𝑓𝑎𝑦𝑧 + 2𝑔𝑎𝑧𝑥 + 2ℎ𝑎𝑥𝑦], if 𝑎 ≠ 0
𝑎
1
= [𝑎2 𝑥 2 + 2𝑎𝑥(𝑔𝑧 + ℎ𝑦) + 𝑎𝑏𝑦 2 + 𝑎𝑐𝑧 2 + 2𝑓𝑎𝑦𝑧]
𝑎
1
= [(𝑎𝑥 + ℎ𝑦 + 𝑔𝑧)2 + 𝑎𝑏𝑦 2 + 𝑎𝑧 2 + 2𝑓𝑎𝑦𝑧 − (𝑔𝑧 + ℎ𝑦)2 ]
𝑎
1
= [(𝑎𝑥 + ℎ𝑦 + 𝑔𝑧)2 + (𝑎𝑏 − ℎ2 )𝑦 2 + 2𝑦𝑧(𝑓𝑎 − 𝑔ℎ) + (𝑎𝑐 − 𝑔2 )𝑧 2 ].
𝑎
Now 𝐼3 will be of the same sign as 𝑎 provided the expression within the square brackets is
positive which will of course be so if
𝑎𝑏 − ℎ2 and {(𝑎𝑏 − ℎ2 )(𝑎𝑐 − 𝑔2 ) − (𝑓𝑎 − 𝑔ℎ)2 }
are both positive i.e., if
𝑎𝑏 − ℎ2 and 𝑎(𝑎𝑏𝑐 + 2𝑓𝑔ℎ − 𝑎𝑓 2 − 𝑏𝑔2 − 𝑐ℎ2 )
are both positive.
Thus 𝐼3 will be positive if
𝑎 ℎ 𝑔
𝑎 ℎ ℎ 𝑏 𝑓|
𝑎, | |,|
ℎ 𝑏
𝑔 𝑓 𝑐

be all positive and will be negative if these three expressions are alternately negative and
positive.
Now we are in a position to derive Lagrange's sufficient conditions for the existence of a
maximum or a minimum of a function of three independent variables at a stationary point.
Let a set of the values of 𝑥, 𝑦, 𝑧 obtained by solving the equations
∂𝑓 ∂𝑓 ∂𝑓
= = = 0 be 𝑎, 𝑏, 𝑐.
∂𝑥 ∂𝑦 ∂𝑧
Let the values of the six second order partial derivatives.
∂2 𝑓 ∂2 𝑓 ∂2 𝑓 ∂2 𝑓 ∂2 𝑓 ∂2 𝑓
, , , , and
∂𝑥 2 ∂𝑦 2 ∂𝑧 2 ∂𝑦 ∂𝑧 ∂𝑧 ∂𝑥 ∂𝑥 ∂𝑦
at the point (𝑎, 𝑏, 𝑐) be denoted by 𝐴, 𝐵, 𝐶, 𝐹, 𝐺 and 𝐻 respectively.
Then, we have

203 | P a g e

© Department of Distance & Continuing Education, Campus of Open Learning,


School of Open Learning, University of Delhi
B.A. (Hons.) Economics

𝑓(𝑎 + ℎ, 𝑏 + 𝑘, 𝑐 + 𝑙) − 𝑓(𝑎, 𝑏, 𝑐)
1
= (𝐴ℎ2 + 𝐵𝑘 2 + 𝐶𝑙 2 + 2𝐹𝑘𝑙 + 2𝐺𝑙ℎ + 2𝐻ℎ𝑘) + 𝑅3 ,
2!
where 𝑅3 consists of terms of third and higher orders of small quantities ℎ, 𝑘 and 𝑙. By taking
ℎ, 𝑘 and 𝑙 sufficiently small, the second-degree terms in ℎ, 𝑘 and 𝑙 can be made to govern the
sign of the right-hand side and therefore of the left hand side of (1). If this group of terms forms
an expression of invariable sign for all such values
of ℎ, 𝑘 and 𝑙, we shall have a maximum or a minimum value of 𝑓(𝑥, 𝑦, 𝑧) at (𝑎, 𝑏, 𝑐)
I-42 according as that sign is negative or positive.
Hence by our lemma 2, if the expressions
𝐴 𝐻 𝐺
𝐴 𝐻
𝐴, | | , |𝐻 𝐵 𝐹|
𝐻 𝐵
𝐺 𝐹 𝐶
be all positive, we shall have a minimum of 𝑓(𝑥, 𝑦, 𝑧) at (𝑎, 𝑏, 𝑐) and if these expressions be
alternately negative and positive, we shall have a maximum of 𝑓(𝑥, 𝑦, 𝑧) at (𝑎, 𝑏, 𝑐), whilst if
these conditions are not satisfied, we shall in general have neither a maximum nor a
minimum of 𝑓(𝑥, 𝑦, 𝑧) at (𝑎, 𝑏, 𝑐).
WORKING RULE FOR FINDING THE MAXIMA AND MINIMA OF A FUNCTION
OF THREE INDEPENDENT VARIABLES.
Suppose 𝑓(𝑥, 𝑦, 𝑧) is a given function of three independent variables 𝑥, 𝑦 and 𝑧. Find
∂𝑓/ ∂𝑥, ∂𝑓/ ∂𝑦 and ∂𝑓/ ∂𝑧 and solve the simultaneous equations ∂𝑓/ ∂𝑥 = 0, ∂𝑓/ ∂𝑦 = 0 and
∂𝑓/ ∂𝑧 = 0. All the triads (𝑎, 𝑏, 𝑐) of the values of 𝑥, 𝑦 and 𝑧 obtained on solving these
equations will give the stationary values of 𝑓(𝑥, 𝑦, 𝑧) i.e., will give the points at which the
function 𝑓(𝑥, 𝑦, 𝑧) may be a maximum or a minimum.
To discuss the maximum or minimum of 𝑓(𝑥, 𝑦, 𝑧) at any point (𝑎, 𝑏, 𝑐) obtained on solving
the equations ∂𝑓/ ∂𝑥 = 0, ∂𝑓/ ∂𝑦 = 0 and ∂𝑓/ ∂𝑧 = 0, we find the values at this point of the
six partial derivatives of second order of 𝑓(𝑥, 𝑦, 𝑧) symbolically denoted as follows :
∂2 𝑓 ∂2 𝑓 ∂2 𝑓 ∂2 𝑓 ∂2 𝑓 ∂2 𝑓
𝐴 = 2 ,𝐵 = 2,𝐶 = 2 ,𝐹 = ,𝐺 = and 𝐻 = .
∂𝑥 ∂𝑦 ∂𝑧 ∂𝑦 ∂𝑧 ∂𝑧 ∂𝑥 ∂𝑥 ∂𝑦
If the expressions

204 | P a g e

© Department of Distance & Continuing Education, Campus of Open Learning,


School of Open Learning, University of Delhi
Intermediate Mathematical Methods for Economics

𝐴 𝐻 𝐺
𝐴 𝐻
𝐴, | | , |𝐻 𝐵 𝐹|
𝐻 𝐵
𝐺 𝐹 𝐶
be all positive, we shall have a minimum of 𝑓(𝑥, 𝑦, 𝑧) at (𝑎, 𝑏, 𝑐) and if these expressions be
alternately negative and positive, we shall have a maximum of 𝑓(𝑥, 𝑦, 𝑧) a (𝑎, 𝑏, 𝑐), whilst if
these conditions are not satisfied, we shall in general have neither a maximum nor a minimum
of 𝑓(𝑥, 𝑦, 𝑧) at (𝑎, 𝑏, 𝑐).
SOLVED EXAMPLES
Ex. 1. Discuss the maximum or minimum values of 𝑢 where
𝑢 = 𝑥 2 + 𝑦 2 + 𝑧 2 + 𝑥 − 2𝑧 − 𝑥𝑦
Sol. For a maximum or a minimum of 𝑢, we must have
∂𝑢
= 2𝑥 − 𝑦 + 1 = 0
∂𝑥
∂𝑢
= −𝑥 + 2𝑦 = 0
∂𝑦
∂𝑢
= 2𝑧 − 2 = 0
∂𝑧
These equations give 𝑥 = −2/3, 𝑦 = −1/3, 𝑧 = 1.
∴ (−2/3, −1/3,1) is the only point at which 𝑢 is statior.ary i.e., at which 𝑢 may have a
maximum or a minimum.
∂2 𝑢 ∂2 𝑢 ∂2 𝑢 ∂2 𝑢 ∂2 𝑢 ∂2 𝑢
Now = 2, ∂𝑦 2 = 2, ∂𝑧 2 = 2, ∂𝑦 ∂𝑧 = 0, ∂𝑧 ∂𝑥 = 0 and = −1.
∂𝑥 2 ∂𝑥 ∂𝑦

If 𝐴, 𝐵, 𝐶, 𝐹, 𝐺 and 𝐻 denote the respective values of these six partial derivatives of second
order at the point (−2/3, −1/3,1), then
𝐴 = 2, 𝐵 = 2, 𝐶 = 2, 𝐹 = 0, 𝐺 = 0, 𝐻 = −1.
Now we have
and
𝐴 𝐻 2 −1
𝐴 = 2, | |=| |=3
𝐻 𝐵 −1 2
𝐴 𝐻 𝐺 2 −1 0
|𝐻 𝐵 𝐹 | = |−1 2 0| = 6.
𝐺 𝐹 𝐶 0 0 2
205 | P a g e

© Department of Distance & Continuing Education, Campus of Open Learning,


School of Open Learning, University of Delhi
B.A. (Hons.) Economics

Since these three expressions are all positive, we have a minimum of 𝑢 when 𝑥 = −2/3, 𝑦 =
−1/3, 𝑧 = 1.
Ex. 2. Show that the point such that the sum of the squares of its distances from 𝑛 given points
shall be minimum, is the centre of the mean position of the given points.
Sol. Let the 𝑛 given points be (𝑎1 , 𝑏1 , 𝑐1 ), (𝑎2 , 𝑏2 , 𝑐2 ), … … , (𝑎𝑛 , 𝑏𝑛 , 𝑐𝑛 ) and let (𝑥, 𝑦, 𝑧) be the
coordinates of the required point.
If 𝑢 denotes the sum of the squares of the distances of (𝑥, 𝑦, 𝑧) from the 𝑛 given points, then
𝑢 = Σ[(𝑥 − 𝑎1 )2 + (𝑦 − 𝑏1 )2 + (𝑧 − 𝑐1 )2 ]
= Σ(𝑥 − 𝑎1 )2 + Σ(𝑦 − 𝑏1 )2 + Σ(𝑧 − 𝑐1 )2 .
For a maximum or a minimum of 𝑢, we must have
∂𝑢
= 2Σ(𝑥 − 𝑎1 ) = 2𝑛𝑥 − 2Σ𝑎1 = 0
∂𝑥
∂𝑢
= 2Σ(𝑦 − 𝑏1 ) = 2𝑛𝑦 − 2Σ𝑏1 = 0
∂𝑦
∂𝑢
= 2Σ(𝑧 − 𝑐1 ) = 2𝑛𝑧 − 2Σ𝑐1 = 0
∂𝑧
Solving these equations, we get
Now
Σ𝑎1 Σ𝑏1 Σ𝑐1
𝑥= ,𝑦 = ,𝑧 = .
𝑛 𝑛 𝑛
∂2 𝑢 ∂2 𝑢 ∂2 𝑢
𝐴 = 2 = 2𝑛, 𝐵 = 2 = 2𝑛, 𝐶 = 2 = 2𝑛,
∂𝑥 ∂𝑦 ∂𝑧
2 2
∂ 𝑢 ∂ 𝑢 ∂2 𝑢
𝐹= = 0, 𝐺 = = 0, 𝐻 = = 0.
∂𝑦 ∂𝑧 ∂𝑧 ∂𝑥 ∂𝑥 ∂𝑦
𝐴 𝐻 2𝑛 0
We have 𝐴 = 2𝑛, | |=| | = 4𝑛2 ,
𝐻 𝐵 0 2𝑛
and
𝐴 𝐻 𝐺 2𝑛 0 0
|𝐻 𝐵 𝐹| = | 0 2𝑛 0 | = 8𝑛3
𝐺 𝐹 𝐶 0 0 2𝑛
Since these three expressions are all positive, 𝑢 is minimum when

206 | P a g e

© Department of Distance & Continuing Education, Campus of Open Learning,


School of Open Learning, University of Delhi
Intermediate Mathematical Methods for Economics

Σ𝑎1 Σ𝑏1 Σ𝑐1


𝑥= ,𝑦 = and 𝑧 =
𝑛 𝑛 𝑛
i.e., 𝑢 is minimum when the point (𝑥, 𝑦, 𝑧) is the centre of the mean position of the 𝑛 given
points.
EX3. Find the maximum value of 𝑢 where
𝑥𝑦𝑧
𝑢 = (𝑎+𝑥)(𝑥+𝑦)(𝑦+𝑧)(𝑧+𝑏).

Sol. We have
log 𝑢 = log 𝑥 + log 𝑦 + log 𝑧 − log (𝑎 + 𝑥) − log (𝑥 + 𝑦) − log (𝑦 + 𝑧)
−log (𝑧 + 𝑏)

1 ∂𝑢 1 1 1 𝑎𝑦 − 𝑥 2
∴ ⋅ = − − =
𝑢 ∂𝑥 𝑥 𝑎 + 𝑥 𝑥 + 𝑦 𝑥(𝑎 + 𝑥)(𝑥 + 𝑦)
∂𝑢 (𝑎𝑦 − 𝑥 2 )𝑢
= .
∂𝑥 𝑥(𝑎 + 𝑥)(𝑥 + 𝑦)
∂𝑢 (𝑥𝑧−𝑦 2 )𝑢
Similarly, ∂𝑦 = 𝑦(𝑥+𝑦)(𝑦+𝑧)

∂𝑢 (𝑏𝑦 − 𝑧 2 )𝑢
=
∂𝑧 𝑧(𝑦 + 𝑧)(𝑧 + 𝑏)
Now for a maximum or a minimum of 𝑢, we must have
∂𝑢
= 0 i.e., 𝑎𝑦 − 𝑥 2 = 0
∂𝑥
∂𝑢
= 0 i.e., 𝑥𝑧 − 𝑦 2 = 0
∂𝑦
∂𝑢
and = 0 i.e., 𝑏𝑦 − 𝑧 2 = 0.
∂𝑧

From the above equations, it follows that 𝑎, 𝑥, 𝑦, 𝑧 and 𝑏 are in geometrical progression. Let 𝑟
be the common ratio of this geometrical progression. Then
𝑎𝑟 4 = 𝑏 or 𝑟 = (𝑏/𝑎)1/4 .
Also 𝑥 = 𝑎𝑟, 𝑦 = 𝑎𝑟 2 , 𝑧 = 𝑎𝑟 3 .

207 | P a g e

© Department of Distance & Continuing Education, Campus of Open Learning,


School of Open Learning, University of Delhi
B.A. (Hons.) Economics

Substituting these values, we get


𝑎𝑟 ⋅ 𝑎𝑟 2 ⋅ 𝑎𝑟 3
𝑢 =
𝑎(1 + 𝑟) ⋅ 𝑎𝑟(1 + 𝑟) ⋅ 𝑎𝑟 2 (1 + 𝑟) ⋅ 𝑎𝑟 3 (1 + 𝑟)
1 1 1
= 4
= 1/4 4
= 1/4 .
𝑎(1 + 𝑟) 𝑎[1 + (𝑏/𝑎) ] (𝑎 + 𝑏1/4 )4
This gives a stationary value of 𝑢. To decide whether this value of 𝑢 is a maximum or a
minimum we proceed to find the second order partial derivatives of
We have
∂2 𝑢 −2𝑥𝑢 2)
∂ 𝑢
= + (𝑎𝑦 − 𝑥 [ ]
∂𝑥 2 𝑥(𝑎 + 𝑥)(𝑥 + 𝑦) ∂𝑥 𝑥(𝑎 + 𝑥)(𝑥 + 𝑦)
∴ when 𝑥 = 𝑎𝑟, 𝑦 = 𝑎𝑟 2 , 𝑧 = 𝑎𝑟 3 , we have
∂2 𝑢 −2⋅𝑎𝑟⋅𝑢 −2𝑢
𝐴 = ∂𝑥 2 = 𝑎𝑟⋅𝑎(1+𝑟)⋅𝑎𝑟(1+𝑟) = 𝑎2 𝑟(1+𝑟)2 , which is negative. Hence the above stationary value
of 𝑢 is a maximum.
1
Ans. Maximum value of 𝑢 = 4 .
(𝑎1/4 +𝑏 1/4 )

Note. In the complicated problems in order to find whether a stationary value of 𝑢 is a


maximum or a minimum, it is sufficient to find the value of a second partial differential
coefficient of 𝑢 with respect to any of the independent variables. The value of 𝑢 will be
maximum or minimum according as the value of this second partial derivative at the stationary
point under consideration is -ve or +ve.
12.3.3 Lagrange Method of Undetermined Multipliers:

Let 𝑢 = 𝑓(𝑥1 , 𝑥2 , … , 𝑥𝑛 )
be a function of 𝑛 variables 𝑥1 , 𝑥2 , … , 𝑥𝑛 . Let these variables be connected by 𝑚 equations.
𝜙1 (𝑥1 , 𝑥2 , … , 𝑥𝑛 ) = 0, 𝜙2 (𝑥1 , 𝑥2 , … , 𝑥˙𝑛 ) = 0, … , 𝜙𝑚 (𝑥1 , 𝑥2 , … , 𝑥𝑛 ) = 0
so that only 𝑛 − 𝑚 of the 𝑛 variables are independent.
For a maximum or a minimum of 𝑢, we have
∂𝑢 ∂𝑢 ∂𝑢 ∂𝑢
𝑑𝑢 = 𝑑𝑥1 + 𝑑𝑥2 + 𝑑𝑥3 + ⋯ + 𝑑𝑥 = 0
∂𝑥1 ∂𝑥2 ∂𝑥3 ∂𝑥𝑛 𝑛
Also differentiating the 𝑚 given equations connecting the variables, we get

208 | P a g e

© Department of Distance & Continuing Education, Campus of Open Learning,


School of Open Learning, University of Delhi
Intermediate Mathematical Methods for Economics

∂𝜙1 ∂𝜙1 ∂𝜙1 ∂𝜙1


𝑑𝜙1 = 𝑑𝑥1 + 𝑑𝑥2 + 𝑑𝑥3 + ⋯ + 𝑑𝑥 = 0,
∂𝑥1 ∂𝑥2 ∂𝑥3 ∂𝑥𝑛 𝑛
∂𝜙2 ∂𝜙2 ∂𝜙2 ∂𝜙2
𝑑𝜙2 = 𝑑𝑥1 + 𝑑𝑥2 + 𝑑𝑥3 + ⋯ + 𝑑𝑥 = 0
∂𝑥1 ∂𝑥2 ∂𝑥3 ∂𝑥𝑛 𝑛
∂𝜙𝑚 ∂𝜙𝑚 ∂𝜙𝑚 ∂𝜙𝑚
𝑑𝜙𝑚 = 𝑑𝑥1 + 𝑑𝑥2 + 𝑑𝑥3 + ⋯ + 𝑑𝑥𝑛 = 0
∂𝑥1 ∂𝑥2 ∂𝑥3 ∂𝑥𝑛

Multiplying the above 𝑚 + 1 equations obtained on differentiation by 1, 𝜆1 , 𝜆2 , … , 𝜆𝑚


respectively and adding, we get an equation which may be written as
𝑃1 𝑑𝑥1 + 𝑃2 𝑑𝑥2 + 𝑃3 𝑑𝑥3 + ⋯ + 𝑃𝑛 𝑑𝑥𝑛 = 0
∂𝑢 ∂𝜙 ∂𝜙 ∂𝜙𝑚
where 𝑃𝑟 = ∂𝑥 + 𝜆1 ∂𝑥 1 + 𝜆2 ∂𝑥 2 + ⋯ + 𝜆𝑚 .
𝑟 𝑟 𝑟 ∂𝑥𝑟

Now the 𝑚 multipliers 𝜆1 , 𝜆2 , … , 𝜆𝑚 are at our choice. We choose them such that they satisfy
the 𝑚 linear equations
𝑃1 = 0, 𝑃2 = 0, … , 𝑃𝑚 = 0
Then the equation (1) reduces to
𝑃𝑚+1 𝑑𝑥𝑚+1 + 𝑃𝑚+2 𝑑𝑥𝑚+2 + ⋯ + 𝑃𝑛 𝑑𝑥𝑛 = 0
It is immaterial which of the 𝑛 − 𝑚 of the 𝑛 variables 𝑥1 , 𝑥2 , … , 𝑥𝑛 are regarded as independent.
Let us regard the 𝑛 − 𝑚 variables 𝑥𝑚+1 , 𝑥𝑚 + 2, … , 𝑥𝑛 as independent. Then since the 𝑛 − 𝑚
quantities 𝑑𝑥𝑚+1 , 𝑑𝑥𝑚+2 , … , 𝑑𝑥𝑛 are all independent of one another, their coefficients must be
separately zero in the relation (2). Hence, we must have
𝑃𝑚+1 = 0, 𝑃𝑚+2 = 0, … , 𝑃𝑛 = 0
Thus we get 𝑚 + 𝑛 equations.
and
𝑃1 = 0, 𝑃2 = 0, … , 𝑃𝑛 = 0
𝜙1 = 0, 𝜙2 = 0, … , 𝜙𝑚 = 0,
which together with the relation 𝑢 = 𝑓(𝑥1 , 𝑥2 , … , 𝑥𝑛 ) determine the 𝑚 multipliers
𝜆1 , 𝜆2 , … , 𝜆𝑚 , the values of 𝑥1 , 𝑥2 , … , 𝑥𝑛 and 𝑢 at the stationary point. This method is known
as Lagrange's method of undetermined multipliers. It is very convenient to apply and it often
gives us the maximum or minimum values of 𝑢 without determining the values of the

209 | P a g e

© Department of Distance & Continuing Education, Campus of Open Learning,


School of Open Learning, University of Delhi
B.A. (Hons.) Economics

multipliers 𝜆1 , … , 𝜆𝑚 . The only drawback of this method is that it does not determine the nature
of the stationary point.
SOLVED EXAMPLES
Ex. 1. If 𝑢 = 𝑥 2 + 𝑦 2 + 𝑧 2 ,
where 𝑎𝑥 2 + 𝑏𝑦 2 + 𝑐𝑧 2 + 2𝑓𝑦𝑧 + 2𝑔𝑧𝑥 + 2ℎ𝑥𝑦 = 1, find the maximum or minimum values
of 𝑢.
Sol. We have 𝑢 = 𝑥 2 + 𝑦 2 + 𝑧 2 , where the variables 𝑥, 𝑦, 𝑧 are connected by the relation
𝑎𝑥 2 + 𝑏𝑦 2 + 𝑐𝑧 2 + 2𝑓𝑦𝑧 + 2𝑔𝑧𝑥 + 2ℎ𝑥𝑦 = 1
For a maximum or a minimum of 𝑢, we have 𝑑𝑢 = 0
⇒ 2𝑥𝑑𝑥 + 2𝑦𝑑𝑦 + 2𝑧𝑑𝑧 = 0.
⇒ 𝑥𝑑𝑥 + 𝑦𝑑𝑦 + 𝑧𝑑𝑧 = 0.
Also differentiating the given relation (2), we get 2𝑎𝑥𝑑𝑥 + 2𝑏𝑦𝑑𝑦 + 2𝑐𝑧𝑑𝑧 + 2𝑓𝑦𝑑𝑧 +
2𝑓𝑧𝑑𝑦 + 2𝑔𝑧𝑑𝑥 + 2𝑔𝑥𝑑𝑧 + 2ℎ𝑥𝑑𝑦 + 2ℎ𝑦𝑑𝑥 = 0 or (𝑎𝑥 + ℎ𝑦 + 𝑔𝑧)𝑑𝑥 + (ℎ𝑥 + 𝑏𝑦 +
𝑓𝑧)𝑑𝑦 + (𝑔𝑥 + 𝑓𝑦 + 𝑐𝑧)𝑑𝑧 = 0.
Multiplying (3) by 1, (4) by 𝜆 and adding, and then equating the coefficients of 𝑑𝑥, 𝑑𝑦, 𝑑𝑧 to
zero, we have
𝑥 + 𝜆(𝑎𝑥 + ℎ𝑦 + 𝑔𝑧) = 0
, 𝑦 + 𝜆(ℎ𝑥 + 𝑏𝑦 + 𝑓𝑧) = 0,
𝑧 + 𝜆(𝑔𝑥 + 𝑓𝑦 + 𝑐𝑧) = 0.
Multiplying (5) by 𝑥, (6) by 𝑦, (7) by 𝑧 and adding, we get
𝑥 2 + 𝑦 2 + 𝑧 2 + 𝜆(𝑎𝑥 2 + 𝑏𝑦 2 + 𝑐𝑧 2 + 2𝑓𝑦𝑧 + 2𝑔𝑧𝑥 + 2ℎ𝑥𝑦) = 0
or 𝑢 + 𝜆. 1 = 0, using (1) and (2).
∴ 𝜆 = −𝑢
Hence from (5), we have
or
𝑥 − 𝑢(𝑎𝑥 + ℎ𝑦 + 𝑔𝑧) = 0
𝑥(1 − 𝑎𝑢) − ℎ𝑢𝑦 − 𝑔𝑢𝑧 = 0

210 | P a g e

© Department of Distance & Continuing Education, Campus of Open Learning,


School of Open Learning, University of Delhi
Intermediate Mathematical Methods for Economics

1
∫ (𝑎 − ) 𝑥 + ℎ𝑦 + 𝑔𝑧 = 0
𝑢
or
Hence from (5), we have
or
𝑥 − 𝑢(𝑎𝑥 + ℎ𝑦 + 𝑔𝑧) = 0
𝑥(1 − 𝑎𝑢) − ℎ𝑢𝑦 − 𝑔𝑢𝑧 = 0
1
(𝑎 − ) 𝑥 + ℎ𝑦 + 𝑔𝑧 = 0.
𝑢
Or
Similarly, from (6) and (7), we have
ℎ𝑥 + (𝑏 − 1/𝑢)𝑦 + 𝑓𝑧 = 0 and 𝑔𝑥 + 𝑓𝑦 + (𝑐 − 1/𝑢)𝑧 = 0
Eliminating 𝑥, 𝑦, 𝑧 from (8), (9), (10), we get
𝑎 − (1/𝑢) ℎ 𝑔
| ℎ 𝑏 − (1/𝑢) 𝑓 | = 0.
𝑔 𝑓 𝑐 − (1/𝑢)

Hence the required maximum or minimum values of 𝑢 are the roots of the equation (11).
Ex. 2. Find the stationary values of 𝑥 2 + 𝑦 2 + 𝑧 2 subject to the conditions 𝑎𝑥 2 + 𝑏𝑦 2 +
𝑐𝑧 2 = 1 and 𝑙𝑥 + 𝑚𝑦 + 𝑛𝑧 = 0
Interpret the result geometrically.
Sol. Let 𝑢 = 𝑥 2 + 𝑦 2 + 𝑧 2 ,
where the variables 𝑥, 𝑦 and 𝑧 are connected by the relations
𝑎𝑥 2 + 𝑏𝑦 2 + 𝑐𝑧 2 = 1
and 𝑥 + 𝑚𝑦 + 𝑛𝑧 = 0.
For a stationary value of 𝑢, we have
𝑑𝑢 = 0
⇒ 2𝑥𝑑𝑥 + 2𝑦𝑑𝑦 + 2𝑧𝑑𝑧 = 0
⇒ 𝑥𝑑𝑥 + 𝑦𝑑𝑦 + 𝑧𝑑𝑧 = 0.

211 | P a g e

© Department of Distance & Continuing Education, Campus of Open Learning,


School of Open Learning, University of Delhi
B.A. (Hons.) Economics

Also differentiating the given relations (2) and (3), we get i.e.,
2𝑎𝑥𝑑𝑥 + 2𝑏𝑦𝑑𝑦 + 2𝑐𝑧𝑑𝑧 = 0
𝑎𝑥𝑑𝑥 + 𝑏𝑦𝑑𝑦 + 𝑐𝑧𝑑𝑧 = 0
{ }
and
𝑙𝑑𝑥 + 𝑚𝑑𝑦 + 𝑛𝑑𝑧 = 0.
Multiplying (4) by 1, (5) by 𝜆 and (6) by 𝜇 and adding, and then equating the coefficients of
𝑑𝑥, 𝑑𝑦, 𝑑𝑧 to zero, we get
𝑥 + 𝜆𝑎𝑥 + 𝜇𝑙 = 0,
and {𝑦 + 𝜆𝑏𝑦 + 𝜇𝑚 = 0,
𝑧 + 𝜆𝑐𝑧 + 𝜇𝑛 = 0.
Multiplying the equations (7), (8) and (9) by 𝑥, 𝑦 and 𝑧 respectively and adding, we get
𝑥 2 + 𝑦 2 + 𝑧 2 + 𝜆(𝑎𝑥 2 + 𝑏𝑦 2 + 𝑐𝑧 2 ) + 𝜇(𝑏𝑥 + 𝑚𝑦 + 𝑛𝑧) = 0
or 𝑢 + 𝜆. 1 + 𝜇 ⋅ 0 = 0, using (1), (2) and (3)
or 𝜆 = −𝑢.
Substituting for 𝜆 in the equations (7), (8) and (9), we get
𝜇𝑙 𝜇𝑚 𝜇𝑛
𝑥1 = ,𝑦 = ,𝑧 = .
𝑎𝑢 − 1 𝑏𝑢 − 1 𝑐𝑢 − .1
Substituting these values of 𝑥, 𝑦, 𝑧 in (3), we get
𝜇𝑙 2 𝜇𝑚2 𝜇𝑛2
+ + =0
𝑎𝑢 − 1 𝑏𝑢 − 1 𝑐𝑢 − 1
Geometrical interpretation. The surface 𝑎𝑥 2 + 𝑏𝑦 2 + 𝑐𝑧 2 = 1 represents an ellipsoid (or a
hyperboloid) whose centre is origin, and 𝑙𝑥 + 𝑚𝑦 + 𝑛𝑧 = 0 is a plane passing through the
origin. Therefore, the point (𝑥, 𝑦, 𝑧) satisfying both the conditions (2) and (3) lies on the conic
in which (2) and (3) intersect. Also 𝑥 2 + 𝑦 2 + 𝑧 2 gives the square of the distance of (𝑥, 𝑦, 𝑧)
from the origin which is also the centre of the conic of intersection. The maximum and
minimum values of this distance are the major and minor semi-axes of the conic. So the
equation (10) gives the squares of the lengths of the semi-axes of the conic of intersection.
Let 𝑢 = 𝑓(𝑥1 , 𝑥2 , … , 𝑥𝑛 )
be a function of 𝑛 variables 𝑥1 , 𝑥2 , … , 𝑥𝑛 . Let these variables be connected by 𝑚 equations.
𝜙1 (𝑥1 , 𝑥2 , … , 𝑥𝑛 ) = 0, 𝜙2 (𝑥1 , 𝑥2 , … , 𝑥˙𝑛 ) = 0, … , 𝜙𝑚 (𝑥1 , 𝑥2 , … , 𝑥𝑛 ) = 0
so that only 𝑛 − 𝑚 of the 𝑛 variables are independent.

212 | P a g e

© Department of Distance & Continuing Education, Campus of Open Learning,


School of Open Learning, University of Delhi
Intermediate Mathematical Methods for Economics

For a maximum or a minimum of 𝑢, we have


∂𝑢 ∂𝑢 ∂𝑢 ∂𝑢
𝑑𝑢 = 𝑑𝑥1 + 𝑑𝑥2 + 𝑑𝑥3 + ⋯ + 𝑑𝑥 = 0
∂𝑥1 ∂𝑥2 ∂𝑥3 ∂𝑥𝑛 𝑛
Also differentiating the 𝑚 given equations connecting the variables, we get
∂𝜙1 ∂𝜙1 ∂𝜙1 ∂𝜙1
𝑑𝜙1 = 𝑑𝑥1 + 𝑑𝑥2 + 𝑑𝑥3 + ⋯ + 𝑑𝑥 = 0,
∂𝑥1 ∂𝑥2 ∂𝑥3 ∂𝑥𝑛 𝑛
∂𝜙2 ∂𝜙2 ∂𝜙2 ∂𝜙2
𝑑𝜙2 = 𝑑𝑥1 + 𝑑𝑥2 + 𝑑𝑥3 + ⋯ + 𝑑𝑥 = 0
∂𝑥1 ∂𝑥2 ∂𝑥3 ∂𝑥𝑛 𝑛
∂𝜙𝑚 ∂𝜙𝑚 ∂𝜙𝑚 ∂𝜙𝑚
𝑑𝜙𝑚 = 𝑑𝑥1 + 𝑑𝑥2 + 𝑑𝑥3 + ⋯ + 𝑑𝑥𝑛 = 0
∂𝑥1 ∂𝑥2 ∂𝑥3 ∂𝑥𝑛

Multiplying the above 𝑚 + 1 equations obtained on differentiation by 1, 𝜆1 , 𝜆2 , … , 𝜆𝑚


respectively and adding, we get an equation which may be written as
𝑃1 𝑑𝑥1 + 𝑃2 𝑑𝑥2 + 𝑃3 𝑑𝑥3 + ⋯ + 𝑃𝑛 𝑑𝑥𝑛 = 0
∂𝑢 ∂𝜙 ∂𝜙 ∂𝜙𝑚
where 𝑃𝑟 = ∂𝑥 + 𝜆1 ∂𝑥 1 + 𝜆2 ∂𝑥 2 + ⋯ + 𝜆𝑚 .
𝑟 𝑟 𝑟 ∂𝑥𝑟

Now the 𝑚 multipliers 𝜆1 , 𝜆2 , … , 𝜆𝑚 are at our choice. We choose them such that they satisfy
the 𝑚 linear equations
𝑃1 = 0, 𝑃2 = 0, … , 𝑃𝑚 = 0
Then the equation (1) reduces to
𝑃𝑚+1 𝑑𝑥𝑚+1 + 𝑃𝑚+2 𝑑𝑥𝑚+2 + ⋯ + 𝑃𝑛 𝑑𝑥𝑛 = 0
It is immaterial which of the 𝑛 − 𝑚 of the 𝑛 variables 𝑥1 , 𝑥2 , … , 𝑥𝑛 are regarded as independent.
Let us regard the 𝑛 − 𝑚 variables 𝑥𝑚+1 , 𝑥𝑚 + 2, … , 𝑥𝑛 as independent. Then since the 𝑛 − 𝑚
quantities 𝑑𝑥𝑚+1 , 𝑑𝑥𝑚+2 , … , 𝑑𝑥𝑛 are all independent of one another, their coefficients must be
separately zero in the relation (2). Hence, we must have
𝑃𝑚+1 = 0, 𝑃𝑚+2 = 0, … , 𝑃𝑛 = 0
Thus, we get 𝑚 + 𝑛 equations.
and
𝑃1 = 0, 𝑃2 = 0, … , 𝑃𝑛 = 0
𝜙1 = 0, 𝜙2 = 0, … , 𝜙𝑚 = 0,

213 | P a g e

© Department of Distance & Continuing Education, Campus of Open Learning,


School of Open Learning, University of Delhi
B.A. (Hons.) Economics

which together with the relation 𝑢 = 𝑓(𝑥1 , 𝑥2 , … , 𝑥𝑛 ) determine the 𝑚 multipliers


𝜆1 , 𝜆2 , … , 𝜆𝑚 , the values of 𝑥1 , 𝑥2 , … , 𝑥𝑛 and 𝑢 at the stationary point. This method is known
as Lagrange's method of undetermined multipliers. It is very convenient to apply and it often
gives us the maximum or minimum values of 𝑢 without determining the values of the
multipliers 𝜆1 , … , 𝜆𝑚 . The only drawback of this method is that it does not determine the nature
of the stationary point.
SOLVED EXAMPLES
Ex. 1. If 𝑢 = 𝑥 2 + 𝑦 2 + 𝑧 2 ,
where 𝑎𝑥 2 + 𝑏𝑦 2 + 𝑐𝑧 2 + 2𝑓𝑦𝑧 + 2𝑔𝑧𝑥 + 2ℎ𝑥𝑦 = 1, find the maximum or minimum
values of 𝑢.
Sol. We have 𝑢 = 𝑥 2 + 𝑦 2 + 𝑧 2 , where the variables 𝑥, 𝑦, 𝑧 are connected by the relation
𝑎𝑥 2 + 𝑏𝑦 2 + 𝑐𝑧 2 + 2𝑓𝑦𝑧 + 2𝑔𝑧𝑥 + 2ℎ𝑥𝑦 = 1
For a maximum or a minimum of 𝑢, we have 𝑑𝑢 = 0
⇒ 2𝑥𝑑𝑥 + 2𝑦𝑑𝑦 + 2𝑧𝑑𝑧 = 0.
⇒ 𝑥𝑑𝑥 + 𝑦𝑑𝑦 + 𝑧𝑑𝑧 = 0.
Also differentiating the given relation (2), we get 2𝑎𝑥𝑑𝑥 + 2𝑏𝑦𝑑𝑦 + 2𝑐𝑧𝑑𝑧 + 2𝑓𝑦𝑑𝑧 +
2𝑓𝑧𝑑𝑦 + 2𝑔𝑧𝑑𝑥 + 2𝑔𝑥𝑑𝑧 + 2ℎ𝑥𝑑𝑦 + 2ℎ𝑦𝑑𝑥 = 0 or (𝑎𝑥 + ℎ𝑦 + 𝑔𝑧)𝑑𝑥 + (ℎ𝑥 + 𝑏𝑦 +
𝑓𝑧)𝑑𝑦 + (𝑔𝑥 + 𝑓𝑦 + 𝑐𝑧)𝑑𝑧 = 0.
Multiplying (3) by 1, (4) by 𝜆 and adding, and then equating the coefficients of 𝑑𝑥, 𝑑𝑦, 𝑑𝑧 to
zero, we have
𝑥 + 𝜆(𝑎𝑥 + ℎ𝑦 + 𝑔𝑧) = 0
, 𝑦 + 𝜆(ℎ𝑥 + 𝑏𝑦 + 𝑓𝑧) = 0,
𝑧 + 𝜆(𝑔𝑥 + 𝑓𝑦 + 𝑐𝑧) = 0.
Multiplying (5) by 𝑥, (6) by 𝑦, (7) by 𝑧 and adding, we get
𝑥 2 + 𝑦 2 + 𝑧 2 + 𝜆(𝑎𝑥 2 + 𝑏𝑦 2 + 𝑐𝑧 2 + 2𝑓𝑦𝑧 + 2𝑔𝑧𝑥 + 2ℎ𝑥𝑦) = 0
or 𝑢 + 𝜆. 1 = 0, using (1) and (2).
∴ 𝜆 = −𝑢
Hence from (5), we have
or

214 | P a g e

© Department of Distance & Continuing Education, Campus of Open Learning,


School of Open Learning, University of Delhi
Intermediate Mathematical Methods for Economics

𝑥 − 𝑢(𝑎𝑥 + ℎ𝑦 + 𝑔𝑧) = 0
𝑥(1 − 𝑎𝑢) − ℎ𝑢𝑦 − 𝑔𝑢𝑧 = 0
1
∫ (𝑎 − ) 𝑥 + ℎ𝑦 + 𝑔𝑧 = 0
𝑢
or
Hence from (5), we have
or
𝑥 − 𝑢(𝑎𝑥 + ℎ𝑦 + 𝑔𝑧) = 0
𝑥(1 − 𝑎𝑢) − ℎ𝑢𝑦 − 𝑔𝑢𝑧 = 0
1
(𝑎 − ) 𝑥 + ℎ𝑦 + 𝑔𝑧 = 0.
𝑢
or
Similarly from (6) and (7), we have
ℎ𝑥 + (𝑏 − 1/𝑢)𝑦 + 𝑓𝑧 = 0 and 𝑔𝑥 + 𝑓𝑦 + (𝑐 − 1/𝑢)𝑧 = 0
Eliminating 𝑥, 𝑦, 𝑧 from (8), (9), (10), we get
𝑎 − (1/𝑢) ℎ 𝑔
| ℎ 𝑏 − (1/𝑢) 𝑓 | = 0.
𝑔 𝑓 𝑐 − (1/𝑢)

Hence the required maximum or minimum values of 𝑢 are the roots of the equation (11).
Ex. 2. Find the stationary values of 𝑥 2 + 𝑦 2 + 𝑧 2 subject to the conditions 𝑎𝑥 2 + 𝑏𝑦 2 +
𝑐𝑧 2 = 1 and 𝑙𝑥 + 𝑚𝑦 + 𝑛𝑧 = 0
Interpret the result geometrically.
Sol. Let 𝑢 = 𝑥 2 + 𝑦 2 + 𝑧 2 ,
where the variables 𝑥, 𝑦 and 𝑧 are connected by the relations.
𝑎𝑥 2 + 𝑏𝑦 2 + 𝑐𝑧 2 = 1
and 𝑥 + 𝑚𝑦 + 𝑛𝑧 = 0.
For a stationary value of 𝑢, we have
𝑑𝑢 = 0
⇒ 2𝑥𝑑𝑥 + 2𝑦𝑑𝑦 + 2𝑧𝑑𝑧 = 0
⇒ 𝑥𝑑𝑥 + 𝑦𝑑𝑦 + 𝑧𝑑𝑧 = 0.

215 | P a g e

© Department of Distance & Continuing Education, Campus of Open Learning,


School of Open Learning, University of Delhi
B.A. (Hons.) Economics

Also differentiating the given relations (2) and (3), we get i.e.,
2𝑎𝑥𝑑𝑥 + 2𝑏𝑦𝑑𝑦 + 2𝑐𝑧𝑑𝑧 = 0
𝑎𝑥𝑑𝑥 + 𝑏𝑦𝑑𝑦 + 𝑐𝑧𝑑𝑧 = 0
{ }
and
𝑙𝑑𝑥 + 𝑚𝑑𝑦 + 𝑛𝑑𝑧 = 0.
Multiplying (4) by 1, (5) by 𝜆 and (6) by 𝜇 and adding, and then equating the coefficients of
𝑑𝑥, 𝑑𝑦, 𝑑𝑧 to zero, we get
𝑥 + 𝜆𝑎𝑥 + 𝜇𝑙 = 0,
and {𝑦 + 𝜆𝑏𝑦 + 𝜇𝑚 = 0,
𝑧 + 𝜆𝑐𝑧 + 𝜇𝑛 = 0.
Multiplying the equations (7), (8) and (9) by 𝑥, 𝑦 and 𝑧 respectively and adding, we get
𝑥 2 + 𝑦 2 + 𝑧 2 + 𝜆(𝑎𝑥 2 + 𝑏𝑦 2 + 𝑐𝑧 2 ) + 𝜇(𝑏𝑥 + 𝑚𝑦 + 𝑛𝑧) = 0
or 𝑢 + 𝜆. 1 + 𝜇 ⋅ 0 = 0, using (1), (2) and (3)
or 𝜆 = −𝑢.
Substituting for 𝜆 in the equations (7), (8) and (9), we get
𝜇𝑙 𝜇𝑚 𝜇𝑛
𝑥1 = ,𝑦 = ,𝑧 = .
𝑎𝑢 − 1 𝑏𝑢 − 1 𝑐𝑢 − .1
Substituting these values of 𝑥, 𝑦, 𝑧 in (3), we get
𝜇𝑙 2 𝜇𝑚2 𝜇𝑛2
+ + =0
𝑎𝑢 − 1 𝑏𝑢 − 1 𝑐𝑢 − 1
Geometrical interpretation. The surface 𝑎𝑥 2 + 𝑏𝑦 2 + 𝑐𝑧 2 = 1 represents an ellipsoid (or a
hyperboloid) whose centre is origin, and 𝑙𝑥 + 𝑚𝑦 + 𝑛𝑧 = 0 is a plane passing through the
origin. Therefore, the point (𝑥, 𝑦, 𝑧) satisfying both the conditions (2) and (3) lies on the conic
in which (2) and (3) intersect. Also 𝑥 2 + 𝑦 2 + 𝑧 2 gives the square of the distance of (𝑥, 𝑦, 𝑧)
from the origin which is also the centre of the conic of intersection. The maximum and
minimum values of this distance are the major and minor semi-axes of the conic. So the
equation (10) gives the squares of the lengths of the semi-axes of the conic of intersection.
SELF-ASSESSMENT QUESTIONS
1 𝑢 = 𝑥𝑦 + 𝑎3 (1/𝑥 + 1/𝑦).
2 𝑢 = 𝑥 3 𝑦 2 (1 − 𝑥 − 𝑦).
3 𝑢 = 𝑥 3 + 𝑦 3 − 3𝑎𝑥𝑦.

216 | P a g e

© Department of Distance & Continuing Education, Campus of Open Learning,


School of Open Learning, University of Delhi
Intermediate Mathematical Methods for Economics

4 𝑢 = 𝑥 2 + 𝑦 2 + 6𝑥 + 12.
5 𝑢 = 𝑥 2 + 𝑦 2 + 2/𝑥 + 2/𝑦.
6 𝑢 = 𝑦 2 + 4𝑥𝑦 + 3𝑥 2 + 𝑥 3 .
7 𝑢 = 3𝑥 2 − 𝑦 2 + 𝑥 3
8 𝑢 = 2𝑥 2 𝑦 + 𝑥 2 − 𝑦 2 + 2𝑦.
1 1
9 𝑢 = 2sin 2 (𝑥 + 𝑦)cos 2 (𝑥 − 𝑦) + cos (𝑥 + 𝑦).

10 𝑢 = sin 𝑥sin 𝑦sin (𝑥 + 𝑦).


[Hint: It is sufficient to consider the values of 𝑥 and 𝑦 between 0 and 𝜋 since the
function 𝑢 is periodic with period 𝜋 both for 𝑥 and 𝑦 ]
11 𝑢 = 𝑥 2 𝑦 2 − 5𝑥 2 − 8𝑥𝑦 − 5𝑦 2
12 𝑢 = 𝑥 4 + 2𝑥 2 𝑦 − 𝑥 2 + 3𝑦 2.
13 If 𝑢 = 𝑎𝑥 3 𝑦 2 − 𝑥 4 𝑦 2 − 𝑥 3 𝑦 3 , prove that 𝑥 = 𝑎/2, 𝑦 = 𝑎/3 make 𝑢 a maximum.
14 Investigate the maxima and minima of
2(𝑥 − 𝑦)2 − 𝑥 4 − 𝑦 4
leaving aside any doubtful case that may arise.
15 Examine the following surface for high and low points:
𝑧 = 𝑥 2 + 𝑥𝑦 + 3𝑥 + 2𝑦 + 5.
Find the saddle points of the surface if there are any.
16 Find all the stationary points of the function
examining whether they 𝑥 3 + 3𝑥𝑦 2 − 15𝑥 2 − 15𝑦 2 + 72𝑥
17 Find the values of 𝑥 and 𝑦 for maxima or minima.
(𝑎1 𝑥 + 𝑏1 𝑦 + 𝑐1 )2 + (𝑎2 𝑥 + 𝑏2 𝑦 + 𝑐2 )2 + ⋯ + (𝑎𝑛 𝑥 + 𝑏𝑛 𝑦 + 𝑐𝑛 )2
becomes a minimum.
18 Examine for maximum and minimum values of the function
𝑧 = 𝑥 2 − 3𝑥𝑦 + 𝑦 2 + 2𝑥
19. Find the points (𝑥, 𝑦) where the function 𝑥𝑦(1 − 𝑥 − 𝑦) is maximum or minimum.
What is the maximum value of the function?

217 | P a g e

© Department of Distance & Continuing Education, Campus of Open Learning,


School of Open Learning, University of Delhi
B.A. (Hons.) Economics

20. Examine the function 𝑧 = 𝑥 2 𝑦 − 𝑦 2 𝑥 − 𝑥 + 𝑦 for maxima and minima. Find the
saddle points of the given surface.
21. Let 𝑓(𝑥, 𝑦) = 𝑥 2 − 2𝑥𝑦 + 𝑦 2 + 𝑥 3 − 𝑦 3 + 𝑥 5 . Show that 𝑓(𝑥, 𝑦) has neither a
maximum nor a minimum at (0,0).
22. Find all the maxima and minima of the function
𝑓(𝑥, 𝑦) = 𝑦 2 + 𝑥 2 𝑦 + 𝑥 4
23. Find a point within a triangle such that the sum of the squares of its distances from
the three vertices is a minimum.
24. Find points on 𝑧 2 = 𝑥𝑦 + 1 nearest to the origin.
ANSWER
1 Minimum at 𝑥 = 𝑦 = 𝑎.
1 1
2 Maximum at 𝑥 = 2 , 𝑦 = 3.

3 𝑥 = 𝑦 = 𝑎 gives a maximum or a minimum according as 𝑎 is negative or positive.


4 Minimum at (−3,0).
5 Minimum at (1,1).
2 4
6 Minimum at 𝑥 = 3 , 𝑦 = − 3.

7 Maximum at (−2,0).
8 No extreme value.
9 Maximum when 𝑥 = 𝑦 = 𝑛𝜋 + (−1)𝑛 𝜋/6 and minimum when 𝑥 = 𝑦 = 2𝑛𝜋 − 𝜋/2.
10 Maximum at 𝑥 = 𝑦 = 𝜋/3 and minimum at 𝑥 = 𝑦 = 2𝜋/3.
11 Maximum at 𝑥 = 𝑦 = 0.
12 Minimum when 𝑥 = ±√3/2, 𝑦 = −1/4.

13 Maximum when 𝑥 = ±√2, 𝑦 = ∓√ 2.


14 No high and low points. The point (−2,1,3) is a saddle point.
15 Maximum at (4,0) and minimum at (6,0).
16 𝑥, 𝑦 satisfy the equations

218 | P a g e

© Department of Distance & Continuing Education, Campus of Open Learning,


School of Open Learning, University of Delhi
Intermediate Mathematical Methods for Economics

(Σ𝑎12 )𝑥 + (Σ𝑎1 𝑏1 )𝑦 + Σ(𝑎1 𝑐1) = 0


and (Σ𝑎1 𝑏1 )𝑥 + (Σ𝑏12 )𝑦 + Σ(𝑏1 𝑐1 ) = 0.
17 The function 𝑧 is stationary at the point (4/5,6/5). But it is neither maximum nor
minimum at this point.
1 1
19 Maximum at the point (3 , 3). Max. value = 1/27.

20 The function is stationary at the points (1,1) and (−1, −1), but it has no exreme
value. The points (1,1,0) and (−1, −1,0) are the two saddle points of the surface 𝑧 =
𝑥 2 𝑦 − 𝑦 2 𝑥 − 𝑥 + 𝑦.
21 The function is minimum at the point (0,0).
22 Centroid of the triangle.
23 (0,0,1) and (0,0, −1).
(OBJECTIVE QUESTIONS)
Fill In the Blanks.
Fill in the blanks "......" so that the following statements are complete and correct.
1) Let 𝑓(𝑥, 𝑦) be a function of two independent variables 𝑥 and 𝑦. The necessary
conditions for the existence of a maximum or a minimum of 𝑓(𝑥, 𝑦) at 𝑥 = 𝑎, 𝑦 = 𝑏
are
∂𝑓 ∂𝑓
= 0, and = ⋯ at 𝑥 = 𝑎, 𝑦 = 𝑏.
∂𝑥 ∂𝑦
2 Let 𝑓(𝑥, 𝑦) be a function of two independent variables 𝑥 and 𝑦. If at the point (𝑎, 𝑏),
we have
2
∂𝑓 ∂𝑓 ∂2 𝑓 ∂2 𝑓 ∂2 𝑓
= 0, = 0, 2 ⋅ 2 − ( ) >0
∂𝑥 ∂𝑦 ∂𝑥 ∂𝑦 ∂𝑥 ∂𝑦
∂2 𝑓
and ∂𝑥 2 > 0, then 𝑓(𝑥, 𝑦) is…… at (𝑎, 𝑏).

∂2 𝑓 ∂2 𝑓
3 Let 𝑓(𝑥, 𝑦) be a function of two independent variables 𝑥 and 𝑦. Let 𝑟 = ∂𝑥 2 , 𝑠 = ∂𝑥 ∂𝑦
∂2 𝑓
and 𝑡 = ∂𝑦 2.
∂𝑓 ∂𝑓
If at the point (𝑎, 𝑏), we have ∂𝑥 = 0, ∂𝑦 = 0, 𝑟𝑡 − 𝑠 2 > 0 and 𝑟 < 0, then 𝑓(𝑥, 𝑦) is
… at (𝑎, 𝑏).

219 | P a g e

© Department of Distance & Continuing Education, Campus of Open Learning,


School of Open Learning, University of Delhi
B.A. (Hons.) Economics

Multiple Choice Questions.


Indicate the correct answer for each question by writing the corresponding letter from
(a), (b), (c) and (d).
∂2 𝑓 ∂2 𝑓
4 Let 𝑓(𝑥, 𝑦) be a function of two independent variables 𝑥 and 𝑦. Let 𝑟 = ∂𝑥 2 , 𝑠 = ∂𝑥 ∂𝑦
∂2 𝑓
and 𝑡 = ∂𝑦 2.
∂𝑓 ∂𝑓
If at the point (𝑎, 𝑏), we have ∂𝑥 = 0 and ∂𝑦 = 0, then 𝑓(𝑥, 𝑦) is maximum at (𝑎, 𝑏) if
at (𝑎, 𝑏) we have
(a) 𝑟𝑡 − 𝑠 2 < 0
(b) 𝑟𝑡 − 𝑠 2 > 0 and 𝑟 < 0
(c) 𝑟𝑡 − 𝑠 2 > 0 and 𝑟 > 0
(d) 𝑟𝑡 − 𝑠 2 < 0 and 𝑟 < 0
5 The function 𝑢 = 𝑥 2 + 𝑦 2 + 6𝑥 + 12 is minimum at
(a) (3,0)
(b) (0,3)
(c) (−3,0)
(d) (3,3)
6. The function 𝑢 = 3𝑥 2 − 𝑦 2 + 𝑥 3 is maximum at (2,0)
(a) (−2,0)
(b) (2,0)
(c) (0,2)
(d) (0, −2)
True or False.
Write ' 𝑻 ' for thie and ' 𝑭 ' for false statement.
∂2 𝑓 ∂2 𝑓
7 Let 𝑓(𝑥, 𝑦) be a function of two independent variables 𝑥 and 𝑦. Let 𝑟 = ∂𝑥 2 , 𝑠 = ∂𝑥 ∂𝑦
∂2 𝑓
and 𝑡 = ∂𝑦 2
∂𝑓 ∂𝑓
If at the point (𝑎, 𝑏), we have ∂𝑥 = 0, ∂𝑦 = 0 and 𝑟𝑡 − 𝑠 2 < 0, then 𝑓(𝑥, 𝑦) has an
extreme value at (𝑎, 𝑏).

220 | P a g e

© Department of Distance & Continuing Education, Campus of Open Learning,


School of Open Learning, University of Delhi
Intermediate Mathematical Methods for Economics

8 Let 𝑓(𝑥, 𝑦) be a fúnction of two independent variables 𝑥 and 𝑦. If at (𝑎, 𝑏), we have
∂𝑓 ∂𝑓
= 0 and ∂𝑦 = 0, then 𝑓(𝑥, 𝑦) must have a maximum or a minimum at (𝑎, 𝑏)
∂𝑥

𝑝2
9 The minimum value of 𝑥 2 + 𝑦 2 + 𝑧 2 when 𝑎𝑥 + 𝑏𝑦 + 𝑐𝑧 = 𝑝 is (𝑎2+𝑏2+𝑐 2).
1 1
10 The function 𝑥𝑦(1 − 𝑥 − 𝑦) has a maximum value at the point (2 , 2).

11 The function 𝑥 3 + 𝑦 3 + 3𝑥𝑦 has a maximum value at the point (−1, −1).
12 The minimum value of the function 𝑥 3 + 𝑦 3 − 6𝑥𝑦 is -8.

Answers
1 0.
2 minimum.
3 maximum.
4 (b).
5 (c). 6. (a).
6 𝐹.
7 𝐹.
8 𝑇. 10. 𝐹.
9 𝑇. 12. 𝑇.
Self-Assessment Questions
1. Show that
𝑢 = (𝑥 + 𝑦 + 𝑧)3 − 3(𝑥 + 𝑦 + 𝑧) − 24𝑥𝑦𝑧 + 𝑎3
has minimum at (1,1,1) and maximum at (−1, −1, −1).
2. Find the maximum or minimum values of 𝑢 where
𝑢 = 𝑎𝑥𝑦 2 𝑧 3 − 𝑥 2 𝑦 2 𝑧 3 − 𝑥𝑦 3 𝑧 3 − 𝑥𝑦 2 𝑧 4
3. Find the maximum value of
2 𝑥 2 +𝛽 2 𝑦 2 +𝛾 2 𝑧 2 )
(𝑎𝑥 + 𝑏𝑦 + 𝑐𝑧) ⋅ 𝑒 −(𝛼
Answers
𝑎 2𝑎 3𝑎 108𝑎7
2. 𝑢 is maximum when 𝑥 = 7 , 𝑦 = ,𝑧 = and the maximum value is
7 7 7

221 | P a g e

© Department of Distance & Continuing Education, Campus of Open Learning,


School of Open Learning, University of Delhi
B.A. (Hons.) Economics

1 𝑎2 𝑐2
3. √2𝑒 (𝛼2 + 𝛽2 + 𝛾2 )}

12.4 SUMMARY

The main points which we have covered in these lessons are what is estimator and what is
consistency, efficiency and sufficiency of the estimator and how to get best estimator.

12.5 GLOSSARY

Motivation: These Problems are very useful in real life and we can use it in data science,
economics as well as social science.
Attention: Think how the best estimator are useful in real world problems.

12.6 REFERENCES

• Sydsaeter, K., Hammond, P. (2002). Mathematics for economics analysis. Pearson


Education.

• Hoy, M., Livernois, J., McKenna, C., Rees, R., Stengos, T, (2001). Mathematics for
Economics, Prentice-Hall India.

12.7 SUGGESTED READINGS

• A.R Vasishtha, Anurag Sharma, Dr. Vipin Vasishtha, Neenu Agarwal, Dr A.K
Vasishtha, Advanced Calculus, Krishna Publication, 5th Edition.
• A.R Vasishtha, Anurag Sharma, Dr. Vipin Vasishtha, Anil Kumar, Dr A.K Vasishtha,
Analysis, Krishna Publication, 3rd Edition.

222 | P a g e

© Department of Distance & Continuing Education, Campus of Open Learning,


School of Open Learning, University of Delhi
978-81-19169-61-0

9 788119 169610

You might also like